Каково ускорСниС: β„– 176. Каково ускорСниС свободного падСния Π½Π° высотС, Ρ€Π°Π²Π½ΠΎΠΉ ΠΏΠΎΠ»ΠΎΠ²ΠΈΠ½Π΅ радиуса Π—Π΅ΠΌΠ»ΠΈ?

2\]

Π‘ΠΎΠ΄Π΅Ρ€ΠΆΠ°Π½ΠΈΠ΅

ΠžΡ‚Π²Π΅Ρ‚: 264,6 ΠΌ/с

2.

Если Π’Ρ‹ Π½Π΅ поняли Ρ€Π΅ΡˆΠ΅Π½ΠΈΠ΅ ΠΈ Ρƒ Вас Π΅ΡΡ‚ΡŒ ΠΊΠ°ΠΊΠΎΠΉ-Ρ‚ΠΎ вопрос ΠΈΠ»ΠΈ Π’Ρ‹ нашли ΠΎΡˆΠΈΠ±ΠΊΡƒ, Ρ‚ΠΎ смСло оставляйтС Π½ΠΈΠΆΠ΅ ΠΊΠΎΠΌΠΌΠ΅Π½Ρ‚Π°Ρ€ΠΈΠΉ.

Π˜Π·ΠΌΠ΅Ρ€Π΅Π½ΠΈΠ΅ ускорСния свободного падСния Π½Π° Ρ€Π°Π·Π»ΠΈΡ‡Π½Ρ‹Ρ… высотах ΠΏΡ€ΠΈ ΠΏΠΎΠΌΠΎΡ‰ΠΈ матСматичСского маятника

  • Участник: МингалССв Артур Π­Π΄ΡƒΠ°Ρ€Π΄ΠΎΠ²ΠΈΡ‡
  • Π ΡƒΠΊΠΎΠ²ΠΎΠ΄ΠΈΡ‚Π΅Π»ΡŒ: Баскова ΠœΠ°Ρ€ΠΈΡ ΠΡ€ΠΊΠ°Π΄ΡŒΠ΅Π²Π½Π°
ЦСль настоящСго исслСдования состояла Π² ΠΏΠΎΠ»ΡƒΡ‡Π΅Π½ΠΈΠΈ значСния ускорСния свободного падСния ΠΏΡ€ΠΈ ΠΏΠΎΠΌΠΎΡ‰ΠΈ матСматичСского маятника Π² условиях Ρ€Π°Π·Π½ΠΎΠ³ΠΎ уровня высоты Π½Π° ΡƒΡ€ΠΎΠ²Π½Π΅ΠΌ моря.

1. Π’Π²Π΅Π΄Π΅Π½ΠΈΠ΅

ΠŸΠ΅Ρ€Π²Ρ‹ΠΌ Ρ‡Π΅Π»ΠΎΠ²Π΅ΠΊΠΎΠΌ, ΠΈΠ·ΡƒΡ‡Π°Π²ΡˆΠΈΠΌ ΠΏΡ€ΠΈΡ€ΠΎΠ΄Ρƒ падСния Ρ‚Π΅Π», Π±Ρ‹Π» грСчСский ΡƒΡ‡Π΅Π½Ρ‹ΠΉ ΠΡ€ΠΈΡΡ‚ΠΎΡ‚Π΅Π»ΡŒ. Π—Π°Ρ‚Π΅ΠΌ Π“Π°Π»ΠΈΠ»Π΅ΠΎ Π“Π°Π»ΠΈΠ»Π΅ΠΉ ΠΎΠ±ΠΎΠ±Ρ‰ΠΈΠ» ΠΈ Π½Π΅ ΠΏΡ€ΠΎΠ°Π½Π°Π»ΠΈΠ·ΠΈΡ€ΠΎΠ²Π°Π» ΠΎΠΏΡ‹Ρ‚ ΠΈ экспСримСнты Π½Π΅ΡΠΊΠΎΠ»ΡŒΠΊΠΈΡ… ΠΏΠΎΠΊΠΎΠ»Π΅Π½ΠΈΠΉ исслСдоватСлСй. Он ΠΏΡ€Π΅Π΄ΠΏΠΎΠ»ΠΎΠΆΠΈΠ», Ρ‡Ρ‚ΠΎ Π² срСдС, свободной ΠΎΡ‚ Π²ΠΎΠ·Π΄ΡƒΡ…Π°, всС Ρ‚Π΅Π»Π° Π±ΡƒΠ΄ΡƒΡ‚ ΠΏΠ°Π΄Π°Ρ‚ΡŒ с ΠΎΠ΄ΠΈΠ½Π°ΠΊΠΎΠ²ΠΎΠΉ ΡΠΊΠΎΡ€ΠΎΡΡ‚ΡŒΡŽ. Π’Π°ΠΊΠΆΠ΅ Π“Π°Π»ΠΈΠ»Π΅ΠΉ ΠΏΡ€Π΅Π΄ΠΏΠΎΠ»ΠΎΠΆΠΈΠ», Ρ‡Ρ‚ΠΎ Π²ΠΎ врСмя падСния ΡΠΊΠΎΡ€ΠΎΡΡ‚ΡŒ Ρ‚Π΅Π» постоянно увСличиваСтся. Π­ΠΊΡΠΏΠ΅Ρ€ΠΈΠΌΠ΅Π½Ρ‚ΠΈΡ€ΠΎΠ²Π°Ρ‚ΡŒ со свободным ΠΏΠ°Π΄Π΅Π½ΠΈΠ΅ΠΌ Ρ‚Π΅Π» ΠΏΡ€ΠΎΠ΄ΠΎΠ»ΠΆΠΈΠ» Исаак ΠΡŒΡŽΡ‚ΠΎΠ½. Π’ Π΅Π³ΠΎ Π²Ρ‹Π²ΠΎΠ΄Π°Ρ… прослСТиваСтся ΠΌΡ‹ΡΠ»ΡŒ, Ρ‡Ρ‚ΠΎ Π½Π° Π›ΡƒΠ½Π΅ ΠΈ Π½Π° Π΄Ρ€ΡƒΠ³ΠΈΡ… ΠΏΠ»Π°Π½Π΅Ρ‚Π°Ρ… сила тяТСсти, Π²ΠΎΠ·Π΄Π΅ΠΉΡΡ‚Π²ΡƒΡŽΡ‰Π°Ρ Π½Π° ΠΎΠ΄Π½ΠΎ ΠΈ Ρ‚ΠΎ ΠΆΠ΅ Ρ‚Π΅Π»ΠΎ, Π±ΡƒΠ΄Π΅Ρ‚ Π½Π΅ΠΎΠ΄ΠΈΠ½Π°ΠΊΠΎΠ²Π°, зависит ΠΎΠ½Π° Π½Π°ΠΏΡ€ΡΠΌΡƒΡŽ ΠΎΡ‚ массы космичСского Ρ‚Π΅Π»Π°. НапримСр, ускорСниС g Π½Π° Π›ΡƒΠ½Π΅ Π² нСсколько Ρ€Π°Π· мСньшС, Ρ‡Π΅ΠΌ Π½Π° Π—Π΅ΠΌΠ»Π΅. Π’Π°ΠΊΠΈΠΌ ΠΎΠ±Ρ€Π°Π·ΠΎΠΌ, зная массу ΠΏΠ»Π°Π½Π΅Ρ‚Ρ‹, ΠΌΠΎΠΆΠ½ΠΎ Π²Ρ‹Ρ‡ΠΈΡΠ»ΠΈΡ‚ΡŒ ускорСниС свободного падСния Ρ‚Π΅Π»Π° Π½Π° этой ΠΏΠ»Π°Π½Π΅Ρ‚Π΅.

ЦСль настоящСго исслСдования состояла Π² ΠΏΠΎΠ»ΡƒΡ‡Π΅Π½ΠΈΠΈ значСния ускорСния свободного падСния ΠΏΡ€ΠΈ ΠΏΠΎΠΌΠΎΡ‰ΠΈ матСматичСского маятника Π² условиях Ρ€Π°Π·Π½ΠΎΠ³ΠΎ уровня высоты Π½Π° ΡƒΡ€ΠΎΠ²Π½Π΅ΠΌ моря. Для достиТСния поставлСнной Ρ†Π΅Π»ΠΈ Π±Ρ‹Π»ΠΈ сформулированы ΡΠ»Π΅Π΄ΡƒΡŽΡ‰ΠΈΠ΅ Π·Π°Π΄Π°Ρ‡ΠΈ исслСдования:

  1. ΠžΠ·Π½Π°ΠΊΠΎΠΌΠΈΡ‚ΡŒΡΡ с историСй открытия свободного падСния Ρ‚Π΅Π»;
  2. Π˜Π·ΡƒΡ‡ΠΈΡ‚ΡŒ ΠΌΠ΅Ρ‚ΠΎΠ΄Ρ‹ измСрСния ускорСния свободного падСния Π½Π° повСрхности Π—Π΅ΠΌΠ»ΠΈ;
  3. ΠŸΡ€ΠΎΠ²Π΅ΡΡ‚ΠΈ ΡΠ°ΠΌΠΎΡΡ‚ΠΎΡΡ‚Π΅Π»ΡŒΠ½Ρ‹Π΅ измСрСния ускорСния свободного падСния ΠΏΡ€ΠΈ ΠΏΠΎΠΌΠΎΡ‰ΠΈ матСматичСского маятника;
  4. ΠŸΡ€ΠΎΠ²Π΅ΡΡ‚ΠΈ измСрСния Π½Π° Ρ€Π°Π·Π»ΠΈΡ‡Π½Ρ‹Ρ… высотах.

Π“ΠΈΠΏΠΎΡ‚Π΅Π·Π° исслСдования: Π»ΠΎΠ³ΠΈΡ‡Π½ΠΎ ΠΏΡ€Π΅Π΄ΠΏΠΎΠ»ΠΎΠΆΠΈΡ‚ΡŒ, Ρ‡Ρ‚ΠΎ ускорСниС свободного падСния, ΠΏΠΎΠ»ΡƒΡ‡Π΅Π½Π½Ρ‹Π΅ Π² Ρ€Π°Π·Π½Ρ‹Ρ… экспСримСнтах, Π΄ΠΎΠ»ΠΆΠ½Ρ‹ Π±Ρ‹Ρ‚ΡŒ Π±Π»ΠΈΠ·ΠΊΠΈ ΠΊ Π·Π½Π°Ρ‡Π΅Π½ΠΈΡŽ 9,8 ΠΌ/с

2 ΠΈ ΠΎΡ‚Π»ΠΈΡ‡Π°Ρ‚ΡŒΡΡ Π½Π° сотыС ΠΈΠ»ΠΈ тысячныС Π΄ΠΎΠ»ΠΈ Π½Π° Π³Π»ΡƒΠ±ΠΈΠ½Π΅ станции ΠΌΠ΅Ρ‚Ρ€ΠΎ ΠšΡ€Π΅ΠΌΠ»Π΅Π²ΡΠΊΠ°Ρ (–34 ΠΌ) ΠΈ Π½Π° высотС нСбоскрСба Β«Π›Π°Π·ΡƒΡ€Π½Ρ‹Π΅ нСбСса» (+120 ΠΌ). Π’Π°ΠΊΠΆΠ΅ Ρ€Π΅Π·ΡƒΠ»ΡŒΡ‚Π°Ρ‚Ρ‹ ΠΈΠ·ΠΌΠ΅Ρ€Π΅Π½ΠΈΠΉ ΠΈ вычислСний ΠΌΠΎΠ³ΡƒΡ‚ ΠΎΡ‚Π»ΠΈΡ‡Π°Ρ‚ΡŒΡΡ ΠΏΠΎΠ³Ρ€Π΅ΡˆΠ½ΠΎΡΡ‚ΡŒΡŽ ΠΈΠ·ΠΌΠ΅Ρ€Π΅Π½ΠΈΠΉ.

ΠœΠ΅Ρ‚ΠΎΠ΄Ρ‹ изучСния: ΡΠ°ΠΌΠΎΡΡ‚ΠΎΡΡ‚Π΅Π»ΡŒΠ½Π°Ρ, ΠΈΠ½Π΄ΠΈΠ²ΠΈΠ΄ΡƒΠ°Π»ΡŒΠ½Π°Ρ Ρ€Π°Π±ΠΎΡ‚Π° Π² сочСтании с тСорСтичСскими ΠΈΡΡΠ»Π΅Π΄ΠΎΠ²Π°Ρ‚Π΅Π»ΡŒΡΠΊΠΈΠΌΠΈ, ΠΏΡ€ΠΎΠ΅ΠΊΡ‚Π½Ρ‹ΠΌΠΈ Ρ„ΠΎΡ€ΠΌΠ°ΠΌΠΈ Ρ€Π°Π±ΠΎΡ‚Ρ‹.

Читая ΠΌΠ½ΠΎΠ³ΠΎ Ρ€Π°Π·Π»ΠΈΡ‡Π½ΠΎΠΉ Π² Ρ‚ΠΎΠΌ числС ΠΈ тСхничСской Π»ΠΈΡ‚Π΅Ρ€Π°Ρ‚ΡƒΡ€Ρ‹, я ΡƒΠ·Π½Π°Π» ΠΎ практичСском ΠΏΡ€ΠΈΠΌΠ΅Π½Π΅Π½ΠΈΠΈ различия ускорСния свободного падСния Π² Ρ€Π°Π·Π½Ρ‹Ρ… Ρ‚ΠΎΡ‡ΠΊΠ°Ρ… Π½Π° повСрхности Π—Π΅ΠΌΠ»ΠΈ. Π― измСрял g Ρ€Π°Π·Π»ΠΈΡ‡Π½Ρ‹ΠΌΠΈ способами, рассчитывал ΠΏΠΎΠ³Ρ€Π΅ΡˆΠ½ΠΎΡΡ‚ΠΈ ΠΈΠ·ΠΌΠ΅Ρ€Π΅Π½ΠΈΠΉ, ΠΎΠΏΠΈΡ€Π°ΡΡΡŒ Π½Π° общСпринятоС Π·Π½Π°Ρ‡Π΅Π½ΠΈΠ΅ g, учился Π³Ρ€Π°ΠΌΠΎΡ‚Π½ΠΎ ΠΏΡ€ΠΎΠ²ΠΎΠ΄ΠΈΡ‚ΡŒ экспСримСнт. Выяснил, Ρ‡Ρ‚ΠΎ свободноС ΠΏΠ°Π΄Π΅Π½ΠΈΠ΅ – Π΄Π²ΠΈΠΆΠ΅Π½ΠΈΠ΅ равноускорСнноС. УскорСниС свободного падСния Π½Π΅ зависит ΠΎΡ‚ массы Ρ‚Π΅Π»Π°.

Π“ΠΈΠΏΠΎΡ‚Π΅Π·Ρƒ ΠΎ Ρ‚ΠΎΠΌ, Ρ‡Ρ‚ΠΎ значСния ускорСния свободного падСния Π΄ΠΎΠ»ΠΆΠ½Ρ‹ Π±Ρ‹Ρ‚ΡŒ Π±Π»ΠΈΠ·ΠΊΠΈ ΠΊ Π·Π½Π°Ρ‡Π΅Π½ΠΈΡŽ 9,8 ΠΌ/с2 ΠΈ ΠΎΡ‚Π»ΠΈΡ‡Π°Ρ‚ΡŒΡΡ Ρ‚ΠΎΠ»ΡŒΠΊΠΎ ΠΏΠΎΠ³Ρ€Π΅ΡˆΠ½ΠΎΡΡ‚ΡŒΡŽ ΠΈΠ·ΠΌΠ΅Ρ€Π΅Π½ΠΈΠΉ ΡƒΠ΄Π°Π»ΠΎΡΡŒ ΠΏΠΎΠ΄Ρ‚Π²Π΅Ρ€Π΄ΠΈΡ‚ΡŒ Ρ€Π°Π·Π½Ρ‹ΠΌΠΈ экспСримСнтами. НаиболСС Ρ‚ΠΎΡ‡Π½Ρ‹ΠΉ Ρ€Π΅Π·ΡƒΠ»ΡŒΡ‚Π°Ρ‚ ускорСния свободного падСния Ρƒ мСня получился с ΠΏΠΎΠΌΠΎΡ‰ΡŒΡŽ матСматичСского маятника. ΠŸΠΎΡΡ‚ΠΎΠΌΡƒ для исслСдования измСнСния значСния ускорСния свободного падСния с высотой я Π²Ρ‹Π±Ρ€Π°Π» ΠΈΠΌΠ΅Π½Π½ΠΎ этот способ измСрСния. ΠŸΠΎΠ³Ρ€Π΅ΡˆΠ½ΠΎΡΡ‚ΡŒ составила Π½Π΅ Π±ΠΎΠ»Π΅Π΅ 10%.

Π’ дальнСйшСм я Ρ…ΠΎΡ‚Π΅Π» Π±Ρ‹ ΡΠ°ΠΌΠΎΡΡ‚ΠΎΡΡ‚Π΅Π»ΡŒΠ½ΠΎ ΠΈΡΡΠ»Π΅Π΄ΠΎΠ²Π°Ρ‚ΡŒ Π·Π°Π²ΠΈΡΠΈΠΌΠΎΡΡ‚ΡŒ значСния ускорСния свободного падСния ΠΎΡ‚ гСографичСского полоТСния.

2. Основная Ρ‡Π°ΡΡ‚ΡŒ

2.1. Π˜ΡΡ‚ΠΎΡ€ΠΈΡ‡Π΅ΡΠΊΠΈΠ΅ свСдСния ΠΎΠ± ΠΎΡ‚ΠΊΡ€Ρ‹Ρ‚ΠΈΠΈ свободного падСния ΠΈ ΠΌΠ΅Ρ‚ΠΎΠ΄Π°Ρ… Π΅Π³ΠΎ измСрСния

Π•Ρ‰Π΅ тысячСлСтия Π½Π°Π·Π°Π΄ люди Π·Π°ΠΌΠ΅Ρ‡Π°Π»ΠΈ, Ρ‡Ρ‚ΠΎ большая Ρ‡Π°ΡΡ‚ΡŒ ΠΏΡ€Π΅Π΄ΠΌΠ΅Ρ‚ΠΎΠ² ΠΏΠ°Π΄Π°Π΅Ρ‚ всС быстрСС ΠΈ быстрСС, Π° Π½Π΅ΠΊΠΎΡ‚ΠΎΡ€Ρ‹Π΅ ΠΏΠ°Π΄Π°ΡŽΡ‚ Ρ€Π°Π²Π½ΠΎΠΌΠ΅Ρ€Π½ΠΎ. Но ΠΊΠ°ΠΊ ΠΈΠΌΠ΅Π½Π½ΠΎ ΠΏΠ°Π΄Π°ΡŽΡ‚ эти ΠΏΡ€Π΅Π΄ΠΌΠ΅Ρ‚Ρ‹ – этот вопрос ΠΏΠ΅Ρ€Π²ΠΎΠ±Ρ‹Ρ‚Π½Ρ‹Ρ… людСй Π½Π΅ Π·Π°Π½ΠΈΠΌΠ°Π». Π’Π΅ΠΌ Π½Π΅ ΠΌΠ΅Π½Π΅Π΅ нашлись люди, ΠΊΠΎΡ‚ΠΎΡ€Ρ‹Π΅ ΠΏΠΎ ΠΌΠ΅Ρ€Π΅ возмоТностСй Π½Π°Ρ‡Π°Π»ΠΈ ΠΈΡΡΠ»Π΅Π΄ΠΎΠ²Π°Ρ‚ΡŒ это явлСниС.

Π‘Π½Π°Ρ‡Π°Π»Π° ΠΎΠ½ΠΈ ΠΏΡ€ΠΎΠ΄Π΅Π»Ρ‹Π²Π°Π»ΠΈ ΠΎΠΏΡ‹Ρ‚Ρ‹ с двумя ΠΏΡ€Π΅Π΄ΠΌΠ΅Ρ‚Π°ΠΌΠΈ. НапримСр, Π±Ρ€Π°Π»ΠΈ Π΄Π²Π° камня, ΠΈ Π΄Π°Π²Π°Π»ΠΈ Π²ΠΎΠ·ΠΌΠΎΠΆΠ½ΠΎΡΡ‚ΡŒ ΠΈΠΌ свободно ΠΏΠ°Π΄Π°Ρ‚ΡŒ, выпустив ΠΈΡ… ΠΈΠ· Ρ€ΡƒΠΊ ΠΎΠ΄Π½ΠΎΠ²Ρ€Π΅ΠΌΠ΅Π½Π½ΠΎ. Π—Π°Ρ‚Π΅ΠΌ снова бросали Π΄Π²Π° камня, Π½ΠΎ ΡƒΠΆΠ΅ Π² стороны ΠΏΠΎ Π³ΠΎΡ€ΠΈΠ·ΠΎΠ½Ρ‚Π°Π»ΠΈ. ΠŸΠΎΡ‚ΠΎΠΌ бросали ΠΎΠ΄ΠΈΠ½ камСнь Π² сторону, ΠΈ Π² Ρ‚ΠΎΡ‚ ΠΆΠ΅ ΠΌΠΎΠΌΠ΅Π½Ρ‚ выпускали ΠΈΠ· Ρ€ΡƒΠΊ Π²Ρ‚ΠΎΡ€ΠΎΠΉ, Π½ΠΎ Ρ‚Π°ΠΊ, Ρ‡Ρ‚ΠΎΠ±Ρ‹ ΠΎΠ½ просто ΠΏΠ°Π΄Π°Π» ΠΏΠΎ Π²Π΅Ρ€Ρ‚ΠΈΠΊΠ°Π»ΠΈ. Π›ΡŽΠ΄ΠΈ ΠΈΠ·Π²Π»Π΅ΠΊΠ»ΠΈ ΠΈΠ· Ρ‚Π°ΠΊΠΈΡ… ΠΎΠΏΡ‹Ρ‚ΠΎΠ² ΠΌΠ½ΠΎΠ³ΠΎ свСдСний ΠΎ ΠΏΡ€ΠΈΡ€ΠΎΠ΄Π΅. Из ΠΎΠΏΡ‹Ρ‚ΠΎΠ² с ΠΏΠ°Π΄Π°ΡŽΡ‰ΠΈΠΌΠΈ Ρ‚Π΅Π»Π°ΠΌΠΈ люди установили, Ρ‡Ρ‚ΠΎ малСнький ΠΈ большой ΠΊΠ°ΠΌΠ½ΠΈ, Π²Ρ‹ΠΏΡƒΡ‰Π΅Π½Π½Ρ‹Π΅ ΠΈΠ· Ρ€ΡƒΠΊ ΠΎΠ΄Π½ΠΎΠ²Ρ€Π΅ΠΌΠ΅Π½Π½ΠΎ, ΠΏΠ°Π΄Π°ΡŽΡ‚ с ΠΎΠ΄ΠΈΠ½Π°ΠΊΠΎΠ²ΠΎΠΉ ΡΠΊΠΎΡ€ΠΎΡΡ‚ΡŒΡŽ. Π’ΠΎ ΠΆΠ΅ самоС ΠΌΠΎΠΆΠ½ΠΎ ΡΠΊΠ°Π·Π°Ρ‚ΡŒ ΠΎ кусках свинца, Π·ΠΎΠ»ΠΎΡ‚Π°, ΠΆΠ΅Π»Π΅Π·Π°, стСкла, ΠΈ Ρ‚.Π΄. самых Ρ€Π°Π·Π½Ρ‹Ρ… Ρ€Π°Π·ΠΌΠ΅Ρ€ΠΎΠ². Из ΠΏΠΎΠ΄ΠΎΠ±Π½Ρ‹Ρ… ΠΎΠΏΡ‹Ρ‚ΠΎΠ² Π²Ρ‹Π²ΠΎΠ΄ΠΈΡ‚ΡŒΡΡ простоС ΠΎΠ±Ρ‰Π΅Π΅ ΠΏΡ€Π°Π²ΠΈΠ»ΠΎ: свободноС ΠΏΠ°Π΄Π΅Π½ΠΈΠ΅ всСх Ρ‚Π΅Π» происходит ΠΎΠ΄ΠΈΠ½Π°ΠΊΠΎΠ²ΠΎ нСзависимо ΠΎΡ‚ Ρ€Π°Π·ΠΌΠ΅Ρ€Π° ΠΈ ΠΌΠ°Ρ‚Π΅Ρ€ΠΈΠ°Π»Π°, ΠΈΠ· ΠΊΠΎΡ‚ΠΎΡ€ΠΎΠ³ΠΎ Ρ‚Π΅Π»Π° сдСланы. ΠœΠ΅ΠΆΠ΄Ρƒ наблюдСниСм Π·Π° ΠΏΡ€ΠΈΡ‡ΠΈΠ½Π½ΠΎΠΉ связью явлСний ΠΈ Ρ‚Ρ‰Π°Ρ‚Π΅Π»ΡŒΠ½ΠΎ Π²Ρ‹ΠΏΠΎΠ»Π½Π΅Π½Π½Ρ‹ΠΌΠΈ экспСримСнтами, вСроятно, Π΄ΠΎΠ»Π³ΠΎ сущСствовал Ρ€Π°Π·Ρ€Ρ‹Π². Π”Π²Π΅ тысячи Π»Π΅Ρ‚ Π½Π°Π·Π°Π΄ Π½Π΅ΠΊΠΎΡ‚ΠΎΡ€Ρ‹Π΅ Π΄Ρ€Π΅Π²Π½ΠΈΠ΅ ΡƒΡ‡Π΅Π½Ρ‹Π΅, ΠΏΠΎ-Π²ΠΈΠ΄ΠΈΠΌΠΎΠΌΡƒ, ΠΏΡ€ΠΎΠ²ΠΎΠ΄ΠΈΠ»ΠΈ Π²ΠΏΠΎΠ»Π½Π΅ Ρ€Π°Π·ΡƒΠΌΠ½Ρ‹Π΅ ΠΎΠΏΡ‹Ρ‚Ρ‹ с ΠΏΠ°Π΄Π°ΡŽΡ‰ΠΈΠΌΠΈ Ρ‚Π΅Π»Π°ΠΌΠΈ.
Π’Π΅Π»ΠΈΠΊΠΈΠΉ грСчСский философ ΠΈ ΡƒΡ‡Π΅Π½Ρ‹ΠΉ ΠΡ€ΠΈΡΡ‚ΠΎΡ‚Π΅Π»ΡŒ, ΠΏΠΎ-Π²ΠΈΠ΄ΠΈΠΌΠΎΠΌΡƒ придСрТивался распространСнного прСдставлСния ΠΎ Ρ‚ΠΎΠΌ, Ρ‡Ρ‚ΠΎ тяТСлыС Ρ‚Π΅Π»Π° ΠΏΠ°Π΄Π°ΡŽΡ‚ быстрСС, Ρ‡Π΅ΠΌ Π»Π΅Π³ΠΊΠΈΠ΅. ΠΡ€ΠΈΡΡ‚ΠΎΡ‚Π΅Π»ΡŒ ΠΈ Π΅Π³ΠΎ послСдоватСли ΡΡ‚Ρ€Π΅ΠΌΠΈΠ»ΠΈΡΡŒ ΠΎΠ±ΡŠΡΡΠ½ΠΈΡ‚ΡŒ, ΠΏΠΎΡ‡Π΅ΠΌΡƒ происходят Ρ‚Π΅ ΠΈΠ»ΠΈ ΠΈΠ½Ρ‹Π΅ явлСния, Π½ΠΎ Π½Π΅ всСгда Π·Π°Π±ΠΎΡ‚ΠΈΠ»ΠΈΡΡŒ ΠΎ Ρ‚ΠΎΠΌ, Ρ‡Ρ‚ΠΎΠ±Ρ‹ ΠΏΡ€ΠΎΠ½Π°Π±Π»ΡŽΠ΄Π°Ρ‚ΡŒ, Ρ‡Ρ‚ΠΎ происходит ΠΈ ΠΊΠ°ΠΊ происходит. Он Π³ΠΎΠ²ΠΎΡ€ΠΈΠ», Ρ‡Ρ‚ΠΎ Ρ‚Π΅Π»Π° стрСмятся Π½Π°ΠΉΡ‚ΠΈ своС СстСствСнноС мСсто Π½Π° повСрхности Π—Π΅ΠΌΠ»ΠΈ. Π’ XIV столСтии Π³Ρ€ΡƒΠΏΠΏΠ° философов ΠΈΠ· ΠŸΠ°Ρ€ΠΈΠΆΠ° восстала ΠΏΡ€ΠΎΡ‚ΠΈΠ² Ρ‚Π΅ΠΎΡ€ΠΈΠΈ АристотСля ΠΈ ΠΏΡ€Π΅Π΄Π»ΠΎΠΆΠΈΠ»Π° Π·Π½Π°Ρ‡ΠΈΡ‚Π΅Π»ΡŒΠ½ΠΎ Π±ΠΎΠ»Π΅Π΅ Ρ€Π°Π·ΡƒΠΌΠ½ΡƒΡŽ схСму, которая ΠΏΠ΅Ρ€Π΅Π΄Π°Π²Π°Π»Π°ΡΡŒ ΠΈΠ· поколСния Π² ΠΏΠΎΠΊΠΎΠ»Π΅Π½ΠΈΠ΅ ΠΈ Ρ€Π°ΡΠΏΡ€ΠΎΡΡ‚Ρ€Π°Π½ΠΈΠ»Π°ΡΡŒ Π΄ΠΎ Π˜Ρ‚Π°Π»ΠΈΠΈ, ΠΎΠΊΠ°Π·Π°Π² двумя столСтиями ΠΏΠΎΠ·Π΄Π½Π΅Π΅ влияниС Π½Π° ГалилСя. ΠŸΠ°Ρ€ΠΈΠΆΡΠΊΠΈΠ΅ философы Π³ΠΎΠ²ΠΎΡ€ΠΈΠ»ΠΈ ΠΎΠ± ускорСнном Π΄Π²ΠΈΠΆΠ΅Π½ΠΈΠΈ ΠΈ Π΄Π°ΠΆΠ΅ ΠΎ постоянном ускорСнии, объясняя эти понятия Π°Ρ€Ρ…Π°ΠΈΡ‡Π½Ρ‹ΠΌ языком. Π’Π΅Π»ΠΈΠΊΠΈΠΉ ΠΈΡ‚Π°Π»ΡŒΡΠ½ΡΠΊΠΈΠΉ ΡƒΡ‡Π΅Π½Ρ‹ΠΉ Π“Π°Π»ΠΈΠ»Π΅ΠΎ Π“Π°Π»ΠΈΠ»Π΅ΠΉ ΠΎΠ±ΠΎΠ±Ρ‰ΠΈΠ» ΠΈΠΌΠ΅ΡŽΡ‰ΠΈΠ΅ΡΡ свСдСния ΠΈ прСдставлСния ΠΈ критичСски ΠΈΡ… ΠΏΡ€ΠΎΠ°Π½Π°Π»ΠΈΠ·ΠΈΡ€ΠΎΠ²Π°Π», Π° Π·Π°Ρ‚Π΅ΠΌ описал ΠΈ Π½Π°Ρ‡Π°Π» Ρ€Π°ΡΠΏΡ€ΠΎΡΡ‚Ρ€Π°Π½ΡΡ‚ΡŒ Ρ‚ΠΎ, Ρ‡Ρ‚ΠΎ считал Π²Π΅Ρ€Π½Ρ‹ΠΌ. Π“Π°Π»ΠΈΠ»Π΅ΠΉ ΠΏΠΎΠ½ΠΈΠΌΠ°Π», Ρ‡Ρ‚ΠΎ послСдоватСлСй АристотСля сбивало с Ρ‚ΠΎΠ»ΠΊΡƒ сопротивлСниС Π²ΠΎΠ·Π΄ΡƒΡ…Π°.
Он ΡƒΠΊΠ°Π·Π°Π», Ρ‡Ρ‚ΠΎ ΠΏΠ»ΠΎΡ‚Π½Ρ‹Π΅ ΠΏΡ€Π΅Π΄ΠΌΠ΅Ρ‚Ρ‹, для ΠΊΠΎΡ‚ΠΎΡ€Ρ‹Ρ… сопротивлСниС Π²ΠΎΠ·Π΄ΡƒΡ…Π° нСсущСствСнно, ΠΏΠ°Π΄Π°ΡŽΡ‚ ΠΏΠΎΡ‡Ρ‚ΠΈ с ΠΎΠ΄ΠΈΠ½Π°ΠΊΠΎΠ²ΠΎΠΉ ΡΠΊΠΎΡ€ΠΎΡΡ‚ΡŒΡŽ.

ΠŸΡ€Π΅Π΄ΠΏΠΎΠ»ΠΎΠΆΠΈΠ², Ρ‡Ρ‚ΠΎ ΠΏΡ€ΠΎΠΈΠ·ΠΎΡˆΠ»ΠΎ Π±Ρ‹ Π² случаС свободного падСния Ρ‚Π΅Π» Π² Π²Π°ΠΊΡƒΡƒΠΌΠ΅, Π“Π°Π»ΠΈΠ»Π΅ΠΉ Π²Ρ‹Π²Π΅Π» ΡΠ»Π΅Π΄ΡƒΡŽΡ‰ΠΈΠ΅ Π·Π°ΠΊΠΎΠ½Ρ‹ падСния Ρ‚Π΅Π» для идСального случая: всС Ρ‚Π΅Π»Π° ΠΏΡ€ΠΈ ΠΏΠ°Π΄Π΅Π½ΠΈΠΈ двиТутся ΠΎΠ΄ΠΈΠ½Π°ΠΊΠΎΠ²ΠΎ; Π½Π°Ρ‡Π°Π² ΠΏΠ°Π΄Π°Ρ‚ΡŒ ΠΎΠ΄Π½ΠΎΠ²Ρ€Π΅ΠΌΠ΅Π½Π½ΠΎ, ΠΎΠ½ΠΈ двиТутся с ΠΎΠ΄ΠΈΠ½Π°ΠΊΠΎΠ²ΠΎΠΉ ΡΠΊΠΎΡ€ΠΎΡΡ‚ΡŒΡŽ; Π΄Π²ΠΈΠΆΠ΅Π½ΠΈΠ΅ происходит с «ΠΏΠΎΡΡ‚оянным ускорСниСм»; Ρ‚Π΅ΠΌΠΏ увСличСния скорости Ρ‚Π΅Π»Π° Π½Π΅ мСняСтся, Ρ‚.Π΅. Π·Π° ΠΊΠ°ΠΆΠ΄ΡƒΡŽ ΠΏΠΎΡΠ»Π΅Π΄ΡƒΡŽΡ‰ΡƒΡŽ сСкунду ΡΠΊΠΎΡ€ΠΎΡΡ‚ΡŒ Ρ‚Π΅Π»Π° возрастаСт Π½Π° ΠΎΠ΄Π½Ρƒ ΠΈ Ρ‚Ρƒ ΠΆΠ΅ Π²Π΅Π»ΠΈΡ‡ΠΈΠ½Ρƒ. БущСствуСт Π»Π΅Π³Π΅Π½Π΄Π°, Π±ΡƒΠ΄Ρ‚ΠΎ Π“Π°Π»ΠΈΠ»Π΅ΠΉ ΠΏΡ€ΠΎΠ΄Π΅Π»Π°Π» большой дСмонстрационный ΠΎΠΏΡ‹Ρ‚, бросая Π»Π΅Π³ΠΊΠΈΠ΅ ΠΈ тяТСлыС ΠΏΡ€Π΅Π΄ΠΌΠ΅Ρ‚Ρ‹ с Π²Π΅Ρ€ΡˆΠΈΠ½Ρ‹ Пизанской ΠΏΠ°Π΄Π°ΡŽΡ‰Π΅ΠΉ башни (ΠΎΠ΄Π½ΠΈ говорят, Ρ‡Ρ‚ΠΎ ΠΎΠ½ бросал ΡΡ‚Π°Π»ΡŒΠ½Ρ‹Π΅ ΠΈ дСрСвянныС ΡˆΠ°Ρ€Ρ‹, Π° Π΄Ρ€ΡƒΠ³ΠΈΠ΅ ΡƒΡ‚Π²Π΅Ρ€ΠΆΠ΄Π°ΡŽΡ‚, Π±ΡƒΠ΄Ρ‚ΠΎ это Π±Ρ‹Π»ΠΈ ΠΆΠ΅Π»Π΅Π·Π½Ρ‹Π΅ ΡˆΠ°Ρ€Ρ‹ вСсом 0,5 ΠΈ 50 ΠΊΠ³). Описаний Ρ‚Π°ΠΊΠΎΠ³ΠΎ ΠΏΡƒΠ±Π»ΠΈΡ‡Π½ΠΎΠ³ΠΎ ΠΎΠΏΡ‹Ρ‚Π° Π½Π΅Ρ‚, ΠΈ Π“Π°Π»ΠΈΠ»Π΅ΠΉ, нСсомнСнно, Π½Π΅ стал Ρ‚Π°ΠΊΠΈΠΌ способом Π΄Π΅ΠΌΠΎΠ½ΡΡ‚Ρ€ΠΈΡ€ΠΎΠ²Π°Ρ‚ΡŒ своС ΠΏΡ€Π°Π²ΠΈΠ»ΠΎ. Π“Π°Π»ΠΈΠ»Π΅ΠΉ Π·Π½Π°Π», Ρ‡Ρ‚ΠΎ дСрСвянный ΡˆΠ°Ρ€ Π½Π°ΠΌΠ½ΠΎΠ³ΠΎ отстал Π±Ρ‹ ΠΏΡ€ΠΈ ΠΏΠ°Π΄Π΅Π½ΠΈΠΈ ΠΎΡ‚ ΠΆΠ΅Π»Π΅Π·Π½ΠΎΠ³ΠΎ, Π½ΠΎ считал, Ρ‡Ρ‚ΠΎ для дСмонстрации Ρ€Π°Π·Π»ΠΈΡ‡Π½ΠΎΠΉ скорости падСния Π΄Π²ΡƒΡ… Π½Π΅ΠΎΠ΄ΠΈΠ½Π°ΠΊΠΎΠ²Ρ‹Ρ… ΠΆΠ΅Π»Π΅Π·Π½Ρ‹Ρ… ΡˆΠ°Ρ€ΠΎΠ² ΠΏΠΎΡ‚Ρ€Π΅Π±ΠΎΠ²Π°Π»Π°ΡΡŒ Π±Ρ‹ Π±ΠΎΠ»Π΅Π΅ высокая башня.

Π˜Ρ‚Π°ΠΊ, ΠΌΠ΅Π»ΠΊΠΈΠ΅ ΠΊΠ°ΠΌΠ½ΠΈ слСгка ΠΎΡ‚ΡΡ‚Π°ΡŽΡ‚ Π² ΠΏΠ°Π΄Π΅Π½ΠΈΠΈ ΠΎΡ‚ ΠΊΡ€ΡƒΠΏΠ½Ρ‹Ρ…, ΠΈ Ρ€Π°Π·Π½ΠΈΡ†Π° становится Ρ‚Π΅ΠΌ Π±ΠΎΠ»Π΅Π΅ Π·Π°ΠΌΠ΅Ρ‚Π½ΠΎΠΉ, Ρ‡Π΅ΠΌ большСС расстояниС ΠΏΡ€ΠΎΠ»Π΅Ρ‚Π°ΡŽΡ‚ ΠΊΠ°ΠΌΠ½ΠΈ. И Π΄Π΅Π»ΠΎ Ρ‚ΡƒΡ‚ Π½Π΅ просто Π² Ρ€Π°Π·ΠΌΠ΅Ρ€Π΅ Ρ‚Π΅Π»: дСрСвянный ΠΈ ΡΡ‚Π°Π»ΡŒΠ½ΠΎΠΉ ΡˆΠ°Ρ€Ρ‹ ΠΎΠ΄ΠΈΠ½Π°ΠΊΠΎΠ²ΠΎΠ³ΠΎ Ρ€Π°Π·ΠΌΠ΅Ρ€Π° ΠΏΠ°Π΄Π°ΡŽΡ‚ Π½Π΅ строго ΠΎΠ΄ΠΈΠ½Π°ΠΊΠΎΠ²ΠΎ. Π“Π°Π»ΠΈΠ»Π΅ΠΉ Π·Π½Π°Π», Ρ‡Ρ‚ΠΎ простому описанию падСния Ρ‚Π΅Π» ΠΌΠ΅ΡˆΠ°Π΅Ρ‚ сопротивлСниС Π²ΠΎΠ·Π΄ΡƒΡ…Π°. Но ΠΎΠ½ ΠΌΠΎΠ³ лишь ΡƒΠΌΠ΅Π½ΡŒΡˆΠΈΡ‚ΡŒ Π΅Π³ΠΎ ΠΈ Π½Π΅ ΠΌΠΎΠ³ ΡƒΡΡ‚Ρ€Π°Π½ΠΈΡ‚ΡŒ Π΅Π³ΠΎ ΠΏΠΎΠ»Π½ΠΎΡΡ‚ΡŒΡŽ. ΠŸΠΎΡΡ‚ΠΎΠΌΡƒ Π΅ΠΌΡƒ ΠΏΡ€ΠΈΡˆΠ»ΠΎΡΡŒ вСсти Π΄ΠΎΠΊΠ°Π·Π°Ρ‚Π΅Π»ΡŒΡΡ‚Π²ΠΎ, пСрСходя ΠΎΡ‚ Ρ€Π΅Π°Π»ΡŒΠ½Ρ‹Ρ… наблюдСний ΠΊ постоянно ΡƒΠΌΠ΅Π½ΡŒΡˆΠ°ΡŽΡ‰ΠΈΠΌΡΡ сопротивлСниСм Π²ΠΎΠ·Π΄ΡƒΡ…Π° ΠΊ ΠΈΠ΄Π΅Π°Π»ΡŒΠ½ΠΎΠΌΡƒ ΡΠ»ΡƒΡ‡Π°ΡŽ, ΠΊΠΎΠ³Π΄Π° сопротивлСниС Π²ΠΎΠ·Π΄ΡƒΡ…Π° отсутствуСт. ПозТС, ΠΎΠ³Π»ΡΠ΄Ρ‹Π²Π°ΡΡΡŒ Π½Π°Π·Π°Π΄, ΠΎΠ½ смог ΠΎΠ±ΡŠΡΡΠ½ΠΈΡ‚ΡŒ различия Π² Ρ€Π΅Π°Π»ΡŒΠ½Ρ‹Ρ… экспСримСнтах, приписав ΠΈΡ… ΡΠΎΠΏΡ€ΠΎΡ‚ΠΈΠ²Π»Π΅Π½ΠΈΡŽ Π²ΠΎΠ·Π΄ΡƒΡ…Π°.

ВскорС послС ГалилСя Π±Ρ‹Π»ΠΈ созданы Π²ΠΎΠ·Π΄ΡƒΡˆΠ½Ρ‹Π΅ насосы, ΠΊΠΎΡ‚ΠΎΡ€Ρ‹Π΅ ΠΏΠΎΠ·Π²ΠΎΠ»ΠΈΠ»ΠΈ произвСсти экспСримСнты со свободным ΠΏΠ°Π΄Π΅Π½ΠΈΠ΅ΠΌ Π² Π²Π°ΠΊΡƒΡƒΠΌΠ΅. Π‘ этой Ρ†Π΅Π»ΡŒΡŽ ΠΡŒΡŽΡ‚ΠΎΠ½ Π²Ρ‹ΠΊΠ°Ρ‡Π°Π» Π²ΠΎΠ·Π΄ΡƒΡ… ΠΈΠ· Π΄Π»ΠΈΠ½Π½ΠΎΠΉ стСклянной Ρ‚Ρ€ΡƒΠ±ΠΊΠΈ ΠΈ бросил свСрху ΠΎΠ΄Π½ΠΎΠ²Ρ€Π΅ΠΌΠ΅Π½Π½ΠΎ ΠΏΡ‚ΠΈΡ‡ΡŒΠ΅ ΠΏΠ΅Ρ€ΠΎ ΠΈ Π·ΠΎΠ»ΠΎΡ‚ΡƒΡŽ ΠΌΠΎΠ½Π΅Ρ‚Ρƒ. Π”Π°ΠΆΠ΅ ΡΡ‚ΠΎΠ»ΡŒ сильно Ρ€Π°Π·Π»ΠΈΡ‡Π°ΡŽΡ‰ΠΈΠ΅ΡΡ ΠΏΠΎ своСй плотности Ρ‚Π΅Π»Π° ΠΏΠ°Π΄Π°Π»ΠΈ с ΠΎΠ΄ΠΈΠ½Π°ΠΊΠΎΠ²ΠΎΠΉ ΡΠΊΠΎΡ€ΠΎΡΡ‚ΡŒΡŽ. ИмСнно этот ΠΎΠΏΡ‹Ρ‚ Π΄Π°Π» Ρ€Π΅ΡˆΠ°ΡŽΡ‰ΡƒΡŽ ΠΏΡ€ΠΎΠ²Π΅Ρ€ΠΊΡƒ прСдполоТСния ГалилСя. ΠžΠΏΡ‹Ρ‚Ρ‹ ΠΈ рассуТдСния ГалилСя ΠΏΡ€ΠΈΠ²Π΅Π»ΠΈ ΠΊ простому ΠΏΡ€Π°Π²ΠΈΠ»Ρƒ, Ρ‚ΠΎΡ‡Π½ΠΎ справСдливому Π² случаС свободного падСния Ρ‚Π΅Π» Π² Π²Π°ΠΊΡƒΡƒΠΌΠ΅. Π­Ρ‚ΠΎ ΠΏΡ€Π°Π²ΠΈΠ»ΠΎ Π² случаС свободного падСния Ρ‚Π΅Π» Π² Π²ΠΎΠ·Π΄ΡƒΡ…Π΅ выполняСтся с ΠΎΠ³Ρ€Π°Π½ΠΈΡ‡Π΅Π½Π½ΠΎΠΉ Ρ‚ΠΎΡ‡Π½ΠΎΡΡ‚ΡŒΡŽ. ΠŸΠΎΡΡ‚ΠΎΠΌΡƒ Π²Π΅Ρ€ΠΈΡ‚ΡŒ Π² Π½Π΅Π³ΠΎ, ΠΊΠ°ΠΊ Π² ΠΈΠ΄Π΅Π°Π»ΡŒΠ½Ρ‹ΠΉ случай нСльзя. Для ΠΏΠΎΠ»Π½ΠΎΠ³ΠΎ изучСния свободного падСния Ρ‚Π΅Π» Π½Π΅ΠΎΠ±Ρ…ΠΎΠ΄ΠΈΠΌΠΎ Π·Π½Π°Ρ‚ΡŒ, ΠΊΠ°ΠΊΠΈΠ΅ ΠΏΡ€ΠΈ ΠΏΠ°Π΄Π΅Π½ΠΈΠΈ происходят измСнСния Ρ‚Π΅ΠΌΠΏΠ΅Ρ€Π°Ρ‚ΡƒΡ€Ρ‹, давлСния, ΠΈ Π΄Ρ€., Ρ‚ΠΎ Π΅ΡΡ‚ΡŒ ΠΈΡΡΠ»Π΅Π΄ΠΎΠ²Π°Ρ‚ΡŒ ΠΈ Π΄Ρ€ΡƒΠ³ΠΈΠ΅ стороны этого явлСния. Π’Π°ΠΊ Π“Π°Π»ΠΈΠ»Π΅ΠΉ установил ΠΏΡ€ΠΈΠ·Π½Π°ΠΊ равноускорСнного двиТСния:

S1 : S2 : S3 : … = 1 : 2 : 3 : … (ΠΏΡ€ΠΈ V0 = 0)

Π’Π°ΠΊΠΈΠΌ ΠΎΠ±Ρ€Π°Π·ΠΎΠΌ, ΠΌΠΎΠΆΠ½ΠΎ ΠΏΡ€Π΅Π΄ΠΏΠΎΠ»ΠΎΠΆΠΈΡ‚ΡŒ, Ρ‡Ρ‚ΠΎ свободноС ΠΏΠ°Π΄Π΅Π½ΠΈΠ΅ Π΅ΡΡ‚ΡŒ равноускорСнноС Π΄Π²ΠΈΠΆΠ΅Π½ΠΈΠ΅. Π’Π°ΠΊ ΠΊΠ°ΠΊ для равноускорСнного двиТСния ΠΏΠ΅Ρ€Π΅ΠΌΠ΅Ρ‰Π΅Π½ΠΈΠ΅ рассчитываСтся ΠΏΠΎ Ρ„ΠΎΡ€ΠΌΡƒΠ»Π΅, Ρ‚ΠΎ Ссли Π²Π·ΡΡ‚ΡŒ Ρ‚Ρ€ΠΈ Π½Π΅ΠΊΠΎΡ‚ΠΎΡ€Ρ‹Π΅ Ρ‚ΠΎΡ‡ΠΊΠΈ 1,2,3 Ρ‡Π΅Ρ€Π΅Π· ΠΊΠΎΡ‚ΠΎΡ€Ρ‹Π΅ ΠΏΡ€ΠΎΡ…ΠΎΠ΄ΠΈΡ‚ Ρ‚Π΅Π»ΠΎ ΠΏΡ€ΠΈ ΠΏΠ°Π΄Π΅Π½ΠΈΠΈ ΠΈ Π·Π°ΠΏΠΈΡΠ°Ρ‚ΡŒ: (ускорСниС ΠΏΡ€ΠΈ свободном ΠΏΠ°Π΄Π΅Π½ΠΈΠΈ для всСх Ρ‚Π΅Π» ΠΎΠ΄ΠΈΠ½Π°ΠΊΠΎΠ²ΠΎ), получится, Ρ‡Ρ‚ΠΎ ΠΎΡ‚Π½ΠΎΡˆΠ΅Π½ΠΈΠ΅ ΠΏΠ΅Ρ€Π΅ΠΌΠ΅Ρ‰Π΅Π½ΠΈΠΉ ΠΏΡ€ΠΈ равноускорСнном Π΄Π²ΠΈΠΆΠ΅Π½ΠΈΠΈ Ρ€Π°Π²Π½ΠΎ:

S1 : S2 : S3 = t12

: t22 : t32 (2)

ΠžΡΡ‚Π°Π΅Ρ‚ΡΡ Π΅Ρ‰Π΅ Π΄ΠΎΠ±Π°Π²ΠΈΡ‚ΡŒ нСбольшой ΠΊΠΎΠΌΠΌΠ΅Π½Ρ‚Π°Ρ€ΠΈΠΉ ΠΎΡ‚Π½ΠΎΡΠΈΡ‚Π΅Π»ΡŒΠ½ΠΎ экспСримСнтов со свободным ΠΏΠ°Π΄Π΅Π½ΠΈΠ΅ΠΌ Ρ‚Π΅Π» Исаака ΠΡŒΡŽΡ‚ΠΎΠ½Π°. Π’ Π΅Π³ΠΎ Π²Ρ‹Π²ΠΎΠ΄Π°Ρ… прослСТиваСтся ΠΌΡ‹ΡΠ»ΡŒ, Ρ‡Ρ‚ΠΎ Π½Π° Π›ΡƒΠ½Π΅ ΠΈ Π½Π° Π΄Ρ€ΡƒΠ³ΠΈΡ… ΠΏΠ»Π°Π½Π΅Ρ‚Π°Ρ… сила тяТСсти, Π²ΠΎΠ·Π΄Π΅ΠΉΡΡ‚Π²ΡƒΡŽΡ‰Π°Ρ Π½Π° ΠΎΠ΄Π½ΠΎ ΠΈ Ρ‚ΠΎ ΠΆΠ΅ Ρ‚Π΅Π»ΠΎ, Π±ΡƒΠ΄Π΅Ρ‚ Π½Π΅ΠΎΠ΄ΠΈΠ½Π°ΠΊΠΎΠ²Π°, зависит ΠΎΠ½Π° Π½Π°ΠΏΡ€ΡΠΌΡƒΡŽ ΠΎΡ‚ массы космичСского Ρ‚Π΅Π»Π°. НапримСр, ускорСниС g Π½Π° Π›ΡƒΠ½Π΅ Π² нСсколько Ρ€Π°Π· мСньшС, Ρ‡Π΅ΠΌ Π½Π° Π—Π΅ΠΌΠ»Π΅. Π’Π°ΠΊΠΈΠΌ ΠΎΠ±Ρ€Π°Π·ΠΎΠΌ, зная массу ΠΏΠ»Π°Π½Π΅Ρ‚Ρ‹, ΠΌΠΎΠΆΠ½ΠΎ Π²Ρ‹Ρ‡ΠΈΡΠ»ΠΈΡ‚ΡŒ ускорСниС свободного падСния Ρ‚Π΅Π»Π° Π½Π° этой ΠΏΠ»Π°Π½Π΅Ρ‚Π΅.

2.2. ΠŸΡ€Π°ΠΊΡ‚ΠΈΡ‡Π΅ΡΠΊΠ°Ρ Π·Π½Π°Ρ‡ΠΈΠΌΠΎΡΡ‚ΡŒ нахоТдСния значСния ускорСния свободного падСния

Π― ΠΌΠ½ΠΎΠ³ΠΎ Ρ‡ΠΈΡ‚Π°ΡŽ ΠΈ, ΠΊΠ°ΠΊ слСдствиС склонСн Ρ„Π°Π½Ρ‚Π°Π·ΠΈΡ€ΠΎΠ²Π°Ρ‚ΡŒ. Для мСня практичСская Π·Π½Π°Ρ‡ΠΈΠΌΠΎΡΡ‚ΡŒ исслСдования Π·Π°ΠΊΠ»ΡŽΡ‡Π°Π΅Ρ‚ΡΡ Π² возмоТности прогнозирования Ρ„ΠΎΡ€ΠΌ ΠΆΠΈΠ·Π½ΠΈ Π½Π° нСбСсных Ρ‚Π΅Π»Π°Ρ…, с ΠΊΠΎΡ‚ΠΎΡ€Ρ‹ΠΌΠΈ чСловСчСство столкнСтся ΠΏΡ€ΠΈ Π½Π΅ΠΈΠ·Π±Π΅ΠΆΠ½ΠΎΠΌ освоСнии космоса. Π’Π΅Π΄ΡŒ ΠΎΡ‚ значСния g Π½Π° Π΄Ρ€ΡƒΠ³ΠΎΠΉ ΠΏΠ»Π°Π½Π΅Ρ‚Π΅ зависит Π½Π΅ Ρ‚ΠΎΠ»ΡŒΠΊΠΎ сила тяТСсти. Π›ΡŽΠ΄ΠΈ Π·Π°Ρ€Π°Π½Π΅Π΅ смогут ΡƒΠ·Π½Π°Ρ‚ΡŒ, ΠΊΠ°ΠΊΠΈΠ΅ сущСства встрСтят ΠΈΡ… Π½Π° Ρ‚ΠΎΠΉ ΠΈΠ»ΠΈ ΠΈΠ½ΠΎΠΉ ΠΏΠ»Π°Π½Π΅Ρ‚Π΅, ΠΊΠ°ΠΊΠΈΠΌΠΈ физичСскими характСристиками ΠΎΠ½ΠΈ Π±ΡƒΠ΄ΡƒΡ‚ ΠΎΠ±Π»Π°Π΄Π°Ρ‚ΡŒ.

2.3. ΠœΠ΅Ρ‚ΠΎΠ΄Ρ‹ измСрСния ускорСния свободного падСния

На самом Π΄Π΅Π»Π΅ ΠΌΠ΅Ρ‚ΠΎΠ΄ΠΎΠ² ΠΏΠΎ ΠΈΠ·ΠΌΠ΅Ρ€Π΅Π½ΠΈΡŽ ускорСния свободного падСния достаточно ΠΌΠ½ΠΎΠ³ΠΎ. ΠŸΡ€ΠΈΠ²Π΅Π΄Ρƒ Ρ‚ΠΎΠ»ΡŒΠΊΠΎ Ρ‚Π΅, ΠΊΠΎΡ‚ΠΎΡ€Ρ‹Π΅ сам испробовал.

1) Π˜Π·ΠΌΠ΅Ρ€Π΅Π½ΠΈΠ΅ ускорСния свободного падСния с ΠΏΠΎΠΌΠΎΡ‰ΡŒΡŽ Π½Π°ΠΊΠ»ΠΎΠ½Π½ΠΎΠΉ плоскости

ΠŸΠΎΠ½Π°Π΄ΠΎΠ±ΠΈΡ‚ΡΡ ΡΠ»Π΅Π΄ΡƒΡŽΡ‰Π΅Π΅ ΠΎΠ±ΠΎΡ€ΡƒΠ΄ΠΎΠ²Π°Π½ΠΈΠ΅:дСрСвянный брусок, Ρ‚Ρ€ΠΈΠ±ΠΎΠΌΠ΅Ρ‚Ρ€, ΡˆΡ‚Π°Ρ‚ΠΈΠ² с ΠΌΡƒΡ„Ρ‚ΠΎΠΉ ΠΈ Π»Π°ΠΏΠΊΠΎΠΉ, элСктронный сСкундомСр, Π΄ΠΈΠ½Π°ΠΌΠΎΠΌΠ΅Ρ‚Ρ€, ΠΈΠ·ΠΌΠ΅Ρ€ΠΈΡ‚Π΅Π»ΡŒΠ½Π°Ρ Π»Π΅Π½Ρ‚Π°, Π»ΠΈΠ½Π΅ΠΉΠΊΠ°. Рассматривая Π΄Π²ΠΈΠΆΠ΅Π½ΠΈΠ΅ бруска Π²Π½ΠΈΠ· ΠΏΠΎ Π½Π°ΠΊΠ»ΠΎΠ½Π½ΠΎΠΉ плоскости, ΠΌΠΎΠΆΠ½ΠΎ Π·Π°ΠΏΠΈΡΠ°Ρ‚ΡŒ Π²Ρ‚ΠΎΡ€ΠΎΠΉ Π·Π°ΠΊΠΎΠ½ ΠΡŒΡŽΡ‚ΠΎΠ½Π° Π² Π²Π΅ΠΊΡ‚ΠΎΡ€Π½ΠΎΠΌ Π²ΠΈΠ΄Π΅:


Записывая Π²Ρ‚ΠΎΡ€ΠΎΠΉ Π·Π°ΠΊΠΎΠ½ ΠΡŒΡŽΡ‚ΠΎΠ½Π° Π² проСкциях Π½Π° оси ΠΊΠΎΠΎΡ€Π΄ΠΈΠ½Π°Ρ‚:

ΠžΡ…: – FΡ‚Ρ€+ mgsinΞ± = ma

Oy: N – mgcosΞ± = 0

ΠΈ учитывая, Ρ‡Ρ‚ΠΎ N = mgcosΞ±; FΡ‚Ρ€ = ΞΌN; ΠΌΠΎΠΆΠ½ΠΎ Ρ€Π΅ΡˆΠΈΡ‚ΡŒ Π΄Π°Π½Π½ΡƒΡŽ систСму ΡƒΡ€Π°Π²Π½Π΅Π½ΠΈΠΉ ΠΈ ΠΏΠΎΠ»ΡƒΡ‡ΠΈΡ‚ΡŒ ускорСниС свободного падСния:

g =Β  a
sinΞ± – ΞΌcosΞ±

ΠŸΡ€ΠΈ этом ускорСниС a ΠΌΠΎΠΆΠ½ΠΎ Π²Ρ‹Ρ‡ΠΈΡΠ»ΠΈΡ‚ΡŒ ΠΈΠ· Ρ„ΠΎΡ€ΠΌΡƒΠ»Ρ‹

Ρ‚Π°ΠΊ ΠΊΠ°ΠΊ Π½Π°Ρ‡Π°Π»ΡŒΠ½Π°Ρ ΡΠΊΠΎΡ€ΠΎΡΡ‚ΡŒ бруска ΠΏΡ€ΠΈ скольТСнии ΠΏΠΎ Π½Π°ΠΊΠ»ΠΎΠ½Π½ΠΎΠΉ плоскости Ρ€Π°Π²Π½Π° 0:

Π’ΠΈΠ΄ΠΈΠΌ, Ρ‡Ρ‚ΠΎ для этого Π½ΡƒΠΆΠ½ΠΎ ΠΈΠ·ΠΌΠ΅Ρ€ΠΈΡ‚ΡŒ Π΄Π»ΠΈΠ½Ρƒ Π½Π°ΠΊΠ»ΠΎΠ½Π½ΠΎΠΉ плоскости ΠΈ врСмя скольТСния ΠΏΠΎ Π½Π΅ΠΉ бруска.

Для вычислСния sinΞ± ΠΈ cosΞ± Π½ΡƒΠΆΠ½ΠΎ Π·Π½Π°Ρ‚ΡŒ Π΄Π»ΠΈΠ½Ρƒ S ΠΈ высоту h Π½Π°ΠΊΠ»ΠΎΠ½Π½ΠΎΠΉ плоскости:


Для опрСдСлСния коэффициСнта трСния скольТСния ΠΏΠΎΠ»ΠΎΠΆΠΈΠΌ Ρ‚Ρ€ΠΈΠ±ΠΎΠΌΠ΅Ρ‚Ρ€ Π½Π° Π³ΠΎΡ€ΠΈΠ·ΠΎΠ½Ρ‚Π°Π»ΡŒΠ½ΡƒΡŽ ΠΏΠΎΠ²Π΅Ρ€Ρ…Π½ΠΎΡΡ‚ΡŒ ΠΈ с ΠΏΠΎΠΌΠΎΡ‰ΡŒΡŽ Π΄ΠΈΠ½Π°ΠΌΠΎΠΌΠ΅Ρ‚Ρ€Π° Ρ€Π°Π²Π½ΠΎΠΌΠ΅Ρ€Π½ΠΎ ΠΏΡ€ΠΎΡ‚Π°Ρ‰ΠΈΠΌ ΠΏΠΎ Π½Π΅ΠΌΡƒ брусок. Π’ этом случаС Π½Π° брусок Π±ΡƒΠ΄ΡƒΡ‚ Π΄Π΅ΠΉΡΡ‚Π²ΠΎΠ²Π°Ρ‚ΡŒ 4 силы: сила тяТСсти, сила упругости ΠΏΡ€ΡƒΠΆΠΈΠ½Ρ‹ Π΄ΠΈΠ½Π°ΠΌΠΎΠΌΠ΅Ρ‚Ρ€Π°, сила трСния, сила Ρ€Π΅Π°ΠΊΡ†ΠΈΠΈ ΠΎΠΏΠΎΡ€Ρ‹.


ΠŸΡ€ΠΈ Ρ€Π°Π²Π½ΠΎΠΌΠ΅Ρ€Π½ΠΎΠΌ Π΄Π²ΠΈΠΆΠ΅Π½ΠΈΠΈ бруска эти силы Π±ΡƒΠ΄ΡƒΡ‚ ΠΏΠΎΠΏΠ°Ρ€Π½ΠΎ Ρ€Π°Π²Π½Ρ‹: FΡ‚Ρ€Β =Β FΡƒΠΏΡ€, FтяТ =Β N, Ρ‚. Π΅. FΡƒΠΏΡ€Β =Β ΞΌFтяТ, Ρ‚ΠΎΠ³Π΄Π° коэффициСнт трСния Ρ€Π°Π²Π΅Π½

Для мСня Π² этом ΠΌΠ΅Ρ‚ΠΎΠ΄Π΅ оказалось слишком ΠΌΠ½ΠΎΠ³ΠΎ матСматичСских дСйствий, с ΠΊΠΎΡ‚ΠΎΡ€Ρ‹ΠΌΠΈ Π² курсС ΠΌΠ°Ρ‚Π΅ΠΌΠ°Ρ‚ΠΈΠΊΠΈ я Π΅Ρ‰Π΅ Π½Π΅ Π·Π½Π°ΠΊΠΎΠΌ. ΠŸΠΎΡΡ‚ΠΎΠΌΡƒ Π΄Π°ΠΆΠ΅ Π½Π΅ Π±ΡƒΠ΄Ρƒ ΠΏΡ€ΠΈΠ²ΠΎΠ΄ΠΈΡ‚ΡŒ Ρ€Π΅Π·ΡƒΠ»ΡŒΡ‚Π°Ρ‚Ρ‹ ΠΏΡ€ΠΎΠ΄Π΅Π»Π°Π½Π½Ρ‹Ρ… ΠΈΠ·ΠΌΠ΅Ρ€Π΅Π½ΠΈΠΉ ΠΈ вычислСний.

2) ΠžΠΏΡ€Π΅Π΄Π΅Π»Π΅Π½ΠΈΠ΅
g благодаря давлСнию Тидкости

Как извСстно Π΄Π°Π²Π»Π΅Π½ΠΈΠ΅ столба Тидкости обусловлСно ΡΠ»Π΅Π΄ΡƒΡŽΡ‰ΠΈΠΌΠΈ Ρ„Π°ΠΊΡ‚ΠΎΡ€Π°ΠΌΠΈ: ΠΏΠ»ΠΎΡ‚Π½ΠΎΡΡ‚ΡŒ Тидкости, нСпосрСдствСнно высота столба Тидкости ΠΈ само Π·Π½Π°Ρ‡Π΅Π½ΠΈΠ΅ ускорСния свободного падСния Π½Π° Π΄Π°Π½Π½ΠΎΠΉ ΠΏΠ»Π°Π½Π΅Ρ‚Π΅.

Если ΠΏΡ€Π΅ΠΎΠ±Ρ€Π°Π·ΠΎΠ²Π°Ρ‚ΡŒ Ρ„ΠΎΡ€ΠΌΡƒΠ»Ρƒ PΒ = ρgh, получится Ρ„ΠΎΡ€ΠΌΡƒΠ»Π° нахоТдСния g. Π­Ρ‚Π° Ρ„ΠΎΡ€ΠΌΡƒΠ»Π° выглядит Ρ‚Π°ΠΊ gΒ =Β PΒ / ρh, Π³Π΄Π΅ Π  – Π΄Π°Π²Π»Π΅Π½ΠΈΠ΅ Π² Тидкости Π½Π° Π³Π»ΡƒΠ±ΠΈΠ½Π΅ h, ΠΊΠΎΡ‚ΠΎΡ€ΠΎΠ΅ ΠΌΠΎΠΆΠ½ΠΎ ΡƒΠ·Π½Π°Ρ‚ΡŒ с ΠΏΠΎΠΌΠΎΡ‰ΡŒΡŽ ΠΌΠ°Π½ΠΎΠΌΠ΅Ρ‚Ρ€Π°, ρ – ΠΏΠ»ΠΎΡ‚Π½ΠΎΡΡ‚ΡŒ Π²ΠΎΠ΄Ρ‹ Ρ€Π°Π²Π½ΠΎΠ΅ 1000Β ΠΊΠ³/ΠΌ3.

ΠŸΡ€ΠΈ ΠΏΠΎΠ΄ΠΎΠ±Π½Ρ‹Ρ… измСрСниях Π½ΡƒΠΆΠ½ΠΎ ΡƒΡ‡ΠΈΡ‚Ρ‹Π²Π°Ρ‚ΡŒ ΠΏΠΎΠ³Ρ€Π΅ΡˆΠ½ΠΎΡΡ‚ΡŒ ΠΈΠ·ΠΌΠ΅Ρ€ΠΈΡ‚Π΅Π»ΡŒΠ½ΠΎΠ³ΠΎ ΠΏΡ€ΠΈΠ±ΠΎΡ€Π°, ΠΌΠ°Π½ΠΎΠΌΠ΅Ρ‚Ρ€Π°. Достаточно Ρ‚ΠΎΡ‡Π½ΠΎΠ³ΠΎ ΠΌΠ½Π΅ Π½Π°ΠΉΡ‚ΠΈ Π½Π΅ ΡƒΠ΄Π°Π»ΠΎΡΡŒ, поэтому для своих исслСдований я Π²Ρ‹Π±Ρ€Π°Π» Π΄Ρ€ΡƒΠ³ΠΎΠΉ ΠΌΠ΅Ρ‚ΠΎΠ΄.

3) Π˜Π·ΠΌΠ΅Ρ€Π΅Π½ΠΈΠ΅ ускорСния свободного падСния с ΠΏΠΎΠΌΠΎΡ‰ΡŒΡŽ матСматичСского маятника

НСобходимоС ΠΎΠ±ΠΎΡ€ΡƒΠ΄ΠΎΠ²Π°Π½ΠΈΠ΅: сСкундомСр, ΡˆΡ‚Π°Ρ‚ΠΈΠ² с ΠΌΡƒΡ„Ρ‚ΠΎΠΉ ΠΈ Π»Π°ΠΏΠΊΠΎΠΉ, ΡˆΠ°Ρ€ΠΈΠΊ Π½Π° нСрастяТимой Π½ΠΈΡ‚ΠΈ, ΠΈΠ·ΠΌΠ΅Ρ€ΠΈΡ‚Π΅Π»ΡŒΠ½Π°Ρ Π»Π΅Π½Ρ‚Π°. ΠŸΡ€ΠΈ ΠΌΠ°Π»Ρ‹Ρ… Ρ€Π°Π·ΠΌΠ΅Ρ€Π°Ρ… ΡˆΠ°Ρ€ΠΈΠΊΠ° ΠΏΠΎ ΡΡ€Π°Π²Π½Π΅Π½ΠΈΡŽ с Π΄Π»ΠΈΠ½ΠΎΠΉ Π½ΠΈΡ‚ΠΈ ΠΈ Π½Π΅Π±ΠΎΠ»ΡŒΡˆΠΈΡ… отклонСниях (Π΄ΠΎ 10Β°) ΠΎΡ‚ полоТСния равновСсия ΠΏΠ΅Ρ€ΠΈΠΎΠ΄ ΠΊΠΎΠ»Π΅Π±Π°Π½ΠΈΠΉ Ρ€Π°Π²Π΅Π½ ΠΏΠ΅Ρ€ΠΈΠΎΠ΄Ρƒ ΠΊΠΎΠ»Π΅Π±Π°Π½ΠΈΠΉ матСматичСского маятника


Π‘ Π΄Ρ€ΡƒΠ³ΠΎΠΉ стороны ΠΏΠ΅Ρ€ΠΈΠΎΠ΄ ΠΊΠΎΠ»Π΅Π±Π°Π½ΠΈΠΉ маятника ΠΌΠΎΠΆΠ½ΠΎ Ρ€Π°ΡΡ‡ΠΈΡ‚Π°Ρ‚ΡŒ ΠΈΠ· опрСдСлСния, вСдь ΠΏΠ΅Ρ€ΠΈΠΎΠ΄ – это врСмя ΠΎΠ΄Π½ΠΎΠ³ΠΎ ΠΏΠΎΠ»Π½ΠΎΠ³ΠΎ колСбания. Π’ΠΎΠ³Π΄Π° ΠΏΠ΅Ρ€ΠΈΠΎΠ΄

ΠΈ ускорСния свободного падСния ΠΌΠΎΠΆΠ΅Ρ‚ Π±Ρ‹Ρ‚ΡŒ вычислСно ΠΏΠΎ Ρ„ΠΎΡ€ΠΌΡƒΠ»Π΅

ΠŸΠΎΠ΄Π³ΠΎΡ‚ΠΎΠ²ΠΊΠ° ΠΊ ΠΏΡ€ΠΎΠ²Π΅Π΄Π΅Π½ΠΈΡŽ Ρ€Π°Π±ΠΎΡ‚Ρ‹

Π’ Ρ€Π°Π±ΠΎΡ‚Π΅ ΠΈΡΠΏΠΎΠ»ΡŒΠ·ΡƒΠ΅Ρ‚ΡΡ ΠΏΡ€ΠΎΡΡ‚Π΅ΠΉΡˆΠΈΠΉ маятник – ΡˆΠ°Ρ€ΠΈΠΊ Π½Π° Π½ΠΈΡ‚ΠΈ. ΠŸΡ€ΠΈ ΠΌΠ°Π»Ρ‹Ρ… Ρ€Π°Π·ΠΌΠ΅Ρ€Π°Ρ… ΠΏΠΎ ΡΡ€Π°Π²Π½Π΅Π½ΠΈΡŽ с Π΄Π»ΠΈΠ½ΠΎΠΉ Π½ΠΈΡ‚ΠΈ ΠΈ Π½Π΅Π±ΠΎΠ»ΡŒΡˆΠΈΡ… отклонСниях ΠΎΡ‚ полоТСния равновСсия ΠΏΠ΅Ρ€ΠΈΠΎΠ΄ ΠΊΠΎΠ»Π΅Π±Π°Π½ΠΈΠΉ Ρ€Π°Π²Π΅Π½ ΠΏΠ΅Ρ€ΠΈΠΎΠ΄Ρƒ ΠΊΠΎΠ»Π΅Π±Π°Π½ΠΈΠΉ матСматичСского маятника


Π’ΠΎΠ³Π΄Π° ΠΏΠ΅Ρ€ΠΈΠΎΠ΄

ΠΈ ускорСния свободного падСния ΠΌΠΎΠΆΠ΅Ρ‚ Π±Ρ‹Ρ‚ΡŒ вычислСно ΠΏΠΎ Ρ„ΠΎΡ€ΠΌΡƒΠ»Π΅

Π Π΅Π·ΡƒΠ»ΡŒΡ‚Π°Ρ‚Ρ‹ ΠΈΠ·ΠΌΠ΅Ρ€Π΅Π½ΠΈΠΉ ΠΈ вычислСний прСдставлСны Π² Ρ€Π°Π·Π΄Π΅Π»Π΅ 2.5

2.4. ВСорСтичСскиС расчСты ΠΏΠΎ ΠΎΠΏΡ€Π΅Π΄Π΅Π»Π΅Π½ΠΈΡŽ ускорСния свободного падСния Ρ€Π°Π·Π»ΠΈΡ‡Π½Ρ‹Ρ… высотах

ВСорСтичСски Π·Π½Π°Ρ‡Π΅Π½ΠΈΠ΅ ускорСния свободного падСния Π½Π° повСрхности ΠΏΠ»Π°Π½Π΅Ρ‚Ρ‹ ЗСмля ΠΌΠΎΠΆΠ½ΠΎ ΠΏΡ€ΠΈΠ±Π»ΠΈΠ·ΠΈΡ‚Π΅Π»ΡŒΠ½ΠΎ ΠΏΠΎΠ΄ΡΡ‡ΠΈΡ‚Π°Ρ‚ΡŒ, прСдставив ΠΏΠ»Π°Π½Π΅Ρ‚Ρƒ Ρ‚ΠΎΡ‡Π΅Ρ‡Π½ΠΎΠΉ массой M, ΠΈ вычислив Π³Ρ€Π°Π²ΠΈΡ‚Π°Ρ†ΠΈΠΎΠ½Π½ΠΎΠ΅ ускорСниС Π½Π° расстоянии Π΅Ρ‘ радиуса R:

Π³Π΄Π΅ G β€” гравитационная постоянная (G = 6,6743Β Β·Β 10–11 (HΒ Β·ΠΌ2)/ΠΊΠ³2).

ΠŸΡ€ΠΈ вычислСниях я примСнял Ρ‚Π°ΠΊΠΈΠ΅ значСния:

RΒ =Β 6370Β Β·Β 103 ΠΌ – радиус Π—Π΅ΠΌΠ»ΠΈ Π½Π° ΡˆΠΈΡ€ΠΎΡ‚Π΅ Казани;

MΒ =Β 5,9722 Β· 1024 ΠΊΠ³ – масса Π—Π΅ΠΌΠ»ΠΈ.

Π’Π°ΠΊΠΈΠΌ ΠΎΠ±Ρ€Π°Π·ΠΎΠΌ тСорСтичСскоС Π·Π½Π°Ρ‡Π΅Π½ΠΈΠ΅ gΡ‚ = 9,823386 ΠΌ/с2.

Богласно Ρ„ΠΎΡ€ΠΌΡƒΠ»Π΅

СстСствСнно ΠΏΡ€Π΅Π΄ΠΏΠΎΠ»ΠΎΠΆΠΈΡ‚ΡŒ, Ρ‡Ρ‚ΠΎ ускорСниС свободного падСния Π½Π° Ρ€Π°Π·Π½Ρ‹Ρ… высотах Π±ΡƒΠ΄Π΅Ρ‚ Π½Π΅ΠΌΠ½ΠΎΠ³ΠΎ ΠΎΡ‚Π»ΠΈΡ‡Π°Ρ‚ΡŒΡΡ: Π½Π° Π³Π»ΡƒΠ±ΠΈΠ½Π΅ Π±ΡƒΠ΄Π΅Ρ‚ большС, Π° Π½Π° высотС мСньшС вычислСнного Π²Ρ‹ΡˆΠ΅.

Π’ΠΎΠ·ΠΌΠΎΠΆΠ½ΠΎ эту Π½Π΅Π±ΠΎΠ»ΡŒΡˆΡƒΡŽ Ρ€Π°Π·Π½ΠΈΡ†Ρƒ ΠΌΠΎΠΆΠ½ΠΎ ΠΎΠ±ΡŠΡΡΠ½ΠΈΡ‚ΡŒ ΠΏΠΎΠ³Ρ€Π΅ΡˆΠ½ΠΎΡΡ‚ΡŒΡŽ ΠΈΠ·ΠΌΠ΅Ρ€Π΅Π½ΠΈΠΉ. ΠŸΡ€ΠΎΠ²Π΅Ρ€ΠΈΠΌ.

Π Π΅Π·ΡƒΠ»ΡŒΡ‚Π°Ρ‚Ρ‹ вычислСний значСния ускорСния свободного падСния Π½Π° Ρ€Π°Π·Π»ΠΈΡ‡Π½Ρ‹Ρ… высотах прСдставлСны Π² Ρ‚Π°Π±Π»ΠΈΡ†Π΅:

Π’ классС

На станции ΠΌΠ΅Ρ‚Ρ€ΠΎ ΠšΡ€Π΅ΠΌΠ»Π΅Π²ΡΠΊΠ°Ρ

На 36-ΠΌ этаТС нСбоскрСба

R = 6370 ΠΊΠΌ,

h = 0

R = 6370 ΠΊΠΌ,

h = –16 ΠΌ

R = 6370 ΠΊΠΌ,

h = +120 ΠΌ

9,8234

9,8231

9,8227

2.

5. Π­ΠΊΡΠΏΠ΅Ρ€ΠΈΠΌΠ΅Π½Ρ‚Π°Π»ΡŒΠ½ΠΎΠ΅ ΠΎΠΏΡ€Π΅Π΄Π΅Π»Π΅Π½ΠΈΠ΅ ускорСния свободного падСния с ΠΏΠΎΠΌΠΎΡ‰ΡŒΡŽ матСматичСского маятника

Как ΡƒΠΆΠ΅ Π³ΠΎΠ²ΠΎΡ€ΠΈΠ»ΠΎΡΡŒ Ρ€Π°Π½Π΅Π΅, ΠΎΠ±ΠΎΡ€ΡƒΠ΄ΠΎΠ²Π°Π½ΠΈΠ΅ для провСдСния ΠΈΠ·ΠΌΠ΅Ρ€Π΅Π½ΠΈΠΉ Ρ‚Ρ€Π΅Π±ΠΎΠ²Π°Π»ΠΎΡΡŒ вСсьма Π½Π΅ замысловатоС: сСкундомСр, ΡˆΡ‚Π°Ρ‚ΠΈΠ² с ΠΌΡƒΡ„Ρ‚ΠΎΠΉ, ΡˆΠ°Ρ€ΠΈΠΊ Π½Π° нСрастяТимой Π½ΠΈΡ‚ΠΈ, ΠΈΠ·ΠΌΠ΅Ρ€ΠΈΡ‚Π΅Π»ΡŒΠ½Π°Ρ Π»Π΅Π½Ρ‚Π°. ΠŸΡ€ΠΈ ΠΌΠ°Π»Ρ‹Ρ… Ρ€Π°Π·ΠΌΠ΅Ρ€Π°Ρ… ΡˆΠ°Ρ€ΠΈΠΊΠ° ΠΏΠΎ ΡΡ€Π°Π²Π½Π΅Π½ΠΈΡŽ с Π΄Π»ΠΈΠ½ΠΎΠΉ Π½ΠΈΡ‚ΠΈ ΠΈ Π½Π΅Π±ΠΎΠ»ΡŒΡˆΠΈΡ… отклонСниях (Π΄ΠΎ 10Β°) ΠΎΡ‚ полоТСния равновСсия ΠΏΠ΅Ρ€ΠΈΠΎΠ΄ ΠΊΠΎΠ»Π΅Π±Π°Π½ΠΈΠΉ Ρ€Π°Π²Π΅Π½ ΠΏΠ΅Ρ€ΠΈΠΎΠ΄Ρƒ ΠΊΠΎΠ»Π΅Π±Π°Π½ΠΈΠΉ матСматичСского маятника


Π‘ Π΄Ρ€ΡƒΠ³ΠΎΠΉ стороны ΠΏΠ΅Ρ€ΠΈΠΎΠ΄ ΠΊΠΎΠ»Π΅Π±Π°Π½ΠΈΠΉ маятника ΠΌΠΎΠΆΠ½ΠΎ Ρ€Π°ΡΡ‡ΠΈΡ‚Π°Ρ‚ΡŒ ΠΈΠ· опрСдСлСния, вСдь ΠΏΠ΅Ρ€ΠΈΠΎΠ΄ – это врСмя ΠΎΠ΄Π½ΠΎΠ³ΠΎ ΠΏΠΎΠ»Π½ΠΎΠ³ΠΎ колСбания. Π’ΠΎΠ³Π΄Π° ΠΏΠ΅Ρ€ΠΈΠΎΠ΄

ΠΈ ускорСния свободного падСния ΠΌΠΎΠΆΠ΅Ρ‚ Π±Ρ‹Ρ‚ΡŒ вычислСно ΠΏΠΎ Ρ„ΠΎΡ€ΠΌΡƒΠ»Π΅

Π₯ΠΎΠ΄ Ρ€Π°Π±ΠΎΡ‚Ρ‹

Для Π½Π°Ρ‡Π°Π»Π° я ΠΏΡ€ΠΎΠ΄Π΅Π»Π°Π» всС Π½Π΅ΠΎΠ±Ρ…ΠΎΠ΄ΠΈΠΌΡ‹Π΅ измСрСния Π² классС, Π² ΠΊΠ°Π±ΠΈΠ½Π΅Ρ‚Π΅ Ρ„ΠΈΠ·ΠΈΠΊΠΈ ЛицСя β„– 110. ΠšΠ°Π±ΠΈΠ½Π΅Ρ‚ находится Π½Π° Π²Ρ‚ΠΎΡ€ΠΎΠΌ этаТС. Учитывая высоту ΠΏΠΎΡ‚ΠΎΠ»ΠΊΠΎΠ² (ΠΎΠΊΠΎΠ»ΠΎ 3Β ΠΌ), Π»ΠΎΠ³ΠΈΡ‡Π½ΠΎ ΠΏΡ€Π΅Π΄ΠΏΠΎΠ»ΠΎΠΆΠΈΡ‚ΡŒ, Ρ‡Ρ‚ΠΎ вычислСнныС значСния g Π΄ΠΎΠ»ΠΆΠ½Ρ‹ Π±Ρ‹Ρ‚ΡŒ Π±Π»ΠΈΠ·ΠΊΠΈ ΠΊ gΡ‚.

  1. Π― установил Π½Π° ΠΊΡ€Π°ΡŽ стола ΡˆΡ‚Π°Ρ‚ΠΈΠ². Π£ Π΅Π³ΠΎ Π²Π΅Ρ€Ρ…Π½Π΅Π³ΠΎ ΠΊΠΎΠ½Ρ†Π° ΡƒΠΊΡ€Π΅ΠΏΠΈΠ» с ΠΏΠΎΠΌΠΎΡ‰ΡŒΡŽ ΠΌΡƒΡ„Ρ‚Ρ‹ ΠΊΠΎΠ»ΡŒΡ†ΠΎ ΠΈ подвСсил ΠΊ Π½Π΅ΠΌΡƒ ΡˆΠ°Ρ€ΠΈΠΊ Π½Π° Π½ΠΈΡ‚ΠΈ. Π¨Π°Ρ€ΠΈΠΊ Π΄ΠΎΠ»ΠΆΠ΅Π½ Π²ΠΈΡΠ΅Ρ‚ΡŒ ΠΈ свободно ΡΠΎΠ²Π΅Ρ€ΡˆΠ°Ρ‚ΡŒ колСбания.
  2. ΠΠΈΡ‚ΡŒ я взял ΠΌΠ΅Ρ‚Ρ€ΠΎΠ²ΠΎΠΉ Π΄Π»ΠΈΠ½Ρ‹ для удобства вычислСний.
  3. ΠžΡ‚ΠΊΠ»ΠΎΠ½ΠΈΠ² ΡˆΠ°Ρ€ΠΈΠΊ Π½Π° нСбольшоС расстояниС (5-8 см), я Π²ΠΎΠ·Π±ΡƒΠ΄ΠΈΠ» колСбания маятника.
  4. Π˜Π·ΠΌΠ΅Ρ€ΠΈΠ» Π² пяти экспСримСнтах врСмя t 20 ΠΊΠΎΠ»Π΅Π±Π°Π½ΠΈΠΉ маятника ΠΈ вычислил tср:
tср =Β  t1 + t2 + t3 + t4 + t5
5
  1. Π—Π°Ρ‚Π΅ΠΌ вычислил ΡΡ€Π΅Π΄Π½ΡŽΡŽ Π°Π±ΡΠΎΠ»ΡŽΡ‚Π½ΡƒΡŽ ΠΏΠΎΠ³Ρ€Π΅ΡˆΠ½ΠΎΡΡ‚ΡŒ измСрСния Π²Ρ€Π΅ΠΌΠ΅Π½ΠΈ:
βˆ†tср =Β  β”‚t1 – tср│ + β”‚t2 – tср│+ β”‚t3– tср│ + β”‚t4– tср│ + β”‚t5– tср│
5
  1. Вычислил ускорСниС свободного падСния ΠΏΠΎ Ρ„ΠΎΡ€ΠΌΡƒΠ»Π΅:
Π’Π°Π±Π»ΠΈΡ†Π° Ρ€Π΅Π·ΡƒΠ»ΡŒΡ‚Π°Ρ‚ΠΎΠ² ΠΈΠ·ΠΌΠ΅Ρ€Π΅Π½ΠΈΠΉ Π² классС
n

N

t, c

tср, с

Ξ”tср, с

g, м/с2

1

20

40,26

39,94

0,36

9,88924

2

20

39,20

3

20

40,30

4

20

40,18

5

20

39,78

  1. Π― ΠΎΠΏΡ€Π΅Π΄Π΅Π»ΠΈΠ» ΠΎΡ‚Π½ΠΎΡΠΈΡ‚Π΅Π»ΡŒΠ½ΡƒΡŽ ΠΏΠΎΠ³Ρ€Π΅ΡˆΠ½ΠΎΡΡ‚ΡŒ измСрСния Π²Ρ€Π΅ΠΌΠ΅Π½ΠΈ Ξ΅t.
Ρ =Β  βˆ†t Β =Β  βˆ†tΠΈ + βˆ†tотсчСта Β =Β  1 с + 1 с Β =Β  2 c Β =Β  2 с Β =Β 0,05Β =Β 5%
t t t tсрСдн 39,94 с
  1. ΠžΠΏΡ€Π΅Π΄Π΅Π»ΠΈΠ» ΠΎΡ‚Π½ΠΎΡΠΈΡ‚Π΅Π»ΡŒΠ½ΡƒΡŽ ΠΏΠΎΠ³Ρ€Π΅ΡˆΠ½ΠΎΡΡ‚ΡŒ измСрСния Π΄Π»ΠΈΠ½Ρ‹ маятника:
Ξ΅lΒ =Β  βˆ†l Β =Β  βˆ†lΠΈ + βˆ†lотсчСта Β =Β  ΠΏΠΎΠ»ΠΎΠ²ΠΈΠ½Π° Ρ†Π΅Π½Ρ‹ дСлСния + Ρ†Π΅Π½Π° дСлСния Β =Β 
l l Π΄Π»ΠΈΠ½Π° маятника

Β 

=Β  0,0005 ΠΌ + 0,001 ΠΌ Β =Β  0,0015 ΠΌ Β =Β  0,0015 ΠΌ Β =Β 0,0015 = 0,15%
l l 1 ΠΌ
  1. Вычислил ΠΎΡ‚Π½ΠΎΡΠΈΡ‚Π΅Π»ΡŒΠ½ΡƒΡŽ ΠΏΠΎΠ³Ρ€Π΅ΡˆΠ½ΠΎΡΡ‚ΡŒ измСрСния g:

    Ξ΅g = Ξ΅l+ 2Ξ΅t = 0,05 + 2 Β· 0,0015 = 0,053 = 5,3%

  2. Β ΠžΠΏΡ€Π΅Π΄Π΅Π»ΠΈΠ» Π°Π±ΡΠΎΠ»ΡŽΡ‚Π½ΡƒΡŽ ΠΏΠΎΠ³Ρ€Π΅ΡˆΠ½ΠΎΡΡ‚ΡŒ вычислСния ускорСния свободного падСния:

    βˆ†g = Ξ΅ggсрСдняя = 0,053 Β· 9,73971 ΠΌ/с2 = 0,5162 ΠΌ/с2 β‰ˆ 0,520

Π˜Ρ‚ΠΎΠ³ ΠΌΠΎΠΈΡ… ΠΈΠ·ΠΌΠ΅Ρ€Π΅Π½ΠΈΠΉ ΠΈ вычислСний:

9,37 ≀ g ≀ 10,41

Π’Π°ΠΊΠΈΠ΅ дСйствия я ΠΏΡ€ΠΎΠ΄Π΅Π»Π°Π» Π² казанском ΠΌΠ΅Ρ‚Ρ€ΠΎΠΏΠΎΠ»ΠΈΡ‚Π΅Π½Π΅, Π½Π° станции ΠΌΠ΅Ρ‚Ρ€ΠΎ ΠšΡ€Π΅ΠΌΠ»Π΅Π²ΡΠΊΠ°Ρ ΠΈ Π½Π° 36-ΠΌ этаТС СдинствСнного Π² Казани нСбоскрСба Β«Π›Π°Π·ΡƒΡ€Π½Ρ‹Π΅ нСбСса».

Π’Π°Π±Π»ΠΈΡ†Π° Ρ€Π΅Π·ΡƒΠ»ΡŒΡ‚Π°Ρ‚ΠΎΠ² ΠΈΠ·ΠΌΠ΅Ρ€Π΅Π½ΠΈΠΉ Π½Π° станции ΠΌΠ΅Ρ‚Ρ€ΠΎ ΠšΡ€Π΅ΠΌΠ»Π΅Π²ΡΠΊΠ°Ρ

n

N

t, c

tср, с

Ξ”tср, с

g, м/с2

1

20

31,80

31,71

0,042

9,96232

2

20

31,72

3

20

31,62

4

20

31,69

5

20

31,71

ΠŸΡ€ΠΈ измСрСниях Π² ΠΌΠ΅Ρ‚Ρ€ΠΎ ΠΏΡ€ΠΈΡˆΠ»ΠΎΡΡŒ ΠΈΡΠΏΠΎΠ»ΡŒΠ·ΠΎΠ²Π°Ρ‚ΡŒ Π΄Π»ΠΈΠ½Ρƒ Π½ΠΈΡ‚ΠΈ 63,5 см.

ΠžΡ‚Π½ΠΎΡΠΈΡ‚Π΅Π»ΡŒΠ½Π°Ρ ΠΏΠΎΠ³Ρ€Π΅ΡˆΠ½ΠΎΡΡ‚ΡŒ измСрСния Π²Ρ€Π΅ΠΌΠ΅Π½ΠΈ Ξ΅t = 0,063 = 6,3%.

ΠžΡ‚Π½ΠΎΡΠΈΡ‚Π΅Π»ΡŒΠ½Π°Ρ ΠΏΠΎΠ³Ρ€Π΅ΡˆΠ½ΠΎΡΡ‚ΡŒ измСрСния Π΄Π»ΠΈΠ½Ρ‹ маятника: Ξ΅l = 0,24%

ΠžΡ‚Π½ΠΎΡΠΈΡ‚Π΅Π»ΡŒΠ½Π°Ρ ΠΏΠΎΠ³Ρ€Π΅ΡˆΠ½ΠΎΡΡ‚ΡŒ измСрСния g: Ξ΅g = 6,78%

ΠΠ±ΡΠΎΠ»ΡŽΡ‚Π½ΡƒΡŽ ΠΏΠΎΠ³Ρ€Π΅ΡˆΠ½ΠΎΡΡ‚ΡŒ вычислСния ускорСния свободного падСния составила: 0,63 ΠΌ/с2.

Π˜Ρ‚ΠΎΠ³ ΠΌΠΎΠΈΡ… ΠΈΠ·ΠΌΠ΅Ρ€Π΅Π½ΠΈΠΉ ΠΈ вычислСний:

9,33 ≀ g ≀ 10,59

Π’Π°Π±Π»ΠΈΡ†Π° Ρ€Π΅Π·ΡƒΠ»ΡŒΡ‚Π°Ρ‚ΠΎΠ² ΠΈΠ·ΠΌΠ΅Ρ€Π΅Π½ΠΈΠΉ Π½Π° 36-ΠΌ этаТС нСбоскрСба Β«Π›Π°Π·ΡƒΡ€Π½Ρ‹Π΅ нСбСса»
n

N

t, c

tср, с

Ξ”tср, с

g, м/с2

1

20

28,59

28,57

0,10

9,85664

2

20

28,56

3

20

28,81

4

20

28,52

5

20

28,39

Π—Π΄Π΅ΡΡŒ ΠΏΡ€ΠΈ измСрСниях ΠΏΡ€ΠΈΡˆΠ»ΠΎΡΡŒ Π΄Π»ΠΈΠ½Ρƒ Π½ΠΈΡ‚ΠΈ Π΅Ρ‰Π΅ ΡΠΎΠΊΡ€Π°Ρ‚ΠΈΡ‚ΡŒ Π΄ΠΎ 51 см.

ΠžΡ‚Π½ΠΎΡΠΈΡ‚Π΅Π»ΡŒΠ½Π°Ρ ΠΏΠΎΠ³Ρ€Π΅ΡˆΠ½ΠΎΡΡ‚ΡŒ измСрСния Π²Ρ€Π΅ΠΌΠ΅Π½ΠΈ Ξ΅t = 7%.

ΠžΡ‚Π½ΠΎΡΠΈΡ‚Π΅Π»ΡŒΠ½Π°Ρ ΠΏΠΎΠ³Ρ€Π΅ΡˆΠ½ΠΎΡΡ‚ΡŒ измСрСния Π΄Π»ΠΈΠ½Ρ‹ маятника: Ξ΅l = 0,29%

ΠžΡ‚Π½ΠΎΡΠΈΡ‚Π΅Π»ΡŒΠ½Π°Ρ ΠΏΠΎΠ³Ρ€Π΅ΡˆΠ½ΠΎΡΡ‚ΡŒ измСрСния g: Ξ΅g = 7,58%

ΠΠ±ΡΠΎΠ»ΡŽΡ‚Π½ΡƒΡŽ ΠΏΠΎΠ³Ρ€Π΅ΡˆΠ½ΠΎΡΡ‚ΡŒ вычислСния ускорСния свободного падСния составила: 0,75 ΠΌ/с2.

Π˜Ρ‚ΠΎΠ³ ΠΌΠΎΠΈΡ… ΠΈΠ·ΠΌΠ΅Ρ€Π΅Π½ΠΈΠΉ ΠΈ вычислСний:

9,11 ≀ g ≀ 10,61

Π’Π°Π±Π»ΠΈΡ†Π° сравнСния тСорСтичСски ΠΏΠΎΠ»ΡƒΡ‡Π΅Π½Π½Ρ‹Ρ… Π·Π½Π°Ρ‡Π΅Π½ΠΈΠΉ g (ΠΌ/с2) ΠΈ ΠΏΠΎΠ»ΡƒΡ‡Π΅Π½Π½Ρ‹Ρ… ΡΠΊΡΠΏΠ΅Ρ€ΠΈΠΌΠ΅Π½Ρ‚Π°Π»ΡŒΠ½ΠΎ

Β 

Π’ классС

На станции ΠΌΠ΅Ρ‚Ρ€ΠΎ ΠšΡ€Π΅ΠΌΠ»Π΅Π²ΡΠΊΠ°Ρ

На 36-ΠΌ этаТС нСбоскрСба

R = 6370 ΠΊΠΌ,

h = 0

R = 6370 ΠΊΠΌ,

h = –16 ΠΌ

R = 6370 ΠΊΠΌ,

h = +120 ΠΌ

ВСория

9,8234

9,8231

9,8227

ЭкспСримСнт

9,8892

9,9623

9,8566

3.

Π—Π°ΠΊΠ»ΡŽΡ‡Π΅Π½ΠΈΠ΅

ΠŸΡ€ΠΈ ΠΏΠΎΠ΄Π³ΠΎΡ‚ΠΎΠ²ΠΊΠ΅ ΠΊ Π·Π°Ρ‰ΠΈΡ‚Π΅ Π΄Π°Π½Π½ΠΎΠΉ Ρ€Π°Π±ΠΎΡ‚Ρ‹ ΠΈ Π² Ρ€Π΅Π·ΡƒΠ»ΡŒΡ‚Π°Ρ‚Π΅ тСорСтичСского исслСдования, чтСния Ρ€Π°Π·Π½Ρ‹Ρ… ΠΊΠ½ΠΈΠ³ ΠΈ статСй я ΡƒΠ·Π½Π°Π» ΠΌΠ½ΠΎΠ³ΠΎΠ΅ ΠΎΠ± ускорСнии свободного падСния. Как ΡƒΠΆΠ΅ ΡƒΠΏΠΎΠΌΠΈΠ½Π°Π», для мСня практичСская Π·Π½Π°Ρ‡ΠΈΠΌΠΎΡΡ‚ΡŒ исслСдования Π·Π°ΠΊΠ»ΡŽΡ‡Π°Π΅Ρ‚ΡΡ Π² возмоТности прогнозирования Ρ„ΠΎΡ€ΠΌ ΠΆΠΈΠ·Π½ΠΈ Π½Π° нСбСсных Ρ‚Π΅Π»Π°Ρ…, с ΠΊΠΎΡ‚ΠΎΡ€Ρ‹ΠΌΠΈ чСловСчСство столкнСтся ΠΏΡ€ΠΈ Π½Π΅ΠΈΠ·Π±Π΅ΠΆΠ½ΠΎΠΌ освоСнии космоса. Π’Π΅Π΄ΡŒ люди Π·Π°Ρ€Π°Π½Π΅Π΅ смогут ΡƒΠ·Π½Π°Ρ‚ΡŒ, ΠΊΠ°ΠΊΠΈΠ΅ сущСства встрСтят ΠΈΡ… Π½Π° Ρ‚ΠΎΠΉ ΠΈΠ»ΠΈ ΠΈΠ½ΠΎΠΉ ΠΏΠ»Π°Π½Π΅Ρ‚Π΅, ΠΊΠ°ΠΊΠΈΠΌΠΈ физичСскими характСристиками ΠΎΠ½ΠΈ Π±ΡƒΠ΄ΡƒΡ‚ ΠΎΠ±Π»Π°Π΄Π°Ρ‚ΡŒ.

Π’Π°ΠΊΠΆΠ΅ я ΡƒΠ·Π½Π°Π», Ρ‡Ρ‚ΠΎ расчСты различия ускорСния свободного падСния Π² Ρ€Π°Π·Π½Ρ‹Ρ… Ρ‚ΠΎΡ‡ΠΊΠ°Ρ… Π½Π° повСрхности Π—Π΅ΠΌΠ»ΠΈ ΠΌΠΎΠ³ΡƒΡ‚ ΡƒΠΊΠ°Π·Ρ‹Π²Π°Ρ‚ΡŒ Π½Π° Π³Ρ€Π°Π²ΠΈΡ‚Π°Ρ†ΠΈΠΎΠ½Π½Ρ‹Π΅ Π°Π½ΠΎΠΌΠ°Π»ΠΈΠΈ.

Π‘Π°ΠΌΠΎΠ΅ Π³Π»Π°Π²Π½ΠΎΠ΅, я научился ΠΈΠ·ΠΌΠ΅Ρ€ΡΡ‚ΡŒ g, Ρ€Π°Π·Π»ΠΈΡ‡Π½Ρ‹ΠΌΠΈ способами, Ρ€Π°ΡΡΡ‡ΠΈΡ‚Ρ‹Π²Π°Ρ‚ΡŒ ΠΏΠΎΠ³Ρ€Π΅ΡˆΠ½ΠΎΡΡ‚ΠΈ ΠΈΠ·ΠΌΠ΅Ρ€Π΅Π½ΠΈΠΉ, Π³Ρ€Π°ΠΌΠΎΡ‚Π½ΠΎ ΠΏΡ€ΠΎΠ²ΠΎΠ΄ΠΈΡ‚ΡŒ экспСримСнт.

Π‘Ρ‡ΠΈΡ‚Π°ΡŽ Ρ†Π΅Π»ΡŒ исслСдования достигнута. Π‘Ρ€Π΅Π΄Π½ΠΈΠ΅ Π·Π½Π°Ρ‡Π΅Π½ΠΈΠ΅ ускорСния свободного падСния Π½Π° Ρ€Π°Π·Π»ΠΈΡ‡Π½Ρ‹Ρ… высотах ΠΎΡ‚Π»ΠΈΡ‡Π°ΡŽΡ‚ΡΡ Π² зависимости ΠΎΡ‚ высоты Π½Π°Π΄ ΡƒΡ€ΠΎΠ²Π½Π΅ΠΌ моря: ΠΏΡ€ΠΈ ΡƒΠ²Π΅Π»ΠΈΡ‡Π΅Π½ΠΈΠΈ высоты Π·Π½Π°Ρ‡Π΅Π½ΠΈΠ΅ g ΡƒΠΌΠ΅Π½ΡŒΡˆΠ°Π΅Ρ‚ΡΡ, ΠΏΡ€ΠΈ ΡƒΠ³Π»ΡƒΠ±Π»Π΅Π½ΠΈΠΈ Π² Π½Π΅Π΄Ρ€Π° Π—Π΅ΠΌΠ»ΠΈ – увСличиваСтся. Π­ΠΊΡΠΏΠ΅Ρ€ΠΈΠΌΠ΅Π½Ρ‚Π°Π»ΡŒΠ½ΠΎ ΠΏΠΎΠ»ΡƒΡ‡Π΅Π½Π½Ρ‹Π΅ значСния Ρ…ΠΎΡ€ΠΎΡˆΠΎ это ΠΏΠΎΠΊΠ°Π·Ρ‹Π²Π°ΡŽΡ‚.

ΠŸΠΎΠ³Ρ€Π΅ΡˆΠ½ΠΎΡΡ‚ΡŒ ΠΈΠ·ΠΌΠ΅Ρ€Π΅Π½ΠΈΠΉ достаточно Π²Π΅Π»ΠΈΠΊΠ°, Π½ΠΎ Π½Π΅ ΠΏΡ€Π΅Π²Ρ‹ΡˆΠ°Π΅Ρ‚ 10%. Π£ΠΌΠ΅Π½ΡŒΡˆΠΈΡ‚ΡŒ ΠΏΠΎΠ³Ρ€Π΅ΡˆΠ½ΠΎΡΡ‚ΡŒ Π²ΠΎΠ·ΠΌΠΎΠΆΠ½ΠΎ ΠΏΡƒΡ‚Π΅ΠΌ провСдСния большСго числа ΠΈΠ·ΠΌΠ΅Ρ€Π΅Π½ΠΈΠΉ: Π½ΠΈ 5, Π° 20; большСго числа ΠΊΠΎΠ»Π΅Π±Π°Π½ΠΈΠΉ: Π½Π΅ 20, Π° 100. Π’Π°ΠΊΠΆΠ΅ ΠΏΡ€ΠΈ расчСтах ΠΌΠΎΠΆΠ½ΠΎ ΡƒΡ‡Π΅ΡΡ‚ΡŒ, Ρ‡Ρ‚ΠΎ Казань находится ΠΏΡ€ΠΈΠΌΠ΅Ρ€Π½ΠΎ Π½Π° ΡƒΡ€ΠΎΠ²Π½Π΅ 250-300 ΠΌ Π½Π°Π΄ ΡƒΡ€ΠΎΠ²Π½Π΅ΠΌ моря.

Π’ дальнСйшСм Ρ…ΠΎΡ‚Π΅Π»ΠΎΡΡŒ Π±Ρ‹ ΡƒΡΠΎΠ²Π΅Ρ€ΡˆΠ΅Π½ΡΡ‚Π²ΠΎΠ²Π°Ρ‚ΡŒ ΡΠΊΡΠΏΠ΅Ρ€ΠΈΠΌΠ΅Π½Ρ‚Π°Π»ΡŒΠ½Ρ‹Π΅ установки, Ρ‡Ρ‚ΠΎΠ±Ρ‹ ΠΈΠ·ΠΌΠ΅Ρ€ΡΡ‚ΡŒ ускорСниС свободного падСния с большСй Ρ‚ΠΎΡ‡Π½ΠΎΡΡ‚ΡŒΡŽ.

ΠŸΠ»Π°Π½ΠΈΡ€ΡƒΡŽ ΡΠ°ΠΌΠΎΡΡ‚ΠΎΡΡ‚Π΅Π»ΡŒΠ½ΠΎ ΠΈΡΡΠ»Π΅Π΄ΠΎΠ²Π°Ρ‚ΡŒ значСния ускорСния свободного падСния Π² Ρ€Π°Π·Π»ΠΈΡ‡Π½Ρ‹Ρ… ΡƒΠ³ΠΎΠ»ΠΊΠ°Ρ… Π·Π΅ΠΌΠ½ΠΎΠ³ΠΎ ΡˆΠ°Ρ€Π°.


ВзаимодСйствиС Ρ‚Π΅Π», ΠΈΠ½Π΅Ρ€Ρ‚Π½ΠΎΡΡ‚ΡŒ, масса β€” Π—Π€Π’Π¨, МЀВИ

Из наблюдСний ΠΌΠΎΠΆΠ½ΠΎ Π·Π°ΠΌΠ΅Ρ‚ΠΈΡ‚ΡŒ, Ρ‡Ρ‚ΠΎ Ρ‚Π΅Π»Π° ΠΈΠ·ΠΌΠ΅Π½ΡΡŽΡ‚ свою ΡΠΊΠΎΡ€ΠΎΡΡ‚ΡŒ Ρ‚ΠΎΠ»ΡŒΠΊΠΎ ΠΏΡ€ΠΈ Π½Π°Π»ΠΈΡ‡ΠΈΠΈ Π½Π΅ скомпСнсированного дСйствия. Π’.Β ΠΊ. быстрота измСнСния скорости характСризуСтся ускорСниСм Ρ‚Π΅Π»Π°, ΠΌΠΎΠΆΠ΅ΠΌ Π·Π°ΠΊΠ»ΡŽΡ‡ΠΈΡ‚ΡŒ, Ρ‡Ρ‚ΠΎ ΠΏΡ€ΠΈΡ‡ΠΈΠ½ΠΎΠΉ ускорСния являСтся нСкомпСнсированноС дСйствиС ΠΎΠ΄Π½ΠΎΠ³ΠΎ Ρ‚Π΅Π»Π° Π½Π° Π΄Ρ€ΡƒΠ³ΠΎΠ΅. Но ΠΎΠ΄Π½ΠΎ Ρ‚Π΅Π»ΠΎ Π½Π΅ ΠΌΠΎΠΆΠ΅Ρ‚ Π΄Π΅ΠΉΡΡ‚Π²ΠΎΠ²Π°Ρ‚ΡŒ Π½Π° Π΄Ρ€ΡƒΠ³ΠΎΠ΅, Π½Π΅ испытывая Π΅Π³ΠΎ дСйствия Π½Π° сСбС. Π‘Π»Π΅Π΄ΠΎΠ²Π°Ρ‚Π΅Π»ΡŒΠ½ΠΎ, ускорСниС появляСтся ΠΏΡ€ΠΈ взаимодСйствии Ρ‚Π΅Π». УскорСниС ΠΏΡ€ΠΈΠΎΠ±Ρ€Π΅Ρ‚Π°ΡŽΡ‚ ΠΎΠ±Π° Π²Π·Π°ΠΈΠΌΠΎΠ΄Π΅ΠΉΡΡ‚Π²ΡƒΡŽΡ‰ΠΈΠ΅ Ρ‚Π΅Π»Π°. Π’Π°ΠΊ ΠΆΠ΅ ΠΈΠ· наблюдСний ΠΌΠΎΠΆΠ½ΠΎ ΡƒΡΡ‚Π°Π½ΠΎΠ²ΠΈΡ‚ΡŒ Π΅Ρ‰Ρ‘ ΠΎΠ΄ΠΈΠ½ Ρ„Π°ΠΊΡ‚: ΠΏΡ€ΠΈ ΠΎΠ΄ΠΈΠ½Π°ΠΊΠΎΠ²ΠΎΠΌ дСйствии Ρ€Π°Π·Π½Ρ‹Π΅ Ρ‚Π΅Π»Π° ΠΏΡ€ΠΈΠΎΠ±Ρ€Π΅Ρ‚Π°ΡŽΡ‚ Ρ€Π°Π·Π½Ρ‹Π΅ ускорСния.

Π£ΡΡ‚Π°Π½ΠΎΠ²ΠΈΠ»ΠΈΡΡŒ ΡΡ‡ΠΈΡ‚Π°Ρ‚ΡŒ: Ρ‡Π΅ΠΌ мСньшС ускорСниС ΠΏΡ€ΠΈΠΎΠ±Ρ€Π΅Ρ‚Π°Π΅Ρ‚ Ρ‚Π΅Π»ΠΎ ΠΏΡ€ΠΈ взаимодСйствии, тСм инСртнСС это Ρ‚Π΅Π»ΠΎ.

Π˜Π½Π΅Ρ€Ρ‚Π½ΠΎΡΡ‚ΡŒΒ β€“ это свойство Ρ‚Π΅Π»Π° ΡΠΎΡ…Ρ€Π°Π½ΡΡ‚ΡŒ свою ΡΠΊΠΎΡ€ΠΎΡΡ‚ΡŒ постоянной (Ρ‚ΠΎ ΠΆΠ΅, Ρ‡Ρ‚ΠΎ ΠΈ инСрция). ΠŸΡ€ΠΎΡΠ²Π»ΡΠ΅Ρ‚ сСбя Π² Ρ‚ΠΎΠΌ, Ρ‡Ρ‚ΠΎ для измСнСния скорости Ρ‚Π΅Π»Π° трСбуСтся Π½Π΅ΠΊΠΎΡ‚ΠΎΡ€ΠΎΠ΅ врСмя. ΠŸΡ€ΠΎΡ†Π΅ΡΡ измСнСния скорости Π½Π΅ ΠΌΠΎΠΆΠ΅Ρ‚ Π±Ρ‹Ρ‚ΡŒ ΠΌΠ³Π½ΠΎΠ²Π΅Π½Π½Ρ‹ΠΌ.

НапримСр, двиТущийся ΠΏΠΎ Π΄ΠΎΡ€ΠΎΠ³Π΅ Π°Π²Ρ‚ΠΎΠΌΠΎΠ±ΠΈΠ»ΡŒ Π½Π΅ ΠΌΠΎΠΆΠ΅Ρ‚ ΠΌΠ³Π½ΠΎΠ²Π΅Π½Π½ΠΎ ΠΎΡΡ‚Π°Π½ΠΎΠ²ΠΈΡ‚ΡŒΡΡ, для ΡƒΠΌΠ΅Π½ΡŒΡˆΠ΅Π½ΠΈΡ скорости трСбуСтся Π½Π΅ΠΊΠΎΡ‚ΠΎΡ€ΠΎΠ΅ врСмя, Π° Π·Π° это врСмя ΠΎΠ½ успСваСт ΠΏΠ΅Ρ€Π΅ΠΌΠ΅ΡΡ‚ΠΈΡ‚ΡŒΡΡ Π½Π° довольно большоС расстояниС (дСсятки ΠΌΠ΅Ρ‚Ρ€ΠΎΠ²). (ΠžΡΡ‚ΠΎΡ€ΠΎΠΆΠ½ΠΎ ΠΏΠ΅Ρ€Π΅Ρ…ΠΎΠ΄ΠΈΡ‚Π΅ Π΄ΠΎΡ€ΠΎΠ³Ρƒ!!!)

ΠœΠ΅Ρ€ΠΎΠΉ инСртности являСтся инСртная масса.

Масса (инСртная) – ΠΌΠ΅Ρ€Π° инСртности Ρ‚Π΅Π»Π°.

Π§Π΅ΠΌ ΠΈΠ½Π΅Ρ€Ρ‚Π½Π΅Π΅ Ρ‚Π΅Π»ΠΎ, Ρ‚Π΅ΠΌ большС Π΅Π³ΠΎ масса. Π§Π΅ΠΌ большС ΠΈΠ½Π΅Ρ€Ρ‚Π½ΠΎΡΡ‚ΡŒ, Ρ‚Π΅ΠΌ мСньшС ускорСниС. Π‘Π»Π΅Π΄ΠΎΠ²Π°Ρ‚Π΅Π»ΡŒΠ½ΠΎ, Ρ‡Π΅ΠΌ большС масса Ρ‚Π΅Π»Π°, Ρ‚Π΅ΠΌ мСньшС Π΅Π³ΠΎ ускорСниС: a∼1m\boxed{a\sim\frac 1m}.

Данная Π·Π°Π²ΠΈΡΠΈΠΌΠΎΡΡ‚ΡŒ записана СдинствСнно ΠΏΡ€Π°Π²ΠΈΠ»ΡŒΠ½Ρ‹ΠΌ способом, Ρ‚. ΠΊ.Β Ρ„ΠΎΡ€ΠΌΠ° m∼1am \sim \frac 1aΒ Π½Π΅ Π²Π΅Ρ€Π½Π°. Масса Π½Π΅ ΠΌΠΎΠΆΠ΅Ρ‚ Π·Π°Π²ΠΈΡΠ΅Ρ‚ΡŒ ΠΎΡ‚ ускорСния, ΠΎΠ½Π° являСтся свойством Ρ‚Π΅Π»Π°, Π° ускорСниС являСтся характСристикой состояния двиТСния Ρ‚Π΅Π»Π°.

Данная Π·Π°Π²ΠΈΡΠΈΠΌΠΎΡΡ‚ΡŒ подтвСрТдаСтся многочислСнными ΠΎΠΏΡ‹Ρ‚Π½Ρ‹ΠΌΠΈ Ρ€Π΅Π·ΡƒΠ»ΡŒΡ‚Π°Ρ‚Π°ΠΌΠΈ.

Рис. 2 Π˜Π·ΠΌΠ΅Ρ€Π΅Π½ΠΈΠ΅ массы ΠΌΠ΅Ρ‚ΠΎΠ΄ΠΎΠΌ взаимодСйствия Ρ‚Π΅Π».

Π”Π²Π° Ρ‚Π΅Π»Π°, скрСплённыС ΠΌΠ΅ΠΆΠ΄Ρƒ собой сТатой ΠΏΡ€ΡƒΠΆΠΈΠ½ΠΎΠΉ, послС пСрСТигания Π½ΠΈΡ‚ΠΈ, ΡƒΠ΄Π΅Ρ€ΠΆΠΈΠ²Π°ΡŽΡ‰Π΅ΠΉ ΠΏΡ€ΡƒΠΆΠΈΠ½Ρƒ, Π½Π°Ρ‡ΠΈΠ½Π°ΡŽΡ‚ Π΄Π²ΠΈΠ³Π°Ρ‚ΡŒΡΡ Π½Π΅ ΠΊΠΎΡ‚ΠΎΡ€ΠΎΠ΅ врСмя с ускорСниСм (рис. 1) . ΠžΠΏΡ‹Ρ‚ ΠΏΠΎΠΊΠ°Π·Ρ‹Π²Π°Π΅Ρ‚, Ρ‡Ρ‚ΠΎ ΠΏΡ€ΠΈ Π»ΡŽΠ±Ρ‹Ρ… взаимодСйствиях Π΄Π°Π½Π½Ρ‹Ρ… Π΄Π²ΡƒΡ… Ρ‚Π΅Π» ΠΎΡ‚Π½ΠΎΡˆΠ΅Π½ΠΈΠ΅ ускорСний Ρ‚Π΅Π» Ρ€Π°Π²Π½ΠΎ ΠΎΠ±Ρ€Π°Ρ‚Π½ΠΎΠΌΡƒ ΠΎΡ‚Π½ΠΎΡˆΠ΅Π½ΠΈΡŽ ΠΈΡ… масс:

\[\frac{a_1}{a_2} = \frac{m_2}{m_1};\]

Ссли Π²Π·ΡΡ‚ΡŒ ΠΏΠ΅Ρ€Π²ΡƒΡŽ массу Π·Π° ΡΡ‚Π°Π»ΠΎΠ½Π½ΡƒΡŽ (m1=mэтm_1 = m_\mathrm{эт}), Ρ‚ΠΎ m2=mэтaэтa2m_2 = m_\mathrm{эт}\frac{a_\mathrm{эт}}{a_2}.

Масса, измСрСнная ΠΏΡƒΡ‚Ρ‘ΠΌ взаимодСйствия (измСрСния ускорСния), называСтся ΠΈΠ½Π΅Ρ€Ρ‚Π½ΠΎΠΉ.

Π˜Π·ΠΌΠ΅Ρ€Π΅Π½ΠΈΠ΅ массы ΠΌΠ΅Ρ‚ΠΎΠ΄ΠΎΠΌ взвСшивания Ρ‚Π΅Π».

Π’Ρ‚ΠΎΡ€ΠΎΠΉ способ измСрСния масс основан Π½Π° сравнСнии дСйствия Π—Π΅ΠΌΠ»ΠΈ Π½Π° Ρ€Π°Π·Π»ΠΈΡ‡Π½Ρ‹Π΅ Ρ‚Π΅Π»Π°. Π’Π°ΠΊΠΎΠ΅ сравнСниС ΠΌΠΎΠΆΠ½ΠΎ ΠΎΡΡƒΡ‰Π΅ΡΡ‚Π²ΠΈΡ‚ΡŒ Π»ΠΈΠ±ΠΎ ΠΏΠΎΡΠ»Π΅Π΄ΠΎΠ²Π°Ρ‚Π΅Π»ΡŒΠ½ΠΎ (сначала ΠΎΠΏΡ€Π΅Π΄Π΅Π»ΡΡŽΡ‚ растяТСниС ΠΏΡ€ΡƒΠΆΠΈΠ½Ρ‹ ΠΏΠΎΠ΄ дСйствиСм эталонных масс, Π° ΠΏΠΎΡ‚ΠΎΠΌ ΠΏΠΎΠ΄ дСйствиСм исслСдуСмого Ρ‚Π΅Π»Π° Π² Ρ‚Π΅Ρ… ΠΆΠ΅ условиях), Π»ΠΈΠ±ΠΎ ΠΎΠ΄Π½ΠΎΠ²Ρ€Π΅ΠΌΠ΅Π½Π½ΠΎ Ρ€Π°ΡΠΏΠΎΠ»Π°Π³Π°ΡŽΡ‚ Π½Π° Ρ€Π°Π²Π½ΠΎΠΏΠ»Π΅Ρ‡ΠΈΡ… Ρ€Ρ‹Ρ‡Π°ΠΆΠ½Ρ‹Ρ… вСсах Π½Π° ΠΎΠ΄Π½ΠΎΠΉ Ρ‡Π°ΡˆΠ΅ исслСдуСмоС Ρ‚Π΅Π»ΠΎ, Π° Π½Π° Π΄Ρ€ΡƒΠ³ΠΎΠΉ эталонныС массы (рис. 2).

Рис. 2

Рис. 3

Масса, измСрСнная ΠΏΡƒΡ‚Ρ‘ΠΌ взвСшивания, называСтся гравитационной.

Π’ качСствС эталона ΠΈ Ρ‚ΠΎΠΉ ΠΈ Π΄Ρ€ΡƒΠ³ΠΎΠΉ массы принята масса Ρ‚Π΅Π»Π°, Π²Ρ‹ΠΏΠΎΠ»Π½Π΅Π½Π½ΠΎΠ³ΠΎ Π² Ρ„ΠΎΡ€ΠΌΠ΅ Ρ†ΠΈΠ»ΠΈΠ½Π΄Ρ€Π° высотой 39Β ΠΌΠΌ39\ \mathrm{ΠΌΠΌ} ΠΈ Π΄ΠΈΠ°ΠΌΠ΅Ρ‚Ρ€ΠΎΠΌ 39Β ΠΌΠΌ39\ \mathrm{ΠΌΠΌ}, ΠΈΠ·Π³ΠΎΡ‚ΠΎΠ²Π»Π΅Π½Π½ΠΎΠ³ΠΎ ΠΈΠ· сплава 10 % иридия ΠΈ 90 % ΠΏΠ»Π°Ρ‚ΠΈΠ½Ρ‹ (рис. {-12} % эти массы Ρ€Π°Π²Π½Ρ‹.

Π”Π°Π½Π½Ρ‹ΠΉΒ Ρ„Π°ΠΊΡ‚ извСстСн Π±Ρ‹Π» ΠΈ Ρ€Π°Π½Π΅Π΅, ΠΈ послуТил основаниСм для Ρ„ΠΎΡ€ΠΌΡƒΠ»ΠΈΡ€ΠΎΠ²ΠΊΠΈ Π­ΠΉΠ½ΡˆΡ‚Π΅ΠΉΠ½ΠΎΠΌ ΠΏΡ€ΠΈΠ½Ρ†ΠΈΠΏΠ° эквивалСнтности.

ΠŸΡ€ΠΈΠ½Ρ†ΠΈΠΏ эквивалСнтности утвСрТдаСт, Ρ‡Ρ‚ΠΎ

1) ускорСниС, Π²Ρ‹Π·Π²Π°Π½Π½ΠΎΠ΅ Π³Ρ€Π°Π²ΠΈΡ‚Π°Ρ†ΠΈΠΎΠ½Π½Ρ‹ΠΌ взаимодСйствиСм Π² ΠΌΠ°Π»ΠΎΠΉ области пространства, ΠΈ Π·Π° нСбольшой ΠΈΠ½Ρ‚Π΅Ρ€Π²Π°Π» Π²Ρ€Π΅ΠΌΠ΅Π½ΠΈ, Π½Π΅ΠΎΡ‚Π»ΠΈΡ‡ΠΈΠΌΠΎ ΠΎΡ‚ ускорСнно двиТущСйся систСмы отсчёта.

2) ускорСнно двиТущССся Ρ‚Π΅Π»ΠΎ эквивалСнтно Π½Π΅ΠΏΠΎΠ΄Π²ΠΈΠΆΠ½ΠΎΠΌΡƒ Ρ‚Π΅Π»Ρƒ, находящСмуся Π² Π³Ρ€Π°Π²ΠΈΡ‚Π°Ρ†ΠΈΠΎΠ½Π½ΠΎΠΌ ΠΏΠΎΠ»Π΅.

ΠŸΡ€ΠΈΠΌΠ΅Ρ€ 1.Β 

Π”Π²Π° Ρ‚Π΅Π»Π° массами 400Β Π³400\ \mathrm{Π³} ΠΈ 600Β Π³600\ \mathrm{Π³} двигались навстрСчу Π΄Ρ€ΡƒΠ³ Π΄Ρ€ΡƒΠ³Ρƒ ΠΈ послС ΡƒΠ΄Π°Ρ€Π° ΠΎΡΡ‚Π°Π½ΠΎΠ²ΠΈΠ»ΠΈΡΡŒ. Какова ΡΠΊΠΎΡ€ΠΎΡΡ‚ΡŒ Π²Ρ‚ΠΎΡ€ΠΎΠ³ΠΎ Ρ‚Π΅Π»Π°, Ссли ΠΏΠ΅Ρ€Π²ΠΎΠ΅ двигалось со ΡΠΊΠΎΡ€ΠΎΡΡ‚ΡŒΡŽ 3Β ΠΌ/с3\ \mathrm{ΠΌ}/\mathrm{с}?

РСшСниС. 

Π‘ΠΈΠ»Π°, Π²ΠΎΠ·Π½ΠΈΠΊΠ°ΡŽΡ‰Π°Ρ ΠΏΡ€ΠΈ взаимодСйствии Ρ‚Π΅Π», ΠΊΠΎΠ½Π΅Ρ‡Π½ΠΎ ΠΆΠ΅, Π½Π΅ остаётся постоянной, ΠΈ ускорСния Ρ‚ΠΎΠΆΠ΅. ΠœΡ‹ Π±ΡƒΠ΄Π΅ΠΌ ΡΡ‡ΠΈΡ‚Π°Ρ‚ΡŒ, Ρ‡Ρ‚ΠΎ ΠΈ силы, ΠΈ ускорСния ΠΏΡ€ΠΈΠ½ΠΈΠΌΠ°ΡŽΡ‚ Π½Π΅ΠΊΠΎΡ‚ΠΎΡ€Ρ‹ Π΅ срСдниС значСния, ΠΏΡ€ΠΈΡ‡Ρ‘ΠΌ ΠΎΠ΄ΠΈΠ½Π°ΠΊΠΎΠ²Ρ‹Π΅ для любого ΠΌΠΎΠΌΠ΅Π½Ρ‚Π° Π²Ρ€Π΅ΠΌΠ΅Π½ΠΈ. ΠžΡ‚Π½ΠΎΡˆΠ΅Π½ΠΈΠ΅ ускорСний Ρ‚Π΅Π» Ρ€Π°Π²Π½ΠΎ ΠΎΠ±Ρ€Π°Ρ‚Π½ΠΎΠΌΡƒ ΠΎΡ‚Π½ΠΎΡˆΠ΅Π½ΠΈΡŽ ΠΈΡ… масс: a1a2=m2m1\frac{a_1}{a_2} = \frac{m_2}{m_1}.Β Π’ свою ΠΎΡ‡Π΅Ρ€Π΅Π΄ΡŒ, ускорСниС Ρ€Π°Π²Π½ΠΎ ΠΎΡ‚Π½ΠΎΡˆΠ΅Π½ΠΈΡŽ измСнСния скорости ΠΊΠΎ Π²Ρ€Π΅ΠΌΠ΅Π½ΠΈ измСнСния. ΠšΠΎΠ½Π΅Ρ‡Π½Ρ‹Π΅ скорости Ρ‚Π΅Π» Ρ€Π°Π²Π½Ρ‹ Π½ΡƒΠ»ΡŽ, Π° врСмя взаимодСйствия ΠΎΠ΄ΠΈΠ½Π°ΠΊΠΎΠ²ΠΎ для ΠΎΠ±ΠΎΠΈΡ… Ρ‚Π΅Π»:

\[\frac{m_2}{m_1} = \frac{a_1}{a_2} = \frac{\frac{\Delta v_1}{\Delta t}}{\frac{\Delta v_2}{\Delta t}} = \frac{v_\mathrm{ΠΊ1}-v_{01}}{v_\mathrm{ΠΊ2}-v_{02}} = \frac{v_{01}}{v_{02}},\]

ΠΎΡ‚ΠΊΡƒΠ΄Π° ΠΏΠΎΠ»ΡƒΡ‡ΠΈΠΌ ΠΈΡΠΊΠΎΠΌΡƒΡŽ ΡΠΊΠΎΡ€ΠΎΡΡ‚ΡŒ: v02=m1m2Β·v01.v_{02} = \frac{m_1}{m_2}\cdot v_{01}.

ΠšΠΎΠ»ΠΈΡ‡Π΅ΡΡ‚Π²Π΅Π½Π½ΠΎ ΠΎΡ‚Π²Π΅Ρ‚ Π±ΡƒΠ΄Π΅Ρ‚ Ρ‚Π°ΠΊΠΈΠΌ: v02=0,4Β ΠΊΠ³0,6Β ΠΊΠ³Β·3 мс=2 мсv_{02} = \frac{0,4\ \mathrm{ΠΊΠ³}}{0,6\ \mathrm{ΠΊΠ³}}\cdot 3\ \frac{\mathrm{ΠΌ}}{\mathrm{с}} = 2\ \frac{\mathrm{ΠΌ}}{\mathrm{с}}.

ΠŸΡ€ΡΠΌΠΎΠ»ΠΈΠ½Π΅ΠΉΠ½ΠΎΠ΅ равноускорСнноС Π΄Π²ΠΈΠΆΠ΅Π½ΠΈΠ΅. УскорСниС, ΡΠΊΠΎΡ€ΠΎΡΡ‚ΡŒ, ΠΏΠ΅Ρ€Π΅ΠΌΠ΅Ρ‰Π΅Π½ΠΈΠ΅

1439. ΠœΠΎΡ‚ΠΎΡ†ΠΈΠΊΠ» Π² Ρ‚Π΅Ρ‡Π΅Π½ΠΈΠ΅ 5 с ΠΌΠΎΠΆΠ΅Ρ‚ ΡƒΠ²Π΅Π»ΠΈΡ‡ΠΈΡ‚ΡŒ ΡΠΊΠΎΡ€ΠΎΡΡ‚ΡŒ ΠΎΡ‚ 0 Π΄ΠΎ 72 ΠΊΠΌ/Ρ‡. ΠžΠΏΡ€Π΅Π΄Π΅Π»ΠΈΡ‚Π΅ ускорСниС ΠΌΠΎΡ‚ΠΎΡ†ΠΈΠΊΠ»Π°.

Β 

1440. ΠžΠΏΡ€Π΅Π΄Π΅Π»ΠΈΡ‚Π΅ ускорСниС Π»ΠΈΡ„Ρ‚Π° Π² высотном Π·Π΄Π°Π½ΠΈΠΈ, Ссли ΠΎΠ½ ΡƒΠ²Π΅Π»ΠΈΡ‡ΠΈΠ²Π°Π΅Ρ‚ свою ΡΠΊΠΎΡ€ΠΎΡΡ‚ΡŒ Π½Π° 3,2 ΠΌ/с Π² Ρ‚Π΅Ρ‡Π΅Π½ΠΈΠ΅ 2 с.

Β 

1441. ΠΠ²Ρ‚ΠΎΠΌΠΎΠ±ΠΈΠ»ΡŒ, двигавшийся со ΡΠΊΠΎΡ€ΠΎΡΡ‚ΡŒΡŽ 72 ΠΊΠΌ/Ρ‡, Ρ€Π°Π²Π½ΠΎΠΌΠ΅Ρ€Π½ΠΎ Ρ‚ΠΎΡ€ΠΌΠΎΠ·ΠΈΡ‚ ΠΈ Ρ‡Π΅Ρ€Π΅Π· 10 с останавливаСтся. Каково ускорСниС автомобиля?

Β 

1442. Как Π½Π°Π·Π²Π°Ρ‚ΡŒ двиТСния, ΠΏΡ€ΠΈ ΠΊΠΎΡ‚ΠΎΡ€Ρ‹Ρ… ускорСниС постоянно? Ρ€Π°Π²Π½ΠΎ Π½ΡƒΠ»ΡŽ?
РавноускорСнноС, Ρ€Π°Π²Π½ΠΎΠΌΠ΅Ρ€Π½ΠΎΠ΅.

1443. Π‘Π°Π½ΠΊΠΈ, ΡΠΊΠ°Ρ‚Ρ‹Π²Π°ΡΡΡŒ с Π³ΠΎΡ€Ρ‹, двиТутся равноускорСнно ΠΈ Π² ΠΊΠΎΠ½Ρ†Π΅ Ρ‚Ρ€Π΅Ρ‚ΡŒΠ΅ΠΉ сСкунды ΠΎΡ‚ Π½Π°Ρ‡Π°Π»Π° двиТСния ΠΈΠΌΠ΅ΡŽΡ‚ ΡΠΊΠΎΡ€ΠΎΡΡ‚ΡŒ 10,8 ΠΊΠΌ/Ρ‡. ΠžΠΏΡ€Π΅Π΄Π΅Π»ΠΈΡ‚Π΅, с ΠΊΠ°ΠΊΠΈΠΌ ускорСниСм двиТутся санки.

Β 

1444. Π‘ΠΊΠΎΡ€ΠΎΡΡ‚ΡŒ автомобиля Π·Π° 1,5 ΠΌΠΈΠ½ двиТСния возросла ΠΎΡ‚ 0 Π΄ΠΎ 60 ΠΊΠΌ/Ρ‡. НайдитС ускорСниС автомобиля Π² ΠΌ/с2 , Π² см/с2.

Β 

1445. ΠœΠΎΡ‚ΠΎΡ†ΠΈΠΊΠ» Β«Π₯ΠΎΠ½Π΄Π°Β», двигавшийся со ΡΠΊΠΎΡ€ΠΎΡΡ‚ΡŒΡŽ 90 ΠΊΠΌ/Ρ‡, Π½Π°Ρ‡Π°Π» Ρ€Π°Π²Π½ΠΎΠΌΠ΅Ρ€Π½ΠΎ Ρ‚ΠΎΡ€ΠΌΠΎΠ·ΠΈΡ‚ΡŒ ΠΈ Ρ‡Π΅Ρ€Π΅Π· 5 с сбросил ΡΠΊΠΎΡ€ΠΎΡΡ‚ΡŒ Π΄ΠΎ 18 ΠΊΠΌ/Ρ‡. Каково ускорСниС ΠΌΠΎΡ‚ΠΎΡ†ΠΈΠΊΠ»Π°?

Β 

1446. ΠžΠ±ΡŠΠ΅ΠΊΡ‚ ΠΈΠ· состояния покоя Π½Π°Ρ‡ΠΈΠ½Π°Π΅Ρ‚ Π΄Π²ΠΈΠ³Π°Ρ‚ΡŒΡΡ с постоянным ускорСниСм, Ρ€Π°Π²Π½Ρ‹ΠΌ 6 β€’ 10-3 ΠΌ/с2. ΠžΠΏΡ€Π΅Π΄Π΅Π»ΠΈΡ‚Π΅ ΡΠΊΠΎΡ€ΠΎΡΡ‚ΡŒ Ρ‡Π΅Ρ€Π΅Π· 5 ΠΌΠΈΠ½ послС Π½Π°Ρ‡Π°Π»Π° двиТСния. Какой ΠΏΡƒΡ‚ΡŒ ΠΏΡ€ΠΎΡˆΠ΅Π» ΠΎΠ±ΡŠΠ΅ΠΊΡ‚ Π·Π° это врСмя?

Β 

1447. Π―Ρ…Ρ‚Ρƒ ΡΠΏΡƒΡΠΊΠ°ΡŽΡ‚ Π½Π° Π²ΠΎΠ΄Ρƒ ΠΏΠΎ Π½Π°ΠΊΠ»ΠΎΠ½Π½Ρ‹ΠΌ стапСлям. ΠŸΠ΅Ρ€Π²Ρ‹Π΅ 80 см ΠΎΠ½Π° ΠΏΡ€ΠΎΡˆΠ»Π° Π·Π° 10 с. Π—Π° ΠΊΠ°ΠΊΠΎΠ΅ врСмя яхта ΠΏΡ€ΠΎΡˆΠ»Π° ΠΎΡΡ‚Π°Π²ΡˆΠΈΠ΅ΡΡ 30 ΠΌ, Ссли Π΅Π΅ Π΄Π²ΠΈΠΆΠ΅Π½ΠΈΠ΅ ΠΎΡΡ‚Π°Π²Π°Π»ΠΎΡΡŒ равноускорСнным?

Β 

1448. Π“Ρ€ΡƒΠ·ΠΎΠ²ΠΈΠΊ трогаСтся с мСста с ускорСниСм 0,6 ΠΌ/с2. Π—Π° ΠΊΠ°ΠΊΠΎΠ΅ врСмя ΠΎΠ½ ΠΏΡ€ΠΎΠΉΠ΄Π΅Ρ‚ ΠΏΡƒΡ‚ΡŒ Π² 30 ΠΌ?

Β 

1449. Π­Π»Π΅ΠΊΡ‚Ρ€ΠΈΡ‡ΠΊΠ° ΠΎΡ‚Ρ…ΠΎΠ΄ΠΈΡ‚ ΠΎΡ‚ станции, двигаясь равноускорСнно Π² Ρ‚Π΅Ρ‡Π΅Π½ΠΈΠ΅ 1 ΠΌΠΈΠ½ 20 с. Каково ускорСниС элСктрички, Ссли Π·Π° это врСмя Π΅Π΅ ΡΠΊΠΎΡ€ΠΎΡΡ‚ΡŒ стала 57,6 ΠΊΠΌ/Ρ‡? Какой ΠΏΡƒΡ‚ΡŒ ΠΎΠ½Π° ΠΏΡ€ΠΎΡˆΠ»Π° Π·Π° ΡƒΠΊΠ°Π·Π°Π½Π½ΠΎΠ΅ врСмя?

Β 

1450. Π‘Π°ΠΌΠΎΠ»Π΅Ρ‚ для Π²Π·Π»Π΅Ρ‚Π° равноускорСнно разгоняСтся Π² Ρ‚Π΅Ρ‡Π΅Π½ΠΈΠ΅ 6 с Π΄ΠΎ скорости 172,8 ΠΊΠΌ/Ρ‡. НайдитС ускорСниС самолСта. КакоС расстояниС ΠΏΡ€ΠΎΡˆΠ΅Π» самолСт ΠΏΡ€ΠΈ Ρ€Π°Π·Π³ΠΎΠ½Π΅?

Β 

1451. Π’ΠΎΠ²Π°Ρ€Π½Ρ‹ΠΉ ΠΏΠΎΠ΅Π·Π΄, Ρ‚Ρ€ΠΎΠ³Π°ΡΡΡŒ с мСста, двигался с ускорСниСм 0,5 ΠΌ/с2 ΠΈ разогнался Π΄ΠΎ скорости 36 ΠΊΠΌ/Ρ‡. Какой ΠΏΡƒΡ‚ΡŒ ΠΎΠ½ ΠΏΡ€ΠΈ этом ΠΏΡ€ΠΎΡˆΠ΅Π»?

Β 

1452. ΠžΡ‚ станции равноускорСнно тронулся скорый ΠΏΠΎΠ΅Π·Π΄ ΠΈ, пройдя 500 ΠΌ, достиг скорости 72 ΠΊΠΌ/Ρ‡. Каково ускорСниС ΠΏΠΎΠ΅Π·Π΄Π°? ΠžΠΏΡ€Π΅Π΄Π΅Π»ΠΈΡ‚Π΅ врСмя Π΅Π³ΠΎ Ρ€Π°Π·Π³ΠΎΠ½Π°.

Β 

1453. ΠŸΡ€ΠΈ Π²Ρ‹Ρ…ΠΎΠ΄Π΅ ΠΈΠ· ствола ΠΏΡƒΡˆΠΊΠΈ снаряд ΠΈΠΌΠ΅Π΅Ρ‚ ΡΠΊΠΎΡ€ΠΎΡΡ‚ΡŒ 1100 ΠΌ/с. Π”Π»ΠΈΠ½Π° ствола ΠΏΡƒΡˆΠΊΠΈ Ρ€Π°Π²Π½Π° 2,5 ΠΌ. Π’Π½ΡƒΡ‚Ρ€ΠΈ ствола снаряд двигался равноускорСнно. Каково Π΅Π³ΠΎ ускорСниС? Π—Π° ΠΊΠ°ΠΊΠΎΠ΅ врСмя снаряд ΠΏΡ€ΠΎΡˆΠ΅Π» всю Π΄Π»ΠΈΠ½Ρƒ ствола?

Β 

1454. Π­Π»Π΅ΠΊΡ‚Ρ€ΠΈΡ‡ΠΊΠ°, шСдшая со ΡΠΊΠΎΡ€ΠΎΡΡ‚ΡŒΡŽ 72 ΠΊΠΌ/Ρ‡, Π½Π°Ρ‡Π°Π»Π° Ρ‚ΠΎΡ€ΠΌΠΎΠ·ΠΈΡ‚ΡŒ с постоянным ускорСниСм, Ρ€Π°Π²Π½Ρ‹ΠΌ ΠΏΠΎ ΠΌΠΎΠ΄ΡƒΠ»ΡŽ 2 ΠΌ/с2. Π§Π΅Ρ€Π΅Π· ΠΊΠ°ΠΊΠΎΠ΅ врСмя ΠΎΠ½Π° остановится? КакоС расстояниС ΠΎΠ½Π° ΠΏΡ€ΠΎΠΉΠ΄Π΅Ρ‚ Π΄ΠΎ ΠΏΠΎΠ»Π½ΠΎΠΉ остановки?

Β 

1455. Городской автобус двигался Ρ€Π°Π²Π½ΠΎΠΌΠ΅Ρ€Π½ΠΎ со ΡΠΊΠΎΡ€ΠΎΡΡ‚ΡŒΡŽ 6 ΠΌ/с, Π° Π·Π°Ρ‚Π΅ΠΌ Π½Π°Ρ‡Π°Π» Ρ‚ΠΎΡ€ΠΌΠΎΠ·ΠΈΡ‚ΡŒ с ускорСниСм, ΠΏΠΎ модуля Ρ€Π°Π²Π½Ρ‹ΠΌ 0,6 ΠΌ/с2. Π—Π° ΠΊΠ°ΠΊΠΎΠ΅ врСмя Π΄ΠΎ остановки ΠΈ Π½Π° ΠΊΠ°ΠΊΠΎΠΌ расстоянии ΠΎΡ‚ Π½Π΅Π΅ Π½Π°Π΄ΠΎ Π½Π°Ρ‡Π°Ρ‚ΡŒ Ρ‚ΠΎΡ€ΠΌΠΎΠΆΠ΅Π½ΠΈΠ΅?

Β 

1456. Π‘Π°Π½ΠΊΠΈ ΡΠΊΠΎΠ»ΡŒΠ·ΡΡ‚ ΠΏΠΎ лСдяной Π΄ΠΎΡ€ΠΎΠΆΠΊΠ΅ с Π½Π°Ρ‡Π°Π»ΡŒΠ½ΠΎΠΉ ΡΠΊΠΎΡ€ΠΎΡΡ‚ΡŒΡŽ 8 ΠΌ/с, ΠΈ Π·Π° ΠΊΠ°ΠΆΠ΄ΡƒΡŽ сСкунду ΠΈΡ… ΡΠΊΠΎΡ€ΠΎΡΡ‚ΡŒ ΡƒΠΌΠ΅Π½ΡŒΡˆΠ°Π΅Ρ‚ΡΡ Π½Π° 0,25 ΠΌ/с. Π§Π΅Ρ€Π΅Π· ΠΊΠ°ΠΊΠΎΠ΅ врСмя санки остановятся?

Β 

1457. ΠœΠΎΡ‚ΠΎΡ€ΠΎΠ»Π»Π΅Ρ€, двигавшийся со ΡΠΊΠΎΡ€ΠΎΡΡ‚ΡŒΡŽ 46,8 ΠΊΠΌ/Ρ‡, останавливаСтся ΠΏΡ€ΠΈ Ρ€Π°Π²Π½ΠΎΠΌΠ΅Ρ€Π½ΠΎΠΌ Ρ‚ΠΎΡ€ΠΌΠΎΠΆΠ΅Π½ΠΈΠΈ Π² Ρ‚Π΅Ρ‡Π΅Π½ΠΈΠ΅ 2 с. Каково ускорСниС ΠΌΠΎΡ‚ΠΎΡ€ΠΎΠ»Π»Π΅Ρ€Π°? Каков Π΅Π³ΠΎ Ρ‚ΠΎΡ€ΠΌΠΎΠ·Π½ΠΎΠΉ ΠΏΡƒΡ‚ΡŒ?

Β 

1458. Π’Π΅ΠΏΠ»ΠΎΡ…ΠΎΠ΄, ΠΏΠ»Ρ‹Π²ΡƒΡ‰ΠΈΠΉ со ΡΠΊΠΎΡ€ΠΎΡΡ‚ΡŒΡŽ 32,4 ΠΊΠΌ/Ρ‡, стал Ρ€Π°Π²Π½ΠΎΠΌΠ΅Ρ€Π½ΠΎ Ρ‚ΠΎΡ€ΠΌΠΎΠ·ΠΈΡ‚ΡŒ ΠΈ, подойдя ΠΊ пристани Ρ‡Π΅Ρ€Π΅Π· 36 с, ΠΏΠΎΠ»Π½ΠΎΡΡ‚ΡŒΡŽ остановился. Π§Π΅ΠΌΡƒ Ρ€Π°Π²Π½ΠΎ ускорСниС Ρ‚Π΅ΠΏΠ»ΠΎΡ…ΠΎΠ΄Π°? Какой ΠΏΡƒΡ‚ΡŒ ΠΎΠ½ ΠΏΡ€ΠΎΡˆΠ΅Π» Π·Π° врСмя тормоТСния?

Β 

1459. Воварняк, проходя ΠΌΠΈΠΌΠΎ ΡˆΠ»Π°Π³Π±Π°ΡƒΠΌΠ°, приступил ΠΊ Ρ‚ΠΎΡ€ΠΌΠΎΠΆΠ΅Π½ΠΈΡŽ. Бпустя 3 ΠΌΠΈΠ½ ΠΎΠ½ остановился Π½Π° Ρ€Π°Π·ΡŠΠ΅Π·Π΄Π΅. Какова Π½Π°Ρ‡Π°Π»ΡŒΠ½Π°Ρ ΡΠΊΠΎΡ€ΠΎΡΡ‚ΡŒ товарняка ΠΈ ΠΌΠΎΠ΄ΡƒΠ»ΡŒ Π΅Π³ΠΎ ускорСния, Ссли ΡˆΠ»Π°Π³Π±Π°ΡƒΠΌ находится Π½Π° расстоянии 1,8 ΠΊΠΌ ΠΎΡ‚ Ρ€Π°Π·ΡŠΠ΅Π·Π΄Π°?

Β 

1460. Π’ΠΎΡ€ΠΌΠΎΠ·Π½ΠΎΠΉ ΠΏΡƒΡ‚ΡŒ ΠΏΠΎΠ΅Π·Π΄Π° 150 ΠΌ, врСмя тормоТСния 30 с. НайдитС Π½Π°Ρ‡Π°Π»ΡŒΠ½ΡƒΡŽ ΡΠΊΠΎΡ€ΠΎΡΡ‚ΡŒ ΠΏΠΎΠ΅Π·Π΄Π° ΠΈ Π΅Π³ΠΎ ускорСниС.

Β 

1461. Π­Π»Π΅ΠΊΡ‚Ρ€ΠΈΡ‡ΠΊΠ°, двигавшаяся со ΡΠΊΠΎΡ€ΠΎΡΡ‚ΡŒΡŽ 64,8 ΠΊΠΌ/Ρ‡, послС Π½Π°Ρ‡Π°Π»Π° тормоТСния Π΄ΠΎ ΠΏΠΎΠ»Π½ΠΎΠΉ остановки ΠΏΡ€ΠΎΡˆΠ»Π° 180 ΠΌ. ΠžΠΏΡ€Π΅Π΄Π΅Π»ΠΈΡ‚Π΅ Π΅Π΅ ускорСниС ΠΈ врСмя тормоТСния.

Β 

1462. Аэроплан Π»Π΅Ρ‚Π΅Π» Ρ€Π°Π²Π½ΠΎΠΌΠ΅Ρ€Π½ΠΎ со ΡΠΊΠΎΡ€ΠΎΡΡ‚ΡŒΡŽ 360 ΠΊΠΌ/Ρ‡, Π·Π°Ρ‚Π΅ΠΌ Π² Ρ‚Π΅Ρ‡Π΅Π½ΠΈΠ΅ 10 с ΠΎΠ½ двигался равноускорСнно: Π΅Π³ΠΎ ΡΠΊΠΎΡ€ΠΎΡΡ‚ΡŒ возрастала Π½Π° 9 ΠΌ/с Π·Π° сСкунду. ΠžΠΏΡ€Π΅Π΄Π΅Π»ΠΈΡ‚Π΅, ΠΊΠ°ΠΊΡƒΡŽ ΡΠΊΠΎΡ€ΠΎΡΡ‚ΡŒ ΠΏΡ€ΠΈΠΎΠ±Ρ€Π΅Π» аэроплан. КакоС расстояниС ΠΎΠ½ ΠΏΡ€ΠΎΠ»Π΅Ρ‚Π΅Π» ΠΏΡ€ΠΈ равноускорСнном Π΄Π²ΠΈΠΆΠ΅Π½ΠΈΠΈ?

Β 

1463. ΠœΠΎΡ‚ΠΎΡ†ΠΈΠΊΠ», двигавшийся со ΡΠΊΠΎΡ€ΠΎΡΡ‚ΡŒΡŽ 27 ΠΊΠΌ/Ρ‡, Π½Π°Ρ‡Π°Π» Ρ€Π°Π²Π½ΠΎΠΌΠ΅Ρ€Π½ΠΎ ΡƒΡΠΊΠΎΡ€ΡΡ‚ΡŒΡΡ ΠΈ Ρ‡Π΅Ρ€Π΅Π· 10 с достиг скорости 63 ΠΊΠΌ/Ρ‡. ΠžΠΏΡ€Π΅Π΄Π΅Π»ΠΈΡ‚Π΅ ΡΡ€Π΅Π΄Π½ΡŽΡŽ ΡΠΊΠΎΡ€ΠΎΡΡ‚ΡŒ ΠΌΠΎΡ‚ΠΎΡ†ΠΈΠΊΠ»Π° ΠΏΡ€ΠΈ равноускорСнном Π΄Π²ΠΈΠΆΠ΅Π½ΠΈΠΈ. Какой ΠΏΡƒΡ‚ΡŒ ΠΎΠ½ ΠΏΡ€ΠΎΠ΅Ρ…Π°Π» Π·Π° врСмя равноускорСнного двиТСния?

Β 

1464. ΠŸΡ€ΠΈΠ±ΠΎΡ€ отсчитываСт ΠΏΡ€ΠΎΠΌΠ΅ΠΆΡƒΡ‚ΠΊΠΈ Π²Ρ€Π΅ΠΌΠ΅Π½ΠΈ, Ρ€Π°Π²Π½Ρ‹Π΅ 0,75 с. Π¨Π°Ρ€ΠΈΠΊ скатываСтся с Π½Π°ΠΊΠ»ΠΎΠ½Π½ΠΎΠ³ΠΎ ΠΆΠ΅Π»ΠΎΠ±Π° Π² Ρ‚Π΅Ρ‡Π΅Π½ΠΈΠ΅ Ρ‚Ρ€Π΅Ρ… Ρ‚Π°ΠΊΠΈΡ… ΠΏΡ€ΠΎΠΌΠ΅ΠΆΡƒΡ‚ΠΊΠΎΠ² Π²Ρ€Π΅ΠΌΠ΅Π½ΠΈ. Π‘ΠΊΠ°Ρ‚ΠΈΠ²ΡˆΠΈΡΡŒ с Π½Π°ΠΊΠ»ΠΎΠ½Π½ΠΎΠ³ΠΎ ΠΆΠ΅Π»ΠΎΠ±Π°, ΠΎΠ½ ΠΏΡ€ΠΎΠ΄ΠΎΠ»ΠΆΠ°Π΅Ρ‚ Π΄Π²ΠΈΠ³Π°Ρ‚ΡŒΡΡ ΠΏΠΎ Π³ΠΎΡ€ΠΈΠ·ΠΎΠ½Ρ‚Π°Π»ΡŒΠ½ΠΎΠΌΡƒ ΠΆΠ΅Π»ΠΎΠ±Ρƒ ΠΈ ΠΏΡ€ΠΎΡ…ΠΎΠ΄ΠΈΡ‚ Π² Ρ‚Π΅Ρ‡Π΅Π½ΠΈΠ΅ ΠΏΠ΅Ρ€Π²ΠΎΠ³ΠΎ ΠΏΡ€ΠΎΠΌΠ΅ΠΆΡƒΡ‚ΠΊΠ° Π²Ρ€Π΅ΠΌΠ΅Π½ΠΈ 45 см. ΠžΠΏΡ€Π΅Π΄Π΅Π»ΠΈΡ‚Π΅ ΠΌΠ³Π½ΠΎΠ²Π΅Π½Π½ΡƒΡŽ ΡΠΊΠΎΡ€ΠΎΡΡ‚ΡŒ ΡˆΠ°Ρ€ΠΈΠΊΠ° Π² ΠΊΠΎΠ½Ρ†Π΅ Π½Π°ΠΊΠ»ΠΎΠ½Π½ΠΎΠ³ΠΎ ΠΆΠ΅Π»ΠΎΠ±Π° ΠΈ ускорСниС ΡˆΠ°Ρ€ΠΈΠΊΠ° ΠΏΡ€ΠΈ Π΄Π²ΠΈΠΆΠ΅Π½ΠΈΠΈ ΠΏΠΎ этому ΠΆΠ΅Π»ΠΎΠ±Ρƒ.

Β 

1465. ΠžΡ‚Ρ…ΠΎΠ΄Ρ ΠΎΡ‚ станции, ΠΏΠΎΠ΅Π·Π΄ двиТСтся равноускорСнно с ускорСниСм 5 см/с2. По ΠΏΡ€ΠΎΡˆΠ΅ΡΡ‚Π²ΠΈΠΈ ΠΊΠ°ΠΊΠΎΠ³ΠΎ Π²Ρ€Π΅ΠΌΠ΅Π½ΠΈ ΠΏΠΎΠ΅Π·Π΄ ΠΏΡ€ΠΈΠΎΠ±Ρ€Π΅Ρ‚Π°Π΅Ρ‚ ΡΠΊΠΎΡ€ΠΎΡΡ‚ΡŒ 36 ΠΊΠΌ/Ρ‡?

Β 

1466. ΠŸΡ€ΠΈ ΠΎΡ‚ΠΏΡ€Π°Π²Π»Π΅Π½ΠΈΠΈ ΠΏΠΎΠ΅Π·Π΄Π° ΠΎΡ‚ станции Π΅Π³ΠΎ ΡΠΊΠΎΡ€ΠΎΡΡ‚ΡŒ Π² Ρ‚Π΅Ρ‡Π΅Π½ΠΈΠ΅ ΠΏΠ΅Ρ€Π²Ρ‹Ρ… 4 с возросла Π΄ΠΎ 0,2 ΠΌ/с, Π² Ρ‚Π΅Ρ‡Π΅Π½ΠΈΠ΅ ΡΠ»Π΅Π΄ΡƒΡŽΡ‰ΠΈΡ… 6 с Π΅Ρ‰Π΅ Π½Π° 30 см/с ΠΈ Π·Π° ΡΠ»Π΅Π΄ΡƒΡŽΡ‰ΠΈΠ΅ 10 с Π½Π° 1,8 ΠΊΠΌ/Ρ‡. Как двигался ΠΏΠΎΠ΅Π·Π΄ Π² Ρ‚Π΅Ρ‡Π΅Π½ΠΈΠ΅ этих 20 с?

Β 

1467. Π‘Π°Π½ΠΊΠΈ, ΡΠΊΠ°Ρ‚Ρ‹Π²Π°ΡΡΡŒ с Π³ΠΎΡ€Ρ‹, двиТутся равноускорСнно. На Π½Π΅ΠΊΠΎΡ‚ΠΎΡ€ΠΎΠΌ участкС ΠΏΡƒΡ‚ΠΈ ΡΠΊΠΎΡ€ΠΎΡΡ‚ΡŒ санок Π² Ρ‚Π΅Ρ‡Π΅Π½ΠΈΠ΅ 4 с возросла ΠΎΡ‚ 0,8 ΠΌ/с Π΄ΠΎ 14,4 ΠΊΠΌ/Ρ‡. ΠžΠΏΡ€Π΅Π΄Π΅Π»ΠΈΡ‚Π΅ ускорСниС санок.

Β 

1468. ВСлосипСдист Π½Π°Ρ‡ΠΈΠ½Π°Π΅Ρ‚ Π΄Π²ΠΈΠ³Π°Ρ‚ΡŒΡΡ с ускорСниСм 20 см/с2. По истСчСнии ΠΊΠ°ΠΊΠΎΠ³ΠΎ Π²Ρ€Π΅ΠΌΠ΅Π½ΠΈ ΡΠΊΠΎΡ€ΠΎΡΡ‚ΡŒ вСлосипСдиста Π±ΡƒΠ΄Π΅Ρ‚ Ρ€Π°Π²Π½Π° 7,2 ΠΊΠΌ/Ρ‡?

Β 

1469. На рисункС 184 Π΄Π°Π½ Π³Ρ€Π°Ρ„ΠΈΠΊ скорости Π½Π΅ΠΊΠΎΡ‚ΠΎΡ€ΠΎΠ³ΠΎ равноускорСнного двиТСния. ΠŸΠΎΠ»ΡŒΠ·ΡƒΡΡΡŒ ΠΌΠ°ΡΡˆΡ‚Π°Π±ΠΎΠΌ, Π΄Π°Π½Π½Ρ‹ΠΌ Π½Π° рисункС, ΠΎΠΏΡ€Π΅Π΄Π΅Π»ΠΈΡ‚Π΅ ΠΏΡƒΡ‚ΡŒ, ΠΏΡ€ΠΎΡ…ΠΎΠ΄ΠΈΠΌΡ‹ΠΉ Π² этом Π΄Π²ΠΈΠΆΠ΅Π½ΠΈΠΈ Π² Ρ‚Π΅Ρ‡Π΅Π½ΠΈΠ΅ 3,5 с.

Β 1470. На рисункС 185 ΠΈΠ·ΠΎΠ±Ρ€Π°ΠΆΠ΅Π½ Π³Ρ€Π°Ρ„ΠΈΠΊ скорости Π½Π΅ΠΊΠΎΡ‚ΠΎΡ€ΠΎΠ³ΠΎ ΠΏΠ΅Ρ€Π΅ΠΌΠ΅Π½Π½ΠΎΠ³ΠΎ двиТСния. ΠŸΠ΅Ρ€Π΅Ρ‡Π΅Ρ€Ρ‚ΠΈΡ‚Π΅ рисунок Π² Ρ‚Π΅Ρ‚Ρ€Π°Π΄ΡŒ ΠΈ ΠΎΠ±ΠΎΠ·Π½Π°Ρ‡ΡŒΡ‚Π΅ ΡˆΡ‚Ρ€ΠΈΡ…ΠΎΠ²ΠΊΠΎΠΉ ΠΏΠ»ΠΎΡ‰Π°Π΄ΡŒ, числСнно Ρ€Π°Π²Π½ΡƒΡŽ ΠΏΡƒΡ‚ΠΈ, ΠΏΡ€ΠΎΡ…ΠΎΠ΄ΠΈΠΌΠΎΠΌΡƒ Π² Ρ‚Π΅Ρ‡Π΅Π½ΠΈΠ΅ 3 с. Π§Π΅ΠΌΡƒ ΠΏΡ€ΠΈΠΌΠ΅Ρ€Π½ΠΎ Ρ€Π°Π²Π΅Π½ этот ΠΏΡƒΡ‚ΡŒ?

1471. Π’ Ρ‚Π΅Ρ‡Π΅Π½ΠΈΠ΅ ΠΏΠ΅Ρ€Π²ΠΎΠ³ΠΎ ΠΏΡ€ΠΎΠΌΠ΅ΠΆΡƒΡ‚ΠΊΠ° Π²Ρ€Π΅ΠΌΠ΅Π½ΠΈ ΠΎΡ‚ Π½Π°Ρ‡Π°Π»Π° равноускорСнного двиТСния ΡˆΠ°Ρ€ΠΈΠΊ ΠΏΡ€ΠΎΡ…ΠΎΠ΄ΠΈΡ‚ ΠΏΠΎ ΠΆΠ΅Π»ΠΎΠ±Ρƒ 8 см. КакоС расстояниС ΠΏΡ€ΠΎΠΉΠ΄Π΅Ρ‚ ΡˆΠ°Ρ€ΠΈΠΊ Π² Ρ‚Π΅Ρ‡Π΅Π½ΠΈΠ΅ Ρ‚Ρ€Π΅Ρ… Ρ‚Π°ΠΊΠΈΡ… ΠΆΠ΅ ΠΏΡ€ΠΎΠΌΠ΅ΠΆΡƒΡ‚ΠΊΠΎΠ², ΠΏΡ€ΠΎΡˆΠ΅Π΄ΡˆΠΈΡ… ΠΎΡ‚ Π½Π°Ρ‡Π°Π»Π° двиТСния?

Β 

1472. Π’ Ρ‚Π΅Ρ‡Π΅Π½ΠΈΠ΅ 10 Ρ€Π°Π²Π½Ρ‹Ρ… ΠΏΡ€ΠΎΠΌΠ΅ΠΆΡƒΡ‚ΠΊΠΎΠ² Π²Ρ€Π΅ΠΌΠ΅Π½ΠΈ ΠΎΡ‚ Π½Π°Ρ‡Π°Π»Π° двиТСния Ρ‚Π΅Π»ΠΎ, двигаясь равноускорСнно, ΠΏΡ€ΠΎΡˆΠ»ΠΎ 75 см. Бколько сантимСтров ΠΏΡ€ΠΎΡˆΠ»ΠΎ это Ρ‚Π΅Π»ΠΎ Π² Ρ‚Π΅Ρ‡Π΅Π½ΠΈΠ΅ Π΄Π²ΡƒΡ… ΠΏΠ΅Ρ€Π²Ρ‹Ρ… Ρ‚Π°ΠΊΠΈΡ… ΠΆΠ΅ ΠΏΡ€ΠΎΠΌΠ΅ΠΆΡƒΡ‚ΠΊΠΎΠ² Π²Ρ€Π΅ΠΌΠ΅Π½ΠΈ?

Β 

1473. ПоСзд, отходя ΠΎΡ‚ станции, двиТСтся равноускорСнно ΠΈ Π² Ρ‚Π΅Ρ‡Π΅Π½ΠΈΠ΅ Π΄Π²ΡƒΡ… ΠΏΠ΅Ρ€Π²Ρ‹Ρ… сСкунд ΠΏΡ€ΠΎΡ…ΠΎΠ΄ΠΈΡ‚ 12 см. КакоС расстояниС ΠΏΡ€ΠΎΠΉΠ΄Π΅Ρ‚ ΠΏΠΎΠ΅Π·Π΄ Π² Ρ‚Π΅Ρ‡Π΅Π½ΠΈΠ΅ 1 ΠΌΠΈΠ½, считая ΠΎΡ‚ Π½Π°Ρ‡Π°Π»Π° двиТСния?

Β 

1474. ПоСзд, отходя ΠΎΡ‚ станции, двиТСтся равноускорСнно с ускорСниСм 5 см/с2. Бколько Π²Ρ€Π΅ΠΌΠ΅Π½ΠΈ потрСбуСтся для развития скорости 28,8 ΠΊΠΌ/Ρ‡ ΠΈ ΠΊΠ°ΠΊΠΎΠ΅ расстояниС ΠΏΡ€ΠΎΠΉΠ΄Π΅Ρ‚ ΠΏΠΎΠ΅Π·Π΄ Π·Π° это врСмя?

Β 

1475. ΠŸΠ°Ρ€ΠΎΠ²ΠΎΠ· ΠΏΠΎ Π³ΠΎΡ€ΠΈΠ·ΠΎΠ½Ρ‚Π°Π»ΡŒΠ½ΠΎΠΌΡƒ ΠΏΡƒΡ‚ΠΈ ΠΏΠΎΠ΄Ρ…ΠΎΠ΄ΠΈΡ‚ ΠΊ ΡƒΠΊΠ»ΠΎΠ½Ρƒ со ΡΠΊΠΎΡ€ΠΎΡΡ‚ΡŒΡŽ 8 ΠΌ/с, Π·Π°Ρ‚Π΅ΠΌ двиТСтся Π²Π½ΠΈΠ· ΠΏΠΎ ΡƒΠΊΠ»ΠΎΠ½Ρƒ с ускорСниСм 0,2 ΠΌ/с. ΠžΠΏΡ€Π΅Π΄Π΅Π»ΠΈΡ‚Π΅ Π΄Π»ΠΈΠ½Ρƒ ΡƒΠΊΠ»ΠΎΠ½Π°, Ссли ΠΏΠ°Ρ€ΠΎΠ²ΠΎΠ· ΠΏΡ€ΠΎΡ…ΠΎΠ΄ΠΈΡ‚ Π΅Π³ΠΎ Π·Π° 30 с.

Β 

1476. ΠΠ°Ρ‡Π°Π»ΡŒΠ½Π°Ρ ΡΠΊΠΎΡ€ΠΎΡΡ‚ΡŒ Ρ‚Π΅Π»Π΅ΠΆΠΊΠΈ, двиТущСйся Π²Π½ΠΈΠ· ΠΏΠΎ Π½Π°ΠΊΠ»ΠΎΠ½Π½ΠΎΠΉ доскС, Ρ€Π°Π²Π½Π° 10 см/с. Π’ΡΡŽ Π΄Π»ΠΈΠ½Ρƒ доски, Ρ€Π°Π²Π½ΡƒΡŽ 2 ΠΌ, Ρ‚Π΅Π»Π΅ΠΆΠΊΠ° ΠΏΡ€ΠΎΡˆΠ»Π° Π² Ρ‚Π΅Ρ‡Π΅Π½ΠΈΠ΅ 5 сСк. ΠžΠΏΡ€Π΅Π΄Π΅Π»ΠΈΡ‚Π΅ ускорСниС Ρ‚Π΅Π»Π΅ΠΆΠΊΠΈ.

Β 

1477. ΠŸΡƒΠ»Ρ Π²Ρ‹Π»Π΅Ρ‚Π°Π΅Ρ‚ ΠΈΠ· ствола Ρ€ΡƒΠΆΡŒΡ со ΡΠΊΠΎΡ€ΠΎΡΡ‚ΡŒΡŽ 800 ΠΌ/с. Π”Π»ΠΈΠ½Π° ствола 64 см. ΠŸΡ€Π΅Π΄ΠΏΠΎΠ»Π°Π³Π°Ρ Π΄Π²ΠΈΠΆΠ΅Π½ΠΈΠ΅ ΠΏΡƒΠ»ΠΈ Π²Π½ΡƒΡ‚Ρ€ΠΈ ствола равноускорСнным, ΠΎΠΏΡ€Π΅Π΄Π΅Π»ΠΈΡ‚Π΅ ускорСниС ΠΈ врСмя двиТСния.

Β 

1478. Автобус, двигаясь со ΡΠΊΠΎΡ€ΠΎΡΡ‚ΡŒΡŽ 4 ΠΌ/с, Π½Π°Ρ‡ΠΈΠ½Π°Π΅Ρ‚ Ρ€Π°Π²Π½ΠΎΠΌΠ΅Ρ€Π½ΠΎ ΡƒΡΠΊΠΎΡ€ΡΡ‚ΡŒΡΡ Π½Π° 1 ΠΌ/с Π·Π° сСкунду. Какой ΠΏΡƒΡ‚ΡŒ ΠΏΡ€ΠΎΠΉΠ΄Π΅Ρ‚ автобус Π·Π° ΡˆΠ΅ΡΡ‚ΡƒΡŽ сСкунду?

Β 

1479. Π“Ρ€ΡƒΠ·ΠΎΠ²ΠΈΠΊ, имСя Π½Π΅ΠΊΠΎΡ‚ΠΎΡ€ΡƒΡŽ Π½Π°Ρ‡Π°Π»ΡŒΠ½ΡƒΡŽ ΡΠΊΠΎΡ€ΠΎΡΡ‚ΡŒ, Π½Π°Ρ‡Π°Π» Π΄Π²ΠΈΠ³Π°Ρ‚ΡŒΡΡ равноускорСнно: Π·Π° ΠΏΠ΅Ρ€Π²Ρ‹Π΅ 5 с ΠΏΡ€ΠΎΡˆΠ΅Π» 40 ΠΌ, Π° Π·Π° ΠΏΠ΅Ρ€Π²Ρ‹Π΅ 10 с β€” 130 ΠΌ. НайдитС Π½Π°Ρ‡Π°Π»ΡŒΠ½ΡƒΡŽ ΡΠΊΠΎΡ€ΠΎΡΡ‚ΡŒ Π³Ρ€ΡƒΠ·ΠΎΠ²ΠΈΠΊΠ° ΠΈ Π΅Π³ΠΎ ускорСниС.

Β 

1480. ΠšΠ°Ρ‚Π΅Ρ€, отходя ΠΎΡ‚ пристани, Π½Π°Ρ‡Π°Π» равноускорСнноС Π΄Π²ΠΈΠΆΠ΅Π½ΠΈΠ΅. ΠŸΡ€ΠΎΠΉΠ΄Ρ Π½Π΅ΠΊΠΎΡ‚ΠΎΡ€ΠΎΠ΅ расстояниС, ΠΎΠ½ достиг скорости 20 ΠΌ/с. Какова Π±Ρ‹Π»Π° ΡΠΊΠΎΡ€ΠΎΡΡ‚ΡŒ ΠΊΠ°Ρ‚Π΅Ρ€Π° Π² Ρ‚ΠΎΡ‚ ΠΌΠΎΠΌΠ΅Π½Ρ‚, ΠΊΠΎΠ³Π΄Π° ΠΎΠ½ ΠΏΡ€ΠΎΠΏΠ»Ρ‹Π» ΠΏΠΎΠ»ΠΎΠ²ΠΈΠ½Ρƒ этого расстояния?

Β 

1481. Π›Ρ‹ΠΆΠ½ΠΈΠΊ скатываСтся с Π³ΠΎΡ€Ρ‹ с Π½ΡƒΠ»Π΅Π²ΠΎΠΉ Π½Π°Ρ‡Π°Π»ΡŒΠ½ΠΎΠΉ ΡΠΊΠΎΡ€ΠΎΡΡ‚ΡŒΡŽ. На сСрСдинС Π³ΠΎΡ€Ρ‹ Π΅Π³ΠΎ ΡΠΊΠΎΡ€ΠΎΡΡ‚ΡŒ Π±Ρ‹Π»Π° 5 ΠΌ/с, Ρ‡Π΅Ρ€Π΅Π· 2 с ΡΠΊΠΎΡ€ΠΎΡΡ‚ΡŒ стала 6 ΠΌ/с. Бчитая, Ρ‡Ρ‚ΠΎ ΠΎΠ½Π° увСличиваСтся Ρ€Π°Π²Π½ΠΎΠΌΠ΅Ρ€Π½ΠΎ, ΠΎΠΏΡ€Π΅Π΄Π΅Π»ΠΈΡ‚Π΅ ΡΠΊΠΎΡ€ΠΎΡΡ‚ΡŒ Π»Ρ‹ΠΆΠ½ΠΈΠΊΠ° Ρ‡Π΅Ρ€Π΅Π· 8 с послС Π½Π°Ρ‡Π°Π»Π° двиТСния.

Β 

1482. ΠΠ²Ρ‚ΠΎΠΌΠΎΠ±ΠΈΠ»ΡŒ тронулся с мСста ΠΈ двигаСтся равноускорСнно. Π—Π° ΠΊΠ°ΠΊΡƒΡŽ сСкунду ΠΎΡ‚ Π½Π°Ρ‡Π°Π»Π° двиТСния ΠΏΡƒΡ‚ΡŒ, ΠΏΡ€ΠΎΠΉΠ΄Π΅Π½Π½Ρ‹ΠΉ Π°Π²Ρ‚ΠΎΠΌΠΎΠ±ΠΈΠ»Π΅ΠΌ, Π²Π΄Π²ΠΎΠ΅ большС ΠΏΡƒΡ‚ΠΈ, ΠΏΡ€ΠΎΠΉΠ΄Π΅Π½Π½ΠΎΠ³ΠΎ ΠΈΠΌ Π² ΠΏΡ€Π΅Π΄Ρ‹Π΄ΡƒΡ‰ΡƒΡŽ сСкунду?

Β 

1483. НайдитС ΠΏΡƒΡ‚ΡŒ, ΠΏΡ€ΠΎΠΉΠ΄Π΅Π½Π½Ρ‹ΠΉ Ρ‚Π΅Π»ΠΎΠΌ Π·Π° Π²ΠΎΡΡŒΠΌΡƒΡŽ сСкунду двиТСния, Ссли ΠΎΠ½ΠΎ Π½Π°Ρ‡ΠΈΠ½Π°Π΅Ρ‚ Π΄Π²ΠΈΠ³Π°Ρ‚ΡŒΡΡ равноускорСнно Π±Π΅Π· Π½Π°Ρ‡Π°Π»ΡŒΠ½ΠΎΠΉ скорости ΠΈ Π·Π° ΠΏΡΡ‚ΡƒΡŽ сСкунду ΠΏΡ€ΠΎΡ…ΠΎΠ΄ΠΈΡ‚ ΠΏΡƒΡ‚ΡŒ 27 ΠΌ.

Β 

1484. ΠŸΡ€ΠΎΠ²ΠΎΠΆΠ°ΡŽΡ‰ΠΈΠ΅ стоят Ρƒ Π½Π°Ρ‡Π°Π»Π° Π³ΠΎΠ»ΠΎΠ²Π½ΠΎΠ³ΠΎ Π²Π°Π³ΠΎΠ½Π° ΠΏΠΎΠ΅Π·Π΄Π°. ПоСзд трогаСтся ΠΈ двиТСтся равноускорСнно. Π—Π° 3 с вСсь Π³ΠΎΠ»ΠΎΠ²Π½ΠΎΠΉ Π²Π°Π³ΠΎΠ½ ΠΏΡ€ΠΎΡ…ΠΎΠ΄ΠΈΡ‚ ΠΌΠΈΠΌΠΎ ΠΏΡ€ΠΎΠ²ΠΎΠΆΠ°ΡŽΡ‰ΠΈΡ…. Π—Π° ΠΊΠ°ΠΊΠΎΠ΅ врСмя ΠΏΡ€ΠΎΠΉΠ΄Π΅Ρ‚ ΠΌΠΈΠΌΠΎ ΠΏΡ€ΠΎΠ²ΠΎΠΆΠ°ΡŽΡ‰ΠΈΡ… вСсь ΠΏΠΎΠ΅Π·Π΄, состоящий ΠΈΠ· 9 Π²Π°Π³ΠΎΠ½ΠΎΠ²?

Β 

1485. ΠœΠ°Ρ‚Π΅Ρ€ΠΈΠ°Π»ΡŒΠ½Π°Ρ Ρ‚ΠΎΡ‡ΠΊΠ° двиТСтся ΠΏΠΎ Π·Π°ΠΊΠΎΠ½Ρƒ x = 0,5tΒ². КакоС это Π΄Π²ΠΈΠΆΠ΅Π½ΠΈΠ΅? Каково ускорСниС Ρ‚ΠΎΡ‡ΠΊΠΈ? ΠŸΠΎΡΡ‚Ρ€ΠΎΠΉΡ‚Π΅ Π³Ρ€Π°Ρ„ΠΈΠΊ зависимости ΠΎΡ‚ Π²Ρ€Π΅ΠΌΠ΅Π½ΠΈ:
Π°) ΠΊΠΎΠΎΡ€Π΄ΠΈΠ½Π°Ρ‚Ρ‹ Ρ‚ΠΎΡ‡ΠΊΠΈ;
Π±) скорости Ρ‚ΠΎΡ‡ΠΊΠΈ;
Π²) ускорСния.

Β 

1486. ПоСзд остановился Ρ‡Π΅Ρ€Π΅Π· 20 с послС Π½Π°Ρ‡Π°Π»Π° тормоТСния, пройдя Π·Π° это врСмя 120 ΠΌ. ΠžΠΏΡ€Π΅Π΄Π΅Π»ΠΈΡ‚Π΅ ΠΏΠ΅Ρ€Π²ΠΎΠ½Π°Ρ‡Π°Π»ΡŒΠ½ΡƒΡŽ ΡΠΊΠΎΡ€ΠΎΡΡ‚ΡŒ ΠΏΠΎΠ΅Π·Π΄Π° ΠΈ ускорСниС ΠΏΠΎΠ΅Π·Π΄Π°.

Β 

1487. ПоСзд, ΠΈΠ΄ΡƒΡ‰ΠΈΠΉ со ΡΠΊΠΎΡ€ΠΎΡΡ‚ΡŒΡŽ 18 ΠΌ/с, Π½Π°Ρ‡Π°Π» Ρ‚ΠΎΡ€ΠΌΠΎΠ·ΠΈΡ‚ΡŒ, ΠΈ Ρ‡Π΅Ρ€Π΅Π· 15 с остановился. Бчитая Π΄Π²ΠΈΠΆΠ΅Π½ΠΈΠ΅ ΠΏΠΎΠ΅Π·Π΄Π° ΠΏΡ€ΠΈ Ρ‚ΠΎΡ€ΠΌΠΎΠΆΠ΅Π½ΠΈΠΈ Ρ€Π°Π²Π½ΠΎΠ·Π°ΠΌΠ΅Π΄Π»Π΅Π½Π½Ρ‹ΠΌ, ΠΎΠΏΡ€Π΅Π΄Π΅Π»ΠΈΡ‚Π΅ ΠΏΡƒΡ‚ΡŒ, ΠΏΡ€ΠΎΠΉΠ΄Π΅Π½Π½Ρ‹ΠΉ ΠΏΠΎΠ΅Π·Π΄ΠΎΠΌ Π·Π° эти 15 с.

Β 

1488. ΠŸΠΎΡΡ‚Ρ€ΠΎΠΉΡ‚Π΅ Π³Ρ€Π°Ρ„ΠΈΠΊΠΈ скорости Ρ€Π°Π²Π½ΠΎΠ·Π°ΠΌΠ΅Π΄Π»Π΅Π½Π½ΠΎΠ³ΠΎ двиТСния для случаСв:
1) V0 = 10 ΠΌ/с, Π° = β€” 1,5 ΠΌ/с2;
2) V0 = 10 ΠΌ/с; Π° = β€” 2 ΠΌ/с2.
ΠœΠ°ΡΡˆΡ‚Π°Π± Π² ΠΎΠ±ΠΎΠΈΡ… случаях ΠΎΠ΄ΠΈΠ½Π°ΠΊΠΎΠ²: 0,5 см – 1 ΠΌ/с; ΠΎ,5 см – 1 сСк.

Β 

1489. Π˜Π·ΠΎΠ±Ρ€Π°Π·ΠΈΡ‚Π΅ ΠΏΡ€ΠΎΠΉΠ΄Π΅Π½Π½Ρ‹ΠΉ ΠΏΡƒΡ‚ΡŒ Π·Π° врСмя t Π½Π° Π³Ρ€Π°Ρ„ΠΈΠΊΠ΅ скорости Ρ€Π°Π²Π½ΠΎΠ·Π°ΠΌΠ΅Π΄Π»Π΅Π½Π½ΠΎΠ³ΠΎ двиТСния. ΠŸΡ€ΠΈΠ½ΡΡ‚ΡŒ V0 = 10 ΠΌ/с, Π° = 2 ΠΌ/с2.

Β 

1490. ΠžΠΏΠΈΡˆΠΈΡ‚Π΅ двиТСния, Π³Ρ€Π°Ρ„ΠΈΠΊΠΈ скоростСй ΠΊΠΎΡ‚ΠΎΡ€Ρ‹Ρ… Π΄Π°Π½Ρ‹ Π½Π° рисункС 186, Π° ΠΈ Π±.
Π°) Π΄Π²ΠΈΠΆΠ΅Π½ΠΈΠ΅ Π±ΡƒΠ΄Π΅Ρ‚ Ρ€Π°Π²Π½ΠΎΠ·Π°ΠΌΠ΅Π΄Π»Π΅Π½Π½Ρ‹ΠΌ;
Π±) сначала Ρ‚Π΅Π»ΠΎ Π±ΡƒΠ΄Π΅Ρ‚ Π΄Π²ΠΈΠ³Π°Ρ‚ΡŒΡΡ равноускорСнно, Π·Π°Ρ‚Π΅ΠΌ Ρ€Π°Π²Π½ΠΎΠΌΠ΅Ρ€Π½ΠΎ. На 3ΠΌ участкС Π΄Π²ΠΈΠΆΠ΅Π½ΠΈΠ΅ Π±ΡƒΠ΄Π΅Ρ‚ Ρ€Π°Π²Π½ΠΎΠ·Π°ΠΌΠ΅Π΄Π»Π΅Π½Π½ΠΎΠ΅.

«ΠΠ΅ Π΄ΠΎ ΠΆΠΈΡ€Ρƒ, Π±Ρ‹Ρ‚ΡŒ Π±Ρ‹ ΠΆΠΈΠ²Ρƒ». Каково Π±ΡƒΠ΄ΡƒΡ‰Π΅Π΅ ΠΊΠ²Π°Π½Ρ‚ΠΎΠ²ΠΎΠ³ΠΎ ΠΊΠΎΠΌΠΏΡŒΡŽΡ‚Π΅Ρ€Π° Google — Наука

ВАББ, 2 января. БпСциалисты ΠΊΠΎΠΌΠΏΠ°Π½ΠΈΠΈ Google, ΠΊΠΎΡ‚ΠΎΡ€Ρ‹Π΅ создали ΠΏΡ€ΠΎΡ‚ΠΎΡ‚ΠΈΠΏ ΠΊΠ²Π°Π½Ρ‚ΠΎΠ²ΠΎΠ³ΠΎ ΠΊΠΎΠΌΠΏΡŒΡŽΡ‚Π΅Ρ€Π° Sycamore, ΠΏΠ»Π°Π½ΠΈΡ€ΡƒΡŽΡ‚ ΠΈΡΠΏΠΎΠ»ΡŒΠ·ΠΎΠ²Π°Ρ‚ΡŒ Π΅Π³ΠΎ для Ρ€Π΅ΡˆΠ΅Π½ΠΈΡ ΠΊΠ°ΠΊ тСорСтичСских, Ρ‚Π°ΠΊ ΠΈ практичСских Π½Π°ΡƒΡ‡Π½Ρ‹Ρ… Π·Π°Π΄Π°Ρ‡. Π‘Ρ€Π΅Π΄ΠΈ Π½ΠΈΡ… – Π°Π»Π³ΠΎΡ€ΠΈΡ‚ΠΌΡ‹ для Π°Π²Ρ‚ΠΎΠ½ΠΎΠΌΠ½ΠΎΠ³ΠΎ транспорта, ΠΈΠ·ΡƒΡ‡Π΅Π½ΠΈΠ΅ ΠΊΠ²Π°Π½Ρ‚ΠΎΠ²ΠΎΠ³ΠΎ хаоса ΠΈ Π²Π½ΡƒΡ‚Ρ€Π΅Π½Π½Π΅Π»Π³ΠΎ устройства Ρ‡Π΅Ρ€Π½Ρ‹Ρ… Π΄Ρ‹Ρ€. Об этом ВАББ рассказал ΠΎΠ΄ΠΈΠ½ ΠΈΠ· Ρ€Π°Π·Ρ€Π°Π±ΠΎΡ‚Ρ‡ΠΈΠΊΠΎΠ² этого ΠΊΠΎΠΌΠΏΡŒΡŽΡ‚Π΅Ρ€Π° Π’Π°Π΄ΠΈΠΌ БмСлянский.

Π§Ρ‚ΠΎ Ρ‚Π°ΠΊΠΎΠ΅ ΠΊΠ²Π°Π½Ρ‚ΠΎΠ²Ρ‹ΠΉ ΠΊΠΎΠΌΠΏΡŒΡŽΡ‚Π΅Ρ€?

ΠšΠ²Π°Π½Ρ‚ΠΎΠ²Ρ‹ΠΌΠΈ ΠΊΠΎΠΌΠΏΡŒΡŽΡ‚Π΅Ρ€Π°ΠΌΠΈ Π½Π°Π·Ρ‹Π²Π°ΡŽΡ‚ Π²Ρ‹Ρ‡ΠΈΡΠ»ΠΈΡ‚Π΅Π»ΡŒΠ½Ρ‹Π΅ устройства, Π² основС Ρ€Π°Π±ΠΎΡ‚Ρ‹ ΠΊΠΎΡ‚ΠΎΡ€Ρ‹Ρ… Π»Π΅ΠΆΠ°Ρ‚ ΠΏΡ€ΠΈΠ½Ρ†ΠΈΠΏΡ‹ ΠΊΠ²Π°Π½Ρ‚ΠΎΠ²ΠΎΠΉ ΠΌΠ΅Ρ…Π°Π½ΠΈΠΊΠΈ. Π’ ΠΎΡ‚Π»ΠΈΡ‡ΠΈΠ΅ ΠΎΡ‚ ΠΎΠ±Ρ‹Ρ‡Π½Ρ‹Ρ… ΠΊΠΎΠΌΠΏΡŒΡŽΡ‚Π΅Ρ€ΠΎΠ², Π² ΠΊΠΎΡ‚ΠΎΡ€Ρ‹Ρ… для ΠΏΠ΅Ρ€Π΅Π΄Π°Ρ‡ΠΈ ΠΈ ΠΎΠ±Ρ€Π°Π±ΠΎΡ‚ΠΊΠΈ Π΄Π°Π½Π½Ρ‹Ρ… ΠΈΡΠΏΠΎΠ»ΡŒΠ·ΡƒΡŽΡ‚ΡΡ Π±ΠΈΡ‚Ρ‹ – Π΅Π΄ΠΈΠ½ΠΈΡ†Ρ‹ ΠΈΠ½Ρ„ΠΎΡ€ΠΌΠ°Ρ†ΠΈΠΈ, ΠΊΠΎΡ‚ΠΎΡ€Ρ‹Π΅ содСрТат Π»ΠΈΠ±ΠΎ 1, Π»ΠΈΠ±ΠΎ 0, ΠΊΠ²Π°Π½Ρ‚ΠΎΠ²Ρ‹Π΅ ΠΊΠΎΠΌΠΏΡŒΡŽΡ‚Π΅Ρ€Ρ‹ ΠΎΠΏΠ΅Ρ€ΠΈΡ€ΡƒΡŽΡ‚ ΠΊΡƒΠ±ΠΈΡ‚Π°ΠΌΠΈ – ячСйками памяти ΠΈ ΠΏΡ€ΠΈΠΌΠΈΡ‚ΠΈΠ²Π½Ρ‹ΠΌΠΈ Π²Ρ‹Ρ‡ΠΈΡΠ»ΠΈΡ‚Π΅Π»ΡŒΠ½Ρ‹ΠΌΠΈ модулями, ΠΊΠΎΡ‚ΠΎΡ€Ρ‹Π΅ ΠΌΠΎΠ³ΡƒΡ‚ Ρ…Ρ€Π°Π½ΠΈΡ‚ΡŒ Π² сСбС ΠΎΠ΄Π½ΠΎΠ²Ρ€Π΅ΠΌΠ΅Π½Π½ΠΎ ΠΈ ноль, ΠΈ Π΅Π΄ΠΈΠ½ΠΈΡ†Ρƒ. Благодаря этому ΠΊΠ²Π°Π½Ρ‚ΠΎΠ²Ρ‹Π΅ ΠΊΠΎΠΌΠΏΡŒΡŽΡ‚Π΅Ρ€Ρ‹ ΠΌΠΎΠ³ΡƒΡ‚ ΠΎΠ±Ρ€Π°Π±Π°Ρ‚Ρ‹Π²Π°Ρ‚ΡŒ большиС ΠΎΠ±ΡŠΠ΅ΠΌΡ‹ ΠΈΠ½Ρ„ΠΎΡ€ΠΌΠ°Ρ†ΠΈΠΈ Π²ΠΎ ΠΌΠ½ΠΎΠ³ΠΎ Ρ€Π°Π· быстрСС ΠΎΠ±Ρ‹Ρ‡Π½Ρ‹Ρ… – Π΄Π°ΠΆΠ΅ Ссли это ΡΡƒΠΏΠ΅Ρ€ΠΊΠΎΠΌΠΏΡŒΡŽΡ‚Π΅Ρ€Ρ‹ с ΠΎΠ³Ρ€ΠΎΠΌΠ½Ρ‹ΠΌΠΈ Π²Ρ‹Ρ‡ΠΈΡΠ»ΠΈΡ‚Π΅Π»ΡŒΠ½Ρ‹ΠΌΠΈ мощностями.

БущСствуСт Π΄Π²Π° ΠΏΠΎΠ΄Ρ…ΠΎΠ΄Π° ΠΊ Ρ€Π°Π·Ρ€Π°Π±ΠΎΡ‚ΠΊΠ΅ ΠΏΠΎΠ΄ΠΎΠ±Π½Ρ‹Ρ… устройств – классичСский ΠΈ адиабатичСский. Π’ ΠΏΠ΅Ρ€Π²ΠΎΠΌ случаС ΡƒΡ‡Π΅Π½Ρ‹Π΅ ΠΏΡ‹Ρ‚Π°ΡŽΡ‚ΡΡ ΡΠΎΠ·Π΄Π°Ρ‚ΡŒ Ρ‚Π°ΠΊ Π½Π°Π·Ρ‹Π²Π°Π΅ΠΌΡ‹ΠΉ ΡƒΠ½ΠΈΠ²Π΅Ρ€ΡΠ°Π»ΡŒΠ½Ρ‹ΠΉ ΠΊΠ²Π°Π½Ρ‚ΠΎΠ²Ρ‹ΠΉ ΠΊΠΎΠΌΠΏΡŒΡŽΡ‚Π΅Ρ€, ΠΊΠΎΡ‚ΠΎΡ€Ρ‹ΠΉ ΠΏΠΎ ΠΏΡ€ΠΈΠ½Ρ†ΠΈΠΏΡƒ Ρ€Π°Π±ΠΎΡ‚Ρ‹ ΠΏΠΎΡ…ΠΎΠΆ Π½Π° ΠΎΠ±Ρ‹Ρ‡Π½Ρ‹ΠΉ соврСмСнный ΠΊΠΎΠΌΠΏΡŒΡŽΡ‚Π΅Ρ€. АдиабатичСский ΠΊΠΎΠΌΠΏΡŒΡŽΡ‚Π΅Ρ€Β Π±Π»ΠΈΠΆΠ΅ ΠΊ Π°Π½Π°Π»ΠΎΠ³ΠΎΠ²Ρ‹ΠΌ ΠΊΠΎΠΌΠΏΡŒΡŽΡ‚Π΅Ρ€Π°ΠΌ. Π‘ΠΎΠ·Π΄Π°Π²Π°Ρ‚ΡŒ ΠΈΡ… ΠΏΡ€ΠΎΡ‰Π΅, Π½ΠΎ ΠΏΠΎΠ΄ΠΎΠ±Π½Ρ‹Π΅ устройства ΡƒΠΌΠ΅ΡŽΡ‚ Ρ€Π΅ΡˆΠ°Ρ‚ΡŒ лишь ΠΎΠ³Ρ€Π°Π½ΠΈΡ‡Π΅Π½Π½ΠΎΠ΅ количСство Π·Π°Π΄Π°Ρ‡.

Π˜Π½ΠΆΠ΅Π½Π΅Ρ€Ρ‹ ΠΈ ΡƒΡ‡Π΅Π½Ρ‹Π΅ Google ΠΏΠΎΡ‡Ρ‚ΠΈ ΠΏΡΡ‚ΡŒ Π»Π΅Ρ‚ Ρ€Π°Π·Ρ€Π°Π±Π°Ρ‚Ρ‹Π²Π°ΡŽΡ‚ устройство, ΠΊΠΎΡ‚ΠΎΡ€ΠΎΠ΅ Π΄ΠΎΠ»ΠΆΠ½ΠΎ ΠΎΠ±ΡŠΠ΅Π΄ΠΈΠ½ΠΈΡ‚ΡŒ достоинства ΠΎΠ±ΠΎΠΈΡ… ΠΏΠΎΠ΄Ρ…ΠΎΠ΄ΠΎΠ². ΠŸΠ΅Ρ€Π²Π°Ρ Π΅Π³ΠΎ вСрсия состояла ΠΈΠ· дСвяти ΠΊΡƒΠ±ΠΈΡ‚ΠΎΠ². Главная Π·Π°Π΄Π°Ρ‡Π° спСциалистов ΠΊΠΎΠΌΠΏΠ°Π½ΠΈΠΈ состояла Π² Ρ‚ΠΎΠΌ, Ρ‡Ρ‚ΠΎΠ±Ρ‹ Π΄ΠΎΡΡ‚ΠΈΡ‡ΡŒ Ρ‚Π°ΠΊ Π½Π°Π·Ρ‹Π²Π°Π΅ΠΌΠΎΠ³ΠΎ ΠΊΠ²Π°Π½Ρ‚ΠΎΠ²ΠΎΠ³ΠΎ прСвосходства, Ρ‚ΠΎ Π΅ΡΡ‚ΡŒ ΠΌΠΎΠΌΠ΅Π½Ρ‚Π°, ΠΊΠΎΠ³Π΄Π° Π½ΠΈ ΠΎΠ΄ΠΈΠ½ классичСский ΠΊΠΎΠΌΠΏΡŒΡŽΡ‚Π΅Ρ€ Π½Π΅ ΠΌΠΎΠΆΠ΅Ρ‚ Π΄ΠΎΡΡ‚ΠΈΡ‡ΡŒ Ρ‚Π΅Ρ… ΠΆΠ΅ Ρ€Π΅Π·ΡƒΠ»ΡŒΡ‚Π°Ρ‚ΠΎΠ², Ρ‡Ρ‚ΠΎ ΠΈ ΠΊΠ²Π°Π½Ρ‚ΠΎΠ²Ρ‹ΠΉ.

На эту Ρ‚Π΅ΠΌΡƒ

Π’ ноябрС ΠΏΡ€ΠΎΡˆΠ»ΠΎΠ³ΠΎ Π³ΠΎΠ΄Π° прСдставитСли Google заявили, Ρ‡Ρ‚ΠΎ достигли ΠΊΠ²Π°Π½Ρ‚ΠΎΠ²ΠΎΠ³ΠΎ прСвосходства Π² матСматичСской Π·Π°Π΄Π°Ρ‡Π΅, которая сводится ΠΊ ΠΏΠ΅Ρ€Π΅Π±ΠΎΡ€Ρƒ Π½Π°Π±ΠΎΡ€Π° псСвдослучайных чисСл. Для Π΅Π΅ Ρ€Π΅ΡˆΠ΅Π½ΠΈΡ ΡƒΡ‡Π΅Π½Ρ‹Π΅ использовали ΠΏΡ€ΠΎΡ‚ΠΎΡ‚ΠΈΠΏ ΠΊΠ²Π°Π½Ρ‚ΠΎΠ²ΠΎΠ³ΠΎ ΠΊΠΎΠΌΠΏΡŒΡŽΡ‚Π΅Ρ€Π°, Sycamore, ΠΊΠΎΡ‚ΠΎΡ€Ρ‹ΠΉ состоял ΠΈΠ· 53 свСрхпроводящих ΠΊΡƒΠ±ΠΈΡ‚ΠΎΠ². НСсколько ΠΌΠΈΠ»Π»ΠΈΠΎΠ½ΠΎΠ² Ρ€Π΅ΡˆΠ΅Π½ΠΈΠΉ ΠΎΠ½ нашСл мСньшС Ρ‡Π΅ΠΌ Π·Π° Ρ‡Π΅Ρ‚Ρ‹Ρ€Π΅ ΠΌΠΈΠ½ΡƒΡ‚Ρ‹, Ρ‚ΠΎΠ³Π΄Π° ΠΊΠ°ΠΊ Ρƒ самого ΠΌΠΎΡ‰Π½ΠΎΠ³ΠΎ ΡΡƒΠΏΠ΅Ρ€ΠΊΠΎΠΌΠΏΡŒΡŽΡ‚Π΅Ρ€Π° Π½Π° это ΡƒΡˆΠ»ΠΎ Π±Ρ‹ большС 10 тыс. Π»Π΅Ρ‚.

Π’ 2020Β Π³ΠΎΠ΄Ρƒ ΠΎΠ± Π°Π½Π°Π»ΠΎΠ³ΠΈΡ‡Π½ΠΎΠΌ достиТСнии рассказали ΡƒΡ‡Π΅Π½Ρ‹Π΅ ΠΈΠ· китайского Научно-тСхничСского унивСрситСта. Π˜Ρ… ΠΊΠ²Π°Π½Ρ‚ΠΎΠ²Ρ‹ΠΉ ΠΊΠΎΠΌΠΏΡŒΡŽΡ‚Π΅Ρ€, «Π¦Π·ΡŽ Π§ΠΆΠ°Π½», создан Π½Π° основС 73 Ρ„ΠΎΡ‚ΠΎΠ½Π½Ρ‹Ρ… ΠΊΡƒΠ±ΠΈΡ‚ΠΎΠ². Он Π·Π° 20 сСкунд Ρ€Π΅ΡˆΠΈΠ» Π΅Ρ‰Π΅ ΠΎΠ΄Π½Ρƒ Π·Π°Π΄Π°Ρ‡Ρƒ ΠΏΠΎ Π²Ρ‹Ρ€Π°Π±ΠΎΡ‚ΠΊΠ΅ случайных чисСл, ΠΊΠΎΡ‚ΠΎΡ€ΡƒΡŽ ΠΎΠ±Ρ‹Ρ‡Π½Ρ‹ΠΉ ΠΊΠΎΠΌΠΏΡŒΡŽΡ‚Π΅Ρ€ Π·Π°Π²Π΅Ρ€ΡˆΠΈΠ» Π±Ρ‹ Π·Π° нСсколько ΠΌΠΈΠ»Π»ΠΈΠ°Ρ€Π΄ΠΎΠ² Π»Π΅Ρ‚. Как ΠΈ Π² случаС с Sycamore, эти заявлСния Π²Ρ‹Π·Π²Π°Π»ΠΈ мноТСство споров ΠΎ Ρ‚ΠΎΠΌ, Ρ‡Ρ‚ΠΎ ΠΌΠΎΠΆΠ½ΠΎ ΡΡ‡ΠΈΡ‚Π°Ρ‚ΡŒ Π½Π΅Ρ€Π΅ΡˆΠ°Π΅ΠΌΠΎΠΉ Π·Π°Π΄Π°Ρ‡Π΅ΠΉ ΠΈ ΠΊΠ°ΠΊ Π΅Π΅ ΠΌΠΎΠΆΠ½ΠΎ ΠΏΡ€ΠΎΡΡ‡ΠΈΡ‚Ρ‹Π²Π°Ρ‚ΡŒ Π½Π° ΠΊΠ²Π°Π½Ρ‚ΠΎΠ²ΠΎΠΌ ΠΊΠΎΠΌΠΏΡŒΡŽΡ‚Π΅Ρ€Π΅.Β 

Π Π΅Π²ΠΎΠ»ΡŽΡ†ΠΈΡ Π² Π½Π°ΡƒΠΊΠ΅

ΠŸΠΎΠ΄ΠΎΠ±Π½Ρ‹Π΅ достиТСния заставили ΡƒΡ‡Π΅Π½Ρ‹Ρ… Π·Π°Π΄ΡƒΠΌΠ°Ρ‚ΡŒΡΡ, ΠΊΠ°ΠΊΠΈΠ΅ практичСскиС Π·Π°Π΄Π°Ρ‡ΠΈ смоТСт Ρ€Π΅ΡˆΠΈΡ‚ΡŒ ΠΊΠ²Π°Π½Ρ‚ΠΎΠ²Ρ‹ΠΉ ΠΊΠΎΠΌΠΏΡŒΡŽΡ‚Π΅Ρ€. Одним ΠΈΠ· Π½Π°ΠΏΡ€Π°Π²Π»Π΅Π½ΠΈΠΉ, ΠΊΠ°ΠΊ считаСт Π’Π°Π΄ΠΈΠΌ БмСлянский, ΠΌΠΎΠΆΠ΅Ρ‚ ΡΡ‚Π°Ρ‚ΡŒΒ ΡΠΈΠΌΡƒΠ»ΡΡ†ΠΈΡ физичСских систСм. «Π£ΠΆΠ΅ сСйчас, Π΅Ρ‰Π΅ Π΄ΠΎ появлСния систСм ΠΊΠΎΡ€Ρ€Π΅ΠΊΡ†ΠΈΠΈ ошибок, с ΠΏΠΎΠΌΠΎΡ‰ΡŒΡŽ ΠΊΠ²Π°Π½Ρ‚ΠΎΠ²Ρ‹Ρ… ΠΊΠΎΠΌΠΏΡŒΡŽΡ‚Π΅Ρ€ΠΎΠ² ΠΌΠΎΠΆΠ½ΠΎ ΠΈΠ·ΡƒΡ‡Π°Ρ‚ΡŒ явлСния, связанныС с ΠΊΠ²Π°Π½Ρ‚ΠΎΠ²Ρ‹ΠΌ хаосом ΠΈ исчСзновСниСм ΠΈΠ½Ρ„ΠΎΡ€ΠΌΠ°Ρ†ΠΈΠΈ. Другая ΠΊΠΎΠΌΠ°Π½Π΄Π° Google Π±ΡƒΠ΄Π΅Ρ‚ ΠΈΡΠΏΠΎΠ»ΡŒΠ·ΠΎΠ²Π°Ρ‚ΡŒ ΠΊΠ²Π°Π½Ρ‚ΠΎΠ²Ρ‹Π΅ Ρ†Π΅ΠΏΠΈ для построСния ΠΌΠΎΠ΄Π΅Π»Π΅ΠΉ ΠΊΠ²Π°Π½Ρ‚ΠΎΠ²ΠΎΠΉ Π³Ρ€Π°Π²ΠΈΡ‚Π°Ρ†ΠΈΠΈ», – пояснил ΡƒΡ‡Π΅Π½Ρ‹ΠΉ.

На эту Ρ‚Π΅ΠΌΡƒ

Π’Π°ΠΊΠΆΠ΅ исслСдоватСли ΠΏΠ»Π°Π½ΠΈΡ€ΡƒΡŽΡ‚ ΠΈΡΠΏΠΎΠ»ΡŒΠ·ΠΎΠ²Π°Ρ‚ΡŒ ΠΏΠΎΠ΄ΠΎΠ±Π½Ρ‹Π΅ систСмы, Ρ‡Ρ‚ΠΎΠ±Ρ‹ ΠΈΠ·ΡƒΡ‡Π°Ρ‚ΡŒ, ΠΊΠ°ΠΊ Π½Π΅ΠΊΠΎΡ‚ΠΎΡ€Ρ‹Π΅ ΠΏΠΎΠ»ΡƒΠΏΡ€ΠΎΠ²ΠΎΠ΄Π½ΠΈΠΊΠΈ ΠΈ ΠΌΠ΅Ρ‚Π°Π»Π»Ρ‹ ΠΌΠΎΠ³ΡƒΡ‚ ΠΏΡ€Π΅Π²Ρ€Π°Ρ‰Π°Ρ‚ΡŒΡΡ Π² изоляторы ΠΈΠ·-Π·Π° взаимодСйствий элСктронов Π²Π½ΡƒΡ‚Ρ€ΠΈ Π½ΠΈΡ…. Π£Ρ‡Π΅Π½Ρ‹Π΅ Π½Π΅Π΄Π°Π²Π½ΠΎ ΠΎΠ±Π½Π°Ρ€ΡƒΠΆΠΈΠ»ΠΈ, Ρ‡Ρ‚ΠΎ Π°Π½Π°Π»ΠΎΠ³ΠΈΡ‡Π½Ρ‹Π΅ процСссы, Ρ‚Π°ΠΊ Π½Π°Π·Ρ‹Π²Π°Π΅ΠΌΡ‹Π΅ ΠΏΠ΅Ρ€Π΅Ρ…ΠΎΠ΄Ρ‹ АндСрсона,Β ΠΌΠΎΠ³ΡƒΡ‚ Π²ΠΎΠ·Π½ΠΈΠΊΠ°Ρ‚ΡŒ Π² самых Ρ€Π°Π·Π½Ρ‹Ρ… условиях, Π² Ρ‚ΠΎΠΌ числС ΠΈ Π²Π½ΡƒΡ‚Ρ€ΠΈ кристаллов, элСктромагнитных Π²ΠΎΠ»Π½ ΠΈΠ»ΠΈ ΠΎΠ±Π»Π°ΠΊΠΎΠ² ΠΈΠ· Π°Ρ‚ΠΎΠΌΠΎΠ².

БмСлянский ΠΈ Π΅Π³ΠΎ ΠΊΠΎΠ»Π»Π΅Π³ΠΈ Π½Π°Π΄Π΅ΡŽΡ‚ΡΡ, Ρ‡Ρ‚ΠΎ ΠΊΠ²Π°Π½Ρ‚ΠΎΠ²Ρ‹Π΅ ΠΊΠΎΠΌΠΏΡŒΡŽΡ‚Π΅Ρ€Ρ‹ ΠΏΠΎΠΌΠΎΠ³ΡƒΡ‚ ΠΏΠΎΠ½ΡΡ‚ΡŒ ΠΏΡ€ΠΈΡ€ΠΎΠ΄Ρƒ этих ΠΏΠ΅Ρ€Π΅Ρ…ΠΎΠ΄ΠΎΠ² ΠΈ Π²Ρ‹ΡΡΠ½ΠΈΡ‚ΡŒ, ΠΊΠ°ΠΊΡƒΡŽ Ρ€ΠΎΠ»ΡŒ ΠΎΠ½ΠΈ ΠΈΠ³Ρ€Π°ΡŽΡ‚ Π² Ρ„ΠΎΡ€ΠΌΠΈΡ€ΠΎΠ²Π°Π½ΠΈΠΈ свойств Ρ‚ΠΎΠ³ΠΎ ΠΈΠ»ΠΈ ΠΈΠ½ΠΎΠ³ΠΎ Ρ‚ΠΈΠΏΠ° ΠΌΠ°Ρ‚Π΅Ρ€ΠΈΠΈ. По словам ΡƒΡ‡Π΅Π½ΠΎΠ³ΠΎ, для этого Π½Π΅ΠΎΠ±ΡΠ·Π°Ρ‚Π΅Π»ΡŒΠ½ΠΎ ΠΈΡΠΏΠΎΠ»ΡŒΠ·ΠΎΠ²Π°Ρ‚ΡŒ Sycamore ΠΈ Π΄Ρ€ΡƒΠ³ΠΈΠ΅ свСрхпроводниковыС ΠΊΠ²Π°Π½Ρ‚ΠΎΠ²Ρ‹Π΅ ΠΊΠΎΠΌΠΏΡŒΡŽΡ‚Π΅Ρ€Ρ‹ – достаточно Π±ΡƒΠ΄Π΅Ρ‚ ΠΈ Π΄Ρ€ΡƒΠ³ΠΈΡ… машин с подходящим количСством ΠΊΡƒΠ±ΠΈΡ‚ΠΎΠ² ΠΈ высоким ΡƒΡ€ΠΎΠ²Π½Π΅ΠΌ ΠΈΡ… связанности.

«Π’сС эти явлСния ΠΌΠΎΠΆΠ½ΠΎ ΠΈΠ·ΡƒΡ‡Π°Ρ‚ΡŒ Π² ΠΊΠ²Π°Π½Ρ‚ΠΎΠ²Ρ‹Ρ… систСмах самых Ρ€Π°Π·Π½Ρ‹Ρ… Π²ΠΈΠ΄ΠΎΠ², Π² Ρ‚ΠΎΠΌ числС ΠΈ Π½Π΅ΠΉΡ‚Ρ€Π°Π»ΡŒΠ½Ρ‹Ρ… Π°Ρ‚ΠΎΠΌΠΎΠ² ΠΈ ΠΈΠΎΠ½ΠΎΠ². Однако свСрхпроводящиС ΠΊΡƒΠ±ΠΈΡ‚Ρ‹ ΠΏΠΎΠΊΠ° максимально приспособлСны для Ρ€Π΅ΡˆΠ΅Π½ΠΈΡ этих Π·Π°Π΄Π°Ρ‡. По ΡΡ€Π°Π²Π½Π΅Π½ΠΈΡŽ с «ΠΊΠΎΠ½ΠΊΡƒΡ€Π΅Π½Ρ‚Π°ΠΌΠΈ»Β ΠΎΠ½ΠΈ сильно связаны Π΄Ρ€ΡƒΠ³ с Π΄Ρ€ΡƒΠ³ΠΎΠΌ ΠΈ Ρ€Π°Π±ΠΎΡ‚Π°ΡŽΡ‚ быстро. Π­Ρ‚ΠΎ Π²Π°ΠΆΠ½ΠΎ, ΠΊ ΠΏΡ€ΠΈΠΌΠ΅Ρ€Ρƒ, ΠΏΡ€ΠΈ ΠΈΠ·ΡƒΡ‡Π΅Π½ΠΈΠΈ ΠΊΠ²Π°Π½Ρ‚ΠΎΠ²ΠΎΠ³ΠΎ хаоса ΠΈ Ρ€Π°Π·Π»ΠΈΡ‡Π½Ρ‹Ρ… ΠΏΠ΅Ρ€Π΅Ρ…ΠΎΠ΄ΠΎΠ² ΠΌΠ΅ΠΆΠ΄Ρƒ ΠΏΠΎΠ΄ΠΎΠ±Π½Ρ‹ΠΌΠΈ состояниями», – продолТаСт Ρ„ΠΈΠ·ΠΈΠΊ.

НичСйная полоса

Π’ Π±ΡƒΠ΄ΡƒΡ‰Π΅ΠΌ, ΠΊΠ°ΠΊ ΠΎΡ‚ΠΌΠ΅Ρ‚ΠΈΠ» БмСлянский, Google ΠΏΠ»Π°Π½ΠΈΡ€ΡƒΠ΅Ρ‚ ΠΏΡ€ΠΈΠΌΠ΅Π½ΡΡ‚ΡŒ ΠΊΠ²Π°Π½Ρ‚ΠΎΠ²Ρ‹Π΅ ΠΊΠΎΠΌΠΏΡŒΡŽΡ‚Π΅Ρ€Ρ‹, Ρ‡Ρ‚ΠΎΠ±Ρ‹ ΡΠΎΠ·Π΄Π°Π²Π°Ρ‚ΡŒ Π½Π°ΡƒΡ‡Π½Ρ‹Π΅ Π·Π°Π΄Π°Ρ‡ΠΈ, Π½Π° ΠΊΠΎΡ‚ΠΎΡ€Ρ‹Ρ… Π±ΡƒΠ΄ΡƒΡ‚ ΠΎΠ±ΡƒΡ‡Π°Ρ‚ΡŒΡΡ систСмы искусствСнного ΠΈΠ½Ρ‚Π΅Π»Π»Π΅ΠΊΡ‚Π°, Π° Ρ‚Π°ΠΊΠΆΠ΅ для химичСских расчСтов Π½Π° ΡƒΡ€ΠΎΠ²Π½Π΅ ΠΎΡ‚Π΄Π΅Π»ΡŒΠ½Ρ‹Ρ… Π°Ρ‚ΠΎΠΌΠΎΠ². Однако это станСт Π²ΠΎΠ·ΠΌΠΎΠΆΠ½Ρ‹ΠΌ Ρ‚ΠΎΠ»ΡŒΠΊΠΎ послС Ρ‚ΠΎΠ³ΠΎ, ΠΊΠ°ΠΊ ΡƒΡ‡Π΅Π½Ρ‹Π΅ Ρ€Π΅ΡˆΠ°Ρ‚ Π΅Ρ‰Π΅ ΠΎΠ΄Π½Ρƒ Π²Π°ΠΆΠ½ΡƒΡŽ Π·Π°Π΄Π°Ρ‡Ρƒ.

На эту Ρ‚Π΅ΠΌΡƒ

«Π•ΡΡ‚ΡŒ Π΅Ρ‰Π΅ Π·Π°Π΄Π°Ρ‡ΠΈ ΠΊΠ²Π°Π½Ρ‚ΠΎΠ²ΠΎΠΉ Ρ…ΠΈΠΌΠΈΠΈ, ΠΎΠ΄Π½Π°ΠΊΠΎ ΠΏΠΎΠΊΠ° ΠΌΡ‹ Π½Π΅ ΠΏΠΎΠ½ΠΈΠΌΠ°Π΅ΠΌ, ΠΌΠΎΠΆΠ½ΠΎ Π»ΠΈ ΠΈΡ… Π±ΡƒΠ΄Π΅Ρ‚ Ρ€Π΅ΡˆΠ°Ρ‚ΡŒ Π±Π΅Π· создания систСм ΠΊΠΎΡ€Ρ€Π΅ΠΊΡ†ΠΈΠΈ ошибок, Π² ΠΎΡ‚Π»ΠΈΡ‡ΠΈΠ΅ ΠΎΡ‚ Ρ‚Π΅Ρ… Π°Π»Π³ΠΎΡ€ΠΈΡ‚ΠΌΠΎΠ², ΠΎ ΠΊΠΎΡ‚ΠΎΡ€Ρ‹Ρ… я ΡƒΠΆΠ΅ Π³ΠΎΠ²ΠΎΡ€ΠΈΠ». По сути, ΠΌΡ‹ ΠΏΠ΅Ρ€Π΅ΠΉΠ΄Π΅ΠΌ Π² Π½ΠΎΠ²ΠΎΠ΅ ΠΈΠ·ΠΌΠ΅Ρ€Π΅Π½ΠΈΠ΅, ΠΊΠΎΠ³Π΄Π° эта Π²Π΅Ρ…Π° Π±ΡƒΠ΄Π΅Ρ‚ достигнута, Ρ‚Π°ΠΊ ΠΊΠ°ΠΊ ΠΌΡ‹ смоТСм ΡΡ‚Π°Π²ΠΈΡ‚ΡŒ ΠΏΠ΅Ρ€Π΅Π΄ собой ΡΠΎΠ²Π΅Ρ€ΡˆΠ΅Π½Π½ΠΎ ΠΈΠ½Ρ‹Π΅, Π±ΠΎΠ»Π΅Π΅ Π°ΠΌΠ±ΠΈΡ†ΠΈΠΎΠ·Π½Ρ‹Π΅ ΠΈ слоТныС ΠΏΡ€ΠΎΠ±Π»Π΅ΠΌΡ‹», – отмСтил Ρ„ΠΈΠ·ΠΈΠΊ.

Π‘ ΠΏΠΎΠΌΠΎΡ‰ΡŒΡŽ Π°Π»Π³ΠΎΡ€ΠΈΡ‚ΠΌΠΎΠ² ΠΊΠΎΡ€Ρ€Π΅ΠΊΡ†ΠΈΠΈ ошибок Ρ„ΠΈΠ·ΠΈΠΊΠΈ ΠΈ Ρ…ΠΈΠΌΠΈΠΊΠΈ ΡΠΌΠΎΠ³ΡƒΡ‚Β ΠΏΡ€ΠΎΡΡ‡ΠΈΡ‚Ρ‹Π²Π°Ρ‚ΡŒ, ΠΊΠ°ΠΊ Π²Π·Π°ΠΈΠΌΠΎΠ΄Π΅ΠΉΡΡ‚Π²ΡƒΡŽΡ‚ ΠΌΠ΅ΠΆΠ΄Ρƒ собой ΠΎΡ‚Π΄Π΅Π»ΡŒΠ½Ρ‹Π΅ Π°Ρ‚ΠΎΠΌΡ‹ ΠΈ элСктроны Π²Π½ΡƒΡ‚Ρ€ΠΈ самых слоТных ΠΌΠΎΠ»Π΅ΠΊΡƒΠ». Благодаря этому ΡƒΡ‡Π΅Π½Ρ‹Π΅ смогут Ρ‚ΠΎΡ‡Π½ΠΎ ΠΏΡ€Π΅Π΄ΡΠΊΠ°Π·Ρ‹Π²Π°Ρ‚ΡŒ, ΠΊΠ°ΠΊΠΈΠ΅ свойства Π΅ΡΡ‚ΡŒ Ρƒ Ρ‚ΠΎ ΠΈΠ»ΠΈ ΠΈΠ½ΠΎΠ³ΠΎ – Π΄Π°ΠΆΠ΅ ΠΏΠΎΠΊΠ° Π½Π΅ ΡΡƒΡ‰Π΅ΡΡ‚Π²ΡƒΡŽΡ‰Π΅Π³ΠΎ – соСдинСния, Π° Ρ‚Π°ΠΊΠΆΠ΅ Ρ†Π΅Π»Π΅Π½Π°ΠΏΡ€Π°Π²Π»Π΅Π½Π½ΠΎ ΡΠΈΠ½Ρ‚Π΅Π·ΠΈΡ€ΠΎΠ²Π°Ρ‚ΡŒ вСщСства с ΠΊΠΎΠ½ΠΊΡ€Π΅Ρ‚Π½Ρ‹ΠΌΠΈ свойствами.

По словам БмСлянского, для этого ΠΏΠΎΠΊΠ° Π½Π΅ Ρ…Π²Π°Ρ‚Π°Π΅Ρ‚ Π΄Π²ΡƒΡ… Π²Π΅Ρ‰Π΅ΠΉ. Π‘ ΠΎΠ΄Π½ΠΎΠΉ стороны, количСство ΠΊΡƒΠ±ΠΈΡ‚ΠΎΠ² Π² ΠΊΠ²Π°Π½Ρ‚ΠΎΠ²Ρ‹Ρ… ΠΊΠΎΠΌΠΏΡŒΡŽΡ‚Π΅Ρ€Π°Ρ… ΠΏΠΎΠΊΠ° слишком Π½Π΅Π²Π΅Π»ΠΈΠΊΠΎ для Ρ‚ΠΎΠ³ΠΎ, Ρ‡Ρ‚ΠΎΠ±Ρ‹ Π½Π° Π½ΠΈΡ… ΠΌΠΎΠ³Π»ΠΈ Ρ€Π°Π±ΠΎΡ‚Π°Ρ‚ΡŒ ΡƒΠΆΠ΅ ΡΡƒΡ‰Π΅ΡΡ‚Π²ΡƒΡŽΡ‰ΠΈΠ΅ Π°Π»Π³ΠΎΡ€ΠΈΡ‚ΠΌΡ‹ ΠΊΠ²Π°Π½Ρ‚ΠΎΠ²ΠΎΠΉ ΠΊΠΎΡ€Ρ€Π΅ΠΊΡ†ΠΈΠΈ ошибок. Π‘ другой стороны ΠΊΡƒΠ±ΠΈΡ‚Ρ‹ Π²ΠΎ врСмя простых Π²Ρ‹Ρ‡ΠΈΡΠ»ΠΈΡ‚Π΅Π»ΡŒΠ½Ρ‹Ρ… ΠΎΠΏΠ΅Ρ€Π°Ρ†ΠΈΠΉΒ ΠΏΠΎΠΊΠ° Π΄Π΅Π»Π°ΡŽΡ‚ слишком ΠΌΠ½ΠΎΠ³ΠΎ случайных ошибок.

Из-Π·Π° этого ΡƒΡ‡Π΅Π½Ρ‹Π΅ ΠΏΠΎΠΊΠ° Π½Π΅ ΠΌΠΎΠ³ΡƒΡ‚ ΡΠΎΠ·Π΄Π°Π²Π°Ρ‚ΡŒ достаточно большиС ΠΊΠ²Π°Π½Ρ‚ΠΎΠ²Ρ‹Π΅ ΠΊΠΎΠΌΠΏΡŒΡŽΡ‚Π΅Ρ€Ρ‹, ΠΊΠΎΡ‚ΠΎΡ€Ρ‹Π΅ ΠΌΠΎΠ³Π»ΠΈ Π±Ρ‹ ΡΠ°ΠΌΠΎΡΡ‚ΠΎΡΡ‚Π΅Π»ΡŒΠ½ΠΎ ΠΈΡΠΏΡ€Π°Π²Π»ΡΡ‚ΡŒ ошибки Π² своСй Ρ€Π°Π±ΠΎΡ‚Π΅. Π’ Ρ€Π΅Π·ΡƒΠ»ΡŒΡ‚Π°Ρ‚Π΅ ΠΊΠ²Π°Π½Ρ‚ΠΎΠ²Ρ‹Π΅ ΠΊΠΎΠΌΠΏΡŒΡŽΡ‚Π΅Ρ€Ρ‹ находятся Π² своСобразной «Π½ΠΈΡ‡Π΅ΠΉΠ½ΠΎΠΉ полосС». Π˜Ρ… ΠΌΠΎΠΆΠ½ΠΎ ΠΈΡΠΏΠΎΠ»ΡŒΠ·ΠΎΠ²Π°Ρ‚ΡŒ для Ρ€Π΅ΡˆΠ΅Π½ΠΈΡ ΠΎΠ³Ρ€Π°Π½ΠΈΡ‡Π΅Π½Π½ΠΎΠ³ΠΎ числа слоТных практичСских Π·Π°Π΄Π°Ρ‡, ΠΎΠ΄Π½Π°ΠΊΠΎ ΠΏΠΎΠΊΠ° нСльзя ΠΏΡ€Π΅Π²Ρ€Π°Ρ‚ΠΈΡ‚ΡŒ Π² ΡƒΠ½ΠΈΠ²Π΅Ρ€ΡΠ°Π»ΡŒΠ½Ρ‹Π΅ Π²Ρ‹Ρ‡ΠΈΡΠ»ΠΈΡ‚Π΅Π»ΡŒΠ½Ρ‹Π΅ ΠΌΠ°ΡˆΠΈΠ½Ρ‹.

«ΠœΡ‹ ΠΏΡ€Π΅Π΄ΠΏΠΎΠ»Π°Π³Π°Π΅ΠΌ, Ρ‡Ρ‚ΠΎ Ρ‚ΠΎΠΉ точности Ρ€Π°Π±ΠΎΡ‚Ρ‹ Π½Π°ΡˆΠΈΡ… логичСских схСм, ΠΊΠΎΡ‚ΠΎΡ€ΡƒΡŽ сСйчас ΠΌΠΎΠΆΠ½ΠΎ Π΄ΠΎΡΡ‚ΠΈΡ‡ΡŒ, скорСС всСго, Π½Π΅ Ρ…Π²Π°Ρ‚ΠΈΡ‚ для вСдСния ΠΊΠ²Π°Π½Ρ‚ΠΎΠ²ΠΎ-химичСских расчСтов. Π‘ Π΄Ρ€ΡƒΠ³ΠΎΠΉ стороны, ΠΏΠΎΠΊΠ° этот вопрос ΠΎΡ‚ΠΊΡ€Ρ‹Ρ‚. Π’ΠΏΠΎΠ»Π½Π΅ Π²ΠΎΠ·ΠΌΠΎΠΆΠ½ΠΎ, Ρ‡Ρ‚ΠΎ ΠΌΡ‹ смоТСм Ρ€Π΅Π°Π»ΠΈΠ·ΠΎΠ²Π°Ρ‚ΡŒ ΠΊΠ°ΠΊΠΎΠΉ-Ρ‚ΠΎ Π±ΠΎΠ»Π΅Π΅ простой ΠΏΠΎΠ΄Ρ…ΠΎΠ΄ для ΠΊΠΎΡ€Ρ€Π΅ΠΊΡ†ΠΈΠΈ ошибок, ΠΊΠΎΡ‚ΠΎΡ€Ρ‹ΠΉ Π½Π΅ ΠΏΠΎΡ‚Ρ€Π΅Π±ΡƒΠ΅Ρ‚ ΠΊΡƒΠ±ΠΈΡ‚ΠΎΠ², ΠΆΠΈΠ²ΡƒΡ‰ΠΈΡ… бСсконСчно Π΄ΠΎΠ»Π³ΠΎ, подавлСния ΠΊΠ²Π°Π½Ρ‚ΠΎΠ²Ρ‹Ρ… ΡˆΡƒΠΌΠΎΠ² ΠΈ Π΄Ρ€ΡƒΠ³ΠΈΡ… слоТных Ρ‚Ρ€ΡŽΠΊΠΎΠ². Π’ Ρ‚Π΅Ρ‡Π΅Π½ΠΈΠ΅ Π½Π΅ΡΠΊΠΎΠ»ΡŒΠΊΠΈΡ… Π»Π΅Ρ‚ ΠΌΡ‹ Π±ΡƒΠ΄Π΅ΠΌ этим Π·Π°Π½ΠΈΠΌΠ°Ρ‚ΡŒΡΡ», – ΠΎΡ‚ΠΌΠ΅Ρ‚ΠΈΠ» БмСлянский.

ΠšΠ²Π°Π½Ρ‚ΠΎΠ²ΠΎΠ΅ ускорСниС

ΠŸΡ€ΠΈ этом, ΠΏΠΎ Π΅Π³ΠΎ словам, ΡƒΡ‡Π΅Π½Ρ‹Π΅ Π΄ΠΎΠ»ΠΆΠ½Ρ‹ Ρ„ΠΎΠΊΡƒΡΠΈΡ€ΠΎΠ²Π°Ρ‚ΡŒ свои усилия Π½Π΅ Π½Π° ΡƒΠ²Π΅Π»ΠΈΡ‡Π΅Π½ΠΈΠΈ точности Ρ€Π°Π±ΠΎΡ‚Ρ‹ ΠΊΡƒΠ±ΠΈΡ‚ΠΎΠ², Π° Π½Π° создании Π°Π»Π³ΠΎΡ€ΠΈΡ‚ΠΌΠΎΠ², ΠΊΠΎΡ‚ΠΎΡ€Ρ‹Π΅ смогли Π±Ρ‹ ΠΏΠΎΠ΄Π°Π²Π»ΡΡ‚ΡŒ ΠΏΠΎΠΌΠ΅Ρ…ΠΈ, Π° Ρ‚Π°ΠΊΠΆΠ΅ ΠΎΠ±Π»Π΅Π³Ρ‡Π΅Π½Π½Ρ‹Ρ… схСм, с ΠΏΠΎΠΌΠΎΡ‰ΡŒΡŽ ΠΊΠΎΡ‚ΠΎΡ€Ρ‹Ρ… ΠΌΠΎΠΆΠ½ΠΎ Π±Ρ‹Π»ΠΎ Π±Ρ‹ ΠΊΠΎΡ€Ρ€Π΅ΠΊΡ‚ΠΈΡ€ΠΎΠ²Π°Ρ‚ΡŒ ошибки.

На эту Ρ‚Π΅ΠΌΡƒ

«Π”Π°ΠΆΠ΅ Ссли ΠΌΡ‹ Π² Ρ€Π°Π·Ρ‹ ΡƒΠΌΠ΅Π½ΡŒΡˆΠΈΠΌ ΡƒΡ€ΠΎΠ²Π΅Π½ΡŒ ошибок ΠΈ повысим качСство Ρ€Π°Π±ΠΎΡ‚Ρ‹ связанных Ρ†Π΅ΠΏΠΎΡ‡Π΅ΠΊ ΠΊΡƒΠ±ΠΈΡ‚ΠΎΠ², Ρ‚ΠΎ этого всС Ρ€Π°Π²Π½ΠΎ Π½Π΅ Ρ…Π²Π°Ρ‚ΠΈΡ‚ для Ρ‚ΠΎΠ³ΠΎ, Ρ‡Ρ‚ΠΎΠ±Ρ‹ Ρ€Π΅ΡˆΠ°Ρ‚ΡŒ Π·Π°Π΄Π°Ρ‡ΠΈ ΠΊΠ²Π°Π½Ρ‚ΠΎΠ²ΠΎΠΉ Ρ…ΠΈΠΌΠΈΠΈ. ΠŸΠΎΡΡ‚ΠΎΠΌΡƒ Π½Π°ΠΌ критичСски Π²Π°ΠΆΠ΅Π½ прогрСсс ΠΈΠΌΠ΅Π½Π½ΠΎ Π² области подавлСния ошибок. Π‘Π΅Π· этого ΠΌΡ‹ Π½ΠΈΡ‡Π΅Π³ΠΎ Π½Π΅ смоТСм ΡΠ΄Π΅Π»Π°Ρ‚ΡŒ. И Ссли ΠΌΡ‹ достигнСм всСго Π²Ρ‹ΡˆΠ΅ΠΏΠ΅Ρ€Π΅Ρ‡ΠΈΡΠ»Π΅Π½Π½ΠΎΠ³ΠΎ, Ρ‚ΠΎ всС Ρ€Π°Π²Π½ΠΎ Π½ΠΈΠΊΠ°ΠΊΠΎΠΉ Π³Π°Ρ€Π°Π½Ρ‚ΠΈΠΈ успСха Ρƒ нас Π½Π΅Ρ‚», – ΠΏΠΎΠ΄Ρ‡Π΅Ρ€ΠΊΠ½ΡƒΠ» Ρ„ΠΈΠ·ΠΈΠΊ.

По словам БмСлянского, Π² дальнСйшСм ΡƒΡ‡Π΅Π½Ρ‹Π΅ Π±ΡƒΠ΄ΡƒΡ‚ ΡΠΎΠ²Π΅Ρ€ΡˆΠ΅Π½ΡΡ‚Π²ΠΎΠ²Π°Ρ‚ΡŒ ΡƒΠΆΠ΅ ΡΡƒΡ‰Π΅ΡΡ‚Π²ΡƒΡŽΡ‰ΠΈΠ΅ ΠΈ достаточно Ρ€Π°Π·Π²ΠΈΡ‚Ρ‹Π΅ ΠΏΠΎΠ΄Ρ…ΠΎΠ΄Ρ‹, Π² Ρ‚ΠΎΠΌ числС свСрхпроводящиС Ρ†Π΅ΠΏΠΈ, ΠΏΠΎΠ»ΡƒΠΏΡ€ΠΎΠ²ΠΎΠ΄Π½ΠΈΠΊΠΎΠ²Ρ‹Π΅ схСмы, Ρ…ΠΎΠ»ΠΎΠ΄Π½Ρ‹Π΅ Π°Ρ‚ΠΎΠΌΡ‹ ΠΈΠ»ΠΈ ΠΈΠΎΠ½Ρ‹. Π§Ρ‚ΠΎΠ±Ρ‹ ΡΠΎΠ·Π΄Π°Π²Π°Ρ‚ΡŒ ΠΏΡ€ΠΈΠ½Ρ†ΠΈΠΏΠΈΠ°Π»ΡŒΠ½ΠΎ ΠΈΠ½Ρ‹Π΅ систСмы для ΠΊΠ²Π°Π½Ρ‚ΠΎΠ²Ρ‹Ρ… вычислСний, Π² Ρ‚ΠΎΠΌ числС Π½Π° Π±Π°Π·Π΅ Π°Ρ‚ΠΎΠΌΠ½Ρ‹Ρ… ядСр, Π½ΡƒΠΆΠ½Ρ‹ Π±ΡƒΠ΄ΡƒΡ‚Β ΠΊΠΎΠ»ΠΎΡΡΠ°Π»ΡŒΠ½Ρ‹Π΅ усилия ΠΈ врСмя. Π£Ρ‡Π΅Π½Ρ‹ΠΉ ΠΏΠΎΠΊΠ° Π½Π΅ Π²ΠΈΠ΄ΠΈΡ‚ Π½ΠΈΠΊΠ°ΠΊΠΈΡ… пСрспСктивных, Π½ΠΎ ΠΏΠΎΠΊΠ° Π½Π΅ ΠΈΠ·ΡƒΡ‡Π΅Π½Π½Ρ‹Ρ… Π½Π°ΠΏΡ€Π°Π²Π»Π΅Π½ΠΈΠΉ.

«ΠΠ°ΡˆΠΈ российскиС ΠΊΠΎΠ»Π»Π΅Π³ΠΈ сСйчас говорят ΠΎ Ρ‚ΠΎΠΌ, Ρ‡Ρ‚ΠΎ ΠΎΠ½ΠΈ ΠΏΠ»Π°Π½ΠΈΡ€ΡƒΡŽΡ‚ Ρ€Π°Π·Π²ΠΈΠ²Π°Ρ‚ΡŒ всС Ρ‡Π΅Ρ‚Ρ‹Ρ€Π΅ направлСния Π² Ρ€Π°ΠΌΠΊΠ°Ρ… ΠΊΠ²Π°Π½Ρ‚ΠΎΠ²ΠΎΠ³ΠΎ ΠΏΡ€ΠΎΠ΅ΠΊΡ‚Π° «Π ΠΎΡΠ°Ρ‚ΠΎΠΌΠ°». Однако Π² Π΄Π°Π½Π½ΠΎΠΌ случаС Ρ€Π΅Ρ‡ΡŒ ΠΈΠ΄Π΅Ρ‚ ΠΎ Ρ‚ΠΎΠΌ, Ρ‡Ρ‚ΠΎ ΠΎΠ½ΠΈ Π±ΡƒΠ΄ΡƒΡ‚ ΠΈΠ·ΡƒΡ‡Π°Ρ‚ΡŒ, Ρ‡Ρ‚ΠΎ сдСлали Π΄Ρ€ΡƒΠ³ΠΈΠ΅. Π­Ρ‚ΠΎ Π½Π΅Π²ΠΎΠ·ΠΌΠΎΠΆΠ½ΠΎ Π΄Π΅Π»Π°Ρ‚ΡŒ, просто читая ΠΆΡƒΡ€Π½Π°Π»Ρ‹ – для этого Π½ΡƒΠΆΠ½Ρ‹ Π»Π°Π±ΠΎΡ€Π°Ρ‚ΠΎΡ€ΠΈΠΈ. Как я понимаю, ΠΎΠ½ΠΈ Π±ΡƒΠ΄ΡƒΡ‚ этим Π·Π°Π½ΠΈΠΌΠ°Ρ‚ΡŒΡΡ Π΄ΠΎ Ρ‚Π΅Ρ… ΠΏΠΎΡ€, ΠΏΠΎΠΊΠ° ΠΎΠ½ΠΈ Π½Π΅ ΠΏΡ€ΠΈΠΌΡƒΡ‚ Ρ€Π΅ΡˆΠ΅Π½ΠΈΠ΅ ΡΠΎΡΡ€Π΅Π΄ΠΎΡ‚ΠΎΡ‡ΠΈΡ‚ΡŒΡΡ Π½Π° Ρ‡Π΅ΠΌ-Ρ‚ΠΎ ΠΎΠ΄Π½ΠΎΠΌ», – Π΄ΠΎΠ±Π°Π²ΠΈΠ» БмСлянский.

Π”Π°Π»Π΅ΠΊΠΎ Π½Π΅ всС эти усилия, ΠΊΠ°ΠΊ считаСт спСциалист Google, принСсут Ρ€Π΅Π·ΡƒΠ»ΡŒΡ‚Π°Ρ‚. Π’ частности, ΠΎΠ½ ΠΎΡ‚ΠΌΠ΅Ρ‚ΠΈΠ», Ρ‡Ρ‚ΠΎ считаСт адиабатичСский ΠΏΠΎΠ΄Ρ…ΠΎΠ΄, ΠΊΠΎΡ‚ΠΎΡ€Ρ‹Π΅ Π½Π°Π±ΠΈΡ€Π°Π΅Ρ‚ ΠΏΠΎΠΏΡƒΠ»ΡΡ€Π½ΠΎΡΡ‚ΡŒ срСди ΡƒΡ‡Π΅Π½Ρ‹Ρ… ΠΈ инвСсторов, Π³ΠΎΡ€Π°Π·Π΄ΠΎ ΠΌΠ΅Π½Π΅Π΅ пСрспСктивным ΠΈ интСрСсным, Ρ‡Π΅ΠΌ созданиС ΡƒΠ½ΠΈΠ²Π΅Ρ€ΡΠ°Π»ΡŒΠ½Ρ‹Ρ… ΠΊΠ²Π°Π½Ρ‚ΠΎΠ²Ρ‹Ρ… Π²Ρ‹Ρ‡ΠΈΡΠ»ΠΈΡ‚Π΅Π»ΡŒΠ½Ρ‹Ρ… систСм.

«Π’ΠΊΠ»Π°Π΄Ρ‹Π²Π°Π½ΠΈΠ΅ рСсурсов Π² Ρ‚Π°ΠΊΠΈΠ΅ ΠΏΡ€ΠΎΠ΅ΠΊΡ‚Ρ‹ – большая ошибка. ΠšΠ²Π°Π½Ρ‚ΠΎΠ²Ρ‹Π΅ ΠΎΡ‚ΠΆΠΈΠ³Π°Ρ‚Π΅Π»ΠΈ (Ρ‚Π°ΠΊ Π½Π°Π·Ρ‹Π²Π°ΡŽΡ‚ ΠΏΡ€ΠΎΡ‚ΠΎΡ‚ΠΈΠΏΡ‹ адиабатичСских ΠΊΠ²Π°Π½Ρ‚ΠΎΠ²Ρ‹Ρ… ΠΊΠΎΠΌΠΏΡŒΡŽΡ‚Π΅Ρ€ΠΎΠ², – ΠΏΡ€ΠΈΠΌ. ВАББ) ΠΌΠΎΠΆΠ½ΠΎ Π±ΡƒΠ΄Π΅Ρ‚ ΠΈΡΠΏΠΎΠ»ΡŒΠ·ΠΎΠ²Π°Ρ‚ΡŒ для физичСских экспСримСнтов, ΠΎΠ΄Π½Π°ΠΊΠΎ ΠΎΠ½ΠΈ вряд Π»ΠΈ ΠΏΠΎΠ΄ΠΎΠΉΠ΄ΡƒΡ‚ для Ρ€Π΅ΡˆΠ΅Π½ΠΈΡ ΠΎΠΏΡ‚ΠΈΠΌΠΈΠ·Π°Ρ†ΠΈΠΎΠ½Π½Ρ‹Ρ… Π·Π°Π΄Π°Ρ‡. НСдавно наша Π³Ρ€ΡƒΠΏΠΏΠ° ΠΏΠΎΠΊΠ°Π·Π°Π»Π°, Ρ‡Ρ‚ΠΎ адиабатичСскиС ΠΏΠΎΠ΄Ρ…ΠΎΠ΄Ρ‹ Π²ΠΎ всСх рСалистичных случаях приходят ΠΊ ΠΎΡ‚Π²Π΅Ρ‚Ρƒ Π½Π΅ быстрСС, Ρ‡Π΅ΠΌ классичСский ΠΊΠ²Π°Π½Ρ‚ΠΎΠ²Ρ‹ΠΉ Π°Π»Π³ΠΎΡ€ΠΈΡ‚ΠΌ ΠœΠΎΠ½Ρ‚Π΅-ΠšΠ°Ρ€Π»ΠΎ. По сути, ΠΊΠ²Π°Π½Ρ‚ΠΎΠ²ΠΎΠ³ΠΎ ускорСния здСсь Π½Π΅ Π½Π°Π±Π»ΡŽΠ΄Π°Π΅Ρ‚ΡΡ», – пояснил ΠΈΡΡΠ»Π΅Π΄ΠΎΠ²Π°Ρ‚Π΅Π»ΡŒ.

НовыС пСрспСктивы

Π‘Π°ΠΌ Google, ΠΏΠΎ словам БмСлянского, ΠΏΠ»Π°Π½ΠΈΡ€ΡƒΠ΅Ρ‚ ΠΈΡΠΏΠΎΠ»ΡŒΠ·ΠΎΠ²Π°Ρ‚ΡŒ ΡƒΠ½ΠΈΠ²Π΅Ρ€ΡΠ°Π»ΡŒΠ½Ρ‹Π΅ ΠΊΠ²Π°Π½Ρ‚ΠΎΠ²Ρ‹Π΅ Π²Ρ‹Ρ‡ΠΈΡΠ»ΠΈΡ‚Π΅Π»ΡŒΠ½Ρ‹Π΅ ΠΌΠ°ΡˆΠΈΠ½Ρ‹ Π½Π΅ Ρ‚ΠΎΠ»ΡŒΠΊΠΎ для ΠΏΡ€ΠΎΠ²Π΅Ρ€ΠΊΠΈ Ρ„ΡƒΠ½Π΄Π°ΠΌΠ΅Π½Ρ‚Π°Π»ΡŒΠ½Ρ‹Ρ… Π·Π°ΠΊΠΎΠ½ΠΎΠ² Ρ„ΠΈΠ·ΠΈΠΊΠΈ, ΡƒΠ»ΡƒΡ‡ΡˆΠ΅Π½ΠΈΡ Ρ€Π°Π±ΠΎΡ‚Ρ‹ систСм поиска ΠΈΠ½Ρ„ΠΎΡ€ΠΌΠ°Ρ†ΠΈΠΈ, Ρ‚Ρ€Π΅Π½ΠΈΡ€ΠΎΠ²ΠΊΠΈ искусствСнного ΠΈΠ½Ρ‚Π΅Π»Π»Π΅ΠΊΡ‚Π° ΠΈ Π΄Ρ€ΡƒΠ³ΠΈΡ… абстрактных IT-Π·Π°Π΄Π°Ρ‡, Π½ΠΎ ΠΈ Π½Π° ΠΏΡ€Π°ΠΊΡ‚ΠΈΠΊΠ΅.

На эту Ρ‚Π΅ΠΌΡƒ

Π’ частности, ΡƒΡ‡Π΅Π½Ρ‹Π΅ ΡΠΎΠ±ΠΈΡ€Π°ΡŽΡ‚ΡΡ ΠΏΡ€ΠΈΠΌΠ΅Π½ΡΡ‚ΡŒ ΠΊΠ²Π°Π½Ρ‚ΠΎΠ²Ρ‹Π΅ ΠΊΠΎΠΌΠΏΡŒΡŽΡ‚Π΅Ρ€Ρ‹ для создания Π°Π»Π³ΠΎΡ€ΠΈΡ‚ΠΌΠΎΠ², ΠΊΠΎΡ‚ΠΎΡ€Ρ‹Π΅ ΡƒΠΏΡ€Π°Π²Π»ΡΡŽΡ‚ Ρ€Π°Π±ΠΎΡ‚ΠΎΠΉ Π°Π²Ρ‚ΠΎΠ½ΠΎΠΌΠ½ΠΎΠ³ΠΎ транспорта, Π° Ρ‚Π°ΠΊΠΆΠ΅ для ΡƒΠ»ΡƒΡ‡ΡˆΠ΅Π½ΠΈΡ свойств ΠΊΠΎΠΌΠΏΠΎΠ½Π΅Π½Ρ‚ΠΎΠ² Π»ΠΈΡ‚ΠΈΠ΅Π²Ρ‹Ρ… Π±Π°Ρ‚Π°Ρ€Π΅ΠΉ ΠΈ Π΄Ρ€ΡƒΠ³ΠΈΡ… Ρ‚ΠΈΠΏΠΎΠ² аккумуляторов.

«ΠΠ°ΡΠ»Π΅Π΄Π½ΠΈΠΊΠΎΠ²» Sycamore ΠΏΠ»Π°Π½ΠΈΡ€ΡƒΡŽΡ‚ ΠΏΡ€ΠΈΠ²Π»Π΅Ρ‡ΡŒ для ΠΏΠΎΠΌΠΎΡ‰ΠΈ Π±ΠΈΠΎΠ»ΠΎΠ³Π°ΠΌ ΠΈ Π±ΠΈΠΎΡ…ΠΈΠΌΠΈΠΊΠ°ΠΌ ΠΈΠ· Google, Π° Ρ‚Π°ΠΊΠΆΠ΅ ΠΈΡ… ΠΏΠ°Ρ€Ρ‚Π½Π΅Ρ€Π°ΠΌ Π² Π“Π°Ρ€Π²Π°Ρ€Π΄Π΅ ΠΈ Π² Π΄Ρ€ΡƒΠ³ΠΈΡ… амСриканских унивСрситСтах, Ρ‡Ρ‚ΠΎΠ±Ρ‹ Π·Π½Π°Ρ‡ΠΈΡ‚Π΅Π»ΡŒΠ½ΠΎ ΡƒΡΠΊΠΎΡ€ΠΈΡ‚ΡŒ поиски Ρ€Π°Π·Π»ΠΈΡ‡Π½Ρ‹Ρ… Π½ΠΎΠ²Ρ‹Ρ… ΠΌΠ°Ρ‚Π΅Ρ€ΠΈΠ°Π»ΠΎΠ² ΠΈ ΠΊΠ°Ρ‚Π°Π»ΠΈΠ·Π°Ρ‚ΠΎΡ€ΠΎΠ², Π° Ρ‚Π°ΠΊΠΆΠ΅ лСкарств ΠΎΡ‚ Ρ€Π°ΠΊΠ° ΠΈ Π΄Ρ€ΡƒΠ³ΠΈΡ… Π±ΠΎΠ»Π΅Π·Π½Π΅ΠΉ.

Однако ΠΊΠΎΠ³Π΄Π° ΠΈ ΠΊΠ°ΠΊ всС это Π±ΡƒΠ΄Π΅Ρ‚ Ρ€Π΅Π°Π»ΠΈΠ·ΠΎΠ²Π°Π½ΠΎ, ΠΏΠΎΠΊΠ° ΡΠΊΠ°Π·Π°Ρ‚ΡŒ слоТно, Ρ‚Π°ΠΊ ΠΊΠ°ΠΊ ΡƒΡ‡Π΅Π½Ρ‹Π΅ Π΅Ρ‰Π΅ Π½Π΅ Ρ€Π΅ΡˆΠΈΠ»ΠΈ слоТныС тСхничСскиС вопросы, связанныС с созданиСм ΡƒΠ½ΠΈΠ²Π΅Ρ€ΡΠ°Π»ΡŒΠ½ΠΎΠ³ΠΎ ΠΊΠ²Π°Π½Ρ‚ΠΎΠ²ΠΎΠ³ΠΎ ΠΊΠΎΠΌΠΏΡŒΡŽΡ‚Π΅Ρ€Π° с тысячами логичСских ΠΊΡƒΠ±ΠΈΡ‚ΠΎΠ².

«Π‘Сйчас ΠΌΡ‹ Ρ€Π°Π±ΠΎΡ‚Π°Π΅ΠΌ ΠΈ смотрим Π½Π° эти ΠΏΡ€ΠΎΠ±Π»Π΅ΠΌΡ‹ Π² Ρ„ΠΎΡ€ΠΌΠ°Ρ‚Π΅ «Π½Π΅ Π΄ΠΎ ΠΆΠΈΡ€Ρƒ, Π±Ρ‹Ρ‚ΡŒ Π±Ρ‹ ΠΆΠΈΠ²Ρƒ». ΠŸΠΎΡΡ‚ΠΎΠΌΡƒ пСрвоочСрСдная наша Ρ†Π΅Π»ΡŒ – просто ΠΏΠΎΡΡ‚Ρ€ΠΎΠΈΡ‚ΡŒ ΠΌΠ°ΡˆΠΈΠ½Ρƒ, которая смогла Π±Ρ‹ Ρ€Π΅ΡˆΠ°Ρ‚ΡŒ ΠΏΠΎΠ΄ΠΎΠ±Π½Ρ‹Π΅ Π·Π°Π΄Π°Ρ‡ΠΈ. Волько ΠΏΠΎΡ‚ΠΎΠΌ ΠΌΡ‹ Π±ΡƒΠ΄Π΅ΠΌ Π΄ΡƒΠΌΠ°Ρ‚ΡŒ ΠΎ Ρ‚ΠΎΠΌ, ΠΊΠ°ΠΊ всС эти ΠΏΠ°Ρ€Π°ΠΌΠ΅Ρ‚Ρ€Ρ‹ Π±ΡƒΠ΄ΡƒΡ‚ Π²Π»ΠΈΡΡ‚ΡŒ Π½Π° Π΅Π΅ Ρ€Π°Π±ΠΎΡ‚Ρƒ ΠΈ Ρ€Π°Π·Ρ€Π°Π±Π°Ρ‚Ρ‹Π²Π°Ρ‚ΡŒ ΠΌΠ΅Ρ‚Ρ€ΠΈΠΊΠΈ, ΠΏΠΎ ΠΊΠΎΡ‚ΠΎΡ€Ρ‹ΠΌ ΠΌΠΎΠΆΠ½ΠΎ ΠΎΡ†Π΅Π½ΠΈΠ²Π°Ρ‚ΡŒ эти измСнСния», – подытоТил БмСлянский.

Каково это β€” ΠΊΠ°Ρ‚Π°ΠΏΡƒΠ»ΡŒΡ‚ΠΈΡ€ΠΎΠ²Π°Ρ‚ΡŒΡΡ ΠΈΠ· самолСта

КАПИВАН ДАРРЕН Β«Π‘ΠΠœ-Π‘ΠΠœΒ» Π£Π˜Π—, 29 Π»Π΅Ρ‚, ΠΏΠΈΠ»ΠΎΡ‚ BBC:

Β«ΠšΠ°Ρ‚Π°ΠΏΡƒΠ»ΡŒΡ‚ΠΈΡ€ΠΎΠ²Π°Π½ΠΈΠ΅Β β€” Π»ΡƒΡ‡ΡˆΠΈΠΉ способ ΠΈΡΠΏΡ‹Ρ‚Π°Ρ‚ΡŒ своС Ρ‚Π΅Π»ΠΎ Π½Π°Β ΠΏΡ€ΠΎΡ‡Π½ΠΎΡΡ‚ΡŒ. Иногда люди при этом Π³ΠΈΠ±Π½ΡƒΡ‚. Или Ρ‚Π΅Ρ€ΡΡŽΡ‚ конСчности: Ρ€ΡƒΠΊΠΈ-Π½ΠΎΠ³ΠΈ ΠΎΡ‚Ρ€Ρ‹Π²Π°ΡŽΡ‚ΡΡ ΠΈΠ»ΠΈ гнутся Π²Β Π΄ΡƒΠ³Ρƒ. Π―Β Π»ΠΈΡ‡Π½ΠΎ стал мСньшС ростом: Π±Ρ‹Π» 174 см, а стал Π½Π°Β 2 см ΠΊΠΎΡ€ΠΎΡ‡Π΅. Π”Π΅Π»ΠΎ Π±Ρ‹Π»ΠΎ Π²Β Ρ‚Ρ€Π΅Π½ΠΈΡ€ΠΎΠ²ΠΎΡ‡Π½ΠΎΠΌ ΠΏΠΎΠ»Π΅Ρ‚Π΅ Π²Β Π’ΠΈΡ€Π³ΠΈΠ½ΠΈΠΈ. Π―Β ΠΏΠΈΠ»ΠΎΡ‚ΠΈΡ€ΠΎΠ²Π°Π» F-15E (амСриканский ΠΈΡΡ‚Ρ€Π΅Π±ΠΈΡ‚Π΅Π»ΡŒ. β€”Β Esquire). На борту Π±Ρ‹Π»ΠΈ я и мой стрСлок ΠšΡ€Π°ΡΡ‚ΠΈ. ИдСм Π½Π°Β Π±Ρ€Π΅ΡŽΡ‰Π΅ΠΌ, с высокой ΡΠΊΠΎΡ€ΠΎΡΡ‚ΡŒΡŽ. Π˜Β Π²Π΄Ρ€ΡƒΠ³ Π²ΠΈΠΆΡƒ: справа ΠΏΡ€ΠΎΠΌΠ΅Π»ΡŒΠΊΠ½ΡƒΠ» стСрвятник. Π’ΡƒΡ‚Β ΠΆΠ΅ ΠΌΠ°ΡˆΠΈΠ½Ρƒ сильно встряхнуло, и раздался Π²Π·Ρ€Ρ‹Π². Π­Ρ‚ΠΎ ΠΏΡ‚ΠΈΡ†Ρƒ засосало Π²Β Π΄Π²ΠΈΠ³Π°Ρ‚Π΅Π»ΡŒ. Π§Π΅Ρ€Π΅Π·Β ΠΏΠ°Ρ€Ρƒ сСкунд хвост ΠΈΒ ΠΏΡ€Π°Π²Ρ‹ΠΉ Π±ΠΎΡ€Ρ‚ оказались Π²Β ΠΏΠ»Π°ΠΌΠ΅Π½ΠΈ. ΠΠ°Ρ‡ΠΈΠ½Π°ΡŽ Π½Π°Π±ΠΈΡ€Π°Ρ‚ΡŒ высоту, но машина нС поддаСтся и заваливаСтся Π½Π°Β ΠΏΡ€Π°Π²ΠΎΠ΅ ΠΊΡ€Ρ‹Π»ΠΎ. Надо ΡΠ²Π°Π»ΠΈΠ²Π°Ρ‚ΡŒ. Π‘ΠΎΠΎΠ±Ρ‰Π°ΡŽ ΠΏΠΎΒ Ρ€Π°Π΄ΠΈΠΎ Π²Π΅Π΄ΠΎΠΌΠΎΠΌΡƒ, Ρ‡Ρ‚ΠΎΠ±Ρ‹ дСрТался подальшС, и даю ΠΊΠΎΠΌΠ°Π½Π΄Ρƒ: Β«ΠŸΠΎΠΊΠΈΠ½ΡƒΡ‚ΡŒ самолСт!Β»

Π‘Β ΠΎΠ±Π΅ΠΈΡ… сторон крСсла ΠΏΠΎΒ Ρ€ΡƒΡ‡ΠΊΠ΅. Π”Π΅Ρ€Π³Π°ΡŽ Π·Π°Β Π½ΠΈΡ…. ΠœΠ΅Ρ…Π°Π½ΠΈΠ·ΠΌ срабатываСт — отстрСливаСтся стСкло ΠΊΠ°Π±ΠΈΠ½Ρ‹. Π’Π΅ΠΏΠ΅Ρ€ΡŒ совсСм ΡˆΡƒΠΌΠ½ΠΎ, Π²Π΅Ρ‚Π΅Ρ€ Ρ…Π»Π΅Ρ‰Π΅Ρ‚. Π’Π·Π»Π΅Ρ‚Π°Π΅Ρ‚ Π·Π°Π΄Π½Π΅Π΅ крСсло — ΠšΡ€Π°ΡΡ‚ΠΈ выбросился. ΠžΡΡ‚Π°ΡŽΡΡŒ в машинС ΠΎΠ΄ΠΈΠ½Β β€” каТСтся, Π½Π°Β Ρ†Π΅Π»ΡƒΡŽ Π²Π΅Ρ‡Π½ΠΎΡΡ‚ΡŒ, хотя ΠΏΡ€ΠΎΡˆΠ»ΠΎ мСньшС сСкунды. Π‘Ρ‚ΠΈΡΠΊΠΈΠ²Π°ΡŽ Ρ€ΡƒΡ‡ΠΊΠΈ. Π“Π»Π°Π·Π° Π·Π°ΠΊΡ€Ρ‹Π». Π’Π΅ΡΡŒ напрягся — знаю, сСйчас Ρ‚Π°ΠΊΠΎΠΉ Π»ΡƒΠ½Π°-ΠΏΠ°Ρ€ΠΊ начнСтся! Π‘Π»Ρ‹ΡˆΡƒ: Π²Π΅Π΄ΠΎΠΌΡ‹ΠΉ Ρ€Π°Π΄ΠΈΡ€ΡƒΠ΅Ρ‚ диспСтчСрам, Ρ‡Ρ‚ΠΎ ΠΌΡ‹Β Π²Ρ‹Π½ΡƒΠΆΠ΄Π΅Π½Ρ‹ ΠΊΠ°Ρ‚Π°ΠΏΡƒΠ»ΡŒΡ‚ΠΈΡ€ΠΎΠ²Π°Ρ‚ΡŒΡΡ. Π˜Β Π²Ρ‹Π»Π΅Ρ‚Π°ΡŽ ΠΈΠ·Β ΠΊΠ°Π±ΠΈΠ½Ρ‹ сам.

Π”Π°ΠΆΠ΅ нС знаю, ΠΊΠ°ΠΊ это ΠΎΠΏΠΈΡΠ°Ρ‚ΡŒ. Бидишь в крСслС, ΠΊΒ ΠΊΠΎΡ‚ΠΎΡ€ΠΎΠΌΡƒ ΠΏΡ€ΠΈΠ΄Π΅Π»Π°Π½Π° Ρ€Π°ΠΊΠ΅Ρ‚Π°. Π˜Β ΡΡ‚Π°Ρ€Ρ‚ΡƒΠ΅ΡˆΡŒ из самолСта, ΠΊΠΎΡ‚ΠΎΡ€Ρ‹ΠΉ Π»Π΅Ρ‚ΠΈΡ‚ ΡΠΎΒ ΡΠΊΠΎΡ€ΠΎΡΡ‚ΡŒΡŽ 480 ΠΊΠΌ/час. Π”Π°, к крСслу Π΄Π΅ΠΉΡΡ‚Π²ΠΈΡ‚Π΅Π»ΡŒΠ½ΠΎ ΠΏΡ€ΠΈΠ΄Π΅Π»Π°Π½ Ρ€Π°ΠΊΠ΅Ρ‚Π½Ρ‹ΠΉ Π΄Π²ΠΈΠ³Π°Ρ‚Π΅Π»ΡŒ. ΠžΠ½Β Π²Ρ‹ΠΏΠΈΡ…ΠΈΠ²Π°Π΅Ρ‚ тСбя из самолСта ΠΈΒ Π΅Ρ‰Π΅ ΠΌΠ΅Ρ‚Ρ€ΠΎΠ² 60 проносит ΠΏΠΎΒ Π²ΠΎΠ·Π΄ΡƒΡ…Ρƒ Π½Π°Β Ρ€Π΅Π°ΠΊΡ‚ΠΈΠ²Π½ΠΎΠΉ тягС. Π’Β ΠΎΠ±Ρ‰Π΅ΠΌ, Π½Π°Π±ΠΈΡ€Π°ΡŽ высоту с ускорСниСм Π²Β 22 g. Если ΡΡ€Π°Π²Π½ΠΈΡ‚ΡŒ с аттракционами, Ρ‚ΠΎΒ Ρ‚Π°ΠΌ максимальноС ускорСниС составляСт 2 g. Π§ΡƒΠ²ΡΡ‚Π²ΡƒΡŽ сСбя Ρ‚Π°ΠΊ, словно Π²ΠΎΒ ΠΌΠ½Π΅ ΠΏΠΎΠ»Ρ‚ΠΎΡ€Ρ‹ Ρ‚ΠΎΠ½Π½Ρ‹ вСсу. Ноги, спина ΠΈΒ Π·Π°Π΄Π½ΠΈΡ†Π° отяТСлСли. УскорСниС Π²ΠΆΠΈΠΌΠ°Π΅Ρ‚ мСня в спинку крСсла, пытаСтся ΡΠΎΠ³Π½ΡƒΡ‚ΡŒ Π²Β Π΄ΡƒΠ³ΡƒΒ β€” Π²Ρ‹Π½ΡƒΠΆΠ΄Π°Π΅Ρ‚ ΡƒΡ‚ΠΊΠ½ΡƒΡ‚ΡŒΡΡ Π»Π±ΠΎΠΌ Π²Β ΠΊΠΎΠ»Π΅Π½ΠΈ. Π’Π°ΠΊΠΎΠ΅ ΠΎΡ‰ΡƒΡ‰Π΅Π½ΠΈΠ΅, Π±ΡƒΠ΄Ρ‚ΠΎ на тСбя навалилась ΠΊΡƒΡ‡Π° Π²Ρ€Π°Π³ΠΎΠ²: Π±ΡŒΡŽΡ‚ ΠΊΡƒΠ»Π°ΠΊΠ°ΠΌΠΈ, тянут во всС стороны сразу. Π§ΡƒΠ²ΡΡ‚Π²ΡƒΡŽ, Ρ€Π΅ΠΌΠ΅ΡˆΠΎΠΊ шлСма врСзался Π²Β ΠΏΠΎΠ΄Π±ΠΎΡ€ΠΎΠ΄ΠΎΠΊ. Π¨Π»Π΅ΠΌ прямо срываСт с головы. КоТа Π½Π°Β ΠΏΠΎΠ΄Π±ΠΎΡ€ΠΎΠ΄ΠΊΠ΅ Π»ΠΎΠΏΠ½ΡƒΠ»Π° ΠΏΠΎΠ΄Β Π΄Π°Π²Π»Π΅Π½ΠΈΠ΅ΠΌ ΠΈΠ·Π½ΡƒΡ‚Ρ€ΠΈ. Я аТ застонал. Π˜Β Ρ‚ΡƒΡ‚, ΠΊΠ°ΠΊ и полагаСтся, крСсло подбрасываСт мСня Π²Β Π²ΠΎΠ·Π΄ΡƒΡ…. Π―Β Π²ΠΏΠ΅Ρ€Π²Ρ‹Π΅ ΠΎΡ‚ΠΊΡ€Ρ‹Π²Π°ΡŽ Π³Π»Π°Π·Π°. Π’ΠΈΠΆΡƒ: крСсло ΡƒΠ»Π΅Ρ‚Π°Π΅Ρ‚. АвтоматичСски раскрываСтся ΠΏΠ°Ρ€Π°ΡˆΡŽΡ‚. Π‘ΠΊΠΎΡ€ΠΎΡΡ‚ΡŒ ΠΏΠ°Π΄Π°Π΅Ρ‚. Π‘Ρ‚Ρ€ΠΎΠΏΡ‹ сильно Π΄Π΅Ρ€Π³Π°ΡŽΡ‚ мСня Π·Π°Β ΠΏΠ»Π΅Ρ‡ΠΈ. Π”Π°Π»ΡŒΡˆΠ΅ Π»Π΅Ρ‡Ρƒ ΠΏΠ»Π°Π²Π½ΠΎ. Π’Π΅ΡΡŒ процСсс — с момСнта, ΠΊΠΎΠ³Π΄Π° я дСрнул Π·Π°Β Ρ€ΡƒΡ‡ΠΊΠΈ, до раскрытия ΠΏΠ°Ρ€Π°ΡˆΡŽΡ‚Π°Β β€” продлился сСкунды Ρ‚Ρ€ΠΈ-Ρ‡Π΅Ρ‚Ρ‹Ρ€Π΅. ΠžΡ‚ΠΊΡ€Ρ‹Π² Π³Π»Π°Π·Π°, поискал Π³Π»Π°Π·Π°ΠΌΠΈ ΠšΡ€Π°ΡΡ‚ΠΈ: Π²ΠΎΠ½ он спускаСтся Π½Π°Β ΠΏΠ°Ρ€Π°ΡˆΡŽΡ‚Π΅. Π‘ΠΌΠΎΡ‚Ρ€ΡŽ Π²Β Π΄Ρ€ΡƒΠ³ΡƒΡŽ сторону — горящий самолСт втыкаСтся в зСмлю. Как Π½Π°Π·Π»ΠΎ, ΠΎΠΊΠΎΠ»ΠΎ СдинствСнного Π΄ΠΎΠΌΠ° в этой Π΄ΠΈΠΊΠΎΠΉ Π³ΠΎΡ€Π½ΠΎΠΉ мСстности. ΠšΒ ΡΡ‡Π°ΡΡ‚ΡŒΡŽ, Π½ΠΈΠΊΡ‚ΠΎ нС пострадал».

Π§Ρ‚ΠΎ Ρ‚Π°ΠΊΠΎΠ΅ ΠΏΠ°Ρ€Π°Π΄Π°Π½Ρ‚ΠΎΠ· ΠΈ ΠΊΠ°ΠΊΠΎΠ²Ρ‹ Π΅Π³ΠΎ послСдствия?

ΠŸΠ°Ρ€Π°Π΄Π°Π½Ρ‚ΠΎΠ· ΠΊΡ€Π°ΠΉΠ½Π΅ нСприятноС Π·Π°Π±ΠΎΠ»Π΅Π²Π°Π½ΠΈΠ΅, ΠΊΠΎΡ‚ΠΎΡ€ΠΎΠ΅ Π² ΠΏΠ΅Ρ€Π²ΡƒΡŽ ΠΎΡ‡Π΅Ρ€Π΅Π΄ΡŒ Π·Π°Ρ‚Ρ€Π°Π³ΠΈΠ²Π°Π΅Ρ‚ дСсна. По — Π΄Ρ€ΡƒΠ³ΠΎΠΌΡƒ, Π² ΠΏΡ€ΠΎΡΡ‚ΠΎΠ½Π°Ρ€ΠΎΠ΄ΡŒΠ΅, ΠΏΠ°Ρ€Π°Π΄Π°Π½Ρ‚ΠΎΠ· Π½Π°Π·Ρ‹Π²Π°ΡŽΡ‚ болСзнью дСсСн.

ΠŸΠ΅Ρ€Π²Ρ‹ΠΌ ΠΏΡ€ΠΈΠ·Π½Π°ΠΊΠΎΠΌ возникновСния Ρ‚Π°ΠΊΠΎΠΉ Π±ΠΎΠ»Π΅Π·Π½ΠΈ ΠΌΠΎΠΆΠ΅Ρ‚ ΡΠ»ΡƒΠΆΠΈΡ‚ΡŒ ΠΎΠ±ΠΈΠ»ΡŒΠ½Ρ‹ΠΉ ΠΊΡ€ΠΎΠ²ΠΎΡ‚ΠΎΠΊ дСсСн ΠΏΡ€ΠΈ ΡƒΡ‚Ρ€Π΅Π½Π½Π΅ΠΉ чисткС Π·ΡƒΠ±ΠΎΠ². Π­Ρ‚ΠΎ ΠΎΠ·Π½Π°Ρ‡Π°Π΅Ρ‚, Ρ‡Ρ‚ΠΎ дСсна стали ΠΎΡΠ»Π°Π±Π΅Π²Π°Ρ‚ΡŒ ΠΈ ΠΈΠΌ Π½Π΅ΠΎΠ±Ρ…ΠΎΠ΄ΠΈΠΌΠΎ комплСксноС ΠΈ профилактичСскоС Π»Π΅Ρ‡Π΅Π½ΠΈΠ΅, ΠΊΠΎΡ‚ΠΎΡ€ΠΎΠ΅ Π΄ΠΎΠ»ΠΆΠ½ΠΎ Π½Π°Π·Π½Π°Ρ‡Π°Ρ‚ΡŒΡΡ ΠΊΠ°ΠΊ ΠΌΠΎΠΆΠ½ΠΎ Ρ€Π°Π½ΡŒΡˆΠ΅. Π’Π΅Π΄ΡŒ Ρ‡Π΅ΠΌ Ρ€Π°Π½ΡŒΡˆΠ΅ Π±ΡƒΠ΄Π΅Ρ‚ ΠΎΠ±Π½Π°Ρ€ΡƒΠΆΠ΅Π½ΠΎ Ρ‚Π°ΠΊΠΎΠ΅ Π·Π°Π±ΠΎΠ»Π΅Π²Π°Π½ΠΈΠ΅, Ρ‚Π΅ΠΌ большС становится Π²Π΅Ρ€ΠΎΡΡ‚Π½ΠΎΡΡ‚ΡŒ спасти дСсна ΠΈ привСсти ΠΈΡ… Π² Π½ΡƒΠΆΠ½ΠΎΠ΅ состояниС.

Π’Ρ‚ΠΎΡ€Ρ‹ΠΌ ΠΏΡ€ΠΈΠ·Π½Π°ΠΊΠΎΠΌ, довольно нСприятным для Ρ‡Π΅Π»ΠΎΠ²Π΅ΠΊΠ°, ΠΌΠΎΠΆΠ΅Ρ‚ ΡΠ»ΡƒΠΆΠΈΡ‚ΡŒ Π²ΠΎΠ·Π½ΠΈΠΊΠ½ΠΎΠ²Π΅Π½ΠΈΠ΅ спСцифичСского Π·Π°ΠΏΠ°Ρ…Π° ΠΈΠ· Ρ€ΠΎΡ‚ΠΎΠ²ΠΎΠΉ полости, Ρ‡Ρ‚ΠΎ Π²Ρ‹Π·Ρ‹Π²Π°Π΅Ρ‚ Ρƒ ΠΌΠ½ΠΎΠ³ΠΈΡ… людСй, ΡΡ‚Ρ€Π°Π΄Π°ΡŽΡ‰ΠΈΡ… Ρ‚Π°ΠΊΠΎΠΉ болСзнью ΠΊΡ€Π°ΠΉΠ½Π΅Π΅ нСудобство. ЗапущСнная Ρ„ΠΎΡ€ΠΌΠ° ΠΏΠ°Ρ€Π°Π΄Π°Π½Ρ‚ΠΎΠ·Π° способствуСт ΡƒΠΌΠ΅Π½ΡŒΡˆΠ΅Π½ΠΈΡŽ ΠΏΠ»ΠΎΡ‰Π°Π΄ΠΈ дСсСн, Ρ‡Ρ‚ΠΎ Π²Π΅Π΄Π΅Ρ‚ ΠΊ ΠΏΠΎΡ‚Π΅Ρ€Π΅ Π·ΡƒΠ±ΠΎΠ². Но, стоит Ρ€Π°ΡΡΠΌΠΎΡ‚Ρ€Π΅Ρ‚ΡŒ ΠΏΡ€ΠΈΡ‡ΠΈΠ½Ρ‹, Π²Π΅Π΄ΡƒΡ‰ΠΈΠ΅ ΠΊ заболСванию дСсСн.

Π’ ΠΏΠ΅Ρ€Π²ΡƒΡŽ ΠΎΡ‡Π΅Ρ€Π΅Π΄ΡŒ, риск возникновСния Ρ‚Π°ΠΊΠΎΠ³ΠΎ заболСвания ΠΊΠ°ΠΊ ΠΏΠ°Ρ€Π°Π΄Π°Π½Ρ‚ΠΎΠ· Π²Ρ‹Π·Ρ‹Π²Π°Π΅Ρ‚ ΠΊΡƒΡ€Π΅Π½ΠΈΠ΅ ΠΈ Ρ‡Ρ€Π΅Π·ΠΌΠ΅Ρ€Π½ΠΎΠ΅ ΡƒΠΏΠΎΡ‚Ρ€Π΅Π±Π»Π΅Π½ΠΈΠ΅ ΠΊΠΎΡ„Π΅ ΠΈ чая. ΠšΡƒΡ€Π΅Π½ΠΈΠ΅ суТаСт малСнькиС сосуды, располоТСнныС Π² дСснах, Π² связи с Ρ‡Π΅ΠΌ, ΠΊΡ€ΠΎΠ²ΠΎΠΎΠ±Ρ€Π°Ρ‰Π΅Π½ΠΈΠ΅ замСдляСтся, ΠΈΠ»ΠΈ вовсС пСрСстаСт Ρ„ΡƒΠ½ΠΊΡ†ΠΈΠΎΠ½ΠΈΡ€ΠΎΠ²Π°Ρ‚ΡŒ. Π­Ρ‚ΠΎ Π½Π΅ ΠΌΠΎΠΆΠ΅Ρ‚ Π±Ρ‹Ρ‚ΡŒ Π½Π΅Π·Π°ΠΌΠ΅Ρ‡Π΅Π½Π½Ρ‹ΠΌ ΠΈΠ»ΠΈ Π½Π΅ нСсти Π½ΠΈΠΊΠ°ΠΊΠΈΡ… послСдствий. Как ΠΏΡ€Π°Π²ΠΈΠ»ΠΎ, происходит Ρ€Π°Π·Ρ€ΡƒΡˆΠ΅Π½ΠΈΠ΅ дСсСн, Π° слСдом начинаСтся Π²Ρ‹ΠΏΠ°Π΄Π΅Π½ΠΈΠ΅ Π·ΡƒΠ±ΠΎΠ².

Π§Ρ‚ΠΎΠ±Ρ‹ ΠΏΡ€Π΅Π΄ΠΎΡ‚Π²Ρ€Π°Ρ‚ΠΈΡ‚ΡŒ ΡΡ‚ΠΎΠ»ΡŒ Π½Π΅ΠΆΠ΅Π»Π°Ρ‚Π΅Π»ΡŒΠ½ΡƒΡŽ болСзнь, стоит ΠΊΠ°ΠΊ ΠΌΠΎΠΆΠ½ΠΎ Ρ‡Π°Ρ‰Π΅ ΠΏΠΎΡΠ΅Ρ‰Π°Ρ‚ΡŒ стоматолога ΠΊΠ»ΠΈΠ½ΠΈΠΊΠΈ Банация, Ρ‡ΠΈΡΡ‚ΠΈΡ‚ΡŒ Π·ΡƒΠ±Ρ‹ Π΄Π²Π° Ρ€Π°Π·Π° Π² дСнь – ΡƒΡ‚Ρ€ΠΎΠΌ ΠΈ Π²Π΅Ρ‡Π΅Ρ€ΠΎΠΌ, ΠΎΠ±ΡΠ·Π°Ρ‚Π΅Π»ΡŒΠ½ΠΎ ΠΈΡΠΏΠΎΠ»ΡŒΠ·ΠΎΠ²Π°Ρ‚ΡŒ срСдства для полоскания, Π² состав ΠΊΠΎΡ‚ΠΎΡ€ΠΎΠ³ΠΎ Π΄ΠΎΠ»ΠΆΠ΅Π½ Π²Ρ…ΠΎΠ΄ΠΈΡ‚ хлоргСксидин. ΠšΡ€ΠΎΠΌΠ΅ Ρ‚ΠΎΠ³ΠΎ, стоматологи ΠΊΠ»ΠΈΠ½ΠΈΠΊ Банация Ρ€Π΅ΠΊΠΎΠΌΠ΅Π½Π΄ΡƒΡŽΡ‚ ΠΈΡΠΏΠΎΠ»ΡŒΠ·ΠΎΠ²Π°Ρ‚ΡŒ Π·ΡƒΠ±Π½ΡƒΡŽ Π½ΠΈΡ‚ΡŒ послС ΠΊΠ°ΠΆΠ΄ΠΎΠ³ΠΎ ΠΏΡ€ΠΈΠ΅ΠΌΠ° ΠΏΠΈΡ‰ΠΈ. Π’Π°ΠΊΠΈΠΌ ΠΎΠ±Ρ€Π°Π·ΠΎΠΌ, Π½Π΅ΠΎΠ±Ρ…ΠΎΠ΄ΠΈΠΌΠΎ ΠΈΠ·Π±Π°Π²Π»ΡΡ‚ΡŒΡΡ ΠΎΡ‚ остатков ΠΏΠΈΡ‰ΠΈ, ΠΊΠΎΡ‚ΠΎΡ€Ρ‹Π΅ Π² свою ΠΎΡ‡Π΅Ρ€Π΅Π΄ΡŒ ΠΌΠΎΠ³ΡƒΡ‚ ΠΏΠΎΠ²Π»ΠΈΡΡ‚ΡŒ Π½Π° ускорСниС процСсс гниСния Π·ΡƒΠ±ΠΎΠ², Ссли Π² Π½ΠΈΡ… Π΅ΡΡ‚ΡŒ Π΄Ρ‹Ρ€ΠΊΠΈ.

Π—ΡƒΠ±Ρ‹ ΠΈ дСсны Π½ΡƒΠΆΠ΄Π°ΡŽΡ‚ΡΡ Π² постоянном ΠΊΠΎΠ½Ρ‚Ρ€ΠΎΠ»Π΅ ΠΈ ΡƒΡ…ΠΎΠ΄Π΅, Ρ‡Ρ‚ΠΎΠ±Ρ‹ ΠΈΠ·Π±Π΅ΠΆΠ°Ρ‚ΡŒ Ρ‚Π°ΠΊΠΈΡ… нСприятных Π±ΠΎΠ»Π΅Π·Π½Π΅ΠΉ, ΠΊΠ°ΠΊ ΠΏΠ°Ρ€Π°Π΄Π°Π½Ρ‚ΠΎΠ·, кариСс ΠΈ Π²ΠΎΠ·Π½ΠΈΠΊΠ½ΠΎΠ²Π΅Π½ΠΈΠ΅ Π·ΡƒΠ±Π½ΠΎΠ³ΠΎ камня. Π“Π»Π°Π²Π½ΠΎΠ΅ Π½Π΅ Π·Π°Ρ‚ΡΠ³ΠΈΠ²Π°Ρ‚ΡŒ Π²ΠΈΠ·ΠΈΡ‚ ΠΊ Π²Ρ€Π°Ρ‡Ρƒ, Ссли ощущаСтся излишниС количСство Π½Π°Π»Π΅Ρ‚Π°, Π·Π°ΠΏΠ°Ρ… ΠΈΠ·ΠΎ Ρ€Ρ‚Π° ΠΈΠ»ΠΈ ΡƒΠΌΠ΅Π½ΡŒΡˆΠ΅Π½ΠΈΠ΅ ΠΏΠ»ΠΎΡ‰Π°Π΄ΠΈ дСсСн. Π’Π΅Π΄ΡŒ здоровая ΡƒΠ»Ρ‹Π±ΠΊΠ° – Π·Π°Π»ΠΎΠ³ успСха Π²ΠΎ ΠΌΠ½ΠΎΠ³ΠΈΡ… повсСднСвных Π΄Π΅Π»Π°Ρ….

ПолСзная информация

ВСрапСвтичСская стоматология

Π Π°Π·Π³ΠΎΠ½

ПослСдняя матСматичСская Π²Π΅Π»ΠΈΡ‡ΠΈΠ½Π°, обсуТдаСмая Π² Π£Ρ€ΠΎΠΊΠ΅ 1, — это ускорСниС. Часто ΠΏΡƒΡ‚Π°ΡŽΡ‚, Ρ‡Ρ‚ΠΎ ускорСниС ΠΈΠΌΠ΅Π΅Ρ‚ Π·Π½Π°Ρ‡Π΅Π½ΠΈΠ΅, сильно ΠΎΡ‚Π»ΠΈΡ‡Π°ΡŽΡ‰Π΅Π΅ΡΡ ΠΎΡ‚ значСния, ΠΊΠΎΡ‚ΠΎΡ€ΠΎΠ΅ ассоциируСтся с Π½ΠΈΠΌ спортивными ΠΊΠΎΠΌΠΌΠ΅Π½Ρ‚Π°Ρ‚ΠΎΡ€Π°ΠΌΠΈ ΠΈ Π΄Ρ€ΡƒΠ³ΠΈΠΌΠΈ людьми. ΠžΠΏΡ€Π΅Π΄Π΅Π»Π΅Π½ΠΈΠ΅ ускорСния:

  • УскорСниС — это вСкторная Π²Π΅Π»ΠΈΡ‡ΠΈΠ½Π°, которая опрСдСляСтся ΠΊΠ°ΠΊ ΡΠΊΠΎΡ€ΠΎΡΡ‚ΡŒ, с ΠΊΠΎΡ‚ΠΎΡ€ΠΎΠΉ ΠΎΠ±ΡŠΠ΅ΠΊΡ‚ измСняСт свою ΡΠΊΠΎΡ€ΠΎΡΡ‚ΡŒ. ΠžΠ±ΡŠΠ΅ΠΊΡ‚ ускоряСтся, Ссли ΠΎΠ½ мСняСт свою ΡΠΊΠΎΡ€ΠΎΡΡ‚ΡŒ.

Π‘ΠΏΠΎΡ€Ρ‚ΠΈΠ²Π½Ρ‹Π΅ ΠΊΠΎΠΌΠΌΠ΅Π½Ρ‚Π°Ρ‚ΠΎΡ€Ρ‹ ΠΈΠ½ΠΎΠ³Π΄Π° говорят, Ρ‡Ρ‚ΠΎ Ρ‡Π΅Π»ΠΎΠ²Π΅ΠΊ ускоряСтся, Ссли ΠΎΠ½ / ΠΎΠ½Π° быстро двиТСтся.И всС ΠΆΠ΅ ускорСниС Π½Π΅ ΠΈΠΌΠ΅Π΅Ρ‚ Π½ΠΈΡ‡Π΅Π³ΠΎ ΠΎΠ±Ρ‰Π΅Π³ΠΎ с быстрым Π΄Π²ΠΈΠΆΠ΅Π½ΠΈΠ΅ΠΌ. Π§Π΅Π»ΠΎΠ²Π΅ΠΊ ΠΌΠΎΠΆΠ΅Ρ‚ Π΄Π²ΠΈΠ³Π°Ρ‚ΡŒΡΡ ΠΎΡ‡Π΅Π½ΡŒ быстро, Π½ΠΎ ΠΏΡ€ΠΈ этом Π½Π΅ ΡƒΡΠΊΠΎΡ€ΡΡ‚ΡŒΡΡ. УскорСниС связано с ΠΈΠ·ΠΌΠ΅Π½Π΅Π½ΠΈΠ΅ΠΌ скорости двиТСния ΠΎΠ±ΡŠΠ΅ΠΊΡ‚Π°. Если ΠΎΠ±ΡŠΠ΅ΠΊΡ‚ Π½Π΅ мСняСт свою ΡΠΊΠΎΡ€ΠΎΡΡ‚ΡŒ, Π·Π½Π°Ρ‡ΠΈΡ‚, ΠΎΠ±ΡŠΠ΅ΠΊΡ‚ Π½Π΅ ускоряСтся. Π”Π°Π½Π½Ρ‹Π΅ справа ΠΏΡ€Π΅Π΄ΡΡ‚Π°Π²Π»ΡΡŽΡ‚ двиТущийся Π½Π° сСвСр ΡƒΡΠΊΠΎΡ€ΡΡŽΡ‰ΠΈΠΉΡΡ ΠΎΠ±ΡŠΠ΅ΠΊΡ‚. Π‘ΠΊΠΎΡ€ΠΎΡΡ‚ΡŒ мСняСтся с Ρ‚Π΅Ρ‡Π΅Π½ΠΈΠ΅ΠΌ Π²Ρ€Π΅ΠΌΠ΅Π½ΠΈ. ЀактичСски, ΡΠΊΠΎΡ€ΠΎΡΡ‚ΡŒ измСняСтся Π½Π° ΠΏΠΎΡΡ‚ΠΎΡΠ½Π½ΡƒΡŽ Π²Π΅Π»ΠΈΡ‡ΠΈΠ½Ρƒ — 10 ΠΌ / с — ΠΊΠ°ΠΆΠ΄ΡƒΡŽ сСкунду. ΠšΠ°ΠΆΠ΄Ρ‹ΠΉ Ρ€Π°Π·, ΠΊΠΎΠ³Π΄Π° ΡΠΊΠΎΡ€ΠΎΡΡ‚ΡŒ ΠΎΠ±ΡŠΠ΅ΠΊΡ‚Π° измСняСтся, ΠΎΠ±ΡŠΠ΅ΠΊΡ‚ считаСтся ΡƒΡΠΊΠΎΡ€ΡΡŽΡ‰ΠΈΠΌΡΡ; Ρƒ Π½Π΅Π³ΠΎ Π΅ΡΡ‚ΡŒ ускорСниС.


Π—Π½Π°Ρ‡Π΅Π½ΠΈΠ΅ постоянного ускорСния

Иногда ΡƒΡΠΊΠΎΡ€ΡΡŽΡ‰ΠΈΠΉΡΡ ΠΎΠ±ΡŠΠ΅ΠΊΡ‚ мСняСт свою ΡΠΊΠΎΡ€ΠΎΡΡ‚ΡŒ Π½Π° ΠΎΠ΄Π½Ρƒ ΠΈ Ρ‚Ρƒ ΠΆΠ΅ Π²Π΅Π»ΠΈΡ‡ΠΈΠ½Ρƒ ΠΊΠ°ΠΆΠ΄ΡƒΡŽ сСкунду. Как ΡƒΠΏΠΎΠΌΠΈΠ½Π°Π»ΠΎΡΡŒ Π² ΠΏΡ€Π΅Π΄Ρ‹Π΄ΡƒΡ‰Π΅ΠΌ Π°Π±Π·Π°Ρ†Π΅, привСдСнная Π²Ρ‹ΡˆΠ΅ Ρ‚Π°Π±Π»ΠΈΡ†Π° Π΄Π°Π½Π½Ρ‹Ρ… ΠΏΠΎΠΊΠ°Π·Ρ‹Π²Π°Π΅Ρ‚, Ρ‡Ρ‚ΠΎ ΠΎΠ±ΡŠΠ΅ΠΊΡ‚ мСняСт свою ΡΠΊΠΎΡ€ΠΎΡΡ‚ΡŒ Π½Π° 10 ΠΌ / с ΠΊΠ°ΠΆΠ΄ΡƒΡŽ ΠΏΠΎΡΠ»Π΅Π΄ΡƒΡŽΡ‰ΡƒΡŽ сСкунду. Π­Ρ‚ΠΎ называСтся постоянным ускорСниСм, ΠΏΠΎΡΠΊΠΎΠ»ΡŒΠΊΡƒ ΡΠΊΠΎΡ€ΠΎΡΡ‚ΡŒ измСняСтся Π½Π° ΠΏΠΎΡΡ‚ΠΎΡΠ½Π½ΡƒΡŽ Π²Π΅Π»ΠΈΡ‡ΠΈΠ½Ρƒ ΠΊΠ°ΠΆΠ΄ΡƒΡŽ сСкунду. НС слСдуСт ΠΏΡƒΡ‚Π°Ρ‚ΡŒ ΠΎΠ±ΡŠΠ΅ΠΊΡ‚ с постоянным ускорСниСм с ΠΎΠ±ΡŠΠ΅ΠΊΡ‚ΠΎΠΌ с постоянной ΡΠΊΠΎΡ€ΠΎΡΡ‚ΡŒΡŽ.НС Π΄Π°ΠΉΡ‚Π΅ сСбя ΠΎΠ±ΠΌΠ°Π½ΡƒΡ‚ΡŒ! Если ΠΎΠ±ΡŠΠ΅ΠΊΡ‚ мСняСт свою ΡΠΊΠΎΡ€ΠΎΡΡ‚ΡŒ — Π½Π° ΠΏΠΎΡΡ‚ΠΎΡΠ½Π½ΡƒΡŽ ΠΈΠ»ΠΈ ΠΏΠ΅Ρ€Π΅ΠΌΠ΅Π½Π½ΡƒΡŽ Π²Π΅Π»ΠΈΡ‡ΠΈΠ½Ρƒ — Ρ‚ΠΎ это ΡƒΡΠΊΠΎΡ€ΡΡŽΡ‰ΠΈΠΉΡΡ ΠΎΠ±ΡŠΠ΅ΠΊΡ‚. И ΠΎΠ±ΡŠΠ΅ΠΊΡ‚ с постоянной ΡΠΊΠΎΡ€ΠΎΡΡ‚ΡŒΡŽ Π½Π΅ ускоряСтся. ΠŸΡ€ΠΈΠ²Π΅Π΄Π΅Π½Π½Ρ‹Π΅ Π½ΠΈΠΆΠ΅ Ρ‚Π°Π±Π»ΠΈΡ†Ρ‹ Π΄Π°Π½Π½Ρ‹Ρ… ΠΎΡ‚ΠΎΠ±Ρ€Π°ΠΆΠ°ΡŽΡ‚ двиТСния ΠΎΠ±ΡŠΠ΅ΠΊΡ‚ΠΎΠ² с постоянным ускорСниСм ΠΈ ΠΈΠ·ΠΌΠ΅Π½ΡΡŽΡ‰ΠΈΠΌΡΡ ускорСниСм. ΠžΠ±Ρ€Π°Ρ‚ΠΈΡ‚Π΅ Π²Π½ΠΈΠΌΠ°Π½ΠΈΠ΅, Ρ‡Ρ‚ΠΎ ΠΊΠ°ΠΆΠ΄Ρ‹ΠΉ ΠΎΠ±ΡŠΠ΅ΠΊΡ‚ ΠΈΠΌΠ΅Π΅Ρ‚ ΠΈΠ·ΠΌΠ΅Π½ΡΡŽΡ‰ΡƒΡŽΡΡ ΡΠΊΠΎΡ€ΠΎΡΡ‚ΡŒ.

ΠŸΠΎΡΠΊΠΎΠ»ΡŒΠΊΡƒ ΡƒΡΠΊΠΎΡ€ΡΡŽΡ‰ΠΈΠ΅ΡΡ ΠΎΠ±ΡŠΠ΅ΠΊΡ‚Ρ‹ постоянно ΠΌΠ΅Π½ΡΡŽΡ‚ свою ΡΠΊΠΎΡ€ΠΎΡΡ‚ΡŒ, ΠΌΠΎΠΆΠ½ΠΎ ΡΠΊΠ°Π·Π°Ρ‚ΡŒ, Ρ‡Ρ‚ΠΎ ΠΏΡ€ΠΎΠΉΠ΄Π΅Π½Π½ΠΎΠ΅ расстояниС / врСмя Π½Π΅ являСтся постоянной Π²Π΅Π»ΠΈΡ‡ΠΈΠ½ΠΎΠΉ.НапримСр, ΠΏΠ°Π΄Π°ΡŽΡ‰ΠΈΠΉ ΠΎΠ±ΡŠΠ΅ΠΊΡ‚ ΠΎΠ±Ρ‹Ρ‡Π½ΠΎ ускоряСтся ΠΏΡ€ΠΈ ΠΏΠ°Π΄Π΅Π½ΠΈΠΈ. Если Π±Ρ‹ ΠΌΡ‹ наблюдали Π΄Π²ΠΈΠΆΠ΅Π½ΠΈΠ΅ свободно ΠΏΠ°Π΄Π°ΡŽΡ‰Π΅Π³ΠΎ ΠΎΠ±ΡŠΠ΅ΠΊΡ‚Π° (Π΄Π²ΠΈΠΆΠ΅Π½ΠΈΠ΅ свободного падСния Π±ΡƒΠ΄Π΅Ρ‚ ΠΏΠΎΠ΄Ρ€ΠΎΠ±Π½ΠΎ обсуТдСно ΠΏΠΎΠ·ΠΆΠ΅), ΠΌΡ‹ Π±Ρ‹ Π·Π°ΠΌΠ΅Ρ‚ΠΈΠ»ΠΈ, Ρ‡Ρ‚ΠΎ ΠΎΠ±ΡŠΠ΅ΠΊΡ‚ ΠΈΠΌΠ΅Π΅Ρ‚ ΡΡ€Π΅Π΄Π½ΡŽΡŽ ΡΠΊΠΎΡ€ΠΎΡΡ‚ΡŒ ΠΏΡ€ΠΈΠΌΠ΅Ρ€Π½ΠΎ 5 ΠΌ / с Π² ΠΏΠ΅Ρ€Π²ΡƒΡŽ сСкунду, ΠΏΡ€ΠΈΠΌΠ΅Ρ€Π½ΠΎ 15 ΠΌ / с. Π²ΠΎ Π²Ρ‚ΠΎΡ€ΠΎΠΉ сСкундС ΠΏΡ€ΠΈΠΌΠ΅Ρ€Π½ΠΎ 25 ΠΌ / с Π² Ρ‚Ρ€Π΅Ρ‚ΡŒΡŽ сСкунду, ΠΏΡ€ΠΈΠΌΠ΅Ρ€Π½ΠΎ 35 ΠΌ / с Π² Ρ‡Π΅Ρ‚Π²Π΅Ρ€Ρ‚ΡƒΡŽ сСкунду ΠΈ Ρ‚. Π΄. Наш свободно ΠΏΠ°Π΄Π°ΡŽΡ‰ΠΈΠΉ ΠΎΠ±ΡŠΠ΅ΠΊΡ‚ Π±ΡƒΠ΄Π΅Ρ‚ постоянно ΡƒΡΠΊΠΎΡ€ΡΡ‚ΡŒΡΡ. Учитывая эти срСдниС значСния скорости Π² Ρ‚Π΅Ρ‡Π΅Π½ΠΈΠ΅ ΠΊΠ°ΠΆΠ΄ΠΎΠ³ΠΎ ΠΏΠΎΡΠ»Π΅Π΄ΠΎΠ²Π°Ρ‚Π΅Π»ΡŒΠ½ΠΎΠ³ΠΎ Π²Ρ€Π΅ΠΌΠ΅Π½Π½ΠΎΠ³ΠΎ ΠΈΠ½Ρ‚Π΅Ρ€Π²Π°Π»Π° Π² 1 сСкунду, ΠΌΡ‹ ΠΌΠΎΠ³Π»ΠΈ Π±Ρ‹ ΡΠΊΠ°Π·Π°Ρ‚ΡŒ, Ρ‡Ρ‚ΠΎ ΠΎΠ±ΡŠΠ΅ΠΊΡ‚ ΡƒΠΏΠ°Π΄Π΅Ρ‚ Π½Π° 5 ΠΌΠ΅Ρ‚Ρ€ΠΎΠ² Π² ΠΏΠ΅Ρ€Π²ΡƒΡŽ сСкунду, 15 ΠΌΠ΅Ρ‚Ρ€ΠΎΠ² Π²ΠΎ Π²Ρ‚ΠΎΡ€ΡƒΡŽ сСкунду (для ΠΎΠ±Ρ‰Π΅Π³ΠΎ расстояния 20 ΠΌΠ΅Ρ‚Ρ€ΠΎΠ²), 25 ΠΌΠ΅Ρ‚Ρ€ΠΎΠ² Π² Ρ‚Ρ€Π΅Ρ‚ΡŒΡŽ. Π²Ρ‚ΠΎΡ€ΠΎΠΉ (для ΠΎΠ±Ρ‰Π΅ΠΉ дистанции 45 ΠΌΠ΅Ρ‚Ρ€ΠΎΠ²), 35 ΠΌΠ΅Ρ‚Ρ€ΠΎΠ² Π² Ρ‡Π΅Ρ‚Π²Π΅Ρ€Ρ‚ΠΎΠΉ сСкундС (для ΠΎΠ±Ρ‰Π΅ΠΉ дистанции 80 ΠΌΠ΅Ρ‚Ρ€ΠΎΠ² Ρ‡Π΅Ρ€Π΅Π· Ρ‡Π΅Ρ‚Ρ‹Ρ€Π΅ сСкунды).Π­Ρ‚ΠΈ числа ΠΏΡ€ΠΈΠ²Π΅Π΄Π΅Π½Ρ‹ Π² Ρ‚Π°Π±Π»ΠΈΡ†Π΅ Π½ΠΈΠΆΠ΅.

ВрСмя
Π˜Π½Ρ‚Π΅Ρ€Π²Π°Π»
ИзмСнСниС скорости
Π’ Ρ‚Π΅Ρ‡Π΅Π½ΠΈΠ΅ ΠΈΠ½Ρ‚Π΅Ρ€Π²Π°Π»Π°
ΠΏΡ€. Π‘ΠΊΠΎΡ€ΠΎΡΡ‚ΡŒ
Π’ Ρ‚Π΅Ρ‡Π΅Π½ΠΈΠ΅ ΠΈΠ½Ρ‚Π΅Ρ€Π²Π°Π»Π°
ΠŸΡ€ΠΎΠΉΠ΄Π΅Π½Π½ΠΎΠ΅ расстояниС
Π’ Ρ‚Π΅Ρ‡Π΅Π½ΠΈΠ΅ ΠΈΠ½Ρ‚Π΅Ρ€Π²Π°Π»Π°
ΠžΠ±Ρ‰Π΅Π΅ ΠΏΡ€ΠΎΠΉΠ΄Π΅Π½Π½ΠΎΠ΅ расстояниС с
ΠžΡ‚ 0 с Π΄ΠΎ ΠΊΠΎΠ½Ρ†Π° ΠΈΠ½Ρ‚Π΅Ρ€Π²Π°Π»Π°
0 — 1.0 с ΠΎΡ‚ 0 Π΄ΠΎ ~ 10 ΠΌ / с ~ 5 ΠΌ / с ~ 5 ΠΌ ~ 5 ΠΌ
1,0 — 2,0 с ~ 10-20 ΠΌ / с ~ 15 ΠΌ / с ~ 15 ΠΌ ~ 20 ΠΌ
2,0 — 3,0 с ~ ΠΎΡ‚ 20 Π΄ΠΎ 30 ΠΌ / с ~ 25 ΠΌ / с ~ 25 ΠΌ ~ 45 ΠΌ
3,0 — 4.0 с ~ 30-40 ΠΌ / с ~ 35 ΠΌ / с ~ 35 ΠΌ ~ 80 ΠΌ

ΠŸΡ€ΠΈΠΌΠ΅Ρ‡Π°Π½ΠΈΠ΅. Π˜ΡΠΏΠΎΠ»ΡŒΠ·ΡƒΠ΅ΠΌΡ‹ΠΉ здСсь символ ~ ΠΎΠ·Π½Π°Ρ‡Π°Π΅Ρ‚ ΠΏΡ€ΠΈΠ±Π»ΠΈΠ·ΠΈΡ‚Π΅Π»ΡŒΠ½ΠΎ.

Π­Ρ‚ΠΎ обсуТдСниС ΠΏΠΎΠΊΠ°Π·Ρ‹Π²Π°Π΅Ρ‚, Ρ‡Ρ‚ΠΎ свободно ΠΏΠ°Π΄Π°ΡŽΡ‰ΠΈΠΉ ΠΎΠ±ΡŠΠ΅ΠΊΡ‚, ΠΊΠΎΡ‚ΠΎΡ€Ρ‹ΠΉ ускоряСтся с постоянной ΡΠΊΠΎΡ€ΠΎΡΡ‚ΡŒΡŽ, Π±ΡƒΠ΄Π΅Ρ‚ ΠΏΡ€Π΅ΠΎΠ΄ΠΎΠ»Π΅Π²Π°Ρ‚ΡŒ Ρ€Π°Π·Π½Ρ‹Π΅ расстояния Π·Π° ΠΊΠ°ΠΆΠ΄ΡƒΡŽ ΠΏΠΎΡΠ»Π΅Π΄ΡƒΡŽΡ‰ΡƒΡŽ сСкунду. Π”Π°Π»ΡŒΠ½Π΅ΠΉΡˆΠΈΠΉ Π°Π½Π°Π»ΠΈΠ· ΠΏΠ΅Ρ€Π²ΠΎΠ³ΠΎ ΠΈ послСднСго столбцов ΠΏΡ€ΠΈΠ²Π΅Π΄Π΅Π½Π½Ρ‹Ρ… Π²Ρ‹ΡˆΠ΅ Π΄Π°Π½Π½Ρ‹Ρ… ΠΏΠΎΠΊΠ°Π·Ρ‹Π²Π°Π΅Ρ‚, Ρ‡Ρ‚ΠΎ сущСствуСт квадратная Π·Π°Π²ΠΈΡΠΈΠΌΠΎΡΡ‚ΡŒ ΠΌΠ΅ΠΆΠ΄Ρƒ ΠΎΠ±Ρ‰ΠΈΠΌ ΠΏΡ€ΠΎΠΉΠ΄Π΅Π½Π½Ρ‹ΠΌ расстояниСм ΠΈ Π²Ρ€Π΅ΠΌΠ΅Π½Π΅ΠΌ ΠΏΡƒΡ‚Π΅ΡˆΠ΅ΡΡ‚Π²ΠΈΡ для ΠΎΠ±ΡŠΠ΅ΠΊΡ‚Π°, Π½Π°Ρ‡ΠΈΠ½Π°ΡŽΡ‰Π΅Π³ΠΎΡΡ ΠΈΠ· состояния покоя ΠΈ двиТущСгося с постоянным ускорСниСм.2) расстояниС; расстояниС, ΠΏΡ€ΠΎΠΉΠ΄Π΅Π½Π½ΠΎΠ΅ Π·Π° Ρ‡Π΅Ρ‚Ρ‹Ρ€Π΅ сСкунды, Π² 16 Ρ€Π°Π· ΠΏΡ€Π΅Π²Ρ‹ΡˆΠ°Π΅Ρ‚ расстояниС, ΠΏΡ€ΠΎΠΉΠ΄Π΅Π½Π½ΠΎΠ΅ Π·Π° ΠΎΠ΄Π½Ρƒ сСкунду. Для ΠΎΠ±ΡŠΠ΅ΠΊΡ‚ΠΎΠ² с постоянным ускорСниСм расстояниС пСрСмСщСния прямо ΠΏΡ€ΠΎΠΏΠΎΡ€Ρ†ΠΈΠΎΠ½Π°Π»ΡŒΠ½ΠΎ ΠΊΠ²Π°Π΄Ρ€Π°Ρ‚Ρƒ Π²Ρ€Π΅ΠΌΠ΅Π½ΠΈ пСрСмСщСния.

РасчСт срСднСго ускорСния

Π‘Ρ€Π΅Π΄Π½Π΅Π΅ ускорСниС (a) любого ΠΎΠ±ΡŠΠ΅ΠΊΡ‚Π° Π·Π° Π·Π°Π΄Π°Π½Π½Ρ‹ΠΉ ΠΈΠ½Ρ‚Π΅Ρ€Π²Π°Π» Π²Ρ€Π΅ΠΌΠ΅Π½ΠΈ (t) ΠΌΠΎΠΆΠ½ΠΎ Ρ€Π°ΡΡΡ‡ΠΈΡ‚Π°Ρ‚ΡŒ с ΠΏΠΎΠΌΠΎΡ‰ΡŒΡŽ уравнСния

Π­Ρ‚ΠΎ ΡƒΡ€Π°Π²Π½Π΅Π½ΠΈΠ΅ ΠΌΠΎΠΆΠ½ΠΎ ΠΈΡΠΏΠΎΠ»ΡŒΠ·ΠΎΠ²Π°Ρ‚ΡŒ для расчСта ускорСния ΠΎΠ±ΡŠΠ΅ΠΊΡ‚Π°, Π΄Π²ΠΈΠΆΠ΅Π½ΠΈΠ΅ ΠΊΠΎΡ‚ΠΎΡ€ΠΎΠ³ΠΎ прСдставлСно ΠΏΡ€ΠΈΠ²Π΅Π΄Π΅Π½Π½ΠΎΠΉ Π²Ρ‹ΡˆΠ΅ Ρ‚Π°Π±Π»ΠΈΡ†Π΅ΠΉ Π΄Π°Π½Π½Ρ‹Ρ… ΡΠΊΠΎΡ€ΠΎΡΡ‚ΡŒ-врСмя.Π”Π°Π½Π½Ρ‹Π΅ скорости-Π²Ρ€Π΅ΠΌΠ΅Π½ΠΈ Π² Ρ‚Π°Π±Π»ΠΈΡ†Π΅ ΠΏΠΎΠΊΠ°Π·Ρ‹Π²Π°ΡŽΡ‚, Ρ‡Ρ‚ΠΎ ΠΎΠ±ΡŠΠ΅ΠΊΡ‚ ΠΈΠΌΠ΅Π΅Ρ‚ ускорСниС 10 ΠΌ / с / с. РасчСт ΠΏΠΎΠΊΠ°Π·Π°Π½ Π½ΠΈΠΆΠ΅.

ЗначСния ускорСния Π²Ρ‹Ρ€Π°ΠΆΠ°ΡŽΡ‚ΡΡ Π² Π΅Π΄ΠΈΠ½ΠΈΡ†Π°Ρ… скорости / Π²Ρ€Π΅ΠΌΠ΅Π½ΠΈ. Π’ΠΈΠΏΠΈΡ‡Π½Ρ‹Π΅ устройства ускорСния Π²ΠΊΠ»ΡŽΡ‡Π°ΡŽΡ‚ Π² сСбя ΡΠ»Π΅Π΄ΡƒΡŽΡ‰ΠΈΠ΅:

м / с / с
миль / час / с
ΠΊΠΌ / час / с
м / с 2

Π­Ρ‚ΠΈ устройства ΠΌΠΎΠ³ΡƒΡ‚ ΠΏΠΎΠΊΠ°Π·Π°Ρ‚ΡŒΡΡ Π½Π΅ΠΌΠ½ΠΎΠ³ΠΎ Π½Π΅ΡƒΠ΄ΠΎΠ±Π½Ρ‹ΠΌΠΈ для Π½Π°Ρ‡ΠΈΠ½Π°ΡŽΡ‰Π΅Π³ΠΎ студСнта-Ρ„ΠΈΠ·ΠΈΠΊΠ°. Π’Π΅ΠΌ Π½Π΅ ΠΌΠ΅Π½Π΅Π΅, это ΠΎΡ‡Π΅Π½ΡŒ Ρ€Π°Π·ΡƒΠΌΠ½Ρ‹Π΅ Π΅Π΄ΠΈΠ½ΠΈΡ†Ρ‹, ΠΊΠΎΠ³Π΄Π° Π²Ρ‹ Π½Π°Ρ‡ΠΈΠ½Π°Π΅Ρ‚Π΅ Ρ€Π°ΡΡΠΌΠ°Ρ‚Ρ€ΠΈΠ²Π°Ρ‚ΡŒ ΠΎΠΏΡ€Π΅Π΄Π΅Π»Π΅Π½ΠΈΠ΅ ΠΈ ΡƒΡ€Π°Π²Π½Π΅Π½ΠΈΠ΅ ускорСния.ΠŸΡ€ΠΈΡ‡ΠΈΠ½Π° появлСния Π΅Π΄ΠΈΠ½ΠΈΡ† становится ΠΎΡ‡Π΅Π²ΠΈΠ΄Π½ΠΎΠΉ послС изучСния уравнСния ускорСния.

ΠŸΠΎΡΠΊΠΎΠ»ΡŒΠΊΡƒ ускорСниС — это ΠΈΠ·ΠΌΠ΅Π½Π΅Π½ΠΈΠ΅ скорости с Ρ‚Π΅Ρ‡Π΅Π½ΠΈΠ΅ΠΌ Π²Ρ€Π΅ΠΌΠ΅Π½ΠΈ, Π΅Π΄ΠΈΠ½ΠΈΡ†Ρ‹ измСрСния ускорСния — это Π΅Π΄ΠΈΠ½ΠΈΡ†Ρ‹ скорости, Π΄Π΅Π»Π΅Π½Π½Ρ‹Π΅ Π½Π° Π΅Π΄ΠΈΠ½ΠΈΡ†Ρ‹ Π²Ρ€Π΅ΠΌΠ΅Π½ΠΈ, Ρ‚ΠΎ Π΅ΡΡ‚ΡŒ (ΠΌ / с) / с ΠΈΠ»ΠΈ (миль / час) / с. Π•Π΄ΠΈΠ½ΠΈΡ†Ρƒ измСрСния (ΠΌ / с) / с ΠΌΠΎΠΆΠ½ΠΎ матСматичСски ΡƒΠΏΡ€ΠΎΡΡ‚ΠΈΡ‚ΡŒ Π΄ΠΎ ΠΌ / с 2 .

НаправлСниС Π²Π΅ΠΊΡ‚ΠΎΡ€Π° ускорСния

ΠŸΠΎΡΠΊΠΎΠ»ΡŒΠΊΡƒ ускорСниС являСтся Π²Π΅ΠΊΡ‚ΠΎΡ€Π½ΠΎΠΉ Π²Π΅Π»ΠΈΡ‡ΠΈΠ½ΠΎΠΉ, с Π½ΠΈΠΌ связано Π½Π°ΠΏΡ€Π°Π²Π»Π΅Π½ΠΈΠ΅.НаправлСниС Π²Π΅ΠΊΡ‚ΠΎΡ€Π° ускорСния зависит ΠΎΡ‚ Π΄Π²ΡƒΡ… Π²Π΅Ρ‰Π΅ΠΉ:

  • , ускоряСтся ΠΈΠ»ΠΈ замСдляСтся ΠΎΠ±ΡŠΠ΅ΠΊΡ‚
  • , двиТСтся Π»ΠΈ ΠΎΠ±ΡŠΠ΅ΠΊΡ‚ Π² + ΠΈΠ»ΠΈ — Π½Π°ΠΏΡ€Π°Π²Π»Π΅Π½ΠΈΠΈ

ΠžΠ±Ρ‰ΠΈΠΉ ΠΏΡ€ΠΈΠ½Ρ†ΠΈΠΏ опрСдСлСния ускорСния:

Если ΠΎΠ±ΡŠΠ΅ΠΊΡ‚ замСдляСтся, Ρ‚ΠΎ Π΅Π³ΠΎ ускорСниС происходит Π² Π½Π°ΠΏΡ€Π°Π²Π»Π΅Π½ΠΈΠΈ, ΠΏΡ€ΠΎΡ‚ΠΈΠ²ΠΎΠΏΠΎΠ»ΠΎΠΆΠ½ΠΎΠΌ Π΅Π³ΠΎ двиТСнию.

Π­Ρ‚ΠΎΡ‚ ΠΎΠ±Ρ‰ΠΈΠΉ ΠΏΡ€ΠΈΠ½Ρ†ΠΈΠΏ ΠΌΠΎΠΆΠ΅Ρ‚ ΠΏΡ€ΠΈΠΌΠ΅Π½ΡΡ‚ΡŒΡΡ для опрСдСлСния Ρ‚ΠΎΠ³ΠΎ, являСтся Π»ΠΈ Π·Π½Π°ΠΊ ускорСния ΠΎΠ±ΡŠΠ΅ΠΊΡ‚Π° ΠΏΠΎΠ»ΠΎΠΆΠΈΡ‚Π΅Π»ΡŒΠ½Ρ‹ΠΌ ΠΈΠ»ΠΈ ΠΎΡ‚Ρ€ΠΈΡ†Π°Ρ‚Π΅Π»ΡŒΠ½Ρ‹ΠΌ, Π²ΠΏΡ€Π°Π²ΠΎ ΠΈΠ»ΠΈ Π²Π»Π΅Π²ΠΎ, Π²Π²Π΅Ρ€Ρ… ΠΈΠ»ΠΈ Π²Π½ΠΈΠ· ΠΈ Ρ‚. Π”.Рассмотрим Π΄Π²Π΅ Ρ‚Π°Π±Π»ΠΈΡ†Ρ‹ Π΄Π°Π½Π½Ρ‹Ρ… Π½ΠΈΠΆΠ΅. Π’ ΠΊΠ°ΠΆΠ΄ΠΎΠΌ случаС ускорСниС ΠΎΠ±ΡŠΠ΅ΠΊΡ‚Π° происходит Π² ΠΏΠΎΠ»ΠΎΠΆΠΈΡ‚Π΅Π»ΡŒΠ½ΠΎΠΌ Π½Π°ΠΏΡ€Π°Π²Π»Π΅Π½ΠΈΠΈ . Π’ ΠŸΡ€ΠΈΠΌΠ΅Ρ€Π΅ A ΠΎΠ±ΡŠΠ΅ΠΊΡ‚ двиТСтся Π² ΠΏΠΎΠ»ΠΎΠΆΠΈΡ‚Π΅Π»ΡŒΠ½ΠΎΠΌ Π½Π°ΠΏΡ€Π°Π²Π»Π΅Π½ΠΈΠΈ (Ρ‚. Π•. Π˜ΠΌΠ΅Π΅Ρ‚ ΠΏΠΎΠ»ΠΎΠΆΠΈΡ‚Π΅Π»ΡŒΠ½ΡƒΡŽ ΡΠΊΠΎΡ€ΠΎΡΡ‚ΡŒ ) ΠΈ ускоряСтся. Когда ΠΎΠ±ΡŠΠ΅ΠΊΡ‚ ускоряСтся, ускорСниС совпадаСт с Π½Π°ΠΏΡ€Π°Π²Π»Π΅Π½ΠΈΠ΅ΠΌ скорости. Π’Π°ΠΊΠΈΠΌ ΠΎΠ±Ρ€Π°Π·ΠΎΠΌ, этот ΠΎΠ±ΡŠΠ΅ΠΊΡ‚ ΠΈΠΌΠ΅Π΅Ρ‚ ΠΏΠΎΠ»ΠΎΠΆΠΈΡ‚Π΅Π»ΡŒΠ½ΠΎΠ΅ ускорСниС. Π’ ΠΏΡ€ΠΈΠΌΠ΅Ρ€Π΅ B ΠΎΠ±ΡŠΠ΅ΠΊΡ‚ двиТСтся Π² ΠΎΡ‚Ρ€ΠΈΡ†Π°Ρ‚Π΅Π»ΡŒΠ½ΠΎΠΌ Π½Π°ΠΏΡ€Π°Π²Π»Π΅Π½ΠΈΠΈ (Ρ‚.Π΅. ΠΈΠΌΠ΅Π΅Ρ‚ ΠΎΡ‚Ρ€ΠΈΡ†Π°Ρ‚Π΅Π»ΡŒΠ½ΡƒΡŽ ΡΠΊΠΎΡ€ΠΎΡΡ‚ΡŒ) ΠΈ замСдляСтся.Богласно Π½Π°ΡˆΠ΅ΠΌΡƒ ΠΎΠ±Ρ‰Π΅ΠΌΡƒ ΠΏΡ€ΠΈΠ½Ρ†ΠΈΠΏΡƒ, ΠΊΠΎΠ³Π΄Π° ΠΎΠ±ΡŠΠ΅ΠΊΡ‚ замСдляСтся, ускорСниС происходит Π² Π½Π°ΠΏΡ€Π°Π²Π»Π΅Π½ΠΈΠΈ, ΠΏΡ€ΠΎΡ‚ΠΈΠ²ΠΎΠΏΠΎΠ»ΠΎΠΆΠ½ΠΎΠΌ скорости. Π’Π°ΠΊΠΈΠΌ ΠΎΠ±Ρ€Π°Π·ΠΎΠΌ, этот ΠΎΠ±ΡŠΠ΅ΠΊΡ‚ Ρ‚Π°ΠΊΠΆΠ΅ ΠΈΠΌΠ΅Π΅Ρ‚ ΠΏΠΎΠ»ΠΎΠΆΠΈΡ‚Π΅Π»ΡŒΠ½ΠΎΠ΅ ускорСниС.

Π’ΠΎΡ‚ ΠΆΠ΅ самый ΠΎΠ±Ρ‰ΠΈΠΉ ΠΏΡ€ΠΈΠ½Ρ†ΠΈΠΏ ΠΌΠΎΠΆΠ΅Ρ‚ Π±Ρ‹Ρ‚ΡŒ ΠΏΡ€ΠΈΠΌΠ΅Π½Π΅Π½ ΠΊ двиТСнию ΠΎΠ±ΡŠΠ΅ΠΊΡ‚ΠΎΠ², прСдставлСнных Π² Π΄Π²ΡƒΡ… Ρ‚Π°Π±Π»ΠΈΡ†Π°Ρ… Π΄Π°Π½Π½Ρ‹Ρ… Π½ΠΈΠΆΠ΅. Π’ ΠΊΠ°ΠΆΠ΄ΠΎΠΌ случаС ускорСниС ΠΎΠ±ΡŠΠ΅ΠΊΡ‚Π° происходит Π² ΠΎΡ‚Ρ€ΠΈΡ†Π°Ρ‚Π΅Π»ΡŒΠ½ΠΎΠΌ Π½Π°ΠΏΡ€Π°Π²Π»Π΅Π½ΠΈΠΈ . Π’ ΠΏΡ€ΠΈΠΌΠ΅Ρ€Π΅ C ΠΎΠ±ΡŠΠ΅ΠΊΡ‚ двиТСтся Π² ΠΏΠΎΠ»ΠΎΠΆΠΈΡ‚Π΅Π»ΡŒΠ½ΠΎΠΌ Π½Π°ΠΏΡ€Π°Π²Π»Π΅Π½ΠΈΠΈ (Ρ‚.Π΅.Π΅., ΠΈΠΌΠ΅Π΅Ρ‚ ΠΏΠΎΠ»ΠΎΠΆΠΈΡ‚Π΅Π»ΡŒΠ½ΡƒΡŽ ΡΠΊΠΎΡ€ΠΎΡΡ‚ΡŒ ) ΠΈ замСдляСтся. Богласно Π½Π°ΡˆΠ΅ΠΌΡƒ ΠΏΡ€ΠΈΠ½Ρ†ΠΈΠΏΡƒ, ΠΊΠΎΠ³Π΄Π° ΠΎΠ±ΡŠΠ΅ΠΊΡ‚ замСдляСтся, ускорСниС происходит Π² Π½Π°ΠΏΡ€Π°Π²Π»Π΅Π½ΠΈΠΈ, ΠΏΡ€ΠΎΡ‚ΠΈΠ²ΠΎΠΏΠΎΠ»ΠΎΠΆΠ½ΠΎΠΌ скорости. Π’Π°ΠΊΠΈΠΌ ΠΎΠ±Ρ€Π°Π·ΠΎΠΌ, этот ΠΎΠ±ΡŠΠ΅ΠΊΡ‚ ΠΈΠΌΠ΅Π΅Ρ‚ ΠΎΡ‚Ρ€ΠΈΡ†Π°Ρ‚Π΅Π»ΡŒΠ½ΠΎΠ΅ ускорСниС. Π’ ΠŸΡ€ΠΈΠΌΠ΅Ρ€Π΅ D ΠΎΠ±ΡŠΠ΅ΠΊΡ‚ двиТСтся Π² ΠΎΡ‚Ρ€ΠΈΡ†Π°Ρ‚Π΅Π»ΡŒΠ½ΠΎΠΌ Π½Π°ΠΏΡ€Π°Π²Π»Π΅Π½ΠΈΠΈ (Ρ‚.Π΅. ΠΈΠΌΠ΅Π΅Ρ‚ ΠΎΡ‚Ρ€ΠΈΡ†Π°Ρ‚Π΅Π»ΡŒΠ½ΡƒΡŽ ΡΠΊΠΎΡ€ΠΎΡΡ‚ΡŒ ) ΠΈ ускоряСтся. Когда ΠΎΠ±ΡŠΠ΅ΠΊΡ‚ ускоряСтся, ускорСниС совпадаСт с Π½Π°ΠΏΡ€Π°Π²Π»Π΅Π½ΠΈΠ΅ΠΌ скорости. Π’Π°ΠΊΠΈΠΌ ΠΎΠ±Ρ€Π°Π·ΠΎΠΌ, этот ΠΎΠ±ΡŠΠ΅ΠΊΡ‚ Ρ‚Π°ΠΊΠΆΠ΅ ΠΈΠΌΠ΅Π΅Ρ‚ ΠΎΡ‚Ρ€ΠΈΡ†Π°Ρ‚Π΅Π»ΡŒΠ½ΠΎΠ΅ ускорСниС.

ΠžΠ±Ρ€Π°Ρ‚ΠΈΡ‚Π΅ Π²Π½ΠΈΠΌΠ°Π½ΠΈΠ΅ Π½Π° использованиС ΠΏΠΎΠ»ΠΎΠΆΠΈΡ‚Π΅Π»ΡŒΠ½Ρ‹Ρ… ΠΈ ΠΎΡ‚Ρ€ΠΈΡ†Π°Ρ‚Π΅Π»ΡŒΠ½Ρ‹Ρ… слов Π² ΠΏΡ€ΠΈΠ²Π΅Π΄Π΅Π½Π½ΠΎΠΌ Π²Ρ‹ΡˆΠ΅ обсуТдСнии (ΠŸΡ€ΠΈΠΌΠ΅Ρ€Ρ‹ A — D). Π’ Ρ„ΠΈΠ·ΠΈΠΊΠ΅ использованиС ΠΏΠΎΠ»ΠΎΠΆΠΈΡ‚Π΅Π»ΡŒΠ½ΠΎΠ³ΠΎ ΠΈ ΠΎΡ‚Ρ€ΠΈΡ†Π°Ρ‚Π΅Π»ΡŒΠ½ΠΎΠ³ΠΎ всСгда ΠΈΠΌΠ΅Π΅Ρ‚ физичСский смысл. Π­Ρ‚ΠΎ большС, Ρ‡Π΅ΠΌ просто матСматичСский символ. ΠŸΠΎΠ»ΠΎΠΆΠΈΡ‚Π΅Π»ΡŒΠ½ΠΎΠ΅ ΠΈ ΠΎΡ‚Ρ€ΠΈΡ†Π°Ρ‚Π΅Π»ΡŒΠ½ΠΎΠ΅, ΠΈΡΠΏΠΎΠ»ΡŒΠ·ΡƒΠ΅ΠΌΡ‹Π΅ здСсь для описания скорости ΠΈ ускорСния двиТущСгося ΠΎΠ±ΡŠΠ΅ΠΊΡ‚Π°, ΠΎΠΏΠΈΡΡ‹Π²Π°ΡŽΡ‚ Π½Π°ΠΏΡ€Π°Π²Π»Π΅Π½ΠΈΠ΅. И ΡΠΊΠΎΡ€ΠΎΡΡ‚ΡŒ, ΠΈ ускорСниС ΡΠ²Π»ΡΡŽΡ‚ΡΡ Π²Π΅ΠΊΡ‚ΠΎΡ€Π½Ρ‹ΠΌΠΈ Π²Π΅Π»ΠΈΡ‡ΠΈΠ½Π°ΠΌΠΈ, ΠΈ ΠΏΠΎΠ»Π½ΠΎΠ΅ описаниС Π²Π΅Π»ΠΈΡ‡ΠΈΠ½Ρ‹ Ρ‚Ρ€Π΅Π±ΡƒΠ΅Ρ‚ использования ΠΏΡ€ΠΈΠ»Π°Π³Π°Ρ‚Π΅Π»ΡŒΠ½ΠΎΠ³ΠΎ Π½Π°ΠΏΡ€Π°Π²Π»Π΅Π½Π½ΠΎΠ³ΠΎ дСйствия.Π‘Π΅Π²Π΅Ρ€, юг, восток, Π·Π°ΠΏΠ°Π΄, Π²ΠΏΡ€Π°Π²ΠΎ, Π²Π»Π΅Π²ΠΎ, Π²Π²Π΅Ρ€Ρ… ΠΈ Π²Π½ΠΈΠ· — всС ΠΏΡ€ΠΈΠ»Π°Π³Π°Ρ‚Π΅Π»ΡŒΠ½Ρ‹Π΅ направлСния. Π€ΠΈΠ·ΠΈΠΊΠ° часто заимствуСт ΠΈΠ· ΠΌΠ°Ρ‚Π΅ΠΌΠ°Ρ‚ΠΈΠΊΠΈ ΠΈ ΠΈΡΠΏΠΎΠ»ΡŒΠ·ΡƒΠ΅Ρ‚ символы + ΠΈ — ΠΊΠ°ΠΊ Π½Π°ΠΏΡ€Π°Π²Π»Π΅Π½Π½Ρ‹Π΅ ΠΏΡ€ΠΈΠ»Π°Π³Π°Ρ‚Π΅Π»ΡŒΠ½Ρ‹Π΅. Π’ соотвСтствии с матСматичСским соглашСниСм, ΠΈΡΠΏΠΎΠ»ΡŒΠ·ΡƒΠ΅ΠΌΡ‹ΠΌ Π² числовых линиях ΠΈ Π³Ρ€Π°Ρ„ΠΈΠΊΠ°Ρ…, ΠΏΠΎΠ»ΠΎΠΆΠΈΡ‚Π΅Π»ΡŒΠ½ΠΎΠ΅ Π·Π½Π°Ρ‡Π΅Π½ΠΈΠ΅ часто ΠΎΠ·Π½Π°Ρ‡Π°Π΅Ρ‚ Π²ΠΏΡ€Π°Π²ΠΎ ΠΈΠ»ΠΈ Π²Π²Π΅Ρ€Ρ…, Π° ΠΎΡ‚Ρ€ΠΈΡ†Π°Ρ‚Π΅Π»ΡŒΠ½ΠΎΠ΅ — Π²Π»Π΅Π²ΠΎ ΠΈΠ»ΠΈ Π²Π½ΠΈΠ·. Π’Π°ΠΊΠΈΠΌ ΠΎΠ±Ρ€Π°Π·ΠΎΠΌ, ΡΠΊΠ°Π·Π°Ρ‚ΡŒ, Ρ‡Ρ‚ΠΎ ΠΎΠ±ΡŠΠ΅ΠΊΡ‚ ΠΈΠΌΠ΅Π΅Ρ‚ ΠΎΡ‚Ρ€ΠΈΡ†Π°Ρ‚Π΅Π»ΡŒΠ½ΠΎΠ΅ ускорСниС, ΠΊΠ°ΠΊ Π² ΠΏΡ€ΠΈΠΌΠ΅Ρ€Π°Ρ… C ΠΈ D, ΠΎΠ·Π½Π°Ρ‡Π°Π΅Ρ‚ просто ΡΠΊΠ°Π·Π°Ρ‚ΡŒ, Ρ‡Ρ‚ΠΎ Π΅Π³ΠΎ ускорСниС ΠΈΠ΄Π΅Ρ‚ Π²Π»Π΅Π²ΠΎ ΠΈΠ»ΠΈ Π²Π½ΠΈΠ· (ΠΈΠ»ΠΈ Π² любом Π½Π°ΠΏΡ€Π°Π²Π»Π΅Π½ΠΈΠΈ, ΠΎΠΏΡ€Π΅Π΄Π΅Π»Π΅Π½Π½ΠΎΠΌ ΠΊΠ°ΠΊ ΠΎΡ‚Ρ€ΠΈΡ†Π°Ρ‚Π΅Π»ΡŒΠ½ΠΎΠ΅).ΠžΡ‚Ρ€ΠΈΡ†Π°Ρ‚Π΅Π»ΡŒΠ½Ρ‹Π΅ ускорСния Π½Π΅ относятся ΠΊ значСниям ускорСния мСньшС 0. УскорСниС -2 ΠΌ / с / с — это ускорСниС с Π²Π΅Π»ΠΈΡ‡ΠΈΠ½ΠΎΠΉ 2 ΠΌ / с / с, ΠΊΠΎΡ‚ΠΎΡ€ΠΎΠ΅ Π½Π°ΠΏΡ€Π°Π²Π»Π΅Π½ΠΎ Π² ΠΎΡ‚Ρ€ΠΈΡ†Π°Ρ‚Π΅Π»ΡŒΠ½ΠΎΠΌ Π½Π°ΠΏΡ€Π°Π²Π»Π΅Π½ΠΈΠΈ.

Π₯ΠΎΡ‚ΠΈΠΌ ΠΏΡ€Π΅Π΄Π»ΠΎΠΆΠΈΡ‚ΡŒ … Иногда просто ΠΏΡ€ΠΎΡ‡ΠΈΡ‚Π°Ρ‚ΡŒ ΠΎΠ± этом нСдостаточно. Π’Ρ‹ Π΄ΠΎΠ»ΠΆΠ½Ρ‹ с Π½ΠΈΠΌ Π²Π·Π°ΠΈΠΌΠΎΠ΄Π΅ΠΉΡΡ‚Π²ΠΎΠ²Π°Ρ‚ΡŒ! И это ΠΈΠΌΠ΅Π½Π½ΠΎ Ρ‚ΠΎ, Ρ‡Ρ‚ΠΎ Π²Ρ‹ Π΄Π΅Π»Π°Π΅Ρ‚Π΅, ΠΊΠΎΠ³Π΄Π° ΠΈΡΠΏΠΎΠ»ΡŒΠ·ΡƒΠ΅Ρ‚Π΅ ΠΎΠ΄ΠΈΠ½ ΠΈΠ· ΠΈΠ½Ρ‚Π΅Ρ€Π°ΠΊΡ‚ΠΈΠ²Π½Ρ‹Ρ… ΠΌΠ°Ρ‚Π΅Ρ€ΠΈΠ°Π»ΠΎΠ² The Physics Classroom. ΠœΡ‹ Ρ…ΠΎΡ‚Π΅Π»ΠΈ Π±Ρ‹ ΠΏΡ€Π΅Π΄Π»ΠΎΠΆΠΈΡ‚ΡŒ Π²Π°ΠΌ ΡΠΎΠ²ΠΌΠ΅ΡΡ‚ΠΈΡ‚ΡŒ Ρ‡Ρ‚Π΅Π½ΠΈΠ΅ этой страницы с использованиСм нашСго Name That Motion Interactive.Он находится Π² Ρ€Π°Π·Π΄Π΅Π»Π΅ Β«Π˜Π½Ρ‚Π΅Ρ€Π°ΠΊΡ‚ΠΈΠ²Π½Π°Ρ Ρ„ΠΈΠ·ΠΈΠΊΠ°Β» ΠΈ позволяСт учащСмуся ΠΏΡ€ΠΈΠΌΠ΅Π½ΡΡ‚ΡŒ ΠΊΠΎΠ½Ρ†Π΅ΠΏΡ†ΠΈΠΈ скорости, скорости ΠΈ ускорСния.

ΠŸΡ€ΠΎΠ²Π΅Ρ€ΡŒΡ‚Π΅ своС ΠΏΠΎΠ½ΠΈΠΌΠ°Π½ΠΈΠ΅

Π§Ρ‚ΠΎΠ±Ρ‹ ΠΏΡ€ΠΎΠ²Π΅Ρ€ΠΈΡ‚ΡŒ своС ΠΏΠΎΠ½ΠΈΠΌΠ°Π½ΠΈΠ΅ ΠΊΠΎΠ½Ρ†Π΅ΠΏΡ†ΠΈΠΈ ускорСния, рассмотритС ΡΠ»Π΅Π΄ΡƒΡŽΡ‰ΠΈΠ΅ ΠΏΡ€ΠΎΠ±Π»Π΅ΠΌΡ‹ ΠΈ ΡΠΎΠΎΡ‚Π²Π΅Ρ‚ΡΡ‚Π²ΡƒΡŽΡ‰ΠΈΠ΅ Ρ€Π΅ΡˆΠ΅Π½ΠΈΡ. Π˜ΡΠΏΠΎΠ»ΡŒΠ·ΡƒΠΉΡ‚Π΅ ΡƒΡ€Π°Π²Π½Π΅Π½ΠΈΠ΅ для ускорСния, Ρ‡Ρ‚ΠΎΠ±Ρ‹ ΠΎΠΏΡ€Π΅Π΄Π΅Π»ΠΈΡ‚ΡŒ ускорСниС для ΡΠ»Π΅Π΄ΡƒΡŽΡ‰ΠΈΡ… Π΄Π²ΡƒΡ… Π΄Π²ΠΈΠΆΠ΅Π½ΠΈΠΉ.


Ρ€Π°Π·Π³ΠΎΠ½ | Π€ΠΈΠ·ΠΈΠΊΠ°

Π¦Π΅Π»ΠΈ обучСния

К ΠΊΠΎΠ½Ρ†Ρƒ этого Ρ€Π°Π·Π΄Π΅Π»Π° Π²Ρ‹ смоТСтС:

  • ΠžΠΏΡ€Π΅Π΄Π΅Π»Π΅Π½ΠΈΠ΅ ΠΈ Ρ€Π°Π·Π»ΠΈΡ‡ΠΈΠ΅ ΠΌΠ΅ΠΆΠ΄Ρƒ ΠΌΠ³Π½ΠΎΠ²Π΅Π½Π½Ρ‹ΠΌ ускорСниСм, срСдним ускорСниСм ΠΈ Π·Π°ΠΌΠ΅Π΄Π»Π΅Π½ΠΈΠ΅ΠΌ.
  • РассчитайтС ускорСниС с ΡƒΡ‡Π΅Ρ‚ΠΎΠΌ Π½Π°Ρ‡Π°Π»ΡŒΠ½ΠΎΠ³ΠΎ Π²Ρ€Π΅ΠΌΠ΅Π½ΠΈ, Π½Π°Ρ‡Π°Π»ΡŒΠ½ΠΎΠΉ скорости, ΠΊΠΎΠ½Π΅Ρ‡Π½ΠΎΠ³ΠΎ Π²Ρ€Π΅ΠΌΠ΅Π½ΠΈ ΠΈ ΠΊΠΎΠ½Π΅Ρ‡Π½ΠΎΠΉ скорости.

Рис. 1. Π‘Π°ΠΌΠΎΠ»Π΅Ρ‚ замСдляСтся ΠΈΠ»ΠΈ замСдляСтся ΠΏΡ€ΠΈ посадкС Π½Π° островС Π‘Π΅Π½-ΠœΠ°Ρ€Ρ‚Π΅Π½. Π•Π³ΠΎ ускорСниС ΠΏΡ€ΠΎΡ‚ΠΈΠ²ΠΎΠΏΠΎΠ»ΠΎΠΆΠ½ΠΎ Π΅Π³ΠΎ скорости. (Π˜ΡΡ‚ΠΎΡ‡Π½ΠΈΠΊ: Π‘Ρ‚ΠΈΠ² ΠšΠΎΠ½Ρ€ΠΈ, Flickr)

Π’ повсСднСвном Ρ€Π°Π·Π³ΠΎΠ²ΠΎΡ€Π΅ ΡƒΡΠΊΠΎΡ€ΡΡ‚ΡŒ ΠΎΠ·Π½Π°Ρ‡Π°Π΅Ρ‚ ΡƒΡΠΊΠΎΡ€ΡΡ‚ΡŒΡΡ. ЀактичСски, ΡƒΡΠΊΠΎΡ€ΠΈΡ‚Π΅Π»ΡŒ Π² Π°Π²Ρ‚ΠΎΠΌΠΎΠ±ΠΈΠ»Π΅ ΠΌΠΎΠΆΠ΅Ρ‚ Π·Π°ΡΡ‚Π°Π²ΠΈΡ‚ΡŒ Π΅Π³ΠΎ Ρ€Π°Π·ΠΎΠ³Π½Π°Ρ‚ΡŒΡΡ. Π§Π΅ΠΌ большС ускорСниС , Ρ‚Π΅ΠΌ большС ΠΈΠ·ΠΌΠ΅Π½Π΅Π½ΠΈΠ΅ скорости Π·Π° Π·Π°Π΄Π°Π½Π½Ρ‹ΠΉ ΠΏΡ€ΠΎΠΌΠ΅ΠΆΡƒΡ‚ΠΎΠΊ Π²Ρ€Π΅ΠΌΠ΅Π½ΠΈ.Π€ΠΎΡ€ΠΌΠ°Π»ΡŒΠ½ΠΎΠ΅ ΠΎΠΏΡ€Π΅Π΄Π΅Π»Π΅Π½ΠΈΠ΅ ускорСния согласуСтся с этими понятиями, Π½ΠΎ являСтся Π±ΠΎΠ»Π΅Π΅ Π²ΡΠ΅ΠΎΠ±ΡŠΠ΅ΠΌΠ»ΡŽΡ‰ΠΈΠΌ.

Π‘Ρ€Π΅Π΄Π½Π΅Π΅ ускорСниС

Π‘Ρ€Π΅Π΄Π½Π΅Π΅ ускорСниС — это ΡΠΊΠΎΡ€ΠΎΡΡ‚ΡŒ, с ΠΊΠΎΡ‚ΠΎΡ€ΠΎΠΉ измСняСтся ΡΠΊΠΎΡ€ΠΎΡΡ‚ΡŒ ,

[латСкс] \ bar {a} = \ frac {\ Delta v} {\ Delta t} = \ frac {{v} _ {f} — {v} _ {0}} {{t} _ {f} — {t} _ {0}} [/ latex]

, Π³Π΄Π΅ [latex] \ bar {a} [/ latex] — срСднСС ускорСниС, v — ΡΠΊΠΎΡ€ΠΎΡΡ‚ΡŒ, Π° t — врСмя.(Полоса Π½Π°Π΄ a ΠΎΠ·Π½Π°Ρ‡Π°Π΅Ρ‚ срСднСС ускорСниС .)

ΠŸΠΎΡΠΊΠΎΠ»ΡŒΠΊΡƒ ускорСниС — это ΡΠΊΠΎΡ€ΠΎΡΡ‚ΡŒ Π² ΠΌ / с, дСлСнная Π½Π° врСмя Π² сСкундах, Π΅Π΄ΠΈΠ½ΠΈΡ†Π°ΠΌΠΈ измСрСния ускорСния Π² систСмС БИ ΡΠ²Π»ΡΡŽΡ‚ΡΡ ΠΌ / с 2 , ΠΊΠ²Π°Π΄Ρ€Π°Ρ‚Π½Ρ‹Π΅ ΠΌΠ΅Ρ‚Ρ€Ρ‹ Π² сСкунду ΠΈΠ»ΠΈ ΠΌΠ΅Ρ‚Ρ€Ρ‹ Π² сСкунду Π² сСкунду, Ρ‡Ρ‚ΠΎ Π±ΡƒΠΊΠ²Π°Π»ΡŒΠ½ΠΎ ΠΎΠ·Π½Π°Ρ‡Π°Π΅Ρ‚, сколько ΠΌΠ΅Ρ‚Ρ€ΠΎΠ² Π² сСкунду ΡΠΊΠΎΡ€ΠΎΡΡ‚ΡŒ мСняСтся ΠΊΠ°ΠΆΠ΄ΡƒΡŽ сСкунду.

Напомним, Ρ‡Ρ‚ΠΎ ΡΠΊΠΎΡ€ΠΎΡΡ‚ΡŒ — это Π²Π΅ΠΊΡ‚ΠΎΡ€, Ρƒ Π½Π΅Π΅ Π΅ΡΡ‚ΡŒ Π²Π΅Π»ΠΈΡ‡ΠΈΠ½Π° ΠΈ Π½Π°ΠΏΡ€Π°Π²Π»Π΅Π½ΠΈΠ΅. Π­Ρ‚ΠΎ ΠΎΠ·Π½Π°Ρ‡Π°Π΅Ρ‚, Ρ‡Ρ‚ΠΎ ΠΈΠ·ΠΌΠ΅Π½Π΅Π½ΠΈΠ΅ скорости ΠΌΠΎΠΆΠ΅Ρ‚ Π±Ρ‹Ρ‚ΡŒ ΠΈΠ·ΠΌΠ΅Π½Π΅Π½ΠΈΠ΅ΠΌ Π²Π΅Π»ΠΈΡ‡ΠΈΠ½Ρ‹ (ΠΈΠ»ΠΈ скорости), Π½ΠΎ это Ρ‚Π°ΠΊΠΆΠ΅ ΠΌΠΎΠΆΠ΅Ρ‚ Π±Ρ‹Ρ‚ΡŒ ΠΈΠ·ΠΌΠ΅Π½Π΅Π½ΠΈΠ΅ Π² Π½Π°ΠΏΡ€Π°Π²Π»Π΅Π½ΠΈΠΈ .НапримСр, Ссли Π°Π²Ρ‚ΠΎΠΌΠΎΠ±ΠΈΠ»ΡŒ ΠΏΠΎΠ²ΠΎΡ€Π°Ρ‡ΠΈΠ²Π°Π΅Ρ‚ с постоянной ΡΠΊΠΎΡ€ΠΎΡΡ‚ΡŒΡŽ, ΠΎΠ½ ускоряСтся, ΠΏΠΎΡ‚ΠΎΠΌΡƒ Ρ‡Ρ‚ΠΎ Π΅Π³ΠΎ Π½Π°ΠΏΡ€Π°Π²Π»Π΅Π½ΠΈΠ΅ мСняСтся. Π§Π΅ΠΌ быстрСС Π²Ρ‹ ΠΏΠΎΠ²ΠΎΡ€Π°Ρ‡ΠΈΠ²Π°Π΅Ρ‚Π΅, Ρ‚Π΅ΠΌ большС ускорСниС. Π’Π°ΠΊΠΈΠΌ ΠΎΠ±Ρ€Π°Π·ΠΎΠΌ, ускорСниС Π²ΠΎΠ·Π½ΠΈΠΊΠ°Π΅Ρ‚, ΠΊΠΎΠ³Π΄Π° ΡΠΊΠΎΡ€ΠΎΡΡ‚ΡŒ измСняСтся Π»ΠΈΠ±ΠΎ ΠΏΠΎ Π²Π΅Π»ΠΈΡ‡ΠΈΠ½Π΅ (ΡƒΠ²Π΅Π»ΠΈΡ‡Π΅Π½ΠΈΠ΅ ΠΈΠ»ΠΈ ΡƒΠΌΠ΅Π½ΡŒΡˆΠ΅Π½ΠΈΠ΅ скорости), Π»ΠΈΠ±ΠΎ ΠΏΠΎ Π½Π°ΠΏΡ€Π°Π²Π»Π΅Π½ΠΈΡŽ, Π»ΠΈΠ±ΠΎ ΠΏΠΎ ΠΎΠ±ΠΎΠΈΠΌ направлСниям.

УскорСниС ΠΊΠ°ΠΊ Π²Π΅ΠΊΡ‚ΠΎΡ€

УскорСниС — это Π²Π΅ΠΊΡ‚ΠΎΡ€ Π² Ρ‚ΠΎΠΌ ΠΆΠ΅ Π½Π°ΠΏΡ€Π°Π²Π»Π΅Π½ΠΈΠΈ, Ρ‡Ρ‚ΠΎ ΠΈ ΠΈΠ·ΠΌΠ΅Π½Π΅Π½ΠΈΠ΅ скорости Π½Π° , Ξ” v . ΠŸΠΎΡΠΊΠΎΠ»ΡŒΠΊΡƒ ΡΠΊΠΎΡ€ΠΎΡΡ‚ΡŒ — это Π²Π΅ΠΊΡ‚ΠΎΡ€, ΠΎΠ½Π° ΠΌΠΎΠΆΠ΅Ρ‚ ΠΌΠ΅Π½ΡΡ‚ΡŒΡΡ ΠΏΠΎ Π²Π΅Π»ΠΈΡ‡ΠΈΠ½Π΅ ΠΈΠ»ΠΈ ΠΏΠΎ Π½Π°ΠΏΡ€Π°Π²Π»Π΅Π½ΠΈΡŽ.Π’Π°ΠΊΠΈΠΌ ΠΎΠ±Ρ€Π°Π·ΠΎΠΌ, ускорСниС — это ΠΈΠ·ΠΌΠ΅Π½Π΅Π½ΠΈΠ΅ скорости ΠΈΠ»ΠΈ направлСния, Π»ΠΈΠ±ΠΎ ΠΈ Ρ‚ΠΎΠ³ΠΎ, ΠΈ Π΄Ρ€ΡƒΠ³ΠΎΠ³ΠΎ.

Π˜ΠΌΠ΅ΠΉΡ‚Π΅ Π² Π²ΠΈΠ΄Ρƒ, Ρ‡Ρ‚ΠΎ хотя ускорСниС происходит Π² Π½Π°ΠΏΡ€Π°Π²Π»Π΅Π½ΠΈΠΈ измСнСния скорости, ΠΎΠ½ΠΎ Π½Π΅ всСгда происходит Π² Π½Π°ΠΏΡ€Π°Π²Π»Π΅Π½ΠΈΠΈ двиТСния . Когда ΠΎΠ±ΡŠΠ΅ΠΊΡ‚ замСдляСтся, Π΅Π³ΠΎ ускорСниС ΠΏΡ€ΠΎΡ‚ΠΈΠ²ΠΎΠΏΠΎΠ»ΠΎΠΆΠ½ΠΎ Π½Π°ΠΏΡ€Π°Π²Π»Π΅Π½ΠΈΡŽ Π΅Π³ΠΎ двиТСния. Π­Ρ‚ΠΎ извСстно ΠΊΠ°ΠΊ Π·Π°ΠΌΠ΅Π΄Π»Π΅Π½ΠΈΠ΅ .

МгновСнноС ускорСниС

МгновСнноС ускорСниС a , ΠΈΠ»ΠΈ ускорСниС Π² ΠΎΠΏΡ€Π΅Π΄Π΅Π»Π΅Π½Π½Ρ‹ΠΉ ΠΌΠΎΠΌΠ΅Π½Ρ‚ Π²Ρ€Π΅ΠΌΠ΅Π½ΠΈ , получаСтся с ΠΏΠΎΠΌΠΎΡ‰ΡŒΡŽ Ρ‚ΠΎΠ³ΠΎ ΠΆΠ΅ процСсса, ΠΊΠΎΡ‚ΠΎΡ€Ρ‹ΠΉ обсуТдался для ΠΌΠ³Π½ΠΎΠ²Π΅Π½Π½ΠΎΠΉ скорости Π²ΠΎ Π²Ρ€Π΅ΠΌΠ΅Π½ΠΈ, скорости ΠΈ скорости, Ρ‚ΠΎ Π΅ΡΡ‚ΡŒ ΠΏΡƒΡ‚Π΅ΠΌ рассмотрСния бСсконСчно ΠΌΠ°Π»ΠΎΠ³ΠΎ ΠΈΠ½Ρ‚Π΅Ρ€Π²Π°Π»Π° врСмя.Как Π½Π°ΠΉΡ‚ΠΈ ΠΌΠ³Π½ΠΎΠ²Π΅Π½Π½ΠΎΠ΅ ускорСниС, ΠΈΡΠΏΠΎΠ»ΡŒΠ·ΡƒΡ Ρ‚ΠΎΠ»ΡŒΠΊΠΎ Π°Π»Π³Π΅Π±Ρ€Ρƒ? ΠžΡ‚Π²Π΅Ρ‚ Π·Π°ΠΊΠ»ΡŽΡ‡Π°Π΅Ρ‚ΡΡ Π² Ρ‚ΠΎΠΌ, Ρ‡Ρ‚ΠΎ ΠΌΡ‹ Π²Ρ‹Π±ΠΈΡ€Π°Π΅ΠΌ срСднСС ускорСниС, ΠΊΠΎΡ‚ΠΎΡ€ΠΎΠ΅ прСдставляСт Π΄Π²ΠΈΠΆΠ΅Π½ΠΈΠ΅. На рисункС 6 ΠΏΠΎΠΊΠ°Π·Π°Π½Ρ‹ Π³Ρ€Π°Ρ„ΠΈΠΊΠΈ ΠΌΠ³Π½ΠΎΠ²Π΅Π½Π½ΠΎΠ³ΠΎ ускорСния Π² зависимости ΠΎΡ‚ Π²Ρ€Π΅ΠΌΠ΅Π½ΠΈ для Π΄Π²ΡƒΡ… ΠΎΡ‡Π΅Π½ΡŒ Ρ€Π°Π·Π½Ρ‹Ρ… Π΄Π²ΠΈΠΆΠ΅Π½ΠΈΠΉ. На РисункС 6 (Π°) ускорСниС Π½Π΅Π·Π½Π°Ρ‡ΠΈΡ‚Π΅Π»ΡŒΠ½ΠΎ мСняСтся, ΠΈ срСднСС Π·Π½Π°Ρ‡Π΅Π½ΠΈΠ΅ Π·Π° вСсь ΠΈΠ½Ρ‚Π΅Ρ€Π²Π°Π» ΠΏΠΎΡ‡Ρ‚ΠΈ Ρ‚Π°ΠΊΠΎΠ΅ ΠΆΠ΅, ΠΊΠ°ΠΊ ΠΌΠ³Π½ΠΎΠ²Π΅Π½Π½ΠΎΠ΅ ускорСниС Π² любой ΠΌΠΎΠΌΠ΅Π½Ρ‚ Π²Ρ€Π΅ΠΌΠ΅Π½ΠΈ. Π’ этом случаС ΠΌΡ‹ Π΄ΠΎΠ»ΠΆΠ½Ρ‹ Ρ€Π°ΡΡΠΌΠ°Ρ‚Ρ€ΠΈΠ²Π°Ρ‚ΡŒ это Π΄Π²ΠΈΠΆΠ΅Π½ΠΈΠ΅, ΠΊΠ°ΠΊ Ссли Π±Ρ‹ ΠΎΠ½ΠΎ ΠΈΠΌΠ΅Π»ΠΎ постоянноС ускорСниС, Ρ€Π°Π²Π½ΠΎΠ΅ срСднСму (Π² Π΄Π°Π½Π½ΠΎΠΌ случаС ΠΎΠΊΠΎΠ»ΠΎ 1.8 ΠΌ / с 2 ). На рисункС 6 (b) ускорСниС Ρ€Π΅Π·ΠΊΠΎ мСняСтся со Π²Ρ€Π΅ΠΌΠ΅Π½Π΅ΠΌ. Π’ Ρ‚Π°ΠΊΠΈΡ… ситуациях Π»ΡƒΡ‡ΡˆΠ΅ всСго Ρ€Π°ΡΡΠΌΠ°Ρ‚Ρ€ΠΈΠ²Π°Ρ‚ΡŒ мСньшиС Π²Ρ€Π΅ΠΌΠ΅Π½Π½Ρ‹Π΅ ΠΈΠ½Ρ‚Π΅Ρ€Π²Π°Π»Ρ‹ ΠΈ Π²Ρ‹Π±ΠΈΡ€Π°Ρ‚ΡŒ для ΠΊΠ°ΠΆΠ΄ΠΎΠ³ΠΎ срСднСС ускорСниС. НапримСр, ΠΌΡ‹ ΠΌΠΎΠΆΠ΅ΠΌ Ρ€Π°ΡΡΠΌΠ°Ρ‚Ρ€ΠΈΠ²Π°Ρ‚ΡŒ Π΄Π²ΠΈΠΆΠ΅Π½ΠΈΠ΅ Π²ΠΎ Π²Ρ€Π΅ΠΌΠ΅Π½Π½Ρ‹Ρ… ΠΈΠ½Ρ‚Π΅Ρ€Π²Π°Π»Π°Ρ… ΠΎΡ‚ 0 Π΄ΠΎ 1,0 с ΠΈ ΠΎΡ‚ 1,0 Π΄ΠΎ 3,0 с ΠΊΠ°ΠΊ ΠΎΡ‚Π΄Π΅Π»ΡŒΠ½Ρ‹Π΅ двиТСния с ускорСниями +3,0 ΠΌ / с 2 ΠΈ –2,0 ΠΌ / с 2 соотвСтствСнно.

Π’ ΡΠ»Π΅Π΄ΡƒΡŽΡ‰ΠΈΡ… Π½Π΅ΡΠΊΠΎΠ»ΡŒΠΊΠΈΡ… ΠΏΡ€ΠΈΠΌΠ΅Ρ€Π°Ρ… рассматриваСтся Π΄Π²ΠΈΠΆΠ΅Π½ΠΈΠ΅ ΠΏΠΎΠ΅Π·Π΄Π° ΠΌΠ΅Ρ‚Ρ€ΠΎ, ​​показанного Π½Π° рисункС 7.Π’ (Π°) Π²ΠΎΠ»Π°Π½ двиТСтся Π²ΠΏΡ€Π°Π²ΠΎ, Π° Π² (Π±) — Π²Π»Π΅Π²ΠΎ. ΠŸΡ€ΠΈΠΌΠ΅Ρ€Ρ‹ ΠΏΡ€ΠΈΠ·Π²Π°Π½Ρ‹ Π΄ΠΎΠΏΠΎΠ»Π½ΠΈΡ‚Π΅Π»ΡŒΠ½ΠΎ ΠΏΡ€ΠΎΠΈΠ»Π»ΡŽΡΡ‚Ρ€ΠΈΡ€ΠΎΠ²Π°Ρ‚ΡŒ аспСкты двиТСния ΠΈ ΠΏΡ€ΠΎΠΈΠ»Π»ΡŽΡΡ‚Ρ€ΠΈΡ€ΠΎΠ²Π°Ρ‚ΡŒ Π½Π΅ΠΊΠΎΡ‚ΠΎΡ€Ρ‹Π΅ Π°Ρ€Π³ΡƒΠΌΠ΅Π½Ρ‚Ρ‹, ΠΊΠΎΡ‚ΠΎΡ€Ρ‹Π΅ ΠΈΡΠΏΠΎΠ»ΡŒΠ·ΡƒΡŽΡ‚ΡΡ ΠΏΡ€ΠΈ Ρ€Π΅ΡˆΠ΅Π½ΠΈΠΈ ΠΏΡ€ΠΎΠ±Π»Π΅ΠΌ.

ΠŸΡ€ΠΈΠΌΠ΅Ρ€ 2. РасчСт смСщСния: ΠΏΠΎΠ΅Π·Π΄ ΠΌΠ΅Ρ‚Ρ€ΠΎ

ΠšΠ°ΠΊΠΎΠ²Ρ‹ Π²Π΅Π»ΠΈΡ‡ΠΈΠ½Π° ΠΈ Π·Π½Π°ΠΊ смСщСний ΠΏΡ€ΠΈ Π΄Π²ΠΈΠΆΠ΅Π½ΠΈΠΈ ΠΏΠΎΠ΅Π·Π΄Π° ΠΌΠ΅Ρ‚Ρ€ΠΎ, ​​показанных Π² частях (Π°) ΠΈ (b) рисунка 7?

БтратСгия

Π§Π΅Ρ€Ρ‚Π΅ΠΆ с систСмой ΠΊΠΎΠΎΡ€Π΄ΠΈΠ½Π°Ρ‚ ΡƒΠΆΠ΅ прСдоставлСн, поэтому Π½Π°ΠΌ Π½Π΅ Π½ΡƒΠΆΠ½ΠΎ Π΄Π΅Π»Π°Ρ‚ΡŒ набросок, Π½ΠΎ ΠΌΡ‹ Π΄ΠΎΠ»ΠΆΠ½Ρ‹ ΠΏΡ€ΠΎΠ°Π½Π°Π»ΠΈΠ·ΠΈΡ€ΠΎΠ²Π°Ρ‚ΡŒ Π΅Π³ΠΎ, Ρ‡Ρ‚ΠΎΠ±Ρ‹ ΡƒΠ±Π΅Π΄ΠΈΡ‚ΡŒΡΡ, Ρ‡Ρ‚ΠΎ ΠΌΡ‹ ΠΏΠΎΠ½ΠΈΠΌΠ°Π΅ΠΌ, Ρ‡Ρ‚ΠΎ ΠΎΠ½ ΠΏΠΎΠΊΠ°Π·Ρ‹Π²Π°Π΅Ρ‚.ΠžΠ±Ρ€Π°Ρ‚ΠΈΡ‚Π΅ особоС Π²Π½ΠΈΠΌΠ°Π½ΠΈΠ΅ Π½Π° систСму ΠΊΠΎΠΎΡ€Π΄ΠΈΠ½Π°Ρ‚. Π§Ρ‚ΠΎΠ±Ρ‹ Π½Π°ΠΉΡ‚ΠΈ смСщСниС, ΠΌΡ‹ ΠΈΡΠΏΠΎΠ»ΡŒΠ·ΡƒΠ΅ΠΌ ΡƒΡ€Π°Π²Π½Π΅Π½ΠΈΠ΅ Ξ” x = x f x 0 . Π­Ρ‚ΠΎ просто, ΠΏΠΎΡΠΊΠΎΠ»ΡŒΠΊΡƒ Π΄Π°Π½Ρ‹ Π½Π°Ρ‡Π°Π»ΡŒΠ½Π°Ρ ΠΈ конСчная ΠΏΠΎΠ·ΠΈΡ†ΠΈΠΈ.

РСшСниС

1. ΠžΠΏΡ€Π΅Π΄Π΅Π»ΠΈΡ‚Π΅ извСстныС. На рисункС ΠΌΡ‹ Π²ΠΈΠ΄ΠΈΠΌ, Ρ‡Ρ‚ΠΎ x f = 6,70 ΠΊΠΌ ΠΈ x 0 = 4,70 ΠΊΠΌ для части (a), ΠΈ x β€² f = 3,75 ΠΊΠΌ ΠΈ x β€² 0 = 5.25 ΠΊΠΌ ΠΏΠΎ части (Π±).

2. НайдитС смСщСниС Π² части (Π°).

[латСкс] \ Delta x = {x} _ {f} — {x} _ {0} = 6,70 \ text {km} -4,70 \ text {km} = \ text {+} 2,00 \ text {km} [ / латСкс]

3. НайдитС смСщСниС Π² части (b).

[латСкс] \ Delta x β€² = {x β€²} _ {f} — {x β€²} _ {0} = \ text {3,75 ΠΊΠΌ} — \ text {5,25 ΠΊΠΌ} = — \ text {1,50 ΠΊΠΌ} [/ латСкс]

ΠžΠ±ΡΡƒΠΆΠ΄Π΅Π½ΠΈΠ΅

НаправлСниС двиТСния Π² (a) — Π²ΠΏΡ€Π°Π²ΠΎ, поэтому Π΅Π³ΠΎ смСщСниС ΠΈΠΌΠ΅Π΅Ρ‚ ΠΏΠΎΠ»ΠΎΠΆΠΈΡ‚Π΅Π»ΡŒΠ½Ρ‹ΠΉ Π·Π½Π°ΠΊ, Ρ‚ΠΎΠ³Π΄Π° ΠΊΠ°ΠΊ Π΄Π²ΠΈΠΆΠ΅Π½ΠΈΠ΅ Π² (b) — Π²Π»Π΅Π²ΠΎ ΠΈ, ΡΠ»Π΅Π΄ΠΎΠ²Π°Ρ‚Π΅Π»ΡŒΠ½ΠΎ, ΠΈΠΌΠ΅Π΅Ρ‚ ΠΎΡ‚Ρ€ΠΈΡ†Π°Ρ‚Π΅Π»ΡŒΠ½Ρ‹ΠΉ Π·Π½Π°ΠΊ.

ΠŸΡ€ΠΈΠΌΠ΅Ρ€ 3. Π‘Ρ€Π°Π²Π½Π΅Π½ΠΈΠ΅ ΠΏΡ€ΠΎΠΉΠ΄Π΅Π½Π½ΠΎΠ³ΠΎ расстояния ΠΈ пСрСмСщСния: ΠΏΠΎΠ΅Π·Π΄ ΠΌΠ΅Ρ‚Ρ€ΠΎ

КакиС расстояния проходят Π·Π° двиТСния, ΠΏΠΎΠΊΠ°Π·Π°Π½Π½Ρ‹Π΅ Π² частях (a) ΠΈ (b) ΠΏΠΎΠ΅Π·Π΄Π° ΠΌΠ΅Ρ‚Ρ€ΠΎ Π½Π° РисункС 7?

БтратСгия

Π§Ρ‚ΠΎΠ±Ρ‹ ΠΎΡ‚Π²Π΅Ρ‚ΠΈΡ‚ΡŒ Π½Π° этот вопрос, ΠΏΠΎΠ΄ΡƒΠΌΠ°ΠΉΡ‚Π΅ ΠΎΠ± опрСдСлСниях расстояния ΠΈ ΠΏΡ€ΠΎΠΉΠ΄Π΅Π½Π½ΠΎΠ³ΠΎ расстояния ΠΈ ΠΎ Ρ‚ΠΎΠΌ, ΠΊΠ°ΠΊ ΠΎΠ½ΠΈ связаны с ΠΏΠ΅Ρ€Π΅ΠΌΠ΅Ρ‰Π΅Π½ΠΈΠ΅ΠΌ. РасстояниС ΠΌΠ΅ΠΆΠ΄Ρƒ двумя полоТСниями опрСдСляСтся ΠΊΠ°ΠΊ Π²Π΅Π»ΠΈΡ‡ΠΈΠ½Π° смСщСния, которая Π±Ρ‹Π»Π° Π½Π°ΠΉΠ΄Π΅Π½Π° Π² ΠŸΡ€ΠΈΠΌΠ΅Ρ€Π΅ 1.ΠŸΡ€ΠΎΠΉΠ΄Π΅Π½Π½ΠΎΠ΅ расстояниС — это общая Π΄Π»ΠΈΠ½Π° ΠΏΡƒΡ‚ΠΈ, ΠΏΡ€ΠΎΠΉΠ΄Π΅Π½Π½ΠΎΠ³ΠΎ ΠΌΠ΅ΠΆΠ΄Ρƒ двумя позициями. (Π‘ΠΌ. Π‘ΠΌΠ΅Ρ‰Π΅Π½ΠΈΠ΅.) Π’ случаС ΠΏΠΎΠ΅Π·Π΄Π° ΠΌΠ΅Ρ‚Ρ€ΠΎ, ​​показанного Π½Π° рисункС 7, ΠΏΡ€ΠΎΠΉΠ΄Π΅Π½Π½ΠΎΠ΅ расстояниС Ρ€Π°Π²Π½ΠΎ Ρ€Π°ΡΡΡ‚ΠΎΡΠ½ΠΈΡŽ ΠΌΠ΅ΠΆΠ΄Ρƒ Π½Π°Ρ‡Π°Π»ΡŒΠ½Ρ‹ΠΌ ΠΈ ΠΊΠΎΠ½Π΅Ρ‡Π½Ρ‹ΠΌ полоТСниями ΠΏΠΎΠ΅Π·Π΄Π°.

РСшСниС

1. Π‘ΠΌΠ΅Ρ‰Π΅Π½ΠΈΠ΅ для части (Π°) составило +2,00 ΠΊΠΌ. Π’Π°ΠΊΠΈΠΌ ΠΎΠ±Ρ€Π°Π·ΠΎΠΌ, расстояниС ΠΌΠ΅ΠΆΠ΄Ρƒ Π½Π°Ρ‡Π°Π»ΡŒΠ½ΠΎΠΉ ΠΈ ΠΊΠΎΠ½Π΅Ρ‡Π½ΠΎΠΉ позициями составило 2,00 ΠΊΠΌ, Π° ΠΏΡ€ΠΎΠΉΠ΄Π΅Π½Π½ΠΎΠ΅ расстояниС — 2,00 ΠΊΠΌ.

2. Π‘ΠΌΠ΅Ρ‰Π΅Π½ΠΈΠ΅ для части (b) Π±Ρ‹Π»ΠΎ -1.5 ΠΊΠΌ. Π’Π°ΠΊΠΈΠΌ ΠΎΠ±Ρ€Π°Π·ΠΎΠΌ, расстояниС ΠΌΠ΅ΠΆΠ΄Ρƒ Π½Π°Ρ‡Π°Π»ΡŒΠ½ΠΎΠΉ ΠΈ ΠΊΠΎΠ½Π΅Ρ‡Π½ΠΎΠΉ позициями составляло 1,50 ΠΊΠΌ, Π° ΠΏΡ€ΠΎΠΉΠ΄Π΅Π½Π½ΠΎΠ΅ расстояниС — 1,50 ΠΊΠΌ.

ΠžΠ±ΡΡƒΠΆΠ΄Π΅Π½ΠΈΠ΅

РасстояниС — скаляр. Π£ Π½Π΅Π³ΠΎ Π΅ΡΡ‚ΡŒ Π²Π΅Π»ΠΈΡ‡ΠΈΠ½Π°, Π½ΠΎ Π½Π΅Ρ‚ Π·Π½Π°ΠΊΠ°, ΡƒΠΊΠ°Π·Ρ‹Π²Π°ΡŽΡ‰Π΅Π³ΠΎ Π½Π°ΠΏΡ€Π°Π²Π»Π΅Π½ΠΈΠ΅.

ΠŸΡ€ΠΈΠΌΠ΅Ρ€ 4. РасчСт ускорСния: ΠΏΠΎΠ΅Π·Π΄ ΠΌΠ΅Ρ‚Ρ€ΠΎ Π½Π°Π±ΠΈΡ€Π°Π΅Ρ‚ ΡΠΊΠΎΡ€ΠΎΡΡ‚ΡŒ

ΠŸΡ€Π΅Π΄ΠΏΠΎΠ»ΠΎΠΆΠΈΠΌ, Ρ‡Ρ‚ΠΎ ΠΏΠΎΠ΅Π·Π΄ Π½Π° рис. 7 (Π°) ускоряСтся ΠΈΠ· состояния покоя Π΄ΠΎ 30,0 ΠΊΠΌ / Ρ‡ Π·Π° ΠΏΠ΅Ρ€Π²Ρ‹Π΅ 20,0 с своСго двиТСния. Каково Π΅Π³ΠΎ срСднСС ускорСниС Π·Π° этот ΠΏΡ€ΠΎΠΌΠ΅ΠΆΡƒΡ‚ΠΎΠΊ Π²Ρ€Π΅ΠΌΠ΅Π½ΠΈ?

БтратСгия

Π—Π΄Π΅ΡΡŒ стоит ΡΠ΄Π΅Π»Π°Ρ‚ΡŒ простой набросок:

РСшСниС

1.ΠžΠΏΡ€Π΅Π΄Π΅Π»ΠΈΡ‚Π΅ извСстныС. v 0 = 0 (ΠΏΠΎΠ΅Π·Π΄Π° Π·Π°ΠΏΡƒΡΠΊΠ°ΡŽΡ‚ΡΡ Π² состоянии покоя), v f = 30,0 ΠΊΠΌ / Ρ‡ ΠΈ Ξ” t = 20,0 с.

2. ВычислитС Ξ” v . ΠŸΠΎΡΠΊΠΎΠ»ΡŒΠΊΡƒ ΠΏΠΎΠ΅Π·Π΄ стартуСт ΠΈΠ· состояния покоя, Π΅Π³ΠΎ ΡΠΊΠΎΡ€ΠΎΡΡ‚ΡŒ измСняСтся Π½Π° [latex] \ Delta v \ text {=} \ text {+} \ text {30,0 ΠΊΠΌ / Ρ‡} [/ latex], Π³Π΄Π΅ Π·Π½Π°ΠΊ плюс ΠΎΠ·Π½Π°Ρ‡Π°Π΅Ρ‚ ΡΠΊΠΎΡ€ΠΎΡΡ‚ΡŒ Π²ΠΏΡ€Π°Π²ΠΎ. .

3. ΠŸΠΎΠ΄ΡΡ‚Π°Π²ΡŒΡ‚Π΅ извСстныС значСния ΠΈ Ρ€Π΅ΡˆΠΈΡ‚Π΅ нСизвСстноС, [latex] \ bar {a} [/ latex].

[латСкс] \ bar {a} = \ frac {\ Delta v} {\ Delta t} = \ frac {+ \ text {30.{2} [/ латСкс]

ΠžΠ±ΡΡƒΠΆΠ΄Π΅Π½ΠΈΠ΅

Π—Π½Π°ΠΊ плюс ΠΎΠ·Π½Π°Ρ‡Π°Π΅Ρ‚, Ρ‡Ρ‚ΠΎ ускорСниС Π½Π°ΠΏΡ€Π°Π²ΠΎ. Π­Ρ‚ΠΎ Ρ€Π°Π·ΡƒΠΌΠ½ΠΎ, ΠΏΠΎΡ‚ΠΎΠΌΡƒ Ρ‡Ρ‚ΠΎ ΠΏΠΎΠ΅Π·Π΄ стартуСт ΠΈΠ· состояния покоя ΠΈ Π·Π°ΠΊΠ°Π½Ρ‡ΠΈΠ²Π°Π΅Ρ‚ со ΡΠΊΠΎΡ€ΠΎΡΡ‚ΡŒΡŽ Π²ΠΏΡ€Π°Π²ΠΎ (Ρ‚ΠΎΠΆΠ΅ ΠΏΠΎΠ»ΠΎΠΆΠΈΡ‚Π΅Π»ΡŒΠ½ΠΎΠΉ). Π’Π°ΠΊΠΈΠΌ ΠΎΠ±Ρ€Π°Π·ΠΎΠΌ, ускорСниС происходит Π² Ρ‚ΠΎΠΌ ΠΆΠ΅ Π½Π°ΠΏΡ€Π°Π²Π»Π΅Π½ΠΈΠΈ, Ρ‡Ρ‚ΠΎ ΠΈ ΠΈΠ·ΠΌΠ΅Π½Π΅Π½ΠΈΠ΅ скорости Π½Π° , ΠΊΠ°ΠΊ всСгда.

ΠŸΡ€ΠΈΠΌΠ΅Ρ€ 5. РасчСт ускорСния: Π·Π°ΠΌΠ΅Π΄Π»Π΅Π½ΠΈΠ΅ ΠΏΠΎΠ΅Π·Π΄Π° ΠΌΠ΅Ρ‚Ρ€ΠΎ

Π’Π΅ΠΏΠ΅Ρ€ΡŒ ΠΏΡ€Π΅Π΄ΠΏΠΎΠ»ΠΎΠΆΠΈΠΌ, Ρ‡Ρ‚ΠΎ Π² ΠΊΠΎΠ½Ρ†Π΅ ΠΏΠΎΠ΅Π·Π΄ΠΊΠΈ ΠΏΠΎΠ΅Π·Π΄ Π½Π° РисункС 7 (Π°) замСдляСтся Π΄ΠΎ остановки со скорости 30.0 ΠΊΠΌ / Ρ‡ Π·Π° 8.00 с. КакоС Ρƒ Π½Π΅Π³ΠΎ срСднСС ускорСниС ΠΏΡ€ΠΈ остановкС?

БтратСгия
РСшСниС

1. ΠžΠΏΡ€Π΅Π΄Π΅Π»ΠΈΡ‚Π΅ извСстныС. v 0 = 30,0 ΠΊΠΌ / Ρ‡, v f = 0 ΠΊΠΌ / Ρ‡ (ΠΏΠΎΠ΅Π·Π΄ остановлСн, поэтому Π΅Π³ΠΎ ΡΠΊΠΎΡ€ΠΎΡΡ‚ΡŒ Ρ€Π°Π²Π½Π° 0), ΠΈ Ξ” t = 8,00 с.

2. НайдитС ΠΈΠ·ΠΌΠ΅Π½Π΅Π½ΠΈΠ΅ скорости Ξ” v .

Ξ” v = v f v 0 = 0-30.{2} \ text {.} [/ Latex]

ΠžΠ±ΡΡƒΠΆΠ΄Π΅Π½ΠΈΠ΅

Π—Π½Π°ΠΊ минус ΡƒΠΊΠ°Π·Ρ‹Π²Π°Π΅Ρ‚ Π½Π° Ρ‚ΠΎ, Ρ‡Ρ‚ΠΎ ускорСниС происходит Π²Π»Π΅Π²ΠΎ. Π­Ρ‚ΠΎΡ‚ Π·Π½Π°ΠΊ Ρ€Π°Π·ΡƒΠΌΠ΅Π½, ΠΏΠΎΡ‚ΠΎΠΌΡƒ Ρ‡Ρ‚ΠΎ ΠΏΠΎΠ΅Π·Π΄ ΠΈΠ·Π½Π°Ρ‡Π°Π»ΡŒΠ½ΠΎ ΠΈΠΌΠ΅Π΅Ρ‚ ΠΏΠΎΠ»ΠΎΠΆΠΈΡ‚Π΅Π»ΡŒΠ½ΡƒΡŽ ΡΠΊΠΎΡ€ΠΎΡΡ‚ΡŒ Π² этой Π·Π°Π΄Π°Ρ‡Π΅, Π° ΠΎΡ‚Ρ€ΠΈΡ†Π°Ρ‚Π΅Π»ΡŒΠ½ΠΎΠ΅ ускорСниС Π±ΡƒΠ΄Π΅Ρ‚ ΠΏΡ€Π΅ΠΏΡΡ‚ΡΡ‚Π²ΠΎΠ²Π°Ρ‚ΡŒ двиТСнию. ΠžΠΏΡΡ‚ΡŒ ΠΆΠ΅, ускорСниС происходит Π² Ρ‚ΠΎΠΌ ΠΆΠ΅ Π½Π°ΠΏΡ€Π°Π²Π»Π΅Π½ΠΈΠΈ, Ρ‡Ρ‚ΠΎ ΠΈ ΠΈΠ·ΠΌΠ΅Π½Π΅Π½ΠΈΠ΅ скорости Π½Π° , ΠΊΠΎΡ‚ΠΎΡ€ΠΎΠ΅ здСсь ΠΎΡ‚Ρ€ΠΈΡ†Π°Ρ‚Π΅Π»ΡŒΠ½ΠΎ. Π­Ρ‚ΠΎ ускорСниС ΠΌΠΎΠΆΠ½ΠΎ Π½Π°Π·Π²Π°Ρ‚ΡŒ Π·Π°ΠΌΠ΅Π΄Π»Π΅Π½ΠΈΠ΅ΠΌ, ΠΏΠΎΡ‚ΠΎΠΌΡƒ Ρ‡Ρ‚ΠΎ ΠΎΠ½ΠΎ ΠΈΠΌΠ΅Π΅Ρ‚ Π½Π°ΠΏΡ€Π°Π²Π»Π΅Π½ΠΈΠ΅, ΠΏΡ€ΠΎΡ‚ΠΈΠ²ΠΎΠΏΠΎΠ»ΠΎΠΆΠ½ΠΎΠ΅ скорости.

Π“Ρ€Π°Ρ„ΠΈΠΊΠΈ полоТСния, скорости ΠΈ ускорСния отВрСмя для ΠΏΠΎΠ΅Π·Π΄ΠΎΠ² Π² ΠŸΡ€ΠΈΠΌΠ΅Ρ€Π΅ 4 ΠΈ ΠŸΡ€ΠΈΠΌΠ΅Ρ€Π΅ 5 ΠΏΠΎΠΊΠ°Π·Π°Π½ΠΎ Π½Π° рисункС 10. (ΠœΡ‹ приняли ΡΠΊΠΎΡ€ΠΎΡΡ‚ΡŒ постоянной ΠΎΡ‚ 20 Π΄ΠΎ 40 с, послС Ρ‡Π΅Π³ΠΎ ΠΏΠΎΠ΅Π·Π΄ замСдляСтся.)

ΠŸΡ€ΠΈΠΌΠ΅Ρ€ 6. РасчСт срСднСй скорости: ΠΏΠΎΠ΅Π·Π΄ ΠΌΠ΅Ρ‚Ρ€ΠΎ

Какова срСдняя ΡΠΊΠΎΡ€ΠΎΡΡ‚ΡŒ ΠΏΠΎΠ΅Π·Π΄Π° Π² части b ΠΏΡ€ΠΈΠΌΠ΅Ρ€Π° 2, снова ΠΏΠΎΠΊΠ°Π·Π°Π½Π½ΠΎΠΉ Π½ΠΈΠΆΠ΅, Ссли ΠΏΠΎΠ΅Π·Π΄ΠΊΠ° Π·Π°Π½ΠΈΠΌΠ°Π΅Ρ‚ 5,00 ΠΌΠΈΠ½ΡƒΡ‚?

БтратСгия

БрСдняя ΡΠΊΠΎΡ€ΠΎΡΡ‚ΡŒ — это смСщСниС, Ρ€Π°Π·Π΄Π΅Π»Π΅Π½Π½ΠΎΠ΅ Π½Π° врСмя. Π—Π΄Π΅ΡΡŒ ΠΎΠ½ Π±ΡƒΠ΄Π΅Ρ‚ ΠΎΡ‚Ρ€ΠΈΡ†Π°Ρ‚Π΅Π»ΡŒΠ½Ρ‹ΠΌ, Ρ‚Π°ΠΊ ΠΊΠ°ΠΊ ΠΏΠΎΠ΅Π·Π΄ двиТСтся Π²Π»Π΅Π²ΠΎ ΠΈ ΠΈΠΌΠ΅Π΅Ρ‚ ΠΎΡ‚Ρ€ΠΈΡ†Π°Ρ‚Π΅Π»ΡŒΠ½ΠΎΠ΅ смСщСниС.

РСшСниС

1. ΠžΠΏΡ€Π΅Π΄Π΅Π»ΠΈΡ‚Π΅ извСстныС. x β€² f = 3,75 ΠΊΠΌ, x β€² 0 = 5,25 ΠΊΠΌ, Ξ” t = 5,00 ΠΌΠΈΠ½.

2. ΠžΠΏΡ€Π΅Π΄Π΅Π»ΠΈΡ‚Π΅ смСщСниС Ξ” x β€². Π’ ΠΏΡ€ΠΈΠΌΠ΅Ρ€Π΅ 2 ΠΌΡ‹ ΠΎΠ±Π½Π°Ρ€ΡƒΠΆΠΈΠ»ΠΈ, Ρ‡Ρ‚ΠΎ Ξ” x β€² составляСт βˆ’1,5 ΠΊΠΌ.

3. НайдитС ΡΡ€Π΅Π΄Π½ΡŽΡŽ ΡΠΊΠΎΡ€ΠΎΡΡ‚ΡŒ.

[латСкс] \ bar {v} = \ frac {\ Delta x β€²} {\ Delta t} = \ frac {- \ text {1,50 ΠΊΠΌ}} {\ text {5,00 ΠΌΠΈΠ½}} [/ latex]

4. ΠŸΠ΅Ρ€Π΅Π²Π΅ΡΡ‚ΠΈ Π΅Π΄ΠΈΠ½ΠΈΡ†Ρ‹.

[латСкс] \ bar {v} = \ frac {\ Delta x β€²} {\ Delta t} = \ left (\ frac {-1 \ text {.} \ text {50 ΠΊΠΌ}} {5 \ text {.} \ text {00 min}} \ right) \ left (\ frac {\ text {60 min}} {1 h} \ right) = — \ text { 18} \ text {.0 ΠΊΠΌ / Ρ‡} [/ latex]

ΠžΠ±ΡΡƒΠΆΠ΄Π΅Π½ΠΈΠ΅

ΠžΡ‚Ρ€ΠΈΡ†Π°Ρ‚Π΅Π»ΡŒΠ½Π°Ρ ΡΠΊΠΎΡ€ΠΎΡΡ‚ΡŒ ΡƒΠΊΠ°Π·Ρ‹Π²Π°Π΅Ρ‚ Π½Π° Π΄Π²ΠΈΠΆΠ΅Π½ΠΈΠ΅ Π²Π»Π΅Π²ΠΎ.

ΠŸΡ€ΠΈΠΌΠ΅Ρ€ 7. РасчСт замСдлСния: ΠΏΠΎΠ΅Π·Π΄ ΠΌΠ΅Ρ‚Ρ€ΠΎ

НаконСц, ΠΏΡ€Π΅Π΄ΠΏΠΎΠ»ΠΎΠΆΠΈΠΌ, Ρ‡Ρ‚ΠΎ ΠΏΠΎΠ΅Π·Π΄ Π½Π° РисункС 2 замСдляСтся Π΄ΠΎ остановки со скорости 20,0 ΠΊΠΌ / Ρ‡ Π·Π° 10,0 с. КакоС Ρƒ Π½Π΅Π³ΠΎ срСднСС ускорСниС?

БтратСгия

Π•Ρ‰Π΅ Ρ€Π°Π· нарисуСм набросок:

Как ΠΈ Ρ€Π°Π½ΡŒΡˆΠ΅, ΠΌΡ‹ Π΄ΠΎΠ»ΠΆΠ½Ρ‹ Π½Π°ΠΉΡ‚ΠΈ ΠΈΠ·ΠΌΠ΅Π½Π΅Π½ΠΈΠ΅ скорости ΠΈ ΠΈΠ·ΠΌΠ΅Π½Π΅Π½ΠΈΠ΅ Π²ΠΎ Π²Ρ€Π΅ΠΌΠ΅Π½ΠΈ, Ρ‡Ρ‚ΠΎΠ±Ρ‹ Π²Ρ‹Ρ‡ΠΈΡΠ»ΠΈΡ‚ΡŒ срСднСС ускорСниС.

РСшСниС

1. ΠžΠΏΡ€Π΅Π΄Π΅Π»ΠΈΡ‚Π΅ извСстныС. v 0 = βˆ’20 ΠΊΠΌ / Ρ‡, v f = 0 ΠΊΠΌ / Ρ‡, Ξ” t = 10,0 с.

2. ВычислитС Ξ” v . ИзмСнСниС скорости здСсь Π΄Π΅ΠΉΡΡ‚Π²ΠΈΡ‚Π΅Π»ΡŒΠ½ΠΎ ΠΏΠΎΠ»ΠΎΠΆΠΈΡ‚Π΅Π»ΡŒΠ½ΠΎΠ΅, Ρ‚Π°ΠΊ ΠΊΠ°ΠΊ

[латСкс] \ Delta v = {v} _ {f} — {v} _ {0} = 0- \ left (- \ text {20 ΠΊΠΌ / Ρ‡} \ right) \ text {=} \ phantom {\ ΠΏΡ€Π°Π²ΠΈΠ»ΠΎ {0.25} {0ex}} \ text {+} \ text {20 ΠΊΠΌ / Ρ‡} [/ latex]

3. Π Π΅ΡˆΠΈΡ‚Π΅ для [латСкс] \ bar {a} [/ latex].

[латСкс] \ bar {a} = \ frac {\ Delta v} {\ Delta t} = \ frac {+ \ text {20} \ text {.{2} [/ латСкс]

ΠžΠ±ΡΡƒΠΆΠ΄Π΅Π½ΠΈΠ΅

Π—Π½Π°ΠΊ плюс ΠΎΠ·Π½Π°Ρ‡Π°Π΅Ρ‚, Ρ‡Ρ‚ΠΎ ускорСниС Π½Π°ΠΏΡ€Π°Π²ΠΎ. Π­Ρ‚ΠΎ Ρ€Π°Π·ΡƒΠΌΠ½ΠΎ, ΠΏΠΎΡ‚ΠΎΠΌΡƒ Ρ‡Ρ‚ΠΎ ΠΏΠΎΠ΅Π·Π΄ ΠΈΠ·Π½Π°Ρ‡Π°Π»ΡŒΠ½ΠΎ ΠΈΠΌΠ΅Π΅Ρ‚ ΠΎΡ‚Ρ€ΠΈΡ†Π°Ρ‚Π΅Π»ΡŒΠ½ΡƒΡŽ ΡΠΊΠΎΡ€ΠΎΡΡ‚ΡŒ (слСва) Π² этой Π·Π°Π΄Π°Ρ‡Π΅, Π° ΠΏΠΎΠ»ΠΎΠΆΠΈΡ‚Π΅Π»ΡŒΠ½ΠΎΠ΅ ускорСниС противодСйствуСт двиТСнию (Ρ‚ΠΎ Π΅ΡΡ‚ΡŒ справа). ΠžΠΏΡΡ‚ΡŒ ΠΆΠ΅, ускорСниС происходит Π² Ρ‚ΠΎΠΌ ΠΆΠ΅ Π½Π°ΠΏΡ€Π°Π²Π»Π΅Π½ΠΈΠΈ, Ρ‡Ρ‚ΠΎ ΠΈ ΠΈΠ·ΠΌΠ΅Π½Π΅Π½ΠΈΠ΅ скорости Π½Π° , Ρ‡Ρ‚ΠΎ здСсь ΠΏΠΎΠ»ΠΎΠΆΠΈΡ‚Π΅Π»ΡŒΠ½ΠΎ. Как ΠΈ Π² ΠΏΡ€ΠΈΠΌΠ΅Ρ€Π΅ 5, это ускорСниС ΠΌΠΎΠΆΠ½ΠΎ Π½Π°Π·Π²Π°Ρ‚ΡŒ Π·Π°ΠΌΠ΅Π΄Π»Π΅Π½ΠΈΠ΅ΠΌ, ΠΏΠΎΡΠΊΠΎΠ»ΡŒΠΊΡƒ ΠΎΠ½ΠΎ происходит Π² Π½Π°ΠΏΡ€Π°Π²Π»Π΅Π½ΠΈΠΈ, ΠΏΡ€ΠΎΡ‚ΠΈΠ²ΠΎΠΏΠΎΠ»ΠΎΠΆΠ½ΠΎΠΌ скорости.

ΠŸΠΎΠΆΠ°Π»ΡƒΠΉ, самоС Π²Π°ΠΆΠ½ΠΎΠ΅, Ρ‡Ρ‚ΠΎ Π½ΡƒΠΆΠ½ΠΎ ΠΎΡ‚ΠΌΠ΅Ρ‚ΠΈΡ‚ΡŒ Π² этих ΠΏΡ€ΠΈΠΌΠ΅Ρ€Π°Ρ…, — это Π·Π½Π°ΠΊΠΈ ΠΎΡ‚Π²Π΅Ρ‚ΠΎΠ². Π’ Π²Ρ‹Π±Ρ€Π°Π½Π½ΠΎΠΉ Π½Π°ΠΌΠΈ систСмС ΠΊΠΎΠΎΡ€Π΄ΠΈΠ½Π°Ρ‚ плюс ΠΎΠ·Π½Π°Ρ‡Π°Π΅Ρ‚, Ρ‡Ρ‚ΠΎ Π²Π΅Π»ΠΈΡ‡ΠΈΠ½Π° находится справа, Π° минус — слСва. Π­Ρ‚ΠΎ Π»Π΅Π³ΠΊΠΎ ΠΏΡ€Π΅Π΄ΡΡ‚Π°Π²ΠΈΡ‚ΡŒ для смСщСния ΠΈ скорости. Но для Ρ€Π°Π·Π³ΠΎΠ½Π° это Π½Π΅ΠΌΠ½ΠΎΠ³ΠΎ ΠΌΠ΅Π½Π΅Π΅ ΠΎΡ‡Π΅Π²ΠΈΠ΄Π½ΠΎ. Π‘ΠΎΠ»ΡŒΡˆΠΈΠ½ΡΡ‚Π²ΠΎ людСй ΠΈΠ½Ρ‚Π΅Ρ€ΠΏΡ€Π΅Ρ‚ΠΈΡ€ΡƒΡŽΡ‚ ΠΎΡ‚Ρ€ΠΈΡ†Π°Ρ‚Π΅Π»ΡŒΠ½ΠΎΠ΅ ускорСниС ΠΊΠ°ΠΊ Π·Π°ΠΌΠ΅Π΄Π»Π΅Π½ΠΈΠ΅ ΠΎΠ±ΡŠΠ΅ΠΊΡ‚Π°. Π­Ρ‚ΠΎΠ³ΠΎ Π½Π΅ Π±Ρ‹Π»ΠΎ Π² ΠŸΡ€ΠΈΠΌΠ΅Ρ€Π΅ 2, Π³Π΄Π΅ ΠΏΠΎΠ»ΠΎΠΆΠΈΡ‚Π΅Π»ΡŒΠ½ΠΎΠ΅ ускорСниС замСдляло ΠΎΡ‚Ρ€ΠΈΡ†Π°Ρ‚Π΅Π»ΡŒΠ½ΡƒΡŽ ΡΠΊΠΎΡ€ΠΎΡΡ‚ΡŒ. ΠšΠ»ΡŽΡ‡Π΅Π²Ρ‹ΠΌ ΠΎΡ‚Π»ΠΈΡ‡ΠΈΠ΅ΠΌ Π±Ρ‹Π»ΠΎ Ρ‚ΠΎ, Ρ‡Ρ‚ΠΎ ускорСниС происходило Π² Π½Π°ΠΏΡ€Π°Π²Π»Π΅Π½ΠΈΠΈ, ΠΏΡ€ΠΎΡ‚ΠΈΠ²ΠΎΠΏΠΎΠ»ΠΎΠΆΠ½ΠΎΠΌ скорости.ЀактичСски, ΠΎΡ‚Ρ€ΠΈΡ†Π°Ρ‚Π΅Π»ΡŒΠ½ΠΎΠ΅ ускорСниС ΡƒΠ²Π΅Π»ΠΈΡ‡ΠΈΡ‚ ΠΎΡ‚Ρ€ΠΈΡ†Π°Ρ‚Π΅Π»ΡŒΠ½ΡƒΡŽ ΡΠΊΠΎΡ€ΠΎΡΡ‚ΡŒ. НапримСр, ΠΏΠΎΠ΅Π·Π΄, двиТущийся Π²Π»Π΅Π²ΠΎ Π½Π° рисункС 11, ускоряСтся Π·Π° счСт ускорСния Π²Π»Π΅Π²ΠΎ. Π’ этом случаС ΠΈ v , ΠΈ a ΠΎΡ‚Ρ€ΠΈΡ†Π°Ρ‚Π΅Π»ΡŒΠ½Ρ‹. Π—Π½Π°ΠΊΠΈ плюс ΠΈ минус ΡƒΠΊΠ°Π·Ρ‹Π²Π°ΡŽΡ‚ направлСния ускорСний. Если ускорСниС ΠΈΠΌΠ΅Π΅Ρ‚ Ρ‚ΠΎΡ‚ ΠΆΠ΅ Π·Π½Π°ΠΊ, Ρ‡Ρ‚ΠΎ ΠΈ ΠΈΠ·ΠΌΠ΅Π½Π΅Π½ΠΈΠ΅ скорости, ΠΎΠ±ΡŠΠ΅ΠΊΡ‚ ускоряСтся. Если ускорСниС ΠΈΠΌΠ΅Π΅Ρ‚ Π·Π½Π°ΠΊ, ΠΏΡ€ΠΎΡ‚ΠΈΠ²ΠΎΠΏΠΎΠ»ΠΎΠΆΠ½Ρ‹ΠΉ измСнСнию скорости, ΠΎΠ±ΡŠΠ΅ΠΊΡ‚ замСдляСтся.

ΠŸΡ€ΠΎΠ²Π΅Ρ€ΡŒΡ‚Π΅ своС ΠΏΠΎΠ½ΠΈΠΌΠ°Π½ΠΈΠ΅

Π‘Π°ΠΌΠΎΠ»Π΅Ρ‚ призСмляСтся Π½Π° Π²Π·Π»Π΅Ρ‚Π½ΠΎ-посадочной полосС, лСтящСй Π½Π° восток.ΠžΠΏΠΈΡˆΠΈΡ‚Π΅ Π΅Π³ΠΎ ускорСниС.

РСшСниС

Если ΠΏΡ€ΠΈΠ½ΡΡ‚ΡŒ восток Π·Π° ΠΏΠΎΠ»ΠΎΠΆΠΈΡ‚Π΅Π»ΡŒΠ½ΠΎΠ΅ Π·Π½Π°Ρ‡Π΅Π½ΠΈΠ΅, Ρ‚ΠΎ самолСт ΠΈΠΌΠ΅Π΅Ρ‚ ΠΎΡ‚Ρ€ΠΈΡ†Π°Ρ‚Π΅Π»ΡŒΠ½ΠΎΠ΅ ускорСниС, Ρ‚Π°ΠΊ ΠΊΠ°ΠΊ ΠΎΠ½ ускоряСтся Π² сторону Π·Π°ΠΏΠ°Π΄Π°. Он Ρ‚Π°ΠΊΠΆΠ΅ замСдляСтся: Π΅Π³ΠΎ ускорСниС ΠΏΡ€ΠΎΡ‚ΠΈΠ²ΠΎΠΏΠΎΠ»ΠΎΠΆΠ½ΠΎ Π΅Π³ΠΎ скорости.

ИсслСдования PhET: ΠΌΠΎΠ΄Π΅Π»ΠΈΡ€ΠΎΠ²Π°Π½ΠΈΠ΅ двиТущСгося Ρ‡Π΅Π»ΠΎΠ²Π΅ΠΊΠ°

Π£Π·Π½Π°ΠΉΡ‚Π΅ ΠΎ Π³Ρ€Π°Ρ„ΠΈΠΊΠ°Ρ… полоТСния, скорости ΠΈ ускорСния. ΠŸΠ΅Ρ€Π΅ΠΌΠ΅Ρ‰Π°ΠΉΡ‚Π΅ Ρ‡Π΅Π»ΠΎΠ²Π΅Ρ‡ΠΊΠ° Π²Π·Π°Π΄ ΠΈ Π²ΠΏΠ΅Ρ€Π΅Π΄ с ΠΏΠΎΠΌΠΎΡ‰ΡŒΡŽ ΠΌΡ‹ΡˆΠΈ ΠΈ Π½Π°ΠΌΠ΅Ρ‚ΡŒΡ‚Π΅ Π΅Π³ΠΎ Π΄Π²ΠΈΠΆΠ΅Π½ΠΈΠ΅. Π—Π°Π΄Π°ΠΉΡ‚Π΅ ΠΏΠΎΠ»ΠΎΠΆΠ΅Π½ΠΈΠ΅, ΡΠΊΠΎΡ€ΠΎΡΡ‚ΡŒ ΠΈΠ»ΠΈ ускорСниС ΠΈ ΠΏΠΎΠ·Π²ΠΎΠ»ΡŒΡ‚Π΅ симуляции ΠΏΠ΅Ρ€Π΅ΠΌΠ΅Ρ‰Π°Ρ‚ΡŒ Ρ‡Π΅Π»ΠΎΠ²Π΅ΠΊΠ° Π·Π° вас.

Π©Π΅Π»ΠΊΠ½ΠΈΡ‚Π΅, Ρ‡Ρ‚ΠΎΠ±Ρ‹ Π·Π°Π³Ρ€ΡƒΠ·ΠΈΡ‚ΡŒ ΡΠΈΠΌΡƒΠ»ΡΡ†ΠΈΡŽ. Π—Π°ΠΏΡƒΡΠΊΠ°Ρ‚ΡŒ Π½Π° Java.

Π‘Π²ΠΎΠ΄ΠΊΠ° Ρ€Π°Π·Π΄Π΅Π»Π°

ΠšΠΎΠ½Ρ†Π΅ΠΏΡ‚ΡƒΠ°Π»ΡŒΠ½Ρ‹Π΅ вопросы

1. Π’ΠΎΠ·ΠΌΠΎΠΆΠ½ΠΎ Π»ΠΈ, Ρ‡Ρ‚ΠΎΠ±Ρ‹ ΡΠΊΠΎΡ€ΠΎΡΡ‚ΡŒ ΠΎΡΡ‚Π°Π²Π°Π»Π°ΡΡŒ постоянной ΠΏΡ€ΠΈ Π½Π΅Π½ΡƒΠ»Π΅Π²ΠΎΠΌ ускорСнии? ΠŸΡ€ΠΈΠ²Π΅Π΄ΠΈΡ‚Π΅ ΠΏΡ€ΠΈΠΌΠ΅Ρ€ Ρ‚Π°ΠΊΠΎΠΉ ситуации.

2. Π’ΠΎΠ·ΠΌΠΎΠΆΠ½ΠΎ Π»ΠΈ, Ρ‡Ρ‚ΠΎΠ±Ρ‹ ΡΠΊΠΎΡ€ΠΎΡΡ‚ΡŒ Π±Ρ‹Π»Π° постоянной, Ссли ускорСниС Π½Π΅ Ρ€Π°Π²Π½ΠΎ Π½ΡƒΠ»ΡŽ? ΠžΠ±ΡŠΡΡΠ½ΡΡ‚ΡŒ.

3. ΠŸΡ€ΠΈΠ²Π΅Π΄ΠΈΡ‚Π΅ ΠΏΡ€ΠΈΠΌΠ΅Ρ€, Π² ΠΊΠΎΡ‚ΠΎΡ€ΠΎΠΌ ΡΠΊΠΎΡ€ΠΎΡΡ‚ΡŒ Ρ€Π°Π²Π½Π° Π½ΡƒΠ»ΡŽ, Π° ускорСниС — Π½Π΅Ρ‚.

4. Если ΠΏΠΎΠ΅Π·Π΄ ΠΌΠ΅Ρ‚Ρ€ΠΎ двиТСтся Π²Π»Π΅Π²ΠΎ (ΠΈΠΌΠ΅Π΅Ρ‚ ΠΎΡ‚Ρ€ΠΈΡ†Π°Ρ‚Π΅Π»ΡŒΠ½ΡƒΡŽ ΡΠΊΠΎΡ€ΠΎΡΡ‚ΡŒ), Π° Π·Π°Ρ‚Π΅ΠΌ останавливаСтся, Π² ΠΊΠ°ΠΊΠΎΠΌ Π½Π°ΠΏΡ€Π°Π²Π»Π΅Π½ΠΈΠΈ ΠΎΠ½ ускоряСтся? УскорСниС ΠΏΠΎΠ»ΠΎΠΆΠΈΡ‚Π΅Π»ΡŒΠ½ΠΎΠ΅ ΠΈΠ»ΠΈ ΠΎΡ‚Ρ€ΠΈΡ†Π°Ρ‚Π΅Π»ΡŒΠ½ΠΎΠ΅?

5.Π—Π½Π°ΠΊΠΈ плюс ΠΈ минус ΠΈΡΠΏΠΎΠ»ΡŒΠ·ΡƒΡŽΡ‚ΡΡ Π² ΠΎΠ΄Π½ΠΎΠΌΠ΅Ρ€Π½ΠΎΠΌ Π΄Π²ΠΈΠΆΠ΅Π½ΠΈΠΈ для обозначСния направлСния. Каков Π·Π½Π°ΠΊ ускорСния, ΡƒΠΌΠ΅Π½ΡŒΡˆΠ°ΡŽΡ‰Π΅Π³ΠΎ Π²Π΅Π»ΠΈΡ‡ΠΈΠ½Ρƒ ΠΎΡ‚Ρ€ΠΈΡ†Π°Ρ‚Π΅Π»ΡŒΠ½ΠΎΠΉ скорости? ΠŸΠΎΠ»ΠΎΠΆΠΈΡ‚Π΅Π»ΡŒΠ½ΠΎΠΉ скорости?

Π—Π°Π΄Π°Ρ‡ΠΈ ΠΈ упраТнСния

1. Π“Π΅ΠΏΠ°Ρ€Π΄ ΠΌΠΎΠΆΠ΅Ρ‚ Ρ€Π°Π·ΠΎΠ³Π½Π°Ρ‚ΡŒΡΡ ΠΎΡ‚ состояния покоя Π΄ΠΎ скорости 30,0 ΠΌ / с Π·Π° 7,00 с. КакоС Ρƒ Π½Π΅Π³ΠΎ ускорСниС?

2. ΠŸΡ€ΠΎΡ„Π΅ΡΡΠΈΠΎΠ½Π°Π»ΡŒΠ½ΠΎΠ΅ ΠΏΡ€ΠΈΠ»ΠΎΠΆΠ΅Π½ΠΈΠ΅. Π”ΠΎΠΊΡ‚ΠΎΡ€ Π”ΠΆΠΎΠ½ Пол Π‘Ρ‚Π°ΠΏΠΏ, ΠΎΡ„ΠΈΡ†Π΅Ρ€ Π’Π’Π‘ БША, ΠΈΠ·ΡƒΡ‡Π°Π» влияниС ΡΠΊΡΡ‚Ρ€Π΅ΠΌΠ°Π»ΡŒΠ½ΠΎΠ³ΠΎ замСдлСния Π½Π° чСловСчСскоС Ρ‚Π΅Π»ΠΎ.10 дСкабря 1954 Π³ΠΎΠ΄Π° Π‘Ρ‚Π°ΠΏΠΏ ΠΏΡ€ΠΎΠ΅Ρ…Π°Π» Π½Π° Ρ€Π°ΠΊΠ΅Ρ‚Π½Ρ‹Ρ… санях, Ρ€Π°Π·ΠΎΠ³Π½Π°Π²ΡˆΠΈΡΡŒ ΠΈΠ· состояния покоя Π΄ΠΎ максимальной скорости 282 ΠΌ / с (1015 ΠΊΠΌ / Ρ‡) Π·Π° 5,00 с, ΠΈ Π±Ρ‹Π» Ρ€Π΅Π·ΠΊΠΎ остановлСн всСго Π·Π° 1,40 с! ВычислитС Π΅Π³ΠΎ (Π°) ускорСниС ΠΈ (Π±) Π·Π°ΠΌΠ΅Π΄Π»Π΅Π½ΠΈΠ΅. Π’Ρ‹Ρ€Π°Π·ΠΈΡ‚Π΅ ΠΊΠ°ΠΆΠ΄ΠΎΠ΅ Π·Π½Π°Ρ‡Π΅Π½ΠΈΠ΅ ΠΊΡ€Π°Ρ‚Π½Ρ‹ΠΌ Π³ (9,80 ΠΌ / с 2 ), взяв Π΅Π³ΠΎ ΠΎΡ‚Π½ΠΎΡˆΠ΅Π½ΠΈΠ΅ ΠΊ ΡƒΡΠΊΠΎΡ€Π΅Π½ΠΈΡŽ свободного падСния.

3. ΠŸΠ°ΡΡΠ°ΠΆΠΈΡ€ Π²Ρ‹Π΅Π·ΠΆΠ°Π΅Ρ‚ Π½Π° машинС Π·Π°Π΄Π½ΠΈΠΌ Ρ…ΠΎΠ΄ΠΎΠΌ ΠΈΠ· Π³Π°Ρ€Π°ΠΆΠ° с ускорСниСм 1,40 ΠΌ / с 2 . (A) Бколько Π²Ρ€Π΅ΠΌΠ΅Π½ΠΈ Π΅ΠΌΡƒ Π½ΡƒΠΆΠ½ΠΎ, Ρ‡Ρ‚ΠΎΠ±Ρ‹ Π½Π°Π±Ρ€Π°Ρ‚ΡŒ ΡΠΊΠΎΡ€ΠΎΡΡ‚ΡŒ 2.00 ΠΌ / с? (b) Если ΠΎΠ½Π° Π·Π°Ρ‚Π΅ΠΌ Ρ‚ΠΎΡ€ΠΌΠΎΠ·ΠΈΡ‚ Π΄ΠΎ остановки Ρ‡Π΅Ρ€Π΅Π· 0,800 с, ΠΊΠ°ΠΊΠΎΠ²ΠΎ Π΅Π΅ Π·Π°ΠΌΠ΅Π΄Π»Π΅Π½ΠΈΠ΅?

4. ΠŸΡ€Π΅Π΄ΠΏΠΎΠ»ΠΎΠΆΠΈΠΌ, Ρ‡Ρ‚ΠΎ ΠΌΠ΅ΠΆΠΊΠΎΠ½Ρ‚ΠΈΠ½Π΅Π½Ρ‚Π°Π»ΡŒΠ½Π°Ρ баллистичСская Ρ€Π°ΠΊΠ΅Ρ‚Π° ΠΏΠ΅Ρ€Π΅Ρ…ΠΎΠ΄ΠΈΡ‚ ΠΈΠ· состояния покоя Π² ΡΡƒΠ±ΠΎΡ€Π±ΠΈΡ‚Π°Π»ΡŒΠ½ΡƒΡŽ ΡΠΊΠΎΡ€ΠΎΡΡ‚ΡŒ 6,50 ΠΊΠΌ / с Π·Π° 60,0 с (фактичСская ΡΠΊΠΎΡ€ΠΎΡΡ‚ΡŒ ΠΈ врСмя засСкрСчСны). Каково Π΅Π³ΠΎ срСднСС ускорСниС Π² ΠΌ / с 2 ΠΈ ΠΊΡ€Π°Ρ‚Π½ΠΎΠ΅ g (9,80 ΠΌ / с 2 ).

Глоссарий

ускорСниС:
ΡΠΊΠΎΡ€ΠΎΡΡ‚ΡŒ измСнСния скорости; ΠΈΠ·ΠΌΠ΅Π½Π΅Π½ΠΈΠ΅ скорости с Ρ‚Π΅Ρ‡Π΅Π½ΠΈΠ΅ΠΌ Π²Ρ€Π΅ΠΌΠ΅Π½ΠΈ
срСднСС ускорСниС:
ΠΈΠ·ΠΌΠ΅Π½Π΅Π½ΠΈΠ΅ скорости, Π΄Π΅Π»Π΅Π½Π½ΠΎΠ΅ Π½Π° врСмя, Π² Ρ‚Π΅Ρ‡Π΅Π½ΠΈΠ΅ ΠΊΠΎΡ‚ΠΎΡ€ΠΎΠ³ΠΎ ΠΎΠ½ΠΎ измСняСтся
ΠΌΠ³Π½ΠΎΠ²Π΅Π½Π½ΠΎΠ΅ ускорСниС:
ускорСниС Π² ΠΎΠΏΡ€Π΅Π΄Π΅Π»Π΅Π½Π½Ρ‹ΠΉ ΠΌΠΎΠΌΠ΅Π½Ρ‚ Π²Ρ€Π΅ΠΌΠ΅Π½ΠΈ
Π·Π°ΠΌΠ΅Π΄Π»Π΅Π½ΠΈΠ΅:
ускорСниС Π² Π½Π°ΠΏΡ€Π°Π²Π»Π΅Π½ΠΈΠΈ, ΠΏΡ€ΠΎΡ‚ΠΈΠ²ΠΎΠΏΠΎΠ»ΠΎΠΆΠ½ΠΎΠΌ скорости; ускорСниС, ΠΊΠΎΡ‚ΠΎΡ€ΠΎΠ΅ ΠΏΡ€ΠΈΠ²ΠΎΠ΄ΠΈΡ‚ ΠΊ ΡƒΠΌΠ΅Π½ΡŒΡˆΠ΅Π½ΠΈΡŽ скорости

Π˜Π·Π±Ρ€Π°Π½Π½Ρ‹Π΅ Ρ€Π΅ΡˆΠ΅Π½ΠΈΡ ΠΏΡ€ΠΎΠ±Π»Π΅ΠΌ ΠΈ упраТнСния

1.4,29 м / с 2

3. (а) 1,43 с (б) -2,50 м / с 2

Acceleration

ΠžΠΏΡ‚ΠΈΠΌΠΈΠ·Π°Ρ†ΠΈΡ Π»Π°Π·Π΅Ρ€Π½ΠΎΠ³ΠΎ ускорСния элСктронов

4 дСкабря 2020 Π³. — Π˜ΡΡΠ»Π΅Π΄ΠΎΠ²Π°Ρ‚Π΅Π»ΠΈ рассмотрСли характСристики ускорСния элСктронов Π² Π²Π°ΠΊΡƒΡƒΠΌΠ΅, Π²Ρ‹Π·Ρ‹Π²Π°Π΅ΠΌΠΎΠ³ΠΎ Π»Π°Π·Π΅Ρ€Π½Ρ‹ΠΌΠΈ ΠΈΠΌΠΏΡƒΠ»ΡŒΡΠ°ΠΌΠΈ максимальной мощности, достиТимыми сСгодня, Π² поисках ΠΊΠ»ΡŽΡ‡Π° ΠΊ максимальной чистой энСргии …


НС Ρ‚Π°ΠΊ быстро! ΠšΠΎΠ½Ρ‚Ρ€ΠΎΠ»ΡŒ скорости свСтовых ΠΏΡƒΠ»ΡŒ

Π”Π΅ΠΊΠ°Π±Ρ€ΡŒ14, 2020 — Π˜ΡΡΠ»Π΅Π΄ΠΎΠ²Π°Ρ‚Π΅Π»ΠΈ Ρ‚ΠΎΡ‡Π½ΠΎ ΠΈ ΠΏΡ€ΠΎΠΈΠ·Π²ΠΎΠ»ΡŒΠ½ΠΎ ΠΊΠΎΠ½Ρ‚Ρ€ΠΎΠ»ΠΈΡ€ΡƒΡŽΡ‚ ΡΠΊΠΎΡ€ΠΎΡΡ‚ΡŒ ΠΏΠΎΠ»Π΅Ρ‚Π° Π»Π΅Π³ΠΊΠΈΡ… ΠΏΡƒΠ»ΡŒ, открывая Π½ΠΎΠ²Ρ‹Π΅ возмоТности для оптичСских ΠΈ физичСских …


ΠžΠ±Π΅ΡΠΏΠ΅Ρ‡Π΅Π½ΠΈΠ΅ Π±ΠΎΠ»Π΅Π΅ Π΄Π»ΠΈΡ‚Π΅Π»ΡŒΠ½Ρ‹Ρ… космичСских ΠΏΠΎΠ»Π΅Ρ‚ΠΎΠ²

20 августа 2019 Π³. — ΠŸΠΎΠ΄Ρ€ΡƒΠ»ΠΈΠ²Π°ΡŽΡ‰Π΅Π΅ устройство Π₯ΠΎΠ»Π»Π° — это Π΄Π²ΠΈΠ³Π°Ρ‚Π΅Π»ΡŒΠ½Π°Ρ установка, которая часто ΠΈΡΠΏΠΎΠ»ΡŒΠ·ΡƒΠ΅Ρ‚ΡΡ космичСскими кораблями, Π²Ρ‹ΠΏΠΎΠ»Π½ΡΡŽΡ‰ΠΈΠΌΠΈ Π±ΠΎΠ»Π΅Π΅ Π΄Π»ΠΈΡ‚Π΅Π»ΡŒΠ½Ρ‹Π΅ миссии. НСдавнСС исслСдованиС ΠΏΠΎΠΊΠ°Π·Π°Π»ΠΎ, ΠΊΠ°ΠΊ ΠΌΠΎΠΆΠ½ΠΎ ΡƒΠ²Π΅Π»ΠΈΡ‡ΠΈΡ‚ΡŒ срок слуТбы этих систСм …


ΠœΠ΅Ρ‚ΠΎΠ΄Ρ‹ пропуска ΠΊΠ°ΠΌΠ½Π΅ΠΉ ΠΌΠΎΠ³ΡƒΡ‚ ΡƒΠ»ΡƒΡ‡ΡˆΠΈΡ‚ΡŒ Π²ΠΎΠ·Π²Ρ€Π°Ρ‰Π΅Π½ΠΈΠ΅ космичСских Π°ΠΏΠΏΠ°Ρ€Π°Ρ‚ΠΎΠ² Π² атмосфСру

Апр.20, 2021 — Π£Ρ‡Π΅Π½Ρ‹Π΅ выявили нСсколько ΠΊΠ»ΡŽΡ‡Π΅Π²Ρ‹Ρ… Ρ„Π°ΠΊΡ‚ΠΎΡ€ΠΎΠ², ΠΊΠΎΡ‚ΠΎΡ€Ρ‹Π΅ Π²Π»ΠΈΡΡŽΡ‚ Π½Π° количСство отскоков, ΠΊΠΎΡ‚ΠΎΡ€Ρ‹ΠΌ ΠΏΡ€Ρ‹Π³Π°Π΅Ρ‚ камСнь ΠΏΡ€ΠΈ ΡƒΠ΄Π°Ρ€Π΅ ΠΎ Π²ΠΎΠ΄Ρƒ. ИсслСдованиС Π²ΠΊΠ»ΡŽΡ‡Π°Π»ΠΎ тСорСтичСскоС ΠΌΠΎΠ΄Π΅Π»ΠΈΡ€ΠΎΠ²Π°Π½ΠΈΠ΅ ΠΈ ΡΠΊΡΠΏΠ΅Ρ€ΠΈΠΌΠ΅Π½Ρ‚Π°Π»ΡŒΠ½ΡƒΡŽ установку …


ΠœΠΈΡ€ΠΎΠ²ΠΎΠΉ Ρ€Π΅ΠΊΠΎΡ€Π΄: ΠΏΠ»Π°Π·ΠΌΠ΅Π½Π½Ρ‹ΠΉ ΡƒΡΠΊΠΎΡ€ΠΈΡ‚Π΅Π»ΡŒ Ρ€Π°Π±ΠΎΡ‚Π°Π΅Ρ‚ круглосуточно

19 августа 2020 Π³. — Π˜ΡΡΠ»Π΅Π΄ΠΎΠ²Π°Ρ‚Π΅Π»ΠΈ достигли Π²Π°ΠΆΠ½ΠΎΠΉ Π²Π΅Ρ…ΠΈ Π½Π° ΠΏΡƒΡ‚ΠΈ ΠΊ ΡƒΡΠΊΠΎΡ€ΠΈΡ‚Π΅Π»ΡŽ элСмСнтарных частиц Π±ΡƒΠ΄ΡƒΡ‰Π΅Π³ΠΎ. Π’ΠΏΠ΅Ρ€Π²Ρ‹Π΅ Π»Π°Π·Π΅Ρ€Π½ΠΎ-ΠΏΠ»Π°Π·ΠΌΠ΅Π½Π½Ρ‹ΠΉ ΡƒΡΠΊΠΎΡ€ΠΈΡ‚Π΅Π»ΡŒ ΠΏΡ€ΠΎΡ€Π°Π±ΠΎΡ‚Π°Π» Π±ΠΎΠ»Π΅Π΅ суток Π² Π½Π΅ΠΏΡ€Π΅Ρ€Ρ‹Π²Π½ΠΎΠΌ Ρ€Π΅ΠΆΠΈΠΌΠ΅…


Π’Π΅Ρ…Π° для экспСримСнтов ΠΏΠΎ ΡƒΡΠΊΠΎΡ€Π΅Π½ΠΈΡŽ Π½ΠΎΠ²ΠΎΠ³ΠΎ поколСния

30 апрСля 2021 Π³. — Π‘ΡƒΠ΄ΡƒΡ‰Π΅Π΅ ускорСния частиц наступило. ΠŸΡ€ΠΎΠ±ΡƒΠΆΠ΄Π΅Π½ΠΈΠ΅ — ΠΌΠ½ΠΎΠ³ΠΎΠΎΠ±Π΅Ρ‰Π°ΡŽΡ‰Π°Ρ концСпция ΡΠΎΠ²Π΅Ρ€ΡˆΠ΅Π½Π½ΠΎ Π½ΠΎΠ²ΠΎΠ³ΠΎ ΠΌΠ΅Ρ‚ΠΎΠ΄Π°, с ΠΏΠΎΠΌΠΎΡ‰ΡŒΡŽ ΠΊΠΎΡ‚ΠΎΡ€ΠΎΠ³ΠΎ частицы ΠΌΠΎΠΆΠ½ΠΎ ΡƒΡΠΊΠΎΡ€ΡΡ‚ΡŒ Π΄Π°ΠΆΠ΅ Π½Π° ΠΊΠΎΡ€ΠΎΡ‚ΠΊΠΈΠ΅ расстояния. ОснованиСм для этого слуТит …


Новый матСматичСский инструмСнт ΠΌΠΎΠΆΠ΅Ρ‚ Π²Ρ‹Π±Ρ€Π°Ρ‚ΡŒ Π»ΡƒΡ‡ΡˆΠΈΠ΅ Π΄Π°Ρ‚Ρ‡ΠΈΠΊΠΈ для Ρ€Π°Π±ΠΎΡ‚Ρ‹

17 сСнтября 2020 Π³. — Π’ Ρ€Π΅Π·ΡƒΠ»ΡŒΡ‚Π°Ρ‚Π΅ авиакатастрофы Boeing 737 Max Π² 2019 Π³ΠΎΠ΄Ρƒ восстановлСнный Ρ‡Π΅Ρ€Π½Ρ‹ΠΉ ящик послС Π°Π²Π°Ρ€ΠΈΠΈ Π½Π°ΠΌΠ΅ΠΊΠ°Π», Ρ‡Ρ‚ΠΎ ΠΎΡ‚ΠΊΠ°Π·Π°Π²ΡˆΠΈΠΉ Π΄Π°Ρ‚Ρ‡ΠΈΠΊ давлСния, Π²ΠΎΠ·ΠΌΠΎΠΆΠ½ΠΎ, ΠΏΡ€ΠΈΠ²Π΅Π» ΠΊ Ρ‚ΠΎΠΌΡƒ, Ρ‡Ρ‚ΠΎ Π·Π»ΠΎΠΏΠΎΠ»ΡƒΡ‡Π½Ρ‹ΠΉ самолСт Π½Ρ‹Ρ€Π½ΡƒΠ» Π² нос.Π­Ρ‚ΠΎΡ‚ ΠΈ Π΄Ρ€ΡƒΠ³ΠΈΠ΅ ΠΈΠ½Ρ†ΠΈΠ΄Π΅Π½Ρ‚Ρ‹ ΠΈΠΌΠ΅ΡŽΡ‚ …


ΠœΠ΅Ρ‚ΠΎΠ΄Ρ‹ количСствСнной ΠΎΡ†Π΅Π½ΠΊΠΈ острых ΠΎΡ‰ΡƒΡ‰Π΅Π½ΠΈΠΉ ΠΈ судороги ΠΈΠ³Ρ€ΠΎΠΊΠ° Π² Π³ΠΎΠ»ΡŒΡ„

19 июля 2018 Π³. — ΠŸΠΎΡ‡Ρ‚ΠΈ ΠΊΠ°ΠΆΠ΄ΠΎΠΌΡƒ Π³ΠΎΠ»ΡŒΡ„ΠΈΡΡ‚Ρƒ Π·Π½Π°ΠΊΠΎΠΌΠΎ это чувство. Π§Π΅Ρ€Π΅Π· нСсколько ΠΌΠΈΠ½ΡƒΡ‚ послС идСальной Π΅Π·Π΄Ρ‹ ΠΏΠΎ Ρ„Π°Ρ€Π²Π°Ρ‚Π΅Ρ€Ρƒ каскад Π½Π΅ΠΎΠ±ΡŠΡΡΠ½ΠΈΠΌΡ‹Ρ… ΠΏΡ€ΠΎΠΏΡƒΡ‰Π΅Π½Π½Ρ‹Ρ… ΠΏΠ°Ρ‚Ρ‚ΠΎΠ² ΠΏΡ€ΠΈΠ²ΠΎΠ΄ΠΈΡ‚ ΠΊ Ρ€Π°Π·ΠΎΡ‡Π°Ρ€ΠΎΠ²Ρ‹Π²Π°ΡŽΡ‰Π΅ΠΌΡƒ Ρ‚Ρ€ΠΎΠΉΠ½ΠΎΠΌΡƒ Ρ€Π΅Π·ΡƒΠ»ΡŒΡ‚Π°Ρ‚Ρƒ …


ΠŸΠ΅Ρ€Π΅Π½ΠΎΡΠ½ΠΎΠΉ Π΄Π°Ρ‚Ρ‡ΠΈΠΊ Π²ΠΈΠ±Ρ€Π°Ρ†ΠΈΠΈ для Ρ‚ΠΎΡ‡Π½ΠΎΠ³ΠΎ распознавания голоса

24 июня 2019 Π³. — Π£Ρ‡Π΅Π½Ρ‹Π΅ Ρ€Π°Π·Ρ€Π°Π±ΠΎΡ‚Π°Π»ΠΈ Π³ΠΈΠ±ΠΊΠΈΠΉ ΠΈ носимый Ρ‡ΡƒΠ²ΡΡ‚Π²ΠΈΡ‚Π΅Π»ΡŒΠ½Ρ‹ΠΉ ΠΊ Π²ΠΈΠ±Ρ€Π°Ρ†ΠΈΠΈ Π΄Π°Ρ‚Ρ‡ΠΈΠΊ.Когда этот Π΄Π°Ρ‚Ρ‡ΠΈΠΊ ΠΏΡ€ΠΈΠΊΡ€Π΅ΠΏΠ»Π΅Π½ ΠΊ шСС, ΠΎΠ½ ΠΌΠΎΠΆΠ΅Ρ‚ Ρ‚ΠΎΡ‡Π½ΠΎ Ρ€Π°ΡΠΏΠΎΠ·Π½Π°Π²Π°Ρ‚ΡŒ голос ΠΏΠΎ Π²ΠΈΠ±Ρ€Π°Ρ†ΠΈΠΈ ΠΊΠΎΠΆΠΈ шСи ΠΈ Π½Π΅ …


Β«ΠžΠΏΡ‚ΠΈΡ‡Π΅ΡΠΊΠ°Ρ Ρ€Π°ΠΊΠ΅Ρ‚Π°Β», созданная с ΠΏΠΎΠΌΠΎΡ‰ΡŒΡŽ интСнсивного Π»Π°Π·Π΅Ρ€Π½ΠΎΠ³ΠΎ излучСния

14 сСнтября 2018 Π³. — ЭкспСримСнт продСмонстрировал, ΠΊΠ°ΠΊ ΠΏΡ€ΠΈΠΌΠ΅Π½Π΅Π½ΠΈΠ΅ интСнсивного свСта ΡƒΠ²Π΅Π»ΠΈΡ‡ΠΈΠ²Π°Π΅Ρ‚ количСство элСктронов Π΄ΠΎ максимально Π²ΠΎΠ·ΠΌΠΎΠΆΠ½ΠΎΠ³ΠΎ …


Π’Ρ€ΠΈ Ρ„ΠΎΡ€ΠΌΡƒΠ»Ρ‹, ΠΊΠΎΡ‚ΠΎΡ€Ρ‹Π΅ Π²Π°ΠΌ Π½ΡƒΠΆΠ½Ρ‹

«ΠžΠ³ΠΎ, Ρ‚Ρ‹ Π΄Π΅ΠΉΡΡ‚Π²ΠΈΡ‚Π΅Π»ΡŒΠ½ΠΎ ΠΏΡ€ΠΎΡˆΠ΅Π» с нуля Π΄ΠΎ ΡˆΠ΅ΡΡ‚ΠΈΠ΄Π΅ΡΡΡ‚ΠΈ!»

Π’Ρ‹ ΠΊΠΎΠ³Π΄Π°-Π½ΠΈΠ±ΡƒΠ΄ΡŒ ΡΠ»Ρ‹ΡˆΠ°Π»ΠΈ, Ρ‡Ρ‚ΠΎΠ±Ρ‹ ΠΊΡ‚ΠΎ-Ρ‚ΠΎ использовал ΠΈΠ΄ΠΈΠΎΠΌΡƒ Β«ΠΎΡ‚ нуля Π΄ΠΎ ΡˆΠ΅ΡΡ‚ΠΈΠ΄Π΅ΡΡΡ‚ΠΈΒ», ΠΊΠ°ΠΊ я Π² ΠΏΡ€ΠΈΠ²Π΅Π΄Π΅Π½Π½ΠΎΠΌ Π²Ρ‹ΡˆΠ΅ ΠΏΡ€ΠΈΠΌΠ΅Ρ€Π΅? Когда ΠΊΡ‚ΠΎ-Ρ‚ΠΎ Π³ΠΎΠ²ΠΎΡ€ΠΈΡ‚, Ρ‡Ρ‚ΠΎ Ρ‡Ρ‚ΠΎ-Ρ‚ΠΎ пошло с нуля Π΄ΠΎ ΡˆΠ΅ΡΡ‚ΠΈΠ΄Π΅ΡΡΡ‚ΠΈ, Π½Π° самом Π΄Π΅Π»Π΅ ΠΎΠ½ΠΈ говорят, Ρ‡Ρ‚ΠΎ всС ΡƒΡΠΊΠΎΡ€ΠΈΠ»ΠΎΡΡŒ ΠΎΡ‡Π΅Π½ΡŒ быстро. УскорСниС — это Π²Π΅Π»ΠΈΡ‡ΠΈΠ½Π°, Π½Π° ΠΊΠΎΡ‚ΠΎΡ€ΡƒΡŽ ΡΠΊΠΎΡ€ΠΎΡΡ‚ΡŒ Ρ‡Π΅Π³ΠΎ-Π»ΠΈΠ±ΠΎ измСняСтся Π² Ρ‚Π΅Ρ‡Π΅Π½ΠΈΠ΅ Π·Π°Π΄Π°Π½Π½ΠΎΠ³ΠΎ ΠΏΠ΅Ρ€ΠΈΠΎΠ΄Π° Π²Ρ€Π΅ΠΌΠ΅Π½ΠΈ.

Π’ этой ΡΡ‚Π°Ρ‚ΡŒΠ΅ ΠΌΡ‹ ΠΏΠΎΠ³ΠΎΠ²ΠΎΡ€ΠΈΠΌ ΠΎΠ± ускорСнии: Ρ‡Ρ‚ΠΎ это Ρ‚Π°ΠΊΠΎΠ΅ ΠΈ ΠΊΠ°ΠΊ Π΅Π³ΠΎ Ρ€Π°ΡΡΡ‡ΠΈΡ‚Π°Ρ‚ΡŒ. ΠŸΡ€ΠΈΡΡ‚Π΅Π³Π½ΠΈΡ‚Π΅ΡΡŒ!

Π§Ρ‚ΠΎ Ρ‚Π°ΠΊΠΎΠ΅ ускорСниС?

УскорСниС — это ΡΠΊΠΎΡ€ΠΎΡΡ‚ΡŒ измСнСния скорости Π·Π° Π·Π°Π΄Π°Π½Π½Ρ‹ΠΉ ΠΏΠ΅Ρ€ΠΈΠΎΠ΄ Π²Ρ€Π΅ΠΌΠ΅Π½ΠΈ. Для расчСта ускорСния Π½Π΅ΠΎΠ±Ρ…ΠΎΠ΄ΠΈΠΌΡ‹ ΡΠΊΠΎΡ€ΠΎΡΡ‚ΡŒ ΠΈ врСмя.

МногиС ΠΏΡƒΡ‚Π°ΡŽΡ‚ ускорСниС со ΡΠΊΠΎΡ€ΠΎΡΡ‚ΡŒΡŽ (ΠΈΠ»ΠΈ ΡΠΊΠΎΡ€ΠΎΡΡ‚ΡŒΡŽ).ΠŸΡ€Π΅ΠΆΠ΄Π΅ всСго, ΡΠΊΠΎΡ€ΠΎΡΡ‚ΡŒ — это просто ΡΠΊΠΎΡ€ΠΎΡΡ‚ΡŒ с Π½Π°ΠΏΡ€Π°Π²Π»Π΅Π½ΠΈΠ΅ΠΌ, поэтому ΠΎΠ½ΠΈ часто ΠΈΡΠΏΠΎΠ»ΡŒΠ·ΡƒΡŽΡ‚ΡΡ ΠΊΠ°ΠΊ синонимы, Π΄Π°ΠΆΠ΅ Ссли ΠΎΠ½ΠΈ ΠΈΠΌΠ΅ΡŽΡ‚ нСбольшиС различия. УскорСниС — это ΡΠΊΠΎΡ€ΠΎΡΡ‚ΡŒ измСнСния скорости, ΠΎΠ·Π½Π°Ρ‡Π°ΡŽΡ‰Π°Ρ, Ρ‡Ρ‚ΠΎ Ρ‡Ρ‚ΠΎ-Ρ‚ΠΎ становится быстрСС ΠΈΠ»ΠΈ ΠΌΠ΅Π΄Π»Π΅Π½Π½Π΅Π΅.

Π§Ρ‚ΠΎ Ρ‚Π°ΠΊΠΎΠ΅ Ρ„ΠΎΡ€ΠΌΡƒΠ»Π° ускорСния?

Для расчСта ускорСния ΠΌΠΎΠΆΠ½ΠΎ ΠΈΡΠΏΠΎΠ»ΡŒΠ·ΠΎΠ²Π°Ρ‚ΡŒ ΡƒΡ€Π°Π²Π½Π΅Π½ΠΈΠ΅ ускорСния. Π’ΠΎΡ‚ Π½Π°ΠΈΠ±ΠΎΠ»Π΅Π΅ распространСнная Ρ„ΠΎΡ€ΠΌΡƒΠ»Π° ускорСния:

$$ a = {Ξ”v} / {Ξ”t} $$

Π³Π΄Π΅ $ Ξ”v $ — ΠΈΠ·ΠΌΠ΅Π½Π΅Π½ΠΈΠ΅ скорости, Π° $ Ξ”t $ — ΠΈΠ·ΠΌΠ΅Π½Π΅Π½ΠΈΠ΅ Π²ΠΎ Π²Ρ€Π΅ΠΌΠ΅Π½ΠΈ.

Π’Ρ‹ Ρ‚Π°ΠΊΠΆΠ΅ ΠΌΠΎΠΆΠ΅Ρ‚Π΅ Π½Π°ΠΏΠΈΡΠ°Ρ‚ΡŒ ΡƒΡ€Π°Π²Π½Π΅Π½ΠΈΠ΅ ускорСния ΡΠ»Π΅Π΄ΡƒΡŽΡ‰ΠΈΠΌ ΠΎΠ±Ρ€Π°Π·ΠΎΠΌ:

$$ a = {v (f) — v (i)} / {t (f) — t (i)} $$

Π’ этом ΡƒΡ€Π°Π²Π½Π΅Π½ΠΈΠΈ ускорСния $ v (f) $ — это конСчная ΡΠΊΠΎΡ€ΠΎΡΡ‚ΡŒ, Π° — Π½Π°Ρ‡Π°Π»ΡŒΠ½Π°Ρ ΡΠΊΠΎΡ€ΠΎΡΡ‚ΡŒ $ v (i) $. $ T (f) $ — это ΠΊΠΎΠ½Π΅Ρ‡Π½ΠΎΠ΅ врСмя, Π° $ t (i) $ — Π½Π°Ρ‡Π°Π»ΡŒΠ½ΠΎΠ΅ врСмя.

Π•Ρ‰Π΅ ΠΎ Π½Π΅ΠΊΠΎΡ‚ΠΎΡ€Ρ‹Ρ… Π²Π΅Ρ‰Π°Ρ…, ΠΎ ΠΊΠΎΡ‚ΠΎΡ€Ρ‹Ρ… слСдуСт ΠΏΠΎΠΌΠ½ΠΈΡ‚ΡŒ ΠΏΡ€ΠΈ использовании уравнСния ускорСния:

  • Π’Π°ΠΌ Π½ΡƒΠΆΠ½ΠΎ Π²Ρ‹Ρ‡Π΅ΡΡ‚ΡŒ Π½Π°Ρ‡Π°Π»ΡŒΠ½ΡƒΡŽ ΡΠΊΠΎΡ€ΠΎΡΡ‚ΡŒ ΠΈΠ· ΠΊΠΎΠ½Π΅Ρ‡Π½ΠΎΠΉ скорости. Если Π²Ρ‹ ΠΏΠ΅Ρ€Π΅Π²Π΅Ρ€Π½Π΅Ρ‚Π΅ ΠΈΡ…, Π²Ρ‹ ΠΏΠΎΠ»ΡƒΡ‡ΠΈΡ‚Π΅ Π½Π΅Π²Π΅Ρ€Π½ΠΎΠ΅ Π½Π°ΠΏΡ€Π°Π²Π»Π΅Π½ΠΈΠ΅ вашСго ускорСния.
  • Если Ρƒ вас Π½Π΅Ρ‚ Π²Ρ€Π΅ΠΌΠ΅Π½ΠΈ Π½Π°Ρ‡Π°Π»Π°, Π²Ρ‹ ΠΌΠΎΠΆΠ΅Ρ‚Π΅ ΠΈΡΠΏΠΎΠ»ΡŒΠ·ΠΎΠ²Π°Ρ‚ΡŒ Β«0Β».
  • Если конСчная ΡΠΊΠΎΡ€ΠΎΡΡ‚ΡŒ мСньшС Π½Π°Ρ‡Π°Π»ΡŒΠ½ΠΎΠΉ, ускорСниС Π±ΡƒΠ΄Π΅Ρ‚ ΠΎΡ‚Ρ€ΠΈΡ†Π°Ρ‚Π΅Π»ΡŒΠ½Ρ‹ΠΌ, Ρ‡Ρ‚ΠΎ ΠΎΠ·Π½Π°Ρ‡Π°Π΅Ρ‚, Ρ‡Ρ‚ΠΎ ΠΎΠ±ΡŠΠ΅ΠΊΡ‚ замСдлился.

Π’Π΅ΠΏΠ΅Ρ€ΡŒ Π΄Π°Π²Π°ΠΉΡ‚Π΅ Ρ€Π°Π·Π±Π΅Ρ€Π΅ΠΌ ΡƒΡ€Π°Π²Π½Π΅Π½ΠΈΠ΅ ускорСния пошагово Π½Π° Ρ€Π΅Π°Π»ΡŒΠ½ΠΎΠΌ ΠΏΡ€ΠΈΠΌΠ΅Ρ€Π΅.

Как Ρ€Π°ΡΡΡ‡ΠΈΡ‚Π°Ρ‚ΡŒ ускорСниС: пошаговая Ρ€Π°Π·Π±ΠΈΠ²ΠΊΠ°

Π’Π΅ΠΏΠ΅Ρ€ΡŒ Π΄Π°Π²Π°ΠΉΡ‚Π΅ Ρ€Π°Π·Π±Π΅Ρ€Π΅ΠΌ Ρ„ΠΎΡ€ΠΌΡƒΠ»Ρƒ ускорСния пошагово Π½Π° Ρ€Π΅Π°Π»ΡŒΠ½ΠΎΠΌ ΠΏΡ€ΠΈΠΌΠ΅Ρ€Π΅. 2 $$

$$ A = 6.2 $$

Π”Π°Π²Π°ΠΉΡ‚Π΅ ΠΏΠΎΠΏΡ€ΠΎΠ±ΡƒΠ΅ΠΌ Π΄Ρ€ΡƒΠ³ΠΎΠΉ ΠΏΡ€ΠΈΠΌΠ΅Ρ€.

ВСлосипСдист, двиТущийся со ΡΠΊΠΎΡ€ΠΎΡΡ‚ΡŒΡŽ 23,2 ΠΌ / с, ΠΏΠΎΠ»Π½ΠΎΡΡ‚ΡŒΡŽ останавливаСтся Π·Π° 1,5 $ с $. КакоС Ρƒ Π½Π΅Π΅ Π±Ρ‹Π»ΠΎ Π·Π°ΠΌΠ΅Π΄Π»Π΅Π½ΠΈΠ΅?

Π‘Π½Π°Ρ‡Π°Π»Π° Π½Π°ΠΏΠΈΡˆΠΈΡ‚Π΅ ΡƒΡ€Π°Π²Π½Π΅Π½ΠΈΠ΅ ускорСния.

$$ a = (v (f) — v (i)) Γ· (t (f) — t (i)) $$

Π—Π°Ρ‚Π΅ΠΌ ΠΎΠΏΡ€Π΅Π΄Π΅Π»ΠΈΡ‚Π΅ свои ΠΏΠ΅Ρ€Π΅ΠΌΠ΅Π½Π½Ρ‹Π΅.

a = Ρ‚ΠΎ, Ρ‡Ρ‚ΠΎ ΠΌΡ‹ Ρ€Π΅ΡˆΠ°Π΅ΠΌ для

$$ V (f) = 0 м / с $$

$$ V (i) = 23,2 м / с $$

$$ T (f) = 1,4 с $$

$$ T (i) = 0 с $$

Π’Π΅ΠΏΠ΅Ρ€ΡŒ ΠΏΠΎΠ΄ΡΡ‚Π°Π²ΡŒΡ‚Π΅ ΠΏΠ΅Ρ€Π΅ΠΌΠ΅Π½Π½Ρ‹Π΅ Π² ΡƒΡ€Π°Π²Π½Π΅Π½ΠΈΠ΅ ΠΈ Ρ€Π΅ΡˆΠΈΡ‚Π΅:

$$ A = {{(0 — 23.2} $$

2 Π”Ρ€ΡƒΠ³ΠΈΠ΅ ΠΎΠ±Ρ‰ΠΈΠ΅ Ρ„ΠΎΡ€ΠΌΡƒΠ»Ρ‹ ускорСния

НС Π·Π½Π°Π΅Ρ‚Π΅, ΠΊΠ°ΠΊ Ρ€Π°ΡΡΡ‡ΠΈΡ‚Π°Ρ‚ΡŒ ускорСниС ΠΏΠΎ Π΄Ρ€ΡƒΠ³ΠΎΠΉ Ρ„ΠΎΡ€ΠΌΡƒΠ»Π΅? Π•ΡΡ‚ΡŒ нСсколько Π΄Ρ€ΡƒΠ³ΠΈΡ… распространСнных Ρ„ΠΎΡ€ΠΌΡƒΠ» ускорСния.

Π€ΠΎΡ€ΠΌΡƒΠ»Π° ΡƒΠ³Π»ΠΎΠ²ΠΎΠ³ΠΎ ускорСния

Π£Π³Π»ΠΎΠ²ΠΎΠ΅ ускорСниС — это ΡΠΊΠΎΡ€ΠΎΡΡ‚ΡŒ, с ΠΊΠΎΡ‚ΠΎΡ€ΠΎΠΉ ΡƒΠ³Π»ΠΎΠ²ΠΎΠ΅ ускорСниС Π²Ρ€Π°Ρ‰Π°ΡŽΡ‰Π΅Π³ΠΎΡΡ ΠΎΠ±ΡŠΠ΅ΠΊΡ‚Π° измСняСтся Π²ΠΎ Π²Ρ€Π΅ΠΌΠ΅Π½ΠΈ.

Π’ΠΎΡ‚ ΡƒΡ€Π°Π²Π½Π΅Π½ΠΈΠ΅ ΡƒΠ³Π»ΠΎΠ²ΠΎΠ³ΠΎ ускорСния:

$$ a = {\ change \ in \ angular \ velocity} / {\ change \ in \ time} $$

Π€ΠΎΡ€ΠΌΡƒΠ»Π° Ρ†Π΅Π½Ρ‚Ρ€ΠΎΡΡ‚Ρ€Π΅ΠΌΠΈΡ‚Π΅Π»ΡŒΠ½ΠΎΠ³ΠΎ ускорСния

Π¦Π΅Π½Ρ‚Ρ€ΠΎΡΡ‚Ρ€Π΅ΠΌΠΈΡ‚Π΅Π»ΡŒΠ½ΠΎΠ΅ ускорСниС — это ΡΠΊΠΎΡ€ΠΎΡΡ‚ΡŒ двиТСния ΠΎΠ±ΡŠΠ΅ΠΊΡ‚Π° Π²Π½ΡƒΡ‚Ρ€ΡŒ ΠΊ Ρ†Π΅Π½Ρ‚Ρ€Ρƒ ΠΊΡ€ΡƒΠ³Π°.2} / r $$

$ a (c) $ = ускорСниС, Ρ†Π΅Π½Ρ‚Ρ€ΠΎΡΡ‚Ρ€Π΅ΠΌΠΈΡ‚Π΅Π»ΡŒΠ½ΠΎΠ΅

$ v $ = ΡΠΊΠΎΡ€ΠΎΡΡ‚ΡŒ

$ r $ = радиус

ΠžΡΠ½ΠΎΠ²Π½Ρ‹Π΅ Π²Ρ‹Π²ΠΎΠ΄Ρ‹

УскорСниС — это ΡΠΊΠΎΡ€ΠΎΡΡ‚ΡŒ измСнСния скорости Π·Π° Π·Π°Π΄Π°Π½Π½Ρ‹ΠΉ ΠΏΠ΅Ρ€ΠΈΠΎΠ΄ Π²Ρ€Π΅ΠΌΠ΅Π½ΠΈ.

Π’Ρ‹ вычисляСтС ускорСниС, Ρ€Π°Π·Π΄Π΅Π»ΠΈΠ² ΠΈΠ·ΠΌΠ΅Π½Π΅Π½ΠΈΠ΅ скорости Π½Π° ΠΈΠ·ΠΌΠ΅Π½Π΅Π½ΠΈΠ΅ Π²ΠΎ Π²Ρ€Π΅ΠΌΠ΅Π½ΠΈ.

Π§Ρ‚ΠΎ дальшС?

Π˜Ρ‰Π΅Ρ‚Π΅ Π΄Ρ€ΡƒΠ³ΠΈΠ΅ Π½Π°ΡƒΡ‡Π½Ρ‹Π΅ объяснСния? ΠœΡ‹ Ρ€Π°Π·Π±ΠΈΠ²Π°Π΅ΠΌ ΡΠ»Π΅ΠΊΡ‚Ρ€ΠΈΡ‡Π΅ΡΠΊΡƒΡŽ ΡΠ½Π΅Ρ€Π³ΠΈΡŽ ΠΈ ΠΊΠ°ΠΊ ΠΎΠΏΡ€Π΅Π΄Π΅Π»ΠΈΡ‚ΡŒ Ρ€Π°Π·Π»ΠΈΡ‡Π½Ρ‹Ρ… Ρ‚ΠΈΠΏΠΎΠ² ΠΎΠ±Π»Π°ΠΊΠΎΠ² с ΠΏΠΎΠΌΠΎΡ‰ΡŒΡŽ Π½Π°ΡˆΠΈΡ… экспСртных руководств.

Π Π°Π±ΠΎΡ‚Π°Π΅Ρ‚Π΅ Π½Π°Π΄ ΠΈΡΡΠ»Π΅Π΄ΠΎΠ²Π°Ρ‚Π΅Π»ΡŒΡΠΊΠΎΠΉ Ρ€Π°Π±ΠΎΡ‚ΠΎΠΉ, Π½ΠΎ Π½Π΅ Π·Π½Π°Π΅Ρ‚Π΅, с Ρ‡Π΅Π³ΠΎ Π½Π°Ρ‡Π°Ρ‚ΡŒ? Π’ΠΎΠ³Π΄Π° ΠΎΠ·Π½Π°ΠΊΠΎΠΌΡŒΡ‚Π΅ΡΡŒ с нашим руководством, Π³Π΄Π΅ ΠΌΡ‹ собрали мноТСство высококачСствСнных Ρ‚Π΅ΠΌ для исслСдований, ΠΊΠΎΡ‚ΠΎΡ€Ρ‹Π΅ Π²Ρ‹ ΠΌΠΎΠΆΠ΅Ρ‚Π΅ ΠΈΡΠΏΠΎΠ»ΡŒΠ·ΠΎΠ²Π°Ρ‚ΡŒ бСсплатно.

НуТна ΠΏΠΎΠΌΠΎΡ‰ΡŒ с ΡƒΡ€ΠΎΠΊΠΎΠΌ английского языка — Π² частности, с ΠΎΠΏΡ€Π΅Π΄Π΅Π»Π΅Π½ΠΈΠ΅ΠΌ Π»ΠΈΡ‚Π΅Ρ€Π°Ρ‚ΡƒΡ€Π½Ρ‹Ρ… ΠΏΡ€ΠΈΠ΅ΠΌΠΎΠ² Π² тСкстах, ΠΊΠΎΡ‚ΠΎΡ€Ρ‹Π΅ Π²Ρ‹ Ρ‡ΠΈΡ‚Π°Π΅Ρ‚Π΅? Π’ΠΎΠ³Π΄Π° Π²Ρ‹ ΠΎΠ±ΡΠ·Π°Ρ‚Π΅Π»ΡŒΠ½ΠΎ Π·Π°Ρ…ΠΎΡ‚ΠΈΡ‚Π΅ Π²Π·Π³Π»ΡΠ½ΡƒΡ‚ΡŒ Π½Π° нашС ΠΈΡΡ‡Π΅Ρ€ΠΏΡ‹Π²Π°ΡŽΡ‰Π΅Π΅ объяснСниС самых Π²Π°ΠΆΠ½Ρ‹Ρ… Π»ΠΈΡ‚Π΅Ρ€Π°Ρ‚ΡƒΡ€Π½Ρ‹Ρ… устройств ΠΈ Ρ‚ΠΎΠ³ΠΎ, ΠΊΠ°ΠΊ ΠΎΠ½ΠΈ ΠΈΡΠΏΠΎΠ»ΡŒΠ·ΡƒΡŽΡ‚ΡΡ.

УскорСниС силы тяТСсти — Frega Physics



УскорСниС силы тяТСсти — это ΡΠΊΠΎΡ€ΠΎΡΡ‚ΡŒ, с ΠΊΠΎΡ‚ΠΎΡ€ΠΎΠΉ ΠΎΠ±ΡŠΠ΅ΠΊΡ‚ измСняСт свою ΡΠΊΠΎΡ€ΠΎΡΡ‚ΡŒ ΠΏΠΎΠ΄ дСйствиСм силы тяТСсти. ВсС ΠΎΠ±ΡŠΠ΅ΠΊΡ‚Ρ‹, ΠΏΠ°Π΄Π°ΡŽΡ‰ΠΈΠ΅ с ΠΎΠ΄Π½ΠΎΠΉ ΠΈ Ρ‚ΠΎΠΉ ΠΆΠ΅ Ρ‚ΠΎΡ‡ΠΊΠΈ, ΡƒΠ΄Π°Ρ€ΡΡŽΡ‚ΡΡ ΠΎ зСмлю Π·Π° ΠΎΠ΄Π½ΠΎ ΠΈ Ρ‚ΠΎ ΠΆΠ΅ врСмя, нСзависимо ΠΎΡ‚ массы. На Π—Π΅ΠΌΠ»Π΅ срСднСС ускорСниС свободного падСния составляСт -9,81 ΠΌ / с 2 *. ΠžΠ±ΡŠΠ΅ΠΊΡ‚Ρ‹, ΠΊΠΎΡ‚ΠΎΡ€Ρ‹Π΅ Π½Π΅ ΡƒΠ΄Π°Ρ€ΡΡŽΡ‚ΡΡ ΠΎ зСмлю, ΠΈΡΠΏΡ‹Ρ‚Ρ‹Π²Π°ΡŽΡ‚ сопротивлСниС Π²ΠΎΠ·Π΄ΡƒΡ…Π° — силу трСния, которая замСдляСт ΠΈΡ… Π΄Π²ΠΈΠΆΠ΅Π½ΠΈΠ΅. Π­Ρ‚ΠΎΠ³ΠΎ ΠΌΠΎΠΆΠ½ΠΎ Π΄ΠΎΠ±ΠΈΡ‚ΡŒΡΡ, ΡƒΠΌΠ΅Π½ΡŒΡˆΠΈΠ² сопротивлСниС Π²ΠΎΠ·Π΄ΡƒΡ…Π° Π·Π° счСт измСнСния Ρ„ΠΎΡ€ΠΌΡ‹ ΠΎΠ±ΡŠΠ΅ΠΊΡ‚Π°.

УскорСниС свободного падСния ВБЕГДА ΠΎΡ‚Ρ€ΠΈΡ†Π°Ρ‚Π΅Π»ΡŒΠ½ΠΎΠ΅. Π›ΡŽΠ±ΠΎΠΉ ΠΎΠ±ΡŠΠ΅ΠΊΡ‚, Π½Π° ΠΊΠΎΡ‚ΠΎΡ€Ρ‹ΠΉ дСйствуСт Ρ‚ΠΎΠ»ΡŒΠΊΠΎ сила тяТСсти (снаряд ΠΈΠ»ΠΈ ΠΎΠ±ΡŠΠ΅ΠΊΡ‚ Π² свободном ΠΏΠ°Π΄Π΅Π½ΠΈΠΈ), ΠΈΠΌΠ΅Π΅Ρ‚ ускорСниС -9.81 ΠΌ / с 2 нСзависимо ΠΎΡ‚ направлСния. УскорСниС ΠΎΡ‚Ρ€ΠΈΡ†Π°Ρ‚Π΅Π»ΡŒΠ½ΠΎΠ΅ ΠΏΡ€ΠΈ подъСмС, ΠΏΠΎΡ‚ΠΎΠΌΡƒ Ρ‡Ρ‚ΠΎ ΡΠΊΠΎΡ€ΠΎΡΡ‚ΡŒ ΡƒΠΌΠ΅Π½ΡŒΡˆΠ°Π΅Ρ‚ΡΡ. УскорСниС ΠΎΡ‚Ρ€ΠΈΡ†Π°Ρ‚Π΅Π»ΡŒΠ½ΠΎΠ΅ ΠΏΡ€ΠΈ спускС, ΠΏΠΎΡ‚ΠΎΠΌΡƒ Ρ‡Ρ‚ΠΎ ΠΎΠ½ двиТСтся Π² ΠΎΡ‚Ρ€ΠΈΡ†Π°Ρ‚Π΅Π»ΡŒΠ½ΠΎΠΌ Π½Π°ΠΏΡ€Π°Π²Π»Π΅Π½ΠΈΠΈ, Π²Π½ΠΈΠ·. Π”Π°ΠΆΠ΅ Π² Π²Π΅Ρ€Ρ…Π½Π΅ΠΉ части Ρ‚Ρ€Π°Π΅ΠΊΡ‚ΠΎΡ€ΠΈΠΈ, Π³Π΄Π΅ мгновСнная ΡΠΊΠΎΡ€ΠΎΡΡ‚ΡŒ Ρ€Π°Π²Π½Π° 0 ΠΌ / с, ускорСниС ΠΏΠΎ-ΠΏΡ€Π΅ΠΆΠ½Π΅ΠΌΡƒ составляСт -9,81 ΠΌ / с 2 .

Когда Π² Π·Π°Π΄Π°Ρ‡Π΅ упоминаСтся, Ρ‡Ρ‚ΠΎ ΠΎΠ±ΡŠΠ΅ΠΊΡ‚ «находится Π² свободном ΠΏΠ°Π΄Π΅Π½ΠΈΠΈΒ», Β«ΠΏΠ°Π΄Π°Π΅Ρ‚Β», Β«Π±Ρ€ΠΎΡˆΠ΅Π½Β», «подбрасываСтся» ΠΈΠ»ΠΈ ΠΊΠ°ΠΊΠΎΠΉ-Π»ΠΈΠ±ΠΎ Π΄Ρ€ΡƒΠ³ΠΎΠΉ синоним, прСдполагаСтся постоянноС Π·Π½Π°Ρ‡Π΅Π½ΠΈΠ΅ ускорСния свободного падСния.Если Π² ΡƒΡ€Π°Π²Π½Π΅Π½ΠΈΠΈ Π΅ΡΡ‚ΡŒ a , Π½Π°ΠΏΡ€ΠΈΠΌΠ΅Ρ€, v = v 0 + at , ускорСниС Π±ΡƒΠ΄Π΅Ρ‚ ΠΎΡ‚Ρ€ΠΈΡ†Π°Ρ‚Π΅Π»ΡŒΠ½Ρ‹ΠΌ . Если Π² ΡƒΡ€Π°Π²Π½Π΅Π½ΠΈΠΈ g , Π½Π°ΠΏΡ€ΠΈΠΌΠ΅Ρ€ W = mg, подразумСваСтся Π½Π°ΠΏΡ€Π°Π²Π»Π΅Π½ΠΈΠ΅ ΠΈ ускорСниС ΠΏΠΎΠ»ΠΎΠΆΠΈΡ‚Π΅Π»ΡŒΠ½ΠΎΠ΅ .

* -10 ΠΌ / с 2 — ΠΏΡ€ΠΈΠ΅ΠΌΠ»Π΅ΠΌΠΎΠ΅ число для Π±ΠΎΠ»ΡŒΡˆΠΈΠ½ΡΡ‚Π²Π° вычислСний Π² Π·Π°Π΄Π°Ρ‡Π°Ρ…, Π½ΠΎ -9,81 — Π±ΠΎΠ»Π΅Π΅ Ρ‚ΠΎΡ‡Π½ΠΎΠ΅ число.

ΠžΠ±ΠΎΠ·Π½Π°Ρ‡Π΅Π½ΠΈΠ΅ ΠΏΠ΅Ρ€Π΅ΠΌΠ΅Π½Π½ΠΎΠΉ
Имя ΠΏΠ΅Ρ€Π΅ΠΌΠ΅Π½Π½ΠΎΠΉ
Π•Π΄ΠΈΠ½ΠΈΡ†Π° БИ
Π”Ρ€ΡƒΠ³ΠΈΠ΅ Π²ΠΎΠ·ΠΌΠΎΠΆΠ½Ρ‹Π΅ Π΅Π΄ΠΈΠ½ΠΈΡ†Ρ‹
Π₯арактСристики
ΠΌΠ΅Ρ‚Ρ€ΠΎΠ² Π² сСкунду Π² сСкунду
(м / с / с)
ΠΈΠ»ΠΈ
ΠΌΠ΅Ρ‚Ρ€ΠΎΠ² Π² сСкунду Π² ΠΊΠ²Π°Π΄Ρ€Π°Ρ‚Π΅ (ΠΌ / с 2 )
миль / с, ΠΊΠΌ / Ρ‡ / с УскорСниС — это ΡΠΊΠΎΡ€ΠΎΡΡ‚ΡŒ, Π² ΠΊΠΎΡ‚ΠΎΡ€ΠΎΠΉ ΡΠΊΠΎΡ€ΠΎΡΡ‚ΡŒ измСнСния; это ΠΈΠ·ΠΌΠ΅Π½Π΅Π½ΠΈΠ΅ скорости Π² Π΅Π΄ΠΈΠ½ΠΈΡ†Ρƒ Π²Ρ€Π΅ΠΌΠ΅Π½ΠΈ.На этом ΡƒΡ€ΠΎΠ²Π½Π΅ ΠŸΡ€Π΅Π΄ΠΏΠΎΠ»ΠΎΠΆΠΈΠΌ, Ρ‡Ρ‚ΠΎ ускорСниС Ρ€Π°Π²Π½ΠΎΠΌΠ΅Ρ€Π½ΠΎΠ΅ ΠΈΠ»ΠΈ постоянноС. ΠŸΠΎΡΠΊΠΎΠ»ΡŒΠΊΡƒ это Π²Π΅ΠΊΡ‚ΠΎΡ€, принимаСтся Π½Π°ΠΏΡ€Π°Π²Π»Π΅Π½ΠΈΠ΅ Π² ΡƒΡ‡Π΅Ρ‚Π½ΡƒΡŽ запись. Π‘ΡƒΠ΄ΡŒΡ‚Π΅ остороТны со своими ΠΎΡ‚Ρ€ΠΈΡ†Π°Ρ‚Π΅Π»ΡŒΠ½Ρ‹ΠΌΠΈ ΠΈ ΠΏΠΎΠ»ΠΎΠΆΠΈΡ‚Π΅Π»ΡŒΠ½Ρ‹ΠΌΠΈ ΠΌΠΎΠΌΠ΅Π½Ρ‚Π°ΠΌΠΈ. ΠŸΠΎΠ»ΠΎΠΆΠΈΡ‚Π΅Π»ΡŒΠ½Ρ‹ΠΉ ускорСниС ΠΌΠΎΠΆΠ΅Ρ‚ ΠΎΠ·Π½Π°Ρ‡Π°Ρ‚ΡŒ ускорСниС, Π΄Π²ΠΈΠΆΠ΅Π½ΠΈΠ΅ Π²ΠΏΠ΅Ρ€Π΅Π΄ ΠΈΠ»ΠΈ Π·Π°ΠΌΠ΅Π΄Π»Π΅Π½ΠΈΠ΅, Π΄Π²ΠΈΠΆΠ΅Π½ΠΈΠ΅ Π½Π°Π·Π°Π΄. ΠžΡ‚Ρ€ΠΈΡ†Π°Ρ‚Π΅Π»ΡŒΠ½ΠΎΠ΅ ускорСниС ΠΌΠΎΠΆΠ΅Ρ‚ ΠΎΠ·Π½Π°Ρ‡Π°Ρ‚ΡŒ Π·Π°ΠΌΠ΅Π΄Π»Π΅Π½ΠΈΠ΅, Π΄Π²ΠΈΠΆΠ΅Π½ΠΈΠ΅ Π²ΠΏΠ΅Ρ€Π΅Π΄ ΠΈΠ»ΠΈ ускорСниС, Π΄Π²ΠΈΠΆΠ΅Π½ΠΈΠ΅ Π½Π°Π·Π°Π΄.
g ускорСниС свободного падСния ΠΌΠ΅Ρ‚Ρ€ΠΎΠ² Π² сСкунду Π² сСкунду
(м / с / с)
ΠΈΠ»ΠΈ
ΠΌΠ΅Ρ‚Ρ€ΠΎΠ² Π² сСкунду Π² ΠΊΠ²Π°Π΄Ρ€Π°Ρ‚Π΅ (ΠΌ / с 2 )

миль / Ρ‡ / с, ΠΊΠΌ / Ρ‡ / с
ΠŸΡ€ΠΈ отсутствии сопротивлСния Π²ΠΎΠ·Π΄ΡƒΡ…Π° всС ΠΏΡ€Π΅Π΄ΠΌΠ΅Ρ‚Ρ‹ Π² свободном ΠΏΠ°Π΄Π΅Π½ΠΈΠΈ Π±ΡƒΠ΄ΡƒΡ‚ ΠΏΠΎΠΏΠ°Π΄Π°Ρ‚ΡŒ Π² Π·Π΅ΠΌΠ»ΠΈ ΠΎΠ΄Π½ΠΎΠ²Ρ€Π΅ΠΌΠ΅Π½Π½ΠΎ ΠΏΡ€ΠΈ ΠΏΠ°Π΄Π΅Π½ΠΈΠΈ с ΠΎΠ΄ΠΈΠ½Π°ΠΊΠΎΠ²ΠΎΠΉ высоты, нСзависимо ΠΎΡ‚ масса.УскорСниС свободного падСния — установлСнноС число для ΠΎΠΏΡ€Π΅Π΄Π΅Π»Π΅Π½Π½ΠΎΠ³ΠΎ мСстополоТСния, ΠΎΠ±Ρ‹Ρ‡Π½ΠΎ ΠΏΠΎ ΠΏΠ»Π°Π½Π΅Ρ‚Π΅ (Π½ΠΎ это число ΠΌΠΎΠΆΠ΅Ρ‚ Π½Π΅Π·Π½Π°Ρ‡ΠΈΡ‚Π΅Π»ΡŒΠ½ΠΎ ΠΎΡ‚Π»ΠΈΡ‡Π°Ρ‚ΡŒΡΡ Π½Π° повСрхности ΠΏΠ»Π°Π½Π΅Ρ‚Ρ‹ Π² зависимости ΠΎΡ‚ расстояния ΠΈΠ· ядра ΠΏΠ»Π°Π½Π΅Ρ‚Ρ‹). На Π—Π΅ΠΌΠ»Π΅ срСднСС ускорСниС Π·Π° счСт гравитация составляСт -9,81 ΠΌ / с 2 (хотя -10 ΠΌ / с 2 ΠΏΡ€ΠΈΠ΅ΠΌΠ»Π΅ΠΌΠΎ для Π±ΠΎΠ»ΡŒΡˆΠΈΠ½ΡΡ‚Π²Π° расчСтов). Когда Π² ΡƒΡ€Π°Π²Π½Π΅Π½ΠΈΠΈ ΠΈΡΠΏΠΎΠ»ΡŒΠ·ΡƒΠ΅Ρ‚ΡΡ символ g , прСдполагаСтся Π½Π°ΠΏΡ€Π°Π²Π»Π΅Π½ΠΈΠ΅, Π° для вычислСний ΠΈΡΠΏΠΎΠ»ΡŒΠ·ΡƒΠ΅Ρ‚ΡΡ Π°Π±ΡΠΎΠ»ΡŽΡ‚Π½ΠΎΠ΅ Π·Π½Π°Ρ‡Π΅Π½ΠΈΠ΅ (+9.81 ΠΈΠ»ΠΈ +10).

Π‘ΠΌ. «УскорСниС (комплСксноС)Β», Ρ‡Ρ‚ΠΎΠ±Ρ‹ ΡƒΠ·Π½Π°Ρ‚ΡŒ, ΠΊΠ°ΠΊ ΠΈΡΠΏΠΎΠ»ΡŒΠ·ΠΎΠ²Π°Ρ‚ΡŒ ускорСниС свободного падСния Π² Π·Π°Π΄Π°Ρ‡Π΅ свободного падСния.


2,4 Π Π°Π·Π³ΠΎΠ½ | Texas Gateway

МгновСнноС ускорСниС

МгновСнноС ускорСниС a, a, ΠΈΠ»ΠΈ ускорСниС Π² ΠΎΠΏΡ€Π΅Π΄Π΅Π»Π΅Π½Π½Ρ‹ΠΉ ΠΌΠΎΠΌΠ΅Π½Ρ‚ Π²Ρ€Π΅ΠΌΠ΅Π½ΠΈ , получаСтся с ΠΏΠΎΠΌΠΎΡ‰ΡŒΡŽ Ρ‚ΠΎΠ³ΠΎ ΠΆΠ΅ процСсса, ΠΊΠΎΡ‚ΠΎΡ€Ρ‹ΠΉ обсуТдался для ΠΌΠ³Π½ΠΎΠ²Π΅Π½Π½ΠΎΠΉ скорости Π²ΠΎ Time, Velocity ΠΈ Speed, Ρ‚ΠΎ Π΅ΡΡ‚ΡŒ ΠΏΡƒΡ‚Π΅ΠΌ рассмотрСния бСсконСчно ΠΌΠ°Π»ΠΎΠ³ΠΎ ΠΈΠ½Ρ‚Π΅Ρ€Π²Π°Π»Π° Π²Ρ€Π΅ΠΌΠ΅Π½ΠΈ.Как Π½Π°ΠΉΡ‚ΠΈ ΠΌΠ³Π½ΠΎΠ²Π΅Π½Π½ΠΎΠ΅ ускорСниС, ΠΈΡΠΏΠΎΠ»ΡŒΠ·ΡƒΡ Ρ‚ΠΎΠ»ΡŒΠΊΠΎ Π°Π»Π³Π΅Π±Ρ€Ρƒ? ΠžΡ‚Π²Π΅Ρ‚ Π·Π°ΠΊΠ»ΡŽΡ‡Π°Π΅Ρ‚ΡΡ Π² Ρ‚ΠΎΠΌ, Ρ‡Ρ‚ΠΎ ΠΌΡ‹ Π²Ρ‹Π±ΠΈΡ€Π°Π΅ΠΌ срСднСС ускорСниС, ΠΊΠΎΡ‚ΠΎΡ€ΠΎΠ΅ прСдставляСт Π΄Π²ΠΈΠΆΠ΅Π½ΠΈΠ΅. На рисункС 2.29 ΠΏΠΎΠΊΠ°Π·Π°Π½Ρ‹ Π³Ρ€Π°Ρ„ΠΈΠΊΠΈ ΠΌΠ³Π½ΠΎΠ²Π΅Π½Π½ΠΎΠ³ΠΎ ускорСния Π² зависимости ΠΎΡ‚ Π²Ρ€Π΅ΠΌΠ΅Π½ΠΈ для Π΄Π²ΡƒΡ… ΠΎΡ‡Π΅Π½ΡŒ Ρ€Π°Π·Π½Ρ‹Ρ… Π΄Π²ΠΈΠΆΠ΅Π½ΠΈΠΉ. На рис. 2.29 (Π°) ускорСниС Π½Π΅ΠΌΠ½ΠΎΠ³ΠΎ мСняСтся, Π° срСднСС Π·Π½Π°Ρ‡Π΅Π½ΠΈΠ΅ Π·Π° вСсь ΠΈΠ½Ρ‚Π΅Ρ€Π²Π°Π» ΠΏΠΎΡ‡Ρ‚ΠΈ Ρ‚Π°ΠΊΠΎΠ΅ ΠΆΠ΅, ΠΊΠ°ΠΊ ΠΈ ΠΌΠ³Π½ΠΎΠ²Π΅Π½Π½ΠΎΠ΅ ускорСниС Π² любой ΠΌΠΎΠΌΠ΅Π½Ρ‚ Π²Ρ€Π΅ΠΌΠ΅Π½ΠΈ. Π’ этом случаС ΠΌΡ‹ Π΄ΠΎΠ»ΠΆΠ½Ρ‹ Ρ€Π°ΡΡΠΌΠ°Ρ‚Ρ€ΠΈΠ²Π°Ρ‚ΡŒ это Π΄Π²ΠΈΠΆΠ΅Π½ΠΈΠ΅, ΠΊΠ°ΠΊ Ссли Π±Ρ‹ ΠΎΠ½ΠΎ ΠΈΠΌΠ΅Π»ΠΎ постоянноС ускорСниС, Ρ€Π°Π²Π½ΠΎΠ΅ срСднСму — Π² Π΄Π°Π½Π½ΠΎΠΌ случаС ΠΎΠΊΠΎΠ»ΠΎ 1.8 ΠΌ / с 21,8 ΠΌ / с2. На рис. 2.29 (b) ускорСниС Ρ€Π΅Π·ΠΊΠΎ мСняСтся со Π²Ρ€Π΅ΠΌΠ΅Π½Π΅ΠΌ. Π’ Ρ‚Π°ΠΊΠΈΡ… ситуациях Π»ΡƒΡ‡ΡˆΠ΅ всСго Ρ€Π°ΡΡΠΌΠ°Ρ‚Ρ€ΠΈΠ²Π°Ρ‚ΡŒ мСньшиС Π²Ρ€Π΅ΠΌΠ΅Π½Π½Ρ‹Π΅ ΠΈΠ½Ρ‚Π΅Ρ€Π²Π°Π»Ρ‹ ΠΈ Π²Ρ‹Π±ΠΈΡ€Π°Ρ‚ΡŒ для ΠΊΠ°ΠΆΠ΄ΠΎΠ³ΠΎ срСднСС ускорСниС. НапримСр, ΠΌΡ‹ ΠΌΠΎΠ³Π»ΠΈ Π±Ρ‹ Ρ€Π°ΡΡΠΌΠ°Ρ‚Ρ€ΠΈΠ²Π°Ρ‚ΡŒ Π΄Π²ΠΈΠΆΠ΅Π½ΠΈΠ΅ Π²ΠΎ Π²Ρ€Π΅ΠΌΠ΅Π½Π½Ρ‹Ρ… ΠΈΠ½Ρ‚Π΅Ρ€Π²Π°Π»Π°Ρ… ΠΎΡ‚ 0 Π΄ΠΎ 1,0 с ΠΈ ΠΎΡ‚ 1,0 Π΄ΠΎ 3,0 с ΠΊΠ°ΠΊ ΠΎΡ‚Π΄Π΅Π»ΡŒΠ½Ρ‹Π΅ двиТСния с ускорСниями +3,0 ΠΌ / с2 + 3,0 ΠΌ / с2 ΠΈ –2,0 ΠΌ / с2, –2,0 ΠΌ / с2. , соотвСтствСнно.

Рис. 2.29. Π“Ρ€Π°Ρ„ΠΈΠΊΠΈ ΠΌΠ³Π½ΠΎΠ²Π΅Π½Π½ΠΎΠ³ΠΎ ускорСния Π² зависимости ΠΎΡ‚ Π²Ρ€Π΅ΠΌΠ΅Π½ΠΈ для Π΄Π²ΡƒΡ… Ρ€Π°Π·Π»ΠΈΡ‡Π½Ρ‹Ρ… ΠΎΠ΄Π½ΠΎΠΌΠ΅Ρ€Π½Ρ‹Ρ… Π΄Π²ΠΈΠΆΠ΅Π½ΠΈΠΉ.(Π°) Π—Π΄Π΅ΡΡŒ ускорСниС измСняСтся Π½Π΅Π·Π½Π°Ρ‡ΠΈΡ‚Π΅Π»ΡŒΠ½ΠΎ ΠΈ всСгда Π² ΠΎΠ΄Π½ΠΎΠΌ ΠΈ Ρ‚ΠΎΠΌ ΠΆΠ΅ Π½Π°ΠΏΡ€Π°Π²Π»Π΅Π½ΠΈΠΈ, ΠΏΠΎΡΠΊΠΎΠ»ΡŒΠΊΡƒ ΠΎΠ½ΠΎ ΠΏΠΎΠ»ΠΎΠΆΠΈΡ‚Π΅Π»ΡŒΠ½ΠΎΠ΅. Π‘Ρ€Π΅Π΄Π½Π΅Π΅ Π·Π½Π°Ρ‡Π΅Π½ΠΈΠ΅ Π·Π° ΠΈΠ½Ρ‚Π΅Ρ€Π²Π°Π» ΠΏΠΎΡ‡Ρ‚ΠΈ Ρ‚Π°ΠΊΠΎΠ΅ ΠΆΠ΅, ΠΊΠ°ΠΊ ΠΈ ускорСниС Π² любой ΠΌΠΎΠΌΠ΅Π½Ρ‚ Π²Ρ€Π΅ΠΌΠ΅Π½ΠΈ. (b) Π—Π΄Π΅ΡΡŒ ускорСниС сильно различаСтся, Π²ΠΎΠ·ΠΌΠΎΠΆΠ½ΠΎ, прСдставляя ΠΏΠ°ΠΊΠ΅Ρ‚ Π½Π° ΠΊΠΎΠ½Π²Π΅ΠΉΠ΅Ρ€Π½ΠΎΠΉ Π»Π΅Π½Ρ‚Π΅ ΠΏΠΎΡ‡Ρ‚ΠΎΠ²ΠΎΠ³ΠΎ отдСлСния, ΠΊΠΎΡ‚ΠΎΡ€Ρ‹ΠΉ ускоряСтся Π²ΠΏΠ΅Ρ€Π΅Π΄ ΠΈ Π½Π°Π·Π°Π΄, ΠΊΠΎΠ³Π΄Π° ΠΎΠ½ натыкаСтся. Π’ Ρ‚Π°ΠΊΠΎΠΉ ситуации Π½Π΅ΠΎΠ±Ρ…ΠΎΠ΄ΠΈΠΌΠΎ ΡƒΡ‡ΠΈΡ‚Ρ‹Π²Π°Ρ‚ΡŒ нСбольшиС ΠΈΠ½Ρ‚Π΅Ρ€Π²Π°Π»Ρ‹ Π²Ρ€Π΅ΠΌΠ΅Π½ΠΈ (Π½Π°ΠΏΡ€ΠΈΠΌΠ΅Ρ€, ΠΎΡ‚ 0 Π΄ΠΎ 1,0 с) с постоянным ΠΈΠ»ΠΈ ΠΏΠΎΡ‡Ρ‚ΠΈ постоянным ускорСниСм.

Π’ ΡΠ»Π΅Π΄ΡƒΡŽΡ‰ΠΈΡ… Π½Π΅ΡΠΊΠΎΠ»ΡŒΠΊΠΈΡ… ΠΏΡ€ΠΈΠΌΠ΅Ρ€Π°Ρ… рассматриваСтся Π΄Π²ΠΈΠΆΠ΅Π½ΠΈΠ΅ ΠΏΠΎΠ΅Π·Π΄Π° ΠΌΠ΅Ρ‚Ρ€ΠΎ, ​​показанного Π½Π° рисункС 2.30. Π’ (Π°) Π²ΠΎΠ»Π°Π½ двиТСтся Π²ΠΏΡ€Π°Π²ΠΎ, Π° Π² (Π±) — Π²Π»Π΅Π²ΠΎ. ΠŸΡ€ΠΈΠΌΠ΅Ρ€Ρ‹ ΠΏΡ€ΠΈΠ·Π²Π°Π½Ρ‹ Π΄ΠΎΠΏΠΎΠ»Π½ΠΈΡ‚Π΅Π»ΡŒΠ½ΠΎ ΠΏΡ€ΠΎΠΈΠ»Π»ΡŽΡΡ‚Ρ€ΠΈΡ€ΠΎΠ²Π°Ρ‚ΡŒ аспСкты двиТСния ΠΈ ΠΏΡ€ΠΎΠΈΠ»Π»ΡŽΡΡ‚Ρ€ΠΈΡ€ΠΎΠ²Π°Ρ‚ΡŒ Π½Π΅ΠΊΠΎΡ‚ΠΎΡ€Ρ‹Π΅ Π°Ρ€Π³ΡƒΠΌΠ΅Π½Ρ‚Ρ‹, ΠΊΠΎΡ‚ΠΎΡ€Ρ‹Π΅ ΠΈΡΠΏΠΎΠ»ΡŒΠ·ΡƒΡŽΡ‚ΡΡ ΠΏΡ€ΠΈ Ρ€Π΅ΡˆΠ΅Π½ΠΈΠΈ ΠΏΡ€ΠΎΠ±Π»Π΅ΠΌ.

Рис. 2.30. ΠžΠ΄Π½ΠΎΠΌΠ΅Ρ€Π½ΠΎΠ΅ Π΄Π²ΠΈΠΆΠ΅Π½ΠΈΠ΅ ΠΏΠΎΠ΅Π·Π΄Π° ΠΌΠ΅Ρ‚Ρ€ΠΎ, ​​рассмотрСнноС Π² ΠŸΡ€ΠΈΠΌΠ΅Ρ€Π΅ 2.2, ΠŸΡ€ΠΈΠΌΠ΅Ρ€Π΅ 2.3, ΠŸΡ€ΠΈΠΌΠ΅Ρ€Π΅ 2.4, ΠŸΡ€ΠΈΠΌΠ΅Ρ€Π΅ 2.5, ΠŸΡ€ΠΈΠΌΠ΅Ρ€Π΅ 2.6 ΠΈ ΠŸΡ€ΠΈΠΌΠ΅Ρ€Π΅ 2.7. Π—Π΄Π΅ΡΡŒ ΠΌΡ‹ Π²Ρ‹Π±Ρ€Π°Π»ΠΈ ось xx Ρ‚Π°ΠΊ, Ρ‡Ρ‚ΠΎ + ΠΎΠ·Π½Π°Ρ‡Π°Π΅Ρ‚ Π²ΠΏΡ€Π°Π²ΠΎ, Π° βˆ’βˆ’ ΠΎΠ·Π½Π°Ρ‡Π°Π΅Ρ‚ Π²Π»Π΅Π²ΠΎ для смСщСний, скоростСй ΠΈ ускорСний.(a) ПоСзд ΠΌΠ΅Ρ‚Ρ€ΠΎ двиТСтся Π²ΠΏΡ€Π°Π²ΠΎ ΠΎΡ‚ x0x0 Π΄ΠΎ xfxf. Π•Π³ΠΎ смСщСниС Ξ”xΞ”x составляСт +2,0 ΠΊΠΌ. (b) ПоСзд двиТСтся Π½Π°Π»Π΅Π²ΠΎ ΠΎΡ‚ xβ€²0xβ€²0 Π΄ΠΎ xβ€²f.xβ€²f. size 12 {{{x}} sup {‘} rSub {size 8 {f}}} {} Π•Π³ΠΎ Π²ΠΎΠ΄ΠΎΠΈΠ·ΠΌΠ΅Ρ‰Π΅Π½ΠΈΠ΅ Ξ”xβ€²Ξ”x β€² Ρ€Π°Π·ΠΌΠ΅Ρ€ 12 {Ξ”x’} {} составляСт -1,5 ΠΊΠΌ. ΠžΠ±Ρ€Π°Ρ‚ΠΈΡ‚Π΅ Π²Π½ΠΈΠΌΠ°Π½ΠΈΠ΅, Ρ‡Ρ‚ΠΎ основной символ [β€²] ΠΈΡΠΏΠΎΠ»ΡŒΠ·ΡƒΠ΅Ρ‚ΡΡ просто для Ρ‚ΠΎΠ³ΠΎ, Ρ‡Ρ‚ΠΎΠ±Ρ‹ Ρ€Π°Π·Π»ΠΈΡ‡Π°Ρ‚ΡŒ смСщСниС Π² Π΄Π²ΡƒΡ… Ρ€Π°Π·Π½Ρ‹Ρ… ситуациях. Расстояния ΠΏΡ€ΠΎΠ΅Π·Π΄Π° ΠΈ Ρ€Π°Π·ΠΌΠ΅Ρ€Ρ‹ машин ΡƒΠΊΠ°Π·Π°Π½Ρ‹ Π² Ρ€Π°Π·Π½Ρ‹Ρ… ΠΌΠ°ΡΡˆΡ‚Π°Π±Π°Ρ…, Ρ‡Ρ‚ΠΎΠ±Ρ‹ всС ΡƒΠΌΠ΅ΡΡ‚ΠΈΠ»ΠΎΡΡŒ Π½Π° Π΄ΠΈΠ°Π³Ρ€Π°ΠΌΠΌΠ΅.

ΠŸΡ€ΠΈΠΌΠ΅Ρ€ 2.2 РасчСт смСщСния: ΠΏΠΎΠ΅Π·Π΄ ΠΌΠ΅Ρ‚Ρ€ΠΎ

ΠšΠ°ΠΊΠΎΠ²Ρ‹ Π²Π΅Π»ΠΈΡ‡ΠΈΠ½Π° ΠΈ Π·Π½Π°ΠΊ смСщСний ΠΏΡ€ΠΈ Π΄Π²ΠΈΠΆΠ΅Π½ΠΈΠΈ ΠΏΠΎΠ΅Π·Π΄Π° ΠΌΠ΅Ρ‚Ρ€ΠΎ, ​​показанных Π² частях (a) ΠΈ (b) Рисунка 2.30?

БтратСгия

Π§Π΅Ρ€Ρ‚Π΅ΠΆ с систСмой ΠΊΠΎΠΎΡ€Π΄ΠΈΠ½Π°Ρ‚ ΡƒΠΆΠ΅ прСдоставлСн, поэтому Π½Π°ΠΌ Π½Π΅ Π½ΡƒΠΆΠ½ΠΎ Π΄Π΅Π»Π°Ρ‚ΡŒ эскиз, Π½ΠΎ ΠΌΡ‹ Π΄ΠΎΠ»ΠΆΠ½Ρ‹ ΠΏΡ€ΠΎΠ°Π½Π°Π»ΠΈΠ·ΠΈΡ€ΠΎΠ²Π°Ρ‚ΡŒ Π΅Π³ΠΎ, Ρ‡Ρ‚ΠΎΠ±Ρ‹ ΡƒΠ±Π΅Π΄ΠΈΡ‚ΡŒΡΡ, Ρ‡Ρ‚ΠΎ ΠΌΡ‹ ΠΏΠΎΠ½ΠΈΠΌΠ°Π΅ΠΌ, Ρ‡Ρ‚ΠΎ ΠΎΠ½ ΠΏΠΎΠΊΠ°Π·Ρ‹Π²Π°Π΅Ρ‚. ΠžΠ±Ρ€Π°Ρ‚ΠΈΡ‚Π΅ особоС Π²Π½ΠΈΠΌΠ°Π½ΠΈΠ΅ Π½Π° систСму ΠΊΠΎΠΎΡ€Π΄ΠΈΠ½Π°Ρ‚. Π§Ρ‚ΠΎΠ±Ρ‹ Π½Π°ΠΉΡ‚ΠΈ смСщСниС, ΠΌΡ‹ ΠΈΡΠΏΠΎΠ»ΡŒΠ·ΡƒΠ΅ΠΌ ΡƒΡ€Π°Π²Π½Π΅Π½ΠΈΠ΅ Ξ”x = xf βˆ’ x0.Ξ”x = xf βˆ’ x0. size 12 {Ξ”x = x rSub {size 8 {f}} — x rSub {size 8 {0}}} {} Π­Ρ‚ΠΎ просто, ΠΏΠΎΡΠΊΠΎΠ»ΡŒΠΊΡƒ Π΄Π°Π½Ρ‹ Π½Π°Ρ‡Π°Π»ΡŒΠ½Π°Ρ ΠΈ конСчная ΠΏΠΎΠ·ΠΈΡ†ΠΈΠΈ.

РСшСниС

1.ΠžΠΏΡ€Π΅Π΄Π΅Π»ΠΈΡ‚Π΅ извСстныС. На рисункС ΠΌΡ‹ Π²ΠΈΠ΄ΠΈΠΌ, Ρ‡Ρ‚ΠΎ xf = 6,70 ΠΊΠΌ xf = 6,70 ΠΊΠΌ ΠΈ x0 = 4,70 ΠΊΠΌ x0 = 4,70 ΠΊΠΌ для части (a) ΠΈ xβ€²f = 3,75 ΠΊΠΌ xβ€²f = 3,75 ΠΊΠΌ ΠΈ xβ€²0 = 5,25 ΠΊΠΌ xβ€²0 = 5,25 ΠΊΠΌ для части (b).

2. НайдитС смСщСниС Π² части (a).

2,12 Ξ”x = xf βˆ’ x0 = 6,70 ΠΊΠΌ βˆ’ 4,70 ΠΊΠΌ = + 2,00 ΠΊΠΌ. Ξ”x = xf βˆ’ x0 = 6,70 ΠΊΠΌ βˆ’ 4,70 ΠΊΠΌ = + 2,00 ΠΊΠΌ. Ρ€Π°Π·ΠΌΠ΅Ρ€ 12 {Ξ”x = x rSub {Ρ€Π°Π·ΠΌΠ΅Ρ€ 8 {f}} — x rSub {Ρ€Π°Π·ΠΌΠ΅Ρ€ 8 {0}} = 6 «.» Β«70 ΠΊΠΌΒ» — 4 дюйма Β». «70 ΠΊΠΌ» «= +» 2 «.» «00 ΠΊΠΌ»} {}

3. НайдитС смСщСниС Π² части (b).

2.13 Ξ”x β€² = xβ€²f βˆ’ xβ€²0 = 3.75 ΠΊΠΌ βˆ’ 5,25 ΠΊΠΌ = βˆ’1,50 ΠΊΠΌ. Ξ”x β€² = xβ€²f βˆ’ xβ€²0 = 3,75 ΠΊΠΌ βˆ’ 5,25 ΠΊΠΌ = βˆ’1,50 ΠΊΠΌ. Ρ€Π°Π·ΠΌΠ΅Ρ€ 12 {Ξ”x ‘= {{x}} sup {‘} rSub {size 8 {f}} — {{x}} sup {‘} rSub {size 8 {0}} = 3 «.» Β«75 ΠΊΠΌΒ» — 5 Β». «25 ΠΊΠΌ» = — 1 «.» «50 ΠΊΠΌ»} {}

ΠžΠ±ΡΡƒΠΆΠ΄Π΅Π½ΠΈΠ΅

НаправлСниС двиТСния Π² (a) — Π²ΠΏΡ€Π°Π²ΠΎ, ΠΈ, ΡΠ»Π΅Π΄ΠΎΠ²Π°Ρ‚Π΅Π»ΡŒΠ½ΠΎ, Π΅Π³ΠΎ смСщСниС ΠΈΠΌΠ΅Π΅Ρ‚ ΠΏΠΎΠ»ΠΎΠΆΠΈΡ‚Π΅Π»ΡŒΠ½Ρ‹ΠΉ Π·Π½Π°ΠΊ, Ρ‚ΠΎΠ³Π΄Π° ΠΊΠ°ΠΊ Π΄Π²ΠΈΠΆΠ΅Π½ΠΈΠ΅ Π² (b) — Π²Π»Π΅Π²ΠΎ ΠΈ, Ρ‚Π°ΠΊΠΈΠΌ ΠΎΠ±Ρ€Π°Π·ΠΎΠΌ, ΠΈΠΌΠ΅Π΅Ρ‚ Π·Π½Π°ΠΊ минус.

ΠŸΡ€ΠΈΠΌΠ΅Ρ€ 2.3 Π‘Ρ€Π°Π²Π½Π΅Π½ΠΈΠ΅ ΠΏΡ€ΠΎΠΉΠ΄Π΅Π½Π½ΠΎΠ³ΠΎ расстояния ΠΈ водоизмСщСния: ΠΏΠΎΠ΅Π·Π΄ ΠΌΠ΅Ρ‚Ρ€ΠΎ

ΠšΠ°ΠΊΠΎΠ²Ρ‹ расстояния, ΠΏΡ€ΠΎΠΉΠ΄Π΅Π½Π½Ρ‹Π΅ Π·Π° двиТСния, ΠΏΠΎΠΊΠ°Π·Π°Π½Π½Ρ‹Π΅ Π² частях (a) ΠΈ (b) ΠΏΠΎΠ΅Π·Π΄Π° ΠΌΠ΅Ρ‚Ρ€ΠΎ Π½Π° РисункС 2.30?

БтратСгия

Π§Ρ‚ΠΎΠ±Ρ‹ ΠΎΡ‚Π²Π΅Ρ‚ΠΈΡ‚ΡŒ Π½Π° этот вопрос, ΠΏΠΎΠ΄ΡƒΠΌΠ°ΠΉΡ‚Π΅ ΠΎΠ± опрСдСлСниях расстояния ΠΈ ΠΏΡ€ΠΎΠΉΠ΄Π΅Π½Π½ΠΎΠ³ΠΎ расстояния ΠΈ ΠΎ Ρ‚ΠΎΠΌ, ΠΊΠ°ΠΊ ΠΎΠ½ΠΈ связаны с ΠΏΠ΅Ρ€Π΅ΠΌΠ΅Ρ‰Π΅Π½ΠΈΠ΅ΠΌ. РасстояниС ΠΌΠ΅ΠΆΠ΄Ρƒ двумя полоТСниями опрСдСляСтся ΠΊΠ°ΠΊ Π²Π΅Π»ΠΈΡ‡ΠΈΠ½Π° смСщСния, которая Π±Ρ‹Π»Π° Π½Π°ΠΉΠ΄Π΅Π½Π° Π² ΠŸΡ€ΠΈΠΌΠ΅Ρ€Π΅ 2.2. ΠŸΡ€ΠΎΠΉΠ΄Π΅Π½Π½ΠΎΠ΅ расстояниС — это общая Π΄Π»ΠΈΠ½Π° ΠΏΡƒΡ‚ΠΈ, ΠΏΡ€ΠΎΠΉΠ΄Π΅Π½Π½ΠΎΠ³ΠΎ ΠΌΠ΅ΠΆΠ΄Ρƒ двумя позициями. Π‘ΠΌ. Π‘ΠΌΠ΅Ρ‰Π΅Π½ΠΈΠ΅. Π’ случаС ΠΏΠΎΠ΅Π·Π΄Π° ΠΌΠ΅Ρ‚Ρ€ΠΎ, ​​показанного Π½Π° рис. 2.30, ΠΏΡ€ΠΎΠΉΠ΄Π΅Π½Π½ΠΎΠ΅ расстояниС Ρ€Π°Π²Π½ΠΎ Ρ€Π°ΡΡΡ‚ΠΎΡΠ½ΠΈΡŽ ΠΌΠ΅ΠΆΠ΄Ρƒ Π½Π°Ρ‡Π°Π»ΡŒΠ½Ρ‹ΠΌ ΠΈ ΠΊΠΎΠ½Π΅Ρ‡Π½Ρ‹ΠΌ полоТСниями ΠΏΠΎΠ΅Π·Π΄Π°.

РСшСниС

1. Π‘ΠΌΠ΅Ρ‰Π΅Π½ΠΈΠ΅ для части (Π°) составило +2,00 ΠΊΠΌ. Π’Π°ΠΊΠΈΠΌ ΠΎΠ±Ρ€Π°Π·ΠΎΠΌ, расстояниС ΠΌΠ΅ΠΆΠ΄Ρƒ Π½Π°Ρ‡Π°Π»ΡŒΠ½ΠΎΠΉ ΠΈ ΠΊΠΎΠ½Π΅Ρ‡Π½ΠΎΠΉ позициями составило 2,00 ΠΊΠΌ, Π° ΠΏΡ€ΠΎΠΉΠ΄Π΅Π½Π½ΠΎΠ΅ расстояниС — 2,00 ΠΊΠΌ.

2. Π‘ΠΌΠ΅Ρ‰Π΅Π½ΠΈΠ΅ части (Π±) составило -1,5 ΠΊΠΌ. Π’Π°ΠΊΠΈΠΌ ΠΎΠ±Ρ€Π°Π·ΠΎΠΌ, расстояниС ΠΌΠ΅ΠΆΠ΄Ρƒ Π½Π°Ρ‡Π°Π»ΡŒΠ½ΠΎΠΉ ΠΈ ΠΊΠΎΠ½Π΅Ρ‡Π½ΠΎΠΉ позициями составляло 1,50 ΠΊΠΌ, Π° ΠΏΡ€ΠΎΠΉΠ΄Π΅Π½Π½ΠΎΠ΅ расстояниС — 1,50 ΠΊΠΌ.

ΠžΠ±ΡΡƒΠΆΠ΄Π΅Π½ΠΈΠ΅

РасстояниС — скаляр.Π£ Π½Π΅Π³ΠΎ Π΅ΡΡ‚ΡŒ Π²Π΅Π»ΠΈΡ‡ΠΈΠ½Π°, Π½ΠΎ Π½Π΅Ρ‚ Π·Π½Π°ΠΊΠ°, ΡƒΠΊΠ°Π·Ρ‹Π²Π°ΡŽΡ‰Π΅Π³ΠΎ Π½Π°ΠΏΡ€Π°Π²Π»Π΅Π½ΠΈΠ΅.

ΠŸΡ€ΠΈΠΌΠ΅Ρ€ 2.4 РасчСт ускорСния: ΠΏΠΎΠ΅Π·Π΄ ΠΌΠ΅Ρ‚Ρ€ΠΎ Π½Π°Π±ΠΈΡ€Π°Π΅Ρ‚ ΡΠΊΠΎΡ€ΠΎΡΡ‚ΡŒ

ΠŸΡ€Π΅Π΄ΠΏΠΎΠ»ΠΎΠΆΠΈΠΌ, Ρ‡Ρ‚ΠΎ ΠΏΠΎΠ΅Π·Π΄ Π½Π° рис. 2.30 (a) ускоряСтся ΠΈΠ· состояния покоя Π΄ΠΎ 30,0 ΠΊΠΌ / Ρ‡ Π·Π° ΠΏΠ΅Ρ€Π²Ρ‹Π΅ 20,0 с своСго двиТСния. Каково Π΅Π³ΠΎ срСднСС ускорСниС Π·Π° этот ΠΏΡ€ΠΎΠΌΠ΅ΠΆΡƒΡ‚ΠΎΠΊ Π²Ρ€Π΅ΠΌΠ΅Π½ΠΈ?

БтратСгия

Π—Π΄Π΅ΡΡŒ стоит ΡΠ΄Π΅Π»Π°Ρ‚ΡŒ простой набросок

Π­Ρ‚Π° ΠΏΡ€ΠΎΠ±Π»Π΅ΠΌΠ° состоит ΠΈΠ· Ρ‚Ρ€Π΅Ρ… этапов.Π‘Π½Π°Ρ‡Π°Π»Π° ΠΌΡ‹ Π΄ΠΎΠ»ΠΆΠ½Ρ‹ ΠΎΠΏΡ€Π΅Π΄Π΅Π»ΠΈΡ‚ΡŒ ΠΈΠ·ΠΌΠ΅Π½Π΅Π½ΠΈΠ΅ скорости, Π·Π°Ρ‚Π΅ΠΌ ΠΌΡ‹ Π΄ΠΎΠ»ΠΆΠ½Ρ‹ ΠΎΠΏΡ€Π΅Π΄Π΅Π»ΠΈΡ‚ΡŒ ΠΈΠ·ΠΌΠ΅Π½Π΅Π½ΠΈΠ΅ Π²ΠΎ Π²Ρ€Π΅ΠΌΠ΅Π½ΠΈ ΠΈ, Π½Π°ΠΊΠΎΠ½Π΅Ρ†, ΠΌΡ‹ Π΄ΠΎΠ»ΠΆΠ½Ρ‹ ΠΈΡΠΏΠΎΠ»ΡŒΠ·ΠΎΠ²Π°Ρ‚ΡŒ эти значСния для расчСта ускорСния.

РСшСниС

1. ΠžΠΏΡ€Π΅Π΄Π΅Π»ΠΈΡ‚Π΅ извСстныС. v0 = 0v0 = 0 Ρ€Π°Π·ΠΌΠ΅Ρ€ 12 {v rSub {size 8 {0}} = 0} {} (ΠΏΠΎΠ΅Π·Π΄Π° Π·Π°ΠΏΡƒΡΠΊΠ°ΡŽΡ‚ΡΡ Π² состоянии покоя), vf = 30,0 ΠΊΠΌ / Ρ‡, vf = 30,0 ΠΊΠΌ / Ρ‡, Ρ€Π°Π·ΠΌΠ΅Ρ€ 12 {v rSub { Ρ€Π°Π·ΠΌΠ΅Ρ€ 8 {f}} = «30» «.» Β«0 ΠΊΠΌ / Ρ‡Β»} {} ΠΈ Ξ”t = 20,0 с.Ξ”t = 20,0 с. Ρ€Π°Π·ΠΌΠ΅Ρ€ 12 {Ξ”t = «20» «.» «0 с»} {}

2. РассчитайтС Ξ”v.Ξ”v. Ρ€Π°Π·ΠΌΠ΅Ρ€ 12 {Ξ”v} {} ΠŸΠΎΡΠΊΠΎΠ»ΡŒΠΊΡƒ ΠΏΠΎΠ΅Π·Π΄ трогаСтся с мСста, Π΅Π³ΠΎ ΡΠΊΠΎΡ€ΠΎΡΡ‚ΡŒ измСняСтся Ξ”v = + 30.0 ΠΊΠΌ / Ρ‡, Ξ”v = + 30,0 ΠΊΠΌ / Ρ‡, Ρ€Π°Π·ΠΌΠ΅Ρ€ 12 {Ξ”v «= +» «30» «.» 0` «km / h»} {} Π³Π΄Π΅ Π·Π½Π°ΠΊ плюс ΠΎΠ·Π½Π°Ρ‡Π°Π΅Ρ‚ ΡΠΊΠΎΡ€ΠΎΡΡ‚ΡŒ Π²ΠΏΡ€Π°Π²ΠΎ.

3. ΠŸΠΎΠ΄ΡΡ‚Π°Π²ΡŒΡ‚Π΅ извСстныС значСния ΠΈ Π½Π°ΠΉΠ΄ΠΈΡ‚Π΅ нСизвСстноС, Ρ€Π°Π·ΠΌΠ΅Ρ€ a-a-12 {{bar {a}}} {}.

2.14 a- = Ξ”vΞ”t = + 30,0 ΠΊΠΌ / Ρ‡ 30,0 с.a- = Ξ”vΞ”t = + 30,0 ΠΊΠΌ / Ρ‡ 30,0 с. Ρ€Π°Π·ΠΌΠ΅Ρ€ 12 {{bar {a}} = {{Ξ”v} over {Ξ”t}} = {{+ «30» «.» 0` «ΠΊΠΌ / Ρ‡»} большС {«20» «.» 0`s}}} {}

4. ΠŸΠΎΡΠΊΠΎΠ»ΡŒΠΊΡƒ Π΅Π΄ΠΈΠ½ΠΈΡ†Ρ‹ измСрСния ΡΠΌΠ΅ΡˆΠ°Π½Π½Ρ‹Π΅ (Ρƒ нас Π΅ΡΡ‚ΡŒ часы ΠΈ сСкунды для Π²Ρ€Π΅ΠΌΠ΅Π½ΠΈ), Π½Π°ΠΌ Π½ΡƒΠΆΠ½ΠΎ ΠΏΡ€Π΅ΠΎΠ±Ρ€Π°Π·ΠΎΠ²Π°Ρ‚ΡŒ всС Π² Π΅Π΄ΠΈΠ½ΠΈΡ†Ρ‹ измСрСния БИ — ΠΌΠ΅Ρ‚Ρ€Ρ‹ ΠΈ сСкунды.Π‘ΠΌ. Π”ΠΎΠΏΠΎΠ»Π½ΠΈΡ‚Π΅Π»ΡŒΠ½Ρ‹Π΅ указания Π² Ρ€Π°Π·Π΄Π΅Π»Π΅ «ЀизичСскиС Π²Π΅Π»ΠΈΡ‡ΠΈΠ½Ρ‹ ΠΈ Π΅Π΄ΠΈΠ½ΠΈΡ†Ρ‹Β».

2,15 a — = + 30 ΠΊΠΌ / Ρ‡ 30,0 с103 ΠΌ1 ΠΊΠΌ1 h4,600 с = 0,417 ΠΌ / с2a — = + 30 ΠΊΠΌ / Ρ‡ 30,0 с103 ΠΌ1 ΠΊΠΌ1 Ρ‡4,600 с = 0,417 ΠΌ / с2 Ρ€Π°Π·ΠΌΠ΅Ρ€ 12 {{bar {a}} = left ({{+ «30 ΠΊΠΌ / Ρ‡»} большС {«20» «.» «0 s»}} справа) left ({{«10» rSup {size 8 {3}} «m «} Π±ΠΎΠ»Π΅Π΅ {» 1 ΠΊΠΌ «}} справа) слСва ({{» 1 Ρ‡ «} Π±ΠΎΠ»Π΅Π΅ {» 3600 с «}} справа) = 0″. » «417 ΠΌ / с» rSup {Ρ€Π°Π·ΠΌΠ΅Ρ€ 8 {2}}} {}

ΠžΠ±ΡΡƒΠΆΠ΄Π΅Π½ΠΈΠ΅

Π—Π½Π°ΠΊ плюс ΠΎΠ·Π½Π°Ρ‡Π°Π΅Ρ‚, Ρ‡Ρ‚ΠΎ ускорСниС Π½Π°ΠΏΡ€Π°Π²ΠΎ.Π­Ρ‚ΠΎ Ρ€Π°Π·ΡƒΠΌΠ½ΠΎ, ΠΏΠΎΡ‚ΠΎΠΌΡƒ Ρ‡Ρ‚ΠΎ ΠΏΠΎΠ΅Π·Π΄ стартуСт ΠΈΠ· состояния покоя ΠΈ Π·Π°ΠΊΠ°Π½Ρ‡ΠΈΠ²Π°Π΅Ρ‚ со ΡΠΊΠΎΡ€ΠΎΡΡ‚ΡŒΡŽ Π²ΠΏΡ€Π°Π²ΠΎ (Ρ‚ΠΎΠΆΠ΅ ΠΏΠΎΠ»ΠΎΠΆΠΈΡ‚Π΅Π»ΡŒΠ½ΠΎΠΉ). Π’Π°ΠΊΠΈΠΌ ΠΎΠ±Ρ€Π°Π·ΠΎΠΌ, ускорСниС происходит Π² Ρ‚ΠΎΠΌ ΠΆΠ΅ Π½Π°ΠΏΡ€Π°Π²Π»Π΅Π½ΠΈΠΈ, Ρ‡Ρ‚ΠΎ ΠΈ , измСняя ΡΠΊΠΎΡ€ΠΎΡΡ‚ΡŒ Π½Π° , ΠΊΠ°ΠΊ всСгда.

ΠŸΡ€ΠΈΠΌΠ΅Ρ€ 2.5 РасчСт ускорСния: Π·Π°ΠΌΠ΅Π΄Π»Π΅Π½ΠΈΠ΅ ΠΏΠΎΠ΅Π·Π΄Π° ΠΌΠ΅Ρ‚Ρ€ΠΎ

Π’Π΅ΠΏΠ΅Ρ€ΡŒ ΠΏΡ€Π΅Π΄ΠΏΠΎΠ»ΠΎΠΆΠΈΠΌ, Ρ‡Ρ‚ΠΎ Π² ΠΊΠΎΠ½Ρ†Π΅ ΠΏΠΎΠ΅Π·Π΄ΠΊΠΈ ΠΏΠΎΠ΅Π·Π΄ Π½Π° рис. 2.30 (a) замСдляСтся Π΄ΠΎ остановки со скорости 30,0 ΠΊΠΌ / Ρ‡ Π·Π° 8,00 с. КакоС Ρƒ Π½Π΅Π³ΠΎ срСднСС ускорСниС ΠΏΡ€ΠΈ остановкС?

БтратСгия

Π’ этом случаС ΠΏΠΎΠ΅Π·Π΄ замСдляСтся, Π° Π΅Π³ΠΎ ускорСниС ΠΎΡ‚Ρ€ΠΈΡ†Π°Ρ‚Π΅Π»ΡŒΠ½ΠΎΠ΅, ΠΏΠΎΡ‚ΠΎΠΌΡƒ Ρ‡Ρ‚ΠΎ ΠΎΠ½ Π½Π°ΠΏΡ€Π°Π²Π»Π΅Π½ Π²Π»Π΅Π²ΠΎ.Как ΠΈ Π² ΠΏΡ€Π΅Π΄Ρ‹Π΄ΡƒΡ‰Π΅ΠΌ ΠΏΡ€ΠΈΠΌΠ΅Ρ€Π΅, ΠΌΡ‹ Π΄ΠΎΠ»ΠΆΠ½Ρ‹ Π½Π°ΠΉΡ‚ΠΈ ΠΈΠ·ΠΌΠ΅Π½Π΅Π½ΠΈΠ΅ скорости ΠΈ ΠΈΠ·ΠΌΠ΅Π½Π΅Π½ΠΈΠ΅ Π²ΠΎ Π²Ρ€Π΅ΠΌΠ΅Π½ΠΈ, Π° Π·Π°Ρ‚Π΅ΠΌ Π²Ρ‹Ρ‡ΠΈΡΠ»ΠΈΡ‚ΡŒ ускорСниС.

РСшСниС

1. ΠžΠΏΡ€Π΅Π΄Π΅Π»ΠΈΡ‚Π΅ извСстныС. v0 = 30,0 ΠΊΠΌ / Ρ‡, v0 = 30,0 ΠΊΠΌ / Ρ‡, vf = 0 ΠΊΠΌ / Ρ‡ — vf = 0 ΠΊΠΌ / Ρ‡ — ΠΏΠΎΠ΅Π·Π΄ остановлСн, поэтому Π΅Π³ΠΎ ΡΠΊΠΎΡ€ΠΎΡΡ‚ΡŒ Ρ€Π°Π²Π½Π° 0 — ΠΈ Ξ”t = 8,00 с.Ξ”t = 8,00 с.

2. НайдитС ΠΈΠ·ΠΌΠ΅Π½Π΅Π½ΠΈΠ΅ скорости Ξ”v.Ξ”v. Ρ€Π°Π·ΠΌΠ΅Ρ€ 12 {Ξ”v} {}

2,16 Ξ”v = vf-v0 = 0-30,0 ΠΊΠΌ / Ρ‡ = -30,0 ΠΊΠΌ / Ρ‡. Ξ”v = vf-v0 = 0-30,0 ΠΊΠΌ / Ρ‡ = -30,0 ΠΊΠΌ / Ρ‡. Ρ€Π°Π·ΠΌΠ΅Ρ€ 12 {Ξ”v = v rSub {Ρ€Π°Π·ΠΌΠ΅Ρ€ 8 {f}} — v rSub {Ρ€Π°Π·ΠΌΠ΅Ρ€ 8 {0}} = 0 — «30» «.»» 0 ΠΊΠΌ / Ρ‡ «= -» 30 «». «» 0 ΠΊΠΌ / Ρ‡ «} {}

3. ΠŸΠΎΠ΄ΠΊΠ»ΡŽΡ‡ΠΈΡ‚Π΅ извСстныС значСния Ξ”vΞ”v, Ρ€Π°Π·ΠΌΠ΅Ρ€ 12 {Ξ”v} {} ΠΈ Ξ”t, Ξ”t, ΠΈ Ρ€Π΅ΡˆΠΈΡ‚Π΅ для aa-

2,17 a- = Ξ”vΞ”t = βˆ’30,0 ΠΊΠΌ / Ρ‡ 8,00 sa- = Ξ”vΞ”t = βˆ’30,0 ΠΊΠΌ / Ρ‡ 8,00 с. Π Π°Π·ΠΌΠ΅Ρ€ 12 {{bar {a}} = {{Ξ”v} over {Ξ”t}} = { {- «30» «.» «0 ΠΊΠΌ / Ρ‡»} Π±ΠΎΠ»Π΅Π΅ {8 «.» «00 с»}}} {}

4. ΠŸΡ€Π΅ΠΎΠ±Ρ€Π°Π·ΠΎΠ²Π°Π½ΠΈΠ΅ Π΅Π΄ΠΈΠ½ΠΈΡ† Π² ΠΌΠ΅Ρ‚Ρ€Ρ‹ ΠΈ сСкунды.

2,18 a- = Ξ”vΞ”t = βˆ’30,0 ΠΊΠΌ / Ρ‡ 8,00 с103 ΠΌ1 ΠΊΠΌ1 h4,600 с = βˆ’1,04 ΠΌ / с2 .a- = Ξ”vΞ”t = βˆ’30,0 ΠΊΠΌ / Ρ‡ 8,00 с103 ΠΌ1 ΠΊΠΌ1 h4,600 с = βˆ’1,04 ΠΌ / с2. Ρ€Π°Π·ΠΌΠ΅Ρ€ 12 {{bar {a}} = {{Ξ”v} over {Ξ”t}} = left ({{- «30» «.»» 0 ΠΊΠΌ / Ρ‡ «} большС {8». «» 00 s «}} справа) слСва ({{» 10 «rSup {size 8 {3}}» m «} большС {» 1 ΠΊΠΌ «}} справа ) слСва ({{«1 час»} большС {«3600 с»}} справа) = — 1 «.» «04 ΠΌ / с» rSup {Ρ€Π°Π·ΠΌΠ΅Ρ€ 8 {2}} «.»} {}

ΠžΠ±ΡΡƒΠΆΠ΄Π΅Π½ΠΈΠ΅

Π—Π½Π°ΠΊ минус ΡƒΠΊΠ°Π·Ρ‹Π²Π°Π΅Ρ‚ Π½Π° Ρ‚ΠΎ, Ρ‡Ρ‚ΠΎ ускорСниС происходит Π²Π»Π΅Π²ΠΎ. Π­Ρ‚ΠΎΡ‚ Π·Π½Π°ΠΊ Ρ€Π°Π·ΡƒΠΌΠ΅Π½, ΠΏΠΎΡ‚ΠΎΠΌΡƒ Ρ‡Ρ‚ΠΎ ΠΏΠΎΠ΅Π·Π΄ ΠΈΠ·Π½Π°Ρ‡Π°Π»ΡŒΠ½ΠΎ ΠΈΠΌΠ΅Π΅Ρ‚ ΠΏΠΎΠ»ΠΎΠΆΠΈΡ‚Π΅Π»ΡŒΠ½ΡƒΡŽ ΡΠΊΠΎΡ€ΠΎΡΡ‚ΡŒ Π² этой Π·Π°Π΄Π°Ρ‡Π΅, Π° ΠΎΡ‚Ρ€ΠΈΡ†Π°Ρ‚Π΅Π»ΡŒΠ½ΠΎΠ΅ ускорСниС Π±ΡƒΠ΄Π΅Ρ‚ ΠΏΡ€Π΅ΠΏΡΡ‚ΡΡ‚Π²ΠΎΠ²Π°Ρ‚ΡŒ двиТСнию. ΠžΠΏΡΡ‚ΡŒ ΠΆΠ΅, ускорСниС происходит Π² Ρ‚ΠΎΠΌ ΠΆΠ΅ Π½Π°ΠΏΡ€Π°Π²Π»Π΅Π½ΠΈΠΈ, Ρ‡Ρ‚ΠΎ ΠΈ ΠΈΠ·ΠΌΠ΅Π½Π΅Π½ΠΈΠ΅ скорости Π½Π° , Ρ‡Ρ‚ΠΎ здСсь ΠΎΡ‚Ρ€ΠΈΡ†Π°Ρ‚Π΅Π»ΡŒΠ½ΠΎ.Π­Ρ‚ΠΎ ускорСниС ΠΌΠΎΠΆΠ½ΠΎ Π½Π°Π·Π²Π°Ρ‚ΡŒ Π·Π°ΠΌΠ΅Π΄Π»Π΅Π½ΠΈΠ΅ΠΌ, ΠΏΠΎΡ‚ΠΎΠΌΡƒ Ρ‡Ρ‚ΠΎ ΠΎΠ½ΠΎ ΠΈΠΌΠ΅Π΅Ρ‚ Π½Π°ΠΏΡ€Π°Π²Π»Π΅Π½ΠΈΠ΅, ΠΏΡ€ΠΎΡ‚ΠΈΠ²ΠΎΠΏΠΎΠ»ΠΎΠΆΠ½ΠΎΠ΅ скорости.

Π“Ρ€Π°Ρ„ΠΈΠΊΠΈ полоТСния, скорости ΠΈ ускорСния Π² зависимости ΠΎΡ‚ Π²Ρ€Π΅ΠΌΠ΅Π½ΠΈ для ΠΏΠΎΠ΅Π·Π΄ΠΎΠ² Π² ΠŸΡ€ΠΈΠΌΠ΅Ρ€Π΅ 2.4 ΠΈ ΠŸΡ€ΠΈΠΌΠ΅Ρ€Π΅ 2.5 ΠΏΠΎΠΊΠ°Π·Π°Π½Ρ‹ Π½Π° РисункС 2.33. ΠœΡ‹ приняли ΡΠΊΠΎΡ€ΠΎΡΡ‚ΡŒ постоянной ΠΎΡ‚ 20 Π΄ΠΎ 40 с, послС Ρ‡Π΅Π³ΠΎ ΠΏΠΎΠ΅Π·Π΄ замСдляСтся.

Рис. 2.33 (a) ПолоТСниС ΠΏΠΎΠ΅Π·Π΄Π° Π²ΠΎ Π²Ρ€Π΅ΠΌΠ΅Π½ΠΈ. ΠžΠ±Ρ€Π°Ρ‚ΠΈΡ‚Π΅ Π²Π½ΠΈΠΌΠ°Π½ΠΈΠ΅, Ρ‡Ρ‚ΠΎ ΠΏΠΎΠ»ΠΎΠΆΠ΅Π½ΠΈΠ΅ ΠΏΠΎΠ΅Π·Π΄Π° мСняСтся ΠΌΠ΅Π΄Π»Π΅Π½Π½ΠΎ Π² Π½Π°Ρ‡Π°Π»Π΅ ΠΏΡƒΡ‚ΠΈ, Π° Π·Π°Ρ‚Π΅ΠΌ всС быстрСС ΠΈ быстрСС, ΠΊΠΎΠ³Π΄Π° ΠΎΠ½ Π½Π°Π±ΠΈΡ€Π°Π΅Ρ‚ ΡΠΊΠΎΡ€ΠΎΡΡ‚ΡŒ.Π—Π°Ρ‚Π΅ΠΌ Π΅Π³ΠΎ ΠΏΠΎΠ»ΠΎΠΆΠ΅Π½ΠΈΠ΅ мСняСтся ΠΌΠ΅Π΄Π»Π΅Π½Π½Π΅Π΅ ΠΏΠΎ ΠΌΠ΅Ρ€Π΅ замСдлСния Π² ΠΊΠΎΠ½Ρ†Π΅ ΠΏΡƒΡ‚ΠΈ. Π’ сСрСдинС ΠΏΡƒΡ‚ΠΈ, ΠΊΠΎΠ³Π΄Π° ΡΠΊΠΎΡ€ΠΎΡΡ‚ΡŒ остаСтся постоянной, ΠΏΠΎΠ»ΠΎΠΆΠ΅Π½ΠΈΠ΅ мСняСтся с постоянной ΡΠΊΠΎΡ€ΠΎΡΡ‚ΡŒΡŽ. (Π±) Π‘ΠΊΠΎΡ€ΠΎΡΡ‚ΡŒ ΠΏΠΎΠ΅Π·Π΄Π° Π²ΠΎ Π²Ρ€Π΅ΠΌΠ΅Π½ΠΈ. Π‘ΠΊΠΎΡ€ΠΎΡΡ‚ΡŒ ΠΏΠΎΠ΅Π·Π΄Π° увСличиваСтся ΠΏΠΎ ΠΌΠ΅Ρ€Π΅ Π΅Π³ΠΎ ускорСния Π² Π½Π°Ρ‡Π°Π»Π΅ ΠΏΡƒΡ‚ΠΈ. Он остаСтся Π½Π΅ΠΈΠ·ΠΌΠ΅Π½Π½Ρ‹ΠΌ Π² сСрСдинС ΠΏΡƒΡ‚ΠΈ, ΠΊΠΎΠ³Π΄Π° Π½Π΅Ρ‚ ускорСния. Оно ΡƒΠΌΠ΅Π½ΡŒΡˆΠ°Π΅Ρ‚ΡΡ ΠΏΠΎ ΠΌΠ΅Ρ€Π΅ замСдлСния ΠΏΠΎΠ΅Π·Π΄Π° Π² ΠΊΠΎΠ½Ρ†Π΅ ΠΏΡƒΡ‚ΠΈ. (c) УскорСниС ΠΏΠΎΠ΅Π·Π΄Π° с Ρ‚Π΅Ρ‡Π΅Π½ΠΈΠ΅ΠΌ Π²Ρ€Π΅ΠΌΠ΅Π½ΠΈ. ПоСзд ΠΈΠΌΠ΅Π΅Ρ‚ ΠΏΠΎΠ»ΠΎΠΆΠΈΡ‚Π΅Π»ΡŒΠ½ΠΎΠ΅ ускорСниС, ΠΏΠΎΡΠΊΠΎΠ»ΡŒΠΊΡƒ ΠΎΠ½ ускоряСтся Π² Π½Π°Ρ‡Π°Π»Π΅ ΠΏΡƒΡ‚ΠΈ.Π£ Π½Π΅Π³ΠΎ Π½Π΅Ρ‚ ускорСния, ΠΏΠΎΡΠΊΠΎΠ»ΡŒΠΊΡƒ ΠΎΠ½ двиТСтся с постоянной ΡΠΊΠΎΡ€ΠΎΡΡ‚ΡŒΡŽ Π² сСрСдинС ΠΏΡƒΡ‚ΠΈ. Π•Π³ΠΎ ускорСниС ΠΎΡ‚Ρ€ΠΈΡ†Π°Ρ‚Π΅Π»ΡŒΠ½ΠΎΠ΅, Ρ‚Π°ΠΊ ΠΊΠ°ΠΊ ΠΎΠ½ замСдляСтся Π² ΠΊΠΎΠ½Ρ†Π΅ ΠΏΡƒΡ‚ΠΈ.

ΠŸΡ€ΠΈΠΌΠ΅Ρ€ 2.6 РасчСт срСднСй скорости: ΠΏΠΎΠ΅Π·Π΄ ΠΌΠ΅Ρ‚Ρ€ΠΎ

Какова срСдняя ΡΠΊΠΎΡ€ΠΎΡΡ‚ΡŒ ΠΏΠΎΠ΅Π·Π΄Π° Π² части b ΠΏΡ€ΠΈΠΌΠ΅Ρ€Π° 2.2, которая снова ΠΏΠΎΠΊΠ°Π·Π°Π½Π° Π½ΠΈΠΆΠ΅, Ссли ΠΏΠΎΠ΅Π·Π΄ΠΊΠ° Π·Π°Π½ΠΈΠΌΠ°Π΅Ρ‚ 5,00 ΠΌΠΈΠ½ΡƒΡ‚?

БтратСгия

БрСдняя ΡΠΊΠΎΡ€ΠΎΡΡ‚ΡŒ — это смСщСниС, Ρ€Π°Π·Π΄Π΅Π»Π΅Π½Π½ΠΎΠ΅ Π½Π° врСмя. Π—Π΄Π΅ΡΡŒ ΠΎΠ½ Π±ΡƒΠ΄Π΅Ρ‚ ΠΎΡ‚Ρ€ΠΈΡ†Π°Ρ‚Π΅Π»ΡŒΠ½Ρ‹ΠΌ, Ρ‚Π°ΠΊ ΠΊΠ°ΠΊ ΠΏΠΎΠ΅Π·Π΄ двиТСтся Π²Π»Π΅Π²ΠΎ ΠΈ ΠΈΠΌΠ΅Π΅Ρ‚ ΠΎΡ‚Ρ€ΠΈΡ†Π°Ρ‚Π΅Π»ΡŒΠ½ΠΎΠ΅ смСщСниС.

РСшСниС

1. ΠžΠΏΡ€Π΅Π΄Π΅Π»ΠΈΡ‚Π΅ извСстныС. xβ€²f = 3,75 ΠΊΠΌ, xβ€²f = 3,75 ΠΊΠΌ, xβ€²0 = 5,25 ΠΊΠΌ, xβ€²0 = 5,25 ΠΊΠΌ, Ξ”t = 5,00 ΠΌΠΈΠ½. Ξ”t = 5,00 ΠΌΠΈΠ½.

2. ΠžΠΏΡ€Π΅Π΄Π΅Π»ΠΈΡ‚Π΅ ΠΏΠ΅Ρ€Π΅ΠΌΠ΅Ρ‰Π΅Π½ΠΈΠ΅ Ξ”xβ€².Ξ”x β€². Π’ ΠΏΡ€ΠΈΠΌΠ΅Ρ€Π΅ 2.2 ΠΌΡ‹ ΠΎΠ±Π½Π°Ρ€ΡƒΠΆΠΈΠ»ΠΈ, Ρ‡Ρ‚ΠΎ Ξ”xβ€²Ξ”x β€² составляСт βˆ’1,5 ΠΊΠΌ.

3. НайдитС ΡΡ€Π΅Π΄Π½ΡŽΡŽ ΡΠΊΠΎΡ€ΠΎΡΡ‚ΡŒ.

2,19 v- = Ξ”xβ€²Ξ”t = βˆ’1,50 ΠΊΠΌ 5,00 ΠΌΠΈΠ½. V- = Ξ”xβ€²Ξ”t = βˆ’1,50 ΠΊΠΌ 5,00 ΠΌΠΈΠ½. Ρ€Π°Π·ΠΌΠ΅Ρ€ 12 {{bar {v}} = {{Ξ” {{x}} sup {‘}} Π½Π°Π΄ {Ξ”t}} = {{- 1 «.» «50 ΠΊΠΌ»} большС {5 «.» «00 ΠΌΠΈΠ½»}}} {}

4.ΠŸΡ€Π΅ΠΎΠ±Ρ€Π°Π·ΠΎΠ²Π°Π½ΠΈΠ΅ Π΅Π΄ΠΈΠ½ΠΈΡ†.

2,20 v- = Ξ”xβ€²Ξ”t = βˆ’1,50 ΠΊΠΌ 5,00 ΠΌΠΈΠ½ 60 ΠΌΠΈΠ½1 h = βˆ’18,0 ΠΊΠΌ / Ρ‡. V- = Ξ”xβ€²Ξ”t = βˆ’1,50 ΠΊΠΌ 5,00 ΠΌΠΈΠ½60 ΠΌΠΈΠ½1 h = βˆ’18,0 ΠΊΠΌ / Ρ‡. Ρ€Π°Π·ΠΌΠ΅Ρ€ 12 {{bar {v}} = {{Ξ”x ‘} большС {Ξ”t}} = слСва ({{- 1 «.» «» 50 «» «ΠΊΠΌ»} большС {5 «.» «00» `» ΠΌΠΈΠ½ «}} right) left ({{» 60 «` «min»} большС {1`h}} right) = — «18» «.» 0` «ΠΊΠΌ / Ρ‡»} {}

ΠžΠ±ΡΡƒΠΆΠ΄Π΅Π½ΠΈΠ΅

ΠžΡ‚Ρ€ΠΈΡ†Π°Ρ‚Π΅Π»ΡŒΠ½Π°Ρ ΡΠΊΠΎΡ€ΠΎΡΡ‚ΡŒ ΡƒΠΊΠ°Π·Ρ‹Π²Π°Π΅Ρ‚ Π½Π° Π΄Π²ΠΈΠΆΠ΅Π½ΠΈΠ΅ Π²Π»Π΅Π²ΠΎ.

ΠŸΡ€ΠΈΠΌΠ΅Ρ€ 2.7 РасчСт замСдлСния: ΠΏΠΎΠ΅Π·Π΄ ΠΌΠ΅Ρ‚Ρ€ΠΎ

НаконСц, ΠΏΡ€Π΅Π΄ΡΡ‚Π°Π²ΡŒΡ‚Π΅ ΠΏΠΎΠ΅Π·Π΄ Π½Π° РисункС 2.34 замСдляСтся Π΄ΠΎ ΠΏΠΎΠ»Π½ΠΎΠΉ остановки со скорости 20,0 ΠΊΠΌ / Ρ‡ Π·Π° 10,0 с. КакоС Ρƒ Π½Π΅Π³ΠΎ срСднСС ускорСниС?

БтратСгия

Π•Ρ‰Π΅ Ρ€Π°Π· нарисуСм скСтч:

Как ΠΈ Ρ€Π°Π½ΡŒΡˆΠ΅, ΠΌΡ‹ Π΄ΠΎΠ»ΠΆΠ½Ρ‹ Π½Π°ΠΉΡ‚ΠΈ ΠΈΠ·ΠΌΠ΅Π½Π΅Π½ΠΈΠ΅ скорости ΠΈ ΠΈΠ·ΠΌΠ΅Π½Π΅Π½ΠΈΠ΅ Π²ΠΎ Π²Ρ€Π΅ΠΌΠ΅Π½ΠΈ, Ρ‡Ρ‚ΠΎΠ±Ρ‹ Π²Ρ‹Ρ‡ΠΈΡΠ»ΠΈΡ‚ΡŒ срСднСС ускорСниС.

РСшСниС

1. ΠžΠΏΡ€Π΅Π΄Π΅Π»ΠΈΡ‚Π΅ извСстныС. v0 = -20 ΠΊΠΌ / Ρ‡, v0 = -20 ΠΊΠΌ / Ρ‡, vf = 0 ΠΊΠΌ / Ρ‡, vf = 0 ΠΊΠΌ / Ρ‡, Ξ”t = 10,0 с. Ξ”t = 10,0 с.

2.РассчитайтС Ξ”vΞ”v Ρ€Π°Π·ΠΌΠ΅Ρ€ 12 {Ξ”v} {}. ИзмСнСниС скорости здСсь фактичСски ΠΏΠΎΠ»ΠΎΠΆΠΈΡ‚Π΅Π»ΡŒΠ½ΠΎΠ΅, Ρ‚Π°ΠΊ ΠΊΠ°ΠΊ

2,21 Ξ”v = vf βˆ’ v0 = 0βˆ’βˆ’20 ΠΊΠΌ / Ρ‡ = + 20 ΠΊΠΌ / час. Ξ”v = vf βˆ’ v0 = 0βˆ’βˆ’20 ΠΊΠΌ / Ρ‡ = + 20 ΠΊΠΌ / Ρ‡. час Ρ€Π°Π·ΠΌΠ΅Ρ€ 12 {Ξ”v = v rSub {size 8 {f}} — v rSub {size 8 {0}} = 0 — left (- «20 ΠΊΠΌ / Ρ‡» справа) «= +» «20 ΠΊΠΌ / Ρ‡»} { }

3. НайдитС Ρ€Π°Π·ΠΌΠ΅Ρ€ aa- 12 {{bar {a}}} {}.

2,22 a- = Ξ”vΞ”t = + 20,0 ΠΊΠΌ / Ρ‡ 20,0 с.a- = Ξ”vΞ”t = + 20,0 ΠΊΠΌ / Ρ‡ 20,0 с.

4. ΠŸΠ΅Ρ€Π΅Π²Π΅ΡΡ‚ΠΈ Π΅Π΄ΠΈΠ½ΠΈΡ†Ρ‹.

2,23 Π° — = + 20,0 ΠΊΠΌ / Ρ‡ 20,0 с103 ΠΌ1 ΠΊΠΌ1 Ρ‡ 4600 с = + 0,556 ΠΌ / с2, Π° — = + 20,0 ΠΊΠΌ / Ρ‡ 20,0 с103 ΠΌ1 ΠΊΠΌ1 Ρ‡ 4600 с = + 0.556 ΠΌ / с2,

ΠžΠ±ΡΡƒΠΆΠ΄Π΅Π½ΠΈΠ΅

Π—Π½Π°ΠΊ плюс ΠΎΠ·Π½Π°Ρ‡Π°Π΅Ρ‚, Ρ‡Ρ‚ΠΎ ускорСниС Π½Π°ΠΏΡ€Π°Π²ΠΎ. Π­Ρ‚ΠΎ Ρ€Π°Π·ΡƒΠΌΠ½ΠΎ, ΠΏΠΎΡ‚ΠΎΠΌΡƒ Ρ‡Ρ‚ΠΎ Ρƒ ΠΏΠΎΠ΅Π·Π΄Π° ΠΈΠ·Π½Π°Ρ‡Π°Π»ΡŒΠ½ΠΎ ΠΎΡ‚Ρ€ΠΈΡ†Π°Ρ‚Π΅Π»ΡŒΠ½Π°Ρ ΡΠΊΠΎΡ€ΠΎΡΡ‚ΡŒ (слСва) Π² этой Π·Π°Π΄Π°Ρ‡Π΅, Π° ΠΏΠΎΠ»ΠΎΠΆΠΈΡ‚Π΅Π»ΡŒΠ½ΠΎΠ΅ ускорСниС противодСйствуСт двиТСнию, Ρ‚ΠΎ Π΅ΡΡ‚ΡŒ справа. ΠžΠΏΡΡ‚ΡŒ ΠΆΠ΅, ускорСниС происходит Π² Ρ‚ΠΎΠΌ ΠΆΠ΅ Π½Π°ΠΏΡ€Π°Π²Π»Π΅Π½ΠΈΠΈ, Ρ‡Ρ‚ΠΎ ΠΈ ΠΈΠ·ΠΌΠ΅Π½Π΅Π½ΠΈΠ΅ скорости Π½Π° , Ρ‡Ρ‚ΠΎ здСсь ΠΏΠΎΠ»ΠΎΠΆΠΈΡ‚Π΅Π»ΡŒΠ½ΠΎ. Как ΠΈ Π² ΠΏΡ€ΠΈΠΌΠ΅Ρ€Π΅ 2.5, это ускорСниС ΠΌΠΎΠΆΠ½ΠΎ Π½Π°Π·Π²Π°Ρ‚ΡŒ Π·Π°ΠΌΠ΅Π΄Π»Π΅Π½ΠΈΠ΅ΠΌ, ΠΏΠΎΡΠΊΠΎΠ»ΡŒΠΊΡƒ ΠΎΠ½ΠΎ происходит Π² Π½Π°ΠΏΡ€Π°Π²Π»Π΅Π½ΠΈΠΈ, ΠΏΡ€ΠΎΡ‚ΠΈΠ²ΠΎΠΏΠΎΠ»ΠΎΠΆΠ½ΠΎΠΌ скорости.

Как ΠΎΠΏΡ€Π΅Π΄Π΅Π»ΠΈΡ‚ΡŒ ускорСниС

УскорСниС — это ΡΠΊΠΎΡ€ΠΎΡΡ‚ΡŒ измСнСния скорости ΠΊΠ°ΠΊ функция Π²Ρ€Π΅ΠΌΠ΅Π½ΠΈ. Π­Ρ‚ΠΎ Π²Π΅ΠΊΡ‚ΠΎΡ€, Ρ‚ΠΎ Π΅ΡΡ‚ΡŒ ΠΎΠ½ ΠΈΠΌΠ΅Π΅Ρ‚ ΠΊΠ°ΠΊ Π²Π΅Π»ΠΈΡ‡ΠΈΠ½Ρƒ, Ρ‚Π°ΠΊ ΠΈ Π½Π°ΠΏΡ€Π°Π²Π»Π΅Π½ΠΈΠ΅. Он измСряСтся Π² ΠΌΠ΅Ρ‚Ρ€Π°Ρ… Π² сСкунду Π² ΠΊΠ²Π°Π΄Ρ€Π°Ρ‚Π΅ ΠΈΠ»ΠΈ Π² ΠΌΠ΅Ρ‚Ρ€Π°Ρ… Π² сСкунду (ΡΠΊΠΎΡ€ΠΎΡΡ‚ΡŒ ΠΈΠ»ΠΈ ΡΠΊΠΎΡ€ΠΎΡΡ‚ΡŒ ΠΎΠ±ΡŠΠ΅ΠΊΡ‚Π°) Π² сСкунду.

Π’ Ρ‚Π΅Ρ€ΠΌΠΈΠ½Π°Ρ… исчислСния ускорСниС — это вторая производная полоТСния ΠΎΡ‚Π½ΠΎΡΠΈΡ‚Π΅Π»ΡŒΠ½ΠΎ Π²Ρ€Π΅ΠΌΠ΅Π½ΠΈ ΠΈΠ»ΠΈ, Π°Π»ΡŒΡ‚Π΅Ρ€Π½Π°Ρ‚ΠΈΠ²Π½ΠΎ, пСрвая производная скорости ΠΎΡ‚Π½ΠΎΡΠΈΡ‚Π΅Π»ΡŒΠ½ΠΎ Π²Ρ€Π΅ΠΌΠ΅Π½ΠΈ.

УскорСниС — ΠΈΠ·ΠΌΠ΅Π½Π΅Π½ΠΈΠ΅ скорости

ΠšΠ°ΠΆΠ΄Ρ‹ΠΉ дСнь ускорСниС происходит Π² Π°Π²Ρ‚ΠΎΠΌΠΎΠ±ΠΈΠ»Π΅.Π’Ρ‹ Π½Π°ΠΆΠΈΠΌΠ°Π΅Ρ‚Π΅ Π½Π° пСдаль аксСлСратора, ΠΈ Π°Π²Ρ‚ΠΎΠΌΠΎΠ±ΠΈΠ»ΡŒ ускоряСтся ΠΏΠΎ ΠΌΠ΅Ρ€Π΅ Ρ‚ΠΎΠ³ΠΎ, ΠΊΠ°ΠΊ Π΄Π²ΠΈΠ³Π°Ρ‚Π΅Π»ΡŒ ΠΏΡ€ΠΈΠΊΠ»Π°Π΄Ρ‹Π²Π°Π΅Ρ‚ всС Π²ΠΎΠ·Ρ€Π°ΡΡ‚Π°ΡŽΡ‰Π΅Π΅ усилиС ΠΊ трансмиссии. Но Π·Π°ΠΌΠ΅Π΄Π»Π΅Π½ΠΈΠ΅ — это Ρ‚ΠΎΠΆΠ΅ ускорСниС — ΡΠΊΠΎΡ€ΠΎΡΡ‚ΡŒ мСняСтся. Если Π²Ρ‹ ΡƒΠ±Π΅Ρ€Π΅Ρ‚Π΅ Π½ΠΎΠ³Ρƒ с ΠΏΠ΅Π΄Π°Π»ΠΈ аксСлСратора, сила ΠΈ ΡΠΊΠΎΡ€ΠΎΡΡ‚ΡŒ ΡƒΠΌΠ΅Π½ΡŒΡˆΠ°Ρ‚ΡΡ со Π²Ρ€Π΅ΠΌΠ΅Π½Π΅ΠΌ. Как Π²ΠΈΠ΄Π½ΠΎ ΠΈΠ· Ρ€Π΅ΠΊΠ»Π°ΠΌΡ‹, ускорСниС слСдуСт ΠΏΡ€Π°Π²ΠΈΠ»Ρƒ измСнСния скорости (миль Π² час) с Ρ‚Π΅Ρ‡Π΅Π½ΠΈΠ΅ΠΌ Π²Ρ€Π΅ΠΌΠ΅Π½ΠΈ, Π½Π°ΠΏΡ€ΠΈΠΌΠ΅Ρ€, ΠΎΡ‚ нуля Π΄ΠΎ 60 миль Π² час Π·Π° сСмь сСкунд.

Π•Π΄. УскорСния

Π•Π΄ΠΈΠ½ΠΈΡ†Ρ‹ измСрСния ускорСния Π² систСмС БИ: ΠΌ / с 2
(ΠΌΠ΅Ρ‚Ρ€Ρ‹ Π² сСкунду Π² ΠΊΠ²Π°Π΄Ρ€Π°Ρ‚Π΅ ΠΈΠ»ΠΈ ΠΌΠ΅Ρ‚Ρ€Π° Π² сСкунду Π² сСкунду).

Π“Π°Π» ΠΈΠ»ΠΈ Π³Π°Π»ΠΈΠ»Π΅ΠΉ (Gal) — это Π΅Π΄ΠΈΠ½ΠΈΡ†Π° ускорСния, ΠΈΡΠΏΠΎΠ»ΡŒΠ·ΡƒΠ΅ΠΌΠ°Ρ Π² Π³Ρ€Π°Π²ΠΈΠΌΠ΅Ρ‚Ρ€ΠΈΠΈ, Π½ΠΎ Π½Π΅ Π΅Π΄ΠΈΠ½ΠΈΡ†Π° БИ. Он опрСдСляСтся ΠΊΠ°ΠΊ 1 сантимСтр Π½Π° сСкунду Π² ΠΊΠ²Π°Π΄Ρ€Π°Ρ‚Π΅. 1 см / с 2

АнглийскиС Π΅Π΄ΠΈΠ½ΠΈΡ†Ρ‹ измСрСния ускорСния — Ρ„ΡƒΡ‚Ρ‹ Π² сСкунду Π² сСкунду, Ρ„ΡƒΡ‚ / с 2

Π‘Ρ‚Π°Π½Π΄Π°Ρ€Ρ‚Π½ΠΎΠ΅ ускорСниС свободного падСния ΠΈΠ»ΠΈ стандартноС ускорСниС свободного падСния g 0 — это ускорСниС свободного падСния ΠΎΠ±ΡŠΠ΅ΠΊΡ‚Π° Π² Π²Π°ΠΊΡƒΡƒΠΌΠ΅ Ρƒ повСрхности Π·Π΅ΠΌΠ»ΠΈ. Он сочСтаСт Π² сСбС дСйствиС силы тяТСсти ΠΈ Ρ†Π΅Π½Ρ‚Ρ€ΠΎΠ±Π΅ΠΆΠ½ΠΎΠ³ΠΎ ускорСния ΠΎΡ‚ вращСния Π—Π΅ΠΌΠ»ΠΈ.

ΠŸΡ€Π΅ΠΎΠ±Ρ€Π°Π·ΠΎΠ²Π°Π½ΠΈΠ΅ Π΅Π΄ΠΈΠ½ΠΈΡ† ускорСния

Π—Π½Π°Ρ‡Π΅Π½ΠΈΠ΅ ΠΌ / с 2
1 галлон, или см / с 2 0,01
1 Ρ„ΡƒΡ‚ / с 2 0,304800
1 Π³ 0 9,80665

Π’Ρ‚ΠΎΡ€ΠΎΠΉ Π·Π°ΠΊΠΎΠ½ ΠΡŒΡŽΡ‚ΠΎΠ½Π° — расчСт ускорСния

Π£Ρ€Π°Π²Π½Π΅Π½ΠΈΠ΅ классичСской ΠΌΠ΅Ρ…Π°Π½ΠΈΠΊΠΈ для ускорСния происходит ΠΈΠ· Π’Ρ‚ΠΎΡ€ΠΎΠ³ΠΎ Π·Π°ΠΊΠΎΠ½Π° ΠΡŒΡŽΡ‚ΠΎΠ½Π°: сумма сил ( F ) Π½Π° ΠΎΠ±ΡŠΠ΅ΠΊΡ‚Π΅ постоянной массы ( ΠΌ ) Ρ€Π°Π²Π½Π° массС ΠΌ , ΡƒΠΌΠ½ΠΎΠΆΠ΅Π½Π½ΠΎΠΉ Π½Π° ускорСниС ΠΎΠ±ΡŠΠ΅ΠΊΡ‚Π° ( a ). .

F = a ΠΌ

Π‘Π»Π΅Π΄ΠΎΠ²Π°Ρ‚Π΅Π»ΡŒΠ½ΠΎ, это ΠΌΠΎΠΆΠ½ΠΎ ΠΈΠ·ΠΌΠ΅Π½ΠΈΡ‚ΡŒ, Ρ‡Ρ‚ΠΎΠ±Ρ‹ ΠΎΠΏΡ€Π΅Π΄Π΅Π»ΠΈΡ‚ΡŒ ускорСниС ΠΊΠ°ΠΊ:

a = F / ΠΌ

Π Π΅Π·ΡƒΠ»ΡŒΡ‚Π°Ρ‚ΠΎΠΌ этого уравнСния являСтся Ρ‚ΠΎ, Ρ‡Ρ‚ΠΎ Ссли Π½Π° ΠΎΠ±ΡŠΠ΅ΠΊΡ‚ Π½Π΅ Π΄Π΅ΠΉΡΡ‚Π²ΡƒΡŽΡ‚ силы ( F = 0), ΠΎΠ½ Π½Π΅ Π±ΡƒΠ΄Π΅Ρ‚ ΡƒΡΠΊΠΎΡ€ΡΡ‚ΡŒΡΡ. Π•Π³ΠΎ ΡΠΊΠΎΡ€ΠΎΡΡ‚ΡŒ останСтся постоянной. Если ΠΊ ΠΎΠ±ΡŠΠ΅ΠΊΡ‚Ρƒ Π΄ΠΎΠ±Π°Π²ΠΈΡ‚ΡŒ массу, ускорСниС Π±ΡƒΠ΄Π΅Ρ‚ мСньшС. Если ΡƒΠ±Ρ€Π°Ρ‚ΡŒ массу с ΠΎΠ±ΡŠΠ΅ΠΊΡ‚Π°, Π΅Π³ΠΎ ускорСниС Π±ΡƒΠ΄Π΅Ρ‚ Π²Ρ‹ΡˆΠ΅.

Π’Ρ‚ΠΎΡ€ΠΎΠΉ Π·Π°ΠΊΠΎΠ½ ΠΡŒΡŽΡ‚ΠΎΠ½Π° — ΠΎΠ΄ΠΈΠ½ ΠΈΠ· Ρ‚Ρ€Π΅Ρ… Π·Π°ΠΊΠΎΠ½ΠΎΠ² двиТСния, ΠΎΠΏΡƒΠ±Π»ΠΈΠΊΠΎΠ²Π°Π½Π½Ρ‹Ρ… Исааком ΠΡŒΡŽΡ‚ΠΎΠ½ΠΎΠΌ Π² 1687 Π³ΠΎΠ΄Ρƒ Π² ΠΊΠ½ΠΈΠ³Π΅ PhilosophiΓ¦ Naturalis Principia Mathematica ( Mathematical Principles of Natural Philosophy ).

УскорСниС ΠΈ ΠΎΡ‚Π½ΠΎΡΠΈΡ‚Π΅Π»ΡŒΠ½ΠΎΡΡ‚ΡŒ

Π₯отя Π·Π°ΠΊΠΎΠ½Ρ‹ двиТСния ΠΡŒΡŽΡ‚ΠΎΠ½Π° ΠΏΡ€ΠΈΠΌΠ΅Π½ΡΡŽΡ‚ΡΡ Π½Π° скоростях, с ΠΊΠΎΡ‚ΠΎΡ€Ρ‹ΠΌΠΈ ΠΌΡ‹ сталкиваСмся Π² повсСднСвной ΠΆΠΈΠ·Π½ΠΈ, ΠΊΠ°ΠΊ Ρ‚ΠΎΠ»ΡŒΠΊΠΎ ΠΎΠ±ΡŠΠ΅ΠΊΡ‚Ρ‹ двиТутся со ΡΠΊΠΎΡ€ΠΎΡΡ‚ΡŒΡŽ, Π±Π»ΠΈΠ·ΠΊΠΎΠΉ ΠΊ скорости свСта, ΠΏΡ€Π°Π²ΠΈΠ»Π° ΠΌΠ΅Π½ΡΡŽΡ‚ΡΡ. ИмСнно Ρ‚ΠΎΠ³Π΄Π° ΡΠΏΠ΅Ρ†ΠΈΠ°Π»ΡŒΠ½Π°Ρ тСория ΠΎΡ‚Π½ΠΎΡΠΈΡ‚Π΅Π»ΡŒΠ½ΠΎΡΡ‚ΠΈ Π­ΠΉΠ½ΡˆΡ‚Π΅ΠΉΠ½Π° Π±ΠΎΠ»Π΅Π΅ Ρ‚ΠΎΡ‡Π½Π°.

Π”ΠΎΠ±Π°Π²ΠΈΡ‚ΡŒ ΠΊΠΎΠΌΠΌΠ΅Π½Ρ‚Π°Ρ€ΠΈΠΉ

Π’Π°Ρˆ адрСс email Π½Π΅ Π±ΡƒΠ΄Π΅Ρ‚ ΠΎΠΏΡƒΠ±Π»ΠΈΠΊΠΎΠ²Π°Π½. ΠžΠ±ΡΠ·Π°Ρ‚Π΅Π»ΡŒΠ½Ρ‹Π΅ поля ΠΏΠΎΠΌΠ΅Ρ‡Π΅Π½Ρ‹ *